Psma Finals

Download as pdf or txt
Download as pdf or txt
You are on page 1of 1002

OUR LADY OF FATIMA UNIVERSITY

College of Pharmacy

PHARMACEUTICAL
SEMINAR 1 (PSMA411)
Week 16-17 – Pharmaceutical Calculations and Techniques
Student Checklist:
 Read course and unit outline
 Read required learning materials
 Proactively participate in online class
 Participate in the Discussion Board
(Canvas)
 Answer and Submit course unit
tasks
UNIT OUTCOMES

At the end of the course, the students are expected to:


1) Demonstrate knowledge, understanding and skills to perform
basic mathematical operations and solve various problems
2) Perform laboratory techniques pertinent to the practice of
pharmacy with accuracy and precision;
3) Work independently and diligently manifesting the values of
analytical thinking, scientific discipline, honesty and patience.
UNIT OUTLINE

I. International System of Units


II. Pharmaceutical Measurement
III. Reducing and Enlarging of Formulas
IV. Density, Specific Gravity
V. Calculation of Doses: Patient Parameters
VI. Percentage, Ratio Strength and other Expressions of
Concentrations
UNIT OUTLINE

VII. Dilution, Concentration and Alligation


VIII. Electrolyte solution: Milliequivalents, Millimoles, and
Milliosmoles
IX. Isotonic Solutions
X. HLB System
XI. Pharmacoeconomic Calculations
• International System of Units
• Also known as the Metric System
• is the internationally recognized decimal
system of weights and measures.
UNITS OF MEASUREMENT USED IN
PHARMACY

Metric System
• Is based on decimal system in which everything is measured in
multiples or fraction of 10.
• Major system of weights and measurement used in medicine.
• Uses standard measures:
• Meter (m) for length or distance
• Gram (g) for mass or weight
• Liter (L) for volume
MEASURE OF LENGTH

• meter (m) – primary unit of length


• Table of metric length:
1 kilometer (km) = 1,000,000 meters 1 meter = 0.001 kilometer
1 hectometer (hm) = 100,000 meters 0.01 hectometer
1 decameter (dam) = 10,000 meters 0.1 decameter
10 decimeters
1 meter (m)
100 centimeters
1 decimeter (dm) = 0.100 meter 1000 millimeters
1 centimeter (cm) = 0.010 meter 1,000,000 micrometers
1 millimeter (mm) = 0.001 meter 1,000,000,000 nanometers
1 micrometer (mm) = 0.000,001 meter
1 nanometer (nm) = 0.000,000,001 meter
MEASURE OF VOLUME

• Liter (L) – primary unit of volume


• Table of metric volume
1 kiloliter (kL) = 1000,000 liters 1 liter = 0.001 kiloliter
1 hectoliter (hL) = 100,000 liters 0.010 hectoliter
0.100 decaliter
1 decaliter (daL) = 10,000 liters
10 deciliters
1 liter (L) 100 centiliters
1 deciliter (dL) = 0.100 liter 1000 milliliters
1 centiliter (cL) = 0.010 liter 1,000,000 microliters
1 milliliter (mL) = 0.001 liter
1 microliter (mL) = 0.000,001 liter
MEASURE OF WEIGHT

• Gram (g) – primary unit of weight.


• Table of metric weight
1 kilogram (kg) = 1,000,000 grams 1 gram = 0.001 kilogram
1 hectogram (hg) = 100,000 grams
0.010 hectogram
0.100 decagram
1 dekagram (dag) = 10,000 grams
10 decigrams
1 gram (g) 100 centigrams
1 decigram (dg) = 0.100 gram 1000 milligrams
1 centigram (cg) = 0.010 gram 1,000,000 micrograms
1 milligram (mg) = 0.001 gram 1,000,000,000 nanograms
1 microgram (mg or mcg) = 0.000,001 gram
1,000,000,000,000 picograms
1,000,000,000,000,000 femtograms
1 nanogram (ng) = 0.000,000,001 gram
1 picogram (pg) = 0.000,000,000,001 gram
1 femtogram (fg) = 0.000,000,000,000,001 gram
Equivalents of length
1 inch = 2.54 cm
1 meter (m) = 39.37 in
Equivalents of volume
1 fluid ounce (fl. oz.) = 29.57 mL

Some useful 1 pint (16 fl. oz.) = 473 mL


1 quart (32 fl. oz.) = 946 mL
equivalents
1 gallon, US (128 fl. oz.) = 3785 mL
1 gallon, UK = 4545 mL
Equivalents of weight
1 pound (lb, avoirdupois) = 454 g
1 ounce (oz, avoirdupois) = 28.35 g
1 kilogram (kg) = 2.2 lb
POP QUIZ:

1. A low-strength aspirin tablet contains 81 mg of aspirin per


tablet. How many tablets may a manufacturer prepare from
0.5 kg of aspirin?
2. An intravenous solution contains 500 mg of a drug
substance in each milliliter. How many milligrams of the
drug would a patient receive from the intravenous infusion
of a liter of the solution?
Answer:
1. A low-strength aspirin tablet contains 81 mg of aspirin per tablet.
How many tablets may a manufacturer prepare from 0.5 kg of
aspirin?

81𝑚𝑔 0.5𝐾𝑔
=
1 𝑡𝑎𝑏𝑙𝑒𝑡 𝑋
1000 𝑔 1000 𝑚𝑔
0.5 𝐾𝑔 𝑥 1𝐾𝑔
𝑥 1𝑔
= 500,000 𝑚𝑔
81𝑚𝑔 500,000 𝑚𝑔
= 𝑋 = 6,172.83 𝑡𝑎𝑏𝑙𝑒𝑡𝑠
1 𝑡𝑎𝑏𝑙𝑒𝑡 𝑋
or 6,172 𝑡𝑎𝑏𝑙𝑒𝑡𝑠
(81 𝑚𝑔)(𝑋) = (500,000 𝑚𝑔)(1 𝑡𝑎𝑏𝑙𝑒𝑡)
81 𝑚𝑔 81 𝑚𝑔
Answer:
2. An intravenous solution contains 500 mg of a drug substance in
each milliliter. How many milligrams of the drug would a patient
receive from the intravenous infusion of a liter of the solution?

500𝑚𝑔 𝑋
=
1 𝑚𝐿 1 𝐿𝑖𝑡𝑒𝑟
1000 𝑚𝑙
1𝐿𝑥 = 1,000 𝑚𝐿
1𝐿
500𝑚𝑔 𝑋
= 𝑋 = 500,000 mg
1 𝑚𝐿 1,000 𝑚𝐿
(500 𝑚𝑔)(1,000 𝑚𝐿) (𝑋)(1 𝑚𝐿)
=
1 𝑚𝐿 1 𝑚𝐿
PHARMACEUTICAL
MEASUREMENT

• employed in community and institutional pharmacies, in


pharmaceutical research, in the development and manufacture
of pharmaceuticals, in chemical and product analysis, and in
quality control.
Weights and measures
The measurement systems in place for pharmacy are:
Metric
Avoirdupois
Apothecary

• International System of Units (SI) – commonly referred to as the


metric system.
• Avoirdupois – the common system of commerce.
• Apothecaries – the traditional system of pharmacy.
• the traditional system of pharmacy
• Developed in England in 18th century
• components of this system are
occasionally found on prescriptions.
• First used by apothecaries/ early
Apothecary pharmacist and moved from Europe
system to colonial America.
• Household system evolved from the
apothecary system
• Older medications are still measured
in apothecary units
Apothecary System

Units of weight: Units of volume:


• Grain • Minims
• Dram • Fluidounce
• Ounce • Pint
• Pound • Quart
• Gallon
Apothecary system

Measure of weight:
20 grains = 1 scruple
3 scruple (60 grains) = 1 drachm or dram
8 drachms (480 grains) = 1 ounce
12 ounces (5760 grains) = 1 pound
Apothecary System

Measure of Volume:
60 minims = 1 fluidrachm
8 fluidrachm (480 minims) = 1 fluidounce
16 fluidounces = 1 pint
2 pints (32 fluidounces) = 1 quart
4 quarts (8 pints) = 1 gallon (gal)
• Originated in Europe
• Common system of
Commerce
AVOIRDUPOIS • Mainly used in measuring bulk
SYSTEM medication encountered in
manufacturing.
• Commonly used to measure
weight
AVOIRDUPOIS SYSTEM

Units of measure:
• Grains
• Ounce
• Pounds
AVOIRDUPOIS SYSTEM

Measure of weight:
437.5 grains = 1 ounce
16 ounces (7000 grains) = 1 pound
• To convert a given weight or
INTERSYSTEM volume from units of one system
CONVERSION to equivalent units of another
system.
INTERSYSTEM CONVERSION

Measure of volume: Measure of length:


1 mL = 1 minims 1m = 39.37 in
1 minims = 0.06 mL
1 inch = 2.54 cm
1 fl dram = 3.69 mL
1 fl oz = 29.57 mL
1 pt = 473 mL
1 gal (US) = 3785 mL
INTERSYSTEM CONVERSION

Measure of weight: Other equivalents:


1g = 14.432 gr
1 oz (avoir) = 437.5 gr
1 Kg = 2.2 lb (avoir.)
1 gr = 0.065 g (65 mg) 1 oz (apoth) = 480 gr
1 oz (avoir) = 28.35 g 1 gal (US) = 128 fl oz.
1 oz (apoth)= 31.1 g
1 lb (avoir) = 454 g
1 lb (apoth) = 373 g
Common household system

• 1 teaspoon = 5 mL
• 1 tablespoon = 15 mL
• 1 teaspoon = 60 drops
• 1 cup = 240 mL
• 2 tablespoon = 1 fluid ounce
• 8 fluid ounces = 1 cup
• 2 cups = 1 pint
• 4 quarts = 1 gallon
Other Household Measurement
POP QUIZ

1. If a child accidentally swallowed 2 fluidounces of FEOSOL


Elixir, containing 2/3 gr of ferrous sulfate per 5 mL, how
many milligrams of ferrous sulfate did the child ingest?
2. A formula for a cough syrup contains 1/8 gr of codeine
phosphate per teaspoonful (5 mL). How many grams of
codeine phosphate should be used in preparing 1 pint of
the cough syrup?
1. If a child accidentally swallowed 2 fluidounces of FEOSOL Elixir,
containing 2/3 gr of ferrous sulfate per 5 mL, how many milligrams of
ferrous sulfate did the child ingest?
2 𝑋 𝑚𝑔
𝑔𝑟
3 =
5 𝑚𝐿 2 𝑓𝑙 𝑜𝑧
29.57 𝑚𝐿
2𝑓𝑙 𝑜𝑧 𝑥 1𝑓𝑙 𝑜𝑧
= 59.14 𝑚𝐿 𝑋 = 7.8853 𝑔𝑟𝑎𝑖𝑛𝑠
0.6667 𝑔𝑟 𝑋 65𝑚𝑔
= 7.8853 𝑔𝑟𝑎𝑖𝑛𝑠 𝑥 1𝑔𝑟𝑎𝑖𝑛
5 𝑚𝐿 59.14 𝑚𝐿
(0.6667𝑔𝑟)(59.14𝑚𝐿)
= (𝑋)(5𝑚𝐿) = 512.55 𝑚𝑔
5 𝑚𝐿 5 𝑚𝐿
2. A formula for a cough syrup contains 1/8 gr of codeine phosphate
per teaspoonful (5 mL). How many grams of codeine phosphate
should be used in preparing 1 pint of the cough syrup?
1 𝑋𝑔
𝑔𝑟
8 =
5 𝑚𝐿 1 𝑝𝑡
473 𝑚𝐿
1𝑝𝑡 𝑥 1𝑝𝑡
= 473 𝑚𝐿 𝑋 = 11.825 𝑔𝑟𝑎𝑖𝑛𝑠
0.125 𝑔𝑟 𝑋 0.065𝑔
= 11.825 𝑔𝑟𝑎𝑖𝑛𝑠 𝑥
5 𝑚𝐿 473 𝑚𝐿 1𝑔𝑟𝑎𝑖𝑛

(0.125𝑔𝑟)(473𝑚𝐿)
= (𝑋)(5𝑚𝐿) = 0.77𝑔
5 𝑚𝐿 5 𝑚𝐿
ALIQUOT METHOD OF
WEIGHING AND MEASURING
ALIQUOT METHOD
A method by which small quantities of a
substance may be obtained within the
desired degree of accuracy by weighing a
larger-than-needed portion of the
substance, diluting it with an inert material,
and then weighing a portion (aliquot) of the
mixture calculated to contain the desired
amount of the needed substance.
ALIQUOT PROCEDURE

Preliminary Step.
Calculate the smallest quantity of a substance that can be weighed on the
balance with the desired precision.
Using the equation:
100% 𝑥 𝑆𝑒𝑛𝑠𝑖𝑡𝑖𝑣𝑖𝑡𝑦 𝑟𝑒𝑞𝑢𝑖𝑟𝑒𝑚𝑒𝑛𝑡𝑠 𝑚𝑔
= Smallest Quantity (mg)
𝐴𝑐𝑐𝑒𝑝𝑡𝑎𝑏𝑙𝑒 𝑒𝑟𝑟𝑜𝑟 (%)
ALIQUOT METHOD

• PROCEDURE
1. Select a multiple of the desired quantity that can be weighed/measured
with the required precision.
2. Dilute the multiple quantity with an inert substance/diluent
3. Weigh/Measure the aliquot portion of the dilution that contains the
desired quantity.
EXAMPLE

On a balance with an SR of 6 mg, and with an acceptable error of no


greater than 5%,
100% 𝑥 𝑆𝑒𝑛𝑠𝑖𝑡𝑖𝑣𝑖𝑡𝑦 𝑟𝑒𝑞𝑢𝑖𝑟𝑒𝑚𝑒𝑛𝑡
= Smallest quantity (mg)
𝐴𝑐𝑐𝑒𝑝𝑡𝑎𝑏𝑙𝑒 𝐸𝑟𝑟𝑜𝑟 (%)
100% 𝑥 6𝑚𝑔
= 120 mg
5%

 a quantity of not less than 120 mg must be weighed.


1. Select a multiple of the desired quantity that can be
weighed with the required precision

On a balance with an SR of 6 mg, and with an acceptable error


of no greater than 5%,

If 5mg required amount on a prescription

120 𝑚𝑔
= 24 or at le𝑎𝑠𝑡 25 (𝑡𝑕𝑒 𝑚𝑢𝑙𝑡𝑖𝑝𝑙𝑒)
5 𝑚𝑔
Required amount x multiple = Desired amount
5mg x 25 = 125 mg
2. Dilute the multiple quantity with an inert substance.

Smallest Quantity x Multiple selected = Total quantity of the drug-diluent


120𝑚𝑔 𝑥 25 = 3000 𝑚𝑔

Total quantity of drug-diluent - Amount of drug = Amount of diluent

3000 𝑚𝑔 − 125 𝑚𝑔 = 2875 𝑚𝑔


3. Weigh the aliquot portion of the dilution that contains
the desired quantity.

Acceptable error x Desired quantity of drug = Amount of drug


0.04 x 125 mg = 5 mg

Acceptable error x Desired quantity of diluent = Amount of diluent


0.04 x 2875 mg = 115 mg

Amount of Drug + Amount of Diluent = Aliquot Part


5 mg + 115 mg = 120 mg
PERCENTAGE
ERROR
PERCENTAGE OF ERROR

• As the maximum potential error multiplied by 100 and divided by the quantity
desired.
• The difference between an experimental and theoretical value, divided by the
theoretical value, multiplied by 100 to give a percent.
• Always expressed as a positive number.
• Equation is:

𝐸𝑟𝑟𝑜𝑟 𝑥 100
Percentage error =
𝑄𝑢𝑎𝑛𝑡𝑖𝑡𝑦 𝑑𝑒𝑠𝑖𝑟𝑒𝑑
Example:

Using a graduated cylinder, a pharmacist measured 30 mL of a liquid. On


subsequent examination, using a narrow-gauge burette, it was
determined that the pharmacist had actually measured 32 mL. What was
the percentage of error in the original measurement?
V𝑜𝑙𝑢𝑚𝑒 𝑜𝑓 𝑒𝑟𝑟𝑜𝑟 = 32 𝑚𝐿 − 30 𝑚𝐿 = 2 𝑚𝐿
𝐸𝑟𝑟𝑜𝑟 𝑥 100
Percentage error =
𝑄𝑢𝑎𝑛𝑡𝑖𝑡𝑦 𝑑𝑒𝑠𝑖𝑟𝑒𝑑

2𝑚𝐿 𝑥 100%
Percentage error = = 6.7%
30𝑚𝐿
Example:

A pharmacist attempts to weigh 0.375 g of morphine sulfate on a balance of


dubious accuracy. When checked on a highly accurate balance, the weight is
found to be 0.400 g. Calculate the percentage of error in the first weighing.

𝑊𝑒𝑖𝑔𝑕𝑡 𝑜𝑓 𝑒𝑟𝑟𝑜𝑟 = 0.400𝑔 − 0.375𝑔 = 0.025 𝑔


𝐸𝑟𝑟𝑜𝑟 𝑥 100
Percentage error =
𝑄𝑢𝑎𝑛𝑡𝑖𝑡𝑦 𝑑𝑒𝑠𝑖𝑟𝑒𝑑

0.025𝑔 𝑥 100%
Percentage error = = 6.67%
0.375
REDUCING AND ENLARGING
OF FORMULAS

• Determine the total weight or volume of


ingredients and convert, if necessary, to
the quantities desired. The quantities in
the original and new formulas will have
the same ratio.
REDUCING AND ENLARGING OF FORMULAS

• Reducing Calculating the amounts to be used in a


pharmaceutical formula to make a smaller amount than the
original formula.
• Enlarging Calculating the proper amounts to be used in a
pharmaceutical formula to make a larger amount than the
original formula.
Ratio and Proportion
METHODS TO
REDUCE OR
ENLARGE Dimensional analysis
FORMULA
Factor Method
Factor Method

• Is based on the relative quantity of the total formula to be


prepared.
Equation used:

𝑄𝑢𝑎𝑛𝑡𝑖𝑡𝑦 𝑜𝑓 𝑓𝑜𝑟𝑚𝑢𝑙𝑎 𝑑𝑒𝑠𝑖𝑟𝑒𝑑


Factor =
𝑄𝑢𝑎𝑛𝑡𝑖𝑡𝑦 𝑜𝑓 𝑓𝑜𝑟𝑚𝑢𝑙𝑎 𝑔𝑖𝑣𝑒𝑛
Steps in Factor Method
1. Using the following equation, determine the factor that defines the multiple
or the decimal fraction of the amount of formula to be prepared:
𝑄𝑢𝑎𝑛𝑡𝑖𝑡𝑦 𝑜𝑓 𝑓𝑜𝑟𝑚𝑢𝑙𝑎 𝑑𝑒𝑠𝑖𝑟𝑒𝑑
Factor = 𝑄𝑢𝑎𝑛𝑡𝑖𝑡𝑦 𝑜𝑓 𝑓𝑜𝑟𝑚𝑢𝑙𝑎 𝑔𝑖𝑣𝑒𝑛

2. Multiply the quantity of each ingredient in the formula by the factor to


determine the amount of each ingredient required in the reduced or enlarged
formula.
Example of factor method
• If a formula for 1000 mL contains 6 g of a drug, how many grams
of drug are needed to prepare 60 mL of the formula?
Step 1 Step 2
𝑄𝑢𝑎𝑛𝑡𝑖𝑡𝑦 𝑜𝑓 𝑓𝑜𝑟𝑚𝑢𝑙𝑎 𝑑𝑒𝑠𝑖𝑟𝑒𝑑
Factor = 𝑄𝑢𝑎𝑛𝑡𝑖𝑡𝑦 𝑜𝑓 𝑓𝑜𝑟𝑚𝑢𝑙𝑎 𝑔𝑖𝑣𝑒𝑛 6 g x 0.06 = 0.36g
60 𝑚𝐿
Factor =
1000 𝑚𝐿

Factor = 0.06
POP QUIZ
1. Calculate the quantity of each ingredient required to make 240 mL of
calamine lotion.
Calamine 80 g
Zinc oxide 80 g
Glycerin 20 mL
Bentonite magma 250 mL
Calcium hydroxide,
to make 1000 mL
2. Calculate the quantity of each ingredient required to prepare
a dozen 30-mL containers.
Polyvinyl alcohol 1.4 g
Povidone 0.6 g
Chlorobutanol 0.5 g
Sterile sodium chloride,
solution 0.9%, ad 100 mL
• Using the following methods:
• Factor method
• Ratio and proportion
• Dimensional analysis
Factor Method Factor =
𝑄𝑢𝑎𝑛𝑡𝑖𝑡𝑦 𝑜𝑓 𝑓𝑜𝑟𝑚𝑢𝑙𝑎 𝑑𝑒𝑠𝑖𝑟𝑒𝑑
𝑄𝑢𝑎𝑛𝑡𝑖𝑡𝑦 𝑜𝑓 𝑓𝑜𝑟𝑚𝑢𝑙𝑎 𝑔𝑖𝑣𝑒𝑛
1. Calculate the quantity of each ingredient 240 𝑚𝐿
required to make 240 mL of calamine
Factor =
1000 𝑚𝐿
lotion.
Factor = 0.24
Calamine 80 g
Zinc oxide 80 g Calamine 80 g x 0.24 = 19.20 g
Glycerin 20 mL
Zinc oxide 80 g x 0.24 = 19.20 g
Bentonite magma 250 mL
Glycerin 20 mL x 0.24 = 4.80 mL
Calcium hydroxide,
Bentonite magma 250 mL x 0.24 = 60 mL
to make 1000 mL
Calcium hydroxide,
to make 1000 mL x 0.24 = 240 mL
Calamine
Ratio and proportion 80 𝑔
1000 𝑚𝐿 =
𝑥
240 𝑚𝐿
Calculate the quantity of each ingredient
required to make 240 mL of calamine lotion. (80 g) (240 mL) = (x) (1000 mL)
(1000 mL) (1000 mL)
Calamine 80 g
x = 19.20 g
Zinc oxide 80 g Zinc oxide
80 𝑔 𝑥
Glycerin 20 g =
1000 𝑚𝐿 240 𝑚𝐿
Bentonite magma 250 mL (80 g) (240 mL) = (x) (1000 mL)
Calcium hydroxide, (1000 mL) (1000 mL)
x = 19.20 g
to make 1000 mL
Glycerin
20 𝑚𝐿 𝑥
=
1000 𝑚𝐿 240 𝑚𝐿
(20 mL) (240 mL) (x) (1000 mL)
=
(1000 mL) (1000 mL)
x = 4.80 mL
Bentonite magma
Ratio and proportion 250 𝑚𝐿
1000 𝑚𝐿 = 240 𝑚𝐿
𝑥

Calculate the quantity of each ingredient


required to make 240 mL of calamine lotion. (250 mL) (240 mL) = (x) (1000 mL)
(1000 mL) (1000 mL)
Calamine 80 g
x = 60 mL
Zinc oxide 80 g Calcium hydroxide
Glycerin 20 mL 1000 𝑚𝐿 𝑥
1000 𝑚𝐿 = 240 𝑚𝐿
Bentonite magma 250 mL
(1000ml) (240 mL) (x) (1000 mL)
Calcium hydroxide, =
(1000 mL) (1000 mL)
to make 1000 mL
x = 240 mL
Calamine
Dimensional analysis 80 𝑔
𝑥 240 𝑚𝐿 = 19.20 𝑔
1000 𝑚𝐿
Calculate the quantity of each ingredient
required to make 240 mL of calamine lotion. Zinc oxide
80 𝑔
Calamine 80 g 𝑥 240 𝑚𝐿 = 19.20 𝑔
1000 𝑚𝐿
Zinc oxide 80 g
Glycerin
Glycerin 20 mL 20 𝑚𝐿
𝑥 240 𝑚𝐿 = 4.80 mL
Bentonite magma 250 mL 1000 𝑚𝐿
Calcium hydroxide, Bentonite magma
to make 1000 mL 250 𝑚𝐿
𝑥 240 𝑚𝐿 = 60 mL
1000 𝑚𝐿
Calcium hydroxide
1000 𝑚𝐿
𝑥 240 𝑚𝐿 = 240 mL
1000 𝑚𝐿
2. Calculate the quantity of each ingredient
required to prepare a dozen 30-mL containers.
𝑄𝑢𝑎𝑛𝑡𝑖𝑡𝑦 𝑜𝑓 𝑓𝑜𝑟𝑚𝑢𝑙𝑎 𝑑𝑒𝑠𝑖𝑟𝑒𝑑 = 12 X 30 mL = 360 mL

Polyvinyl alcohol 1.4 g 𝑄𝑢𝑎𝑛𝑡𝑖𝑡𝑦 𝑜𝑓 𝑓𝑜𝑟𝑚𝑢𝑙𝑎 𝑔𝑖𝑣𝑒𝑛 = 100 mL


360 𝑚𝐿
Povidone 0.6 g Factor = 100 𝑚𝐿
Chlorobutanol 0.5 g Factor = 3.6
Sterile sodium chloride,
solution 0.9%, ad 100 mL

𝑄𝑢𝑎𝑛𝑡𝑖𝑡𝑦 𝑜𝑓 𝑓𝑜𝑟𝑚𝑢𝑙𝑎 𝑑𝑒𝑠𝑖𝑟𝑒𝑑


Factor = 𝑄𝑢𝑎𝑛𝑡𝑖𝑡𝑦 𝑜𝑓 𝑓𝑜𝑟𝑚𝑢𝑙𝑎 𝑔𝑖𝑣𝑒𝑛
• Equation:
𝑀𝑎𝑠𝑠
• 𝐷𝑒𝑛𝑠𝑖𝑡𝑦 = 𝑉𝑜𝑙𝑢𝑚𝑒
Density • Mass = grams (g)
• Volume = cubic centimeter (cc) or
millimeter (mL)
Example:

• If 10 mL of sulfuric acid weighs 18 grams, what is its density?


𝑀𝑎𝑠𝑠
𝐷𝑒𝑛𝑠𝑖𝑡𝑦 =
𝑉𝑜𝑙𝑢𝑚𝑒

18 𝑔
𝐷𝑒𝑛𝑠𝑖𝑡𝑦 =
10 𝑚𝐿

𝐷𝑒𝑛𝑠𝑖𝑡𝑦 = 1.8 g/mL


Example:

• If 250 mL of alcohol weighs 203 g, what is its density?


𝑀𝑎𝑠𝑠
𝐷𝑒𝑛𝑠𝑖𝑡𝑦 =
𝑉𝑜𝑙𝑢𝑚𝑒

203 𝑔
𝐷𝑒𝑛𝑠𝑖𝑡𝑦 =
250 𝑚𝐿

𝐷𝑒𝑛𝑠𝑖𝑡𝑦 = 0.812 g/mL


• - a ratio of the weight of a substance to the
weight of an equal volume of a substance
chosen as a standard, (both substances having
the same temperature or the temperature of
Specific each being definitely known.)
𝑊𝑒𝑖𝑔𝑕𝑡 𝑜𝑓 𝑠𝑢𝑏𝑠𝑡𝑎𝑛𝑐𝑒
Gravity • 𝑆𝑝𝑒𝑐𝑖𝑓𝑖𝑐 𝑔𝑟𝑎𝑣𝑖𝑡𝑦 = 𝑊𝑒𝑖𝑔𝑕𝑡 𝑜𝑓 𝑒𝑞𝑢𝑎𝑙 𝑣𝑜𝑙𝑢𝑚𝑒 𝑜𝑓 𝑤𝑎𝑡𝑒𝑟

• *USP standard for specific gravities is 25 º C


except for ALCOHOL 15.6º C
Specific gravity

• Substances that have sp. Gr. less than 1 are lighter


than water
• Substances that have sp. gr. Greater than 1 are
heavier than water
Example: (Using pycnometer)
A 50 mL pycnometer is found to weigh 120 g when empty, 171 g when filled with
water; and 160 g when filled with an unknown liquid. Calculate the specific gravity of
the unknown liquid. 40 𝑔
𝑆𝑝𝑒𝑐𝑖𝑓𝑖𝑐 𝑔𝑟𝑎𝑣𝑖𝑡𝑦 =
𝑊𝑒𝑖𝑔𝑕𝑡 𝑜𝑓 𝑠𝑢𝑏𝑠𝑡𝑎𝑛𝑐𝑒 𝑆𝑝𝑒𝑐𝑖𝑓𝑖𝑐 𝑔𝑟𝑎𝑣𝑖𝑡𝑦 =
𝑊𝑒𝑖𝑔𝑕𝑡 𝑜𝑓 𝑒𝑞𝑢𝑎𝑙 𝑣𝑜𝑙𝑢𝑚𝑒 𝑜𝑓 𝑤𝑎𝑡𝑒𝑟 51 𝑔
Weight of pycnometer = 120 g 𝑆𝑝𝑒𝑐𝑖𝑓𝑖𝑐 𝑔𝑟𝑎𝑣𝑖𝑡𝑦 = 0.78
Weight of unknown liquid = Weight of pycnometer with unknown liquid – weight of pycnometer
Weight of unknown liquid = 160 g – 120 g
Weight of unknown liquid = 40 g
Weight of water = Weight of pycnometer with water – weight of pycnometer
Weight of water = 171 g – 120 g
Weight of water = 51 g
Example: (Using pycnometer)
A specific gravity bottle weighs 23.66 g. When filled with water, it weighs 72.95 g; when
filled with another liquid, it weighs 73.56 g. What is the specific gravity of the liquid?
𝑊𝑒𝑖𝑔𝑕𝑡 𝑜𝑓 𝑠𝑢𝑏𝑠𝑡𝑎𝑛𝑐𝑒 49.90 𝑔
𝑆𝑝𝑒𝑐𝑖𝑓𝑖𝑐 𝑔𝑟𝑎𝑣𝑖𝑡𝑦 =
𝑊𝑒𝑖𝑔𝑕𝑡 𝑜𝑓 𝑒𝑞𝑢𝑎𝑙 𝑣𝑜𝑙𝑢𝑚𝑒 𝑜𝑓 𝑤𝑎𝑡𝑒𝑟
𝑆𝑝𝑒𝑐𝑖𝑓𝑖𝑐 𝑔𝑟𝑎𝑣𝑖𝑡𝑦 =
49.29 𝑔
Weight of pycnometer = 23.66 g 𝑆𝑝𝑒𝑐𝑖𝑓𝑖𝑐 𝑔𝑟𝑎𝑣𝑖𝑡𝑦 = 1.01
Weight of liquid = Weight of pycnometer with liquid – weight of pycnometer
Weight of unknown liquid = 73.56 g – 23.66 g
Weight of unknown liquid = 49.90 g
Weight of water = Weight of pycnometer with water – weight of pycnometer
Weight of water = 72.95 g – 23.66 g
Weight of water = 49.29 g
Density is a concrete number (1.8 g/mL)
Difference
Density of water can be expressed as
between 1g/mL, 1000 g/L, 455 gr/floz, 62 ½ lb /cu ft
Density and
Specific gravity is an abstract number
Specific (dimensionless)
Gravity
Specific gravity of water is always 1.
• Specific gravity a factor that expresses
Use of how much heavier or lighter a substance
Specific is than water, the standard with a
specific gravity of 1.0.
Gravity in • For example,
Calculations • A liquid with a specific gravity o 1.25 is
of Weight and 1.25 times as heavy as water, and a
liquid with a specif c gravity of 0.85 is
Volume 0.85 times as heavy as water.
Calculating Weight, Knowing the Volume
and Specific Gravity

• Equation:
• Weight of substance = volume of substance x Specific gravity
• Grams = Milliliters × Specific gravity
• Grams other liquid = Grams (of equal volume of water) x Specific gravity (of other
liquid)
Example:

• What is the weight, in grams, of 3620 mL of alcohol with a specific


gravity of 0.82?
Weight of substance = volume of substance x Specific gravity

Volume of substance = 3620 mL alcohol  volume of water = 3620 mL = 3620 g


Specific gravity = 0.82
Weight of substance = 3620 g x
0.82
Weight of substance = 2968.40 g
Example:
• What is the weight, in grams, of 2 fl. oz. of a liquid having a specific
gravity of 1.118?
29.57 𝑚𝐿
2 fl.oz. x = 59.14 mL
1 𝑓𝑙.𝑜𝑧.
Weight of substance = volume of substance x Specific gravity
Volume of substance = 59.14 mL liquid  volume of water = 59.14 mL = 59.14 g
Specific gravity = 1.118
Weight of substance = 59.14 g x 1.118
Weight of substance = 2968.40 g
Calculating Volume, Knowing the Weight
and Specific Gravity

• Equation:

𝐺𝑟𝑎𝑚𝑠
Milliliters =
𝑆𝑝𝑒𝑐𝑖𝑓𝑖𝑐 𝑔𝑟𝑎𝑣𝑖𝑡𝑦
Example:

• What is the volume, in milliliters, of 492 g of a liquid with a specific


gravity of 1.40?
𝑊𝑒𝑖𝑔𝑕𝑡 𝑜𝑓 𝑠𝑢𝑏𝑠𝑡𝑎𝑛𝑐𝑒 (𝑔)
Volume of substance = 𝑆𝑝𝑒𝑐𝑖𝑓𝑖𝑐 𝑔𝑟𝑎𝑣𝑖𝑡𝑦
492 𝑔
Volume of substance = 𝑊𝑒𝑖𝑔𝑕𝑡 𝑜𝑓 𝑠𝑢𝑏𝑠𝑡𝑎𝑛𝑐𝑒 (𝑔)
1.40 Volume of substance = 𝑔
𝑆𝑝𝑒𝑐𝑖𝑓𝑖𝑐 𝑔𝑟𝑎𝑣𝑖𝑡𝑦 ( )
𝑚𝐿
Volume of substance= 351.43 g
Volume of substance = 𝑚𝐿
Volume of substance = 351.43 mL
Example:

• What is the volume, in milliliters, of 1 lb. of a liquid with a specific


gravity of 1.185? 1 lb. = 454 g
𝑊𝑒𝑖𝑔𝑕𝑡 𝑜𝑓 𝑠𝑢𝑏𝑠𝑡𝑎𝑛𝑐𝑒 (𝑔)
Volume of substance = 𝑆𝑝𝑒𝑐𝑖𝑓𝑖𝑐 𝑔𝑟𝑎𝑣𝑖𝑡𝑦
454𝑔
Volume of substance = 𝑊𝑒𝑖𝑔𝑕𝑡 𝑜𝑓 𝑠𝑢𝑏𝑠𝑡𝑎𝑛𝑐𝑒 (𝑔)
1.185 Volume of substance = 𝑔
𝑆𝑝𝑒𝑐𝑖𝑓𝑖𝑐 𝑔𝑟𝑎𝑣𝑖𝑡𝑦 ( )
𝑚𝐿
Volume of substance= 383.12 g
Volume of substance = 𝑚𝐿
Volume of substance = 383.12 mL
is the branch of medicine that deals with disease in
children rom birth through adolescence.

Pediatric Groups:
Patients Neonate/newborn = Birth to 1 month
Infant = 1 month to 1 year
Early childhood = 1 year to 5 years
Late childhood = 6 years to 12 years
Adolescence = 13 years to 17 years
• Doses should be based on accepted clinical studies as
reported in the literature.
• Doses should be age appropriate and generally based on
body weight or body surface area.
Special • Pediatric patients should be weighed as closely as
possible to the time of admittance to a health care facility
Considerations and that weight recorded in kilograms.
in Dose • As available, pediatric formulations rather than those
intended for adults should be administered.
Determinations
• All calculations of dose should be double-checked by a
for Pediatric second health professional.
Patients • All caregivers should be properly advised with regard to
dosage, dose administration, and important clinical signs
to observe.
• Calibrated oral syringes should be used to measure and
administer oral liquids.
Geriatric Patients

• Also known as elderly.


• Medical care for older adults, an age group that is not easy to
define precisely.
• Therapy is often initiated with a lower-than-usual adult
dose.
• Dose adjustment may be required based on the
therapeutic response.
Special • The patient’s physical condition may determine the
Considerations drug dose and the route of administration used.
in Dose • The dose may be determined, in part, on the patient’s
weight, body sur ace area, health and disease status,
Determinations and pharmacokinetic actors.
for Elderly • Concomitant drug therapy may affect drug/dose
Patients effectiveness.
• A drug’s dose may produce undesired adverse effects
and may affect patient adherence.
• Complex dosage regimens of multiple drug therapy
may affect patient adherence.
Rules for Approximate Doses for
Infants and Children

Based on Age
• Young’s Rule (for children 2 years old and older)
• Cowling’s Rule
• Fried’s Rule
Based on Weight
• Clark’s Rule
Drug Dosage Based on Age

Young’s Rule:

𝐴𝑔𝑒 (𝑦𝑒𝑎𝑟𝑠)
Dose for a Child = x Adult Dose
𝐴𝑔𝑒 𝑦𝑒𝑎𝑟𝑠 :12
Example:
Determine the dose for a 2–year old child if the
usual adult dose is 250 mg. (use young’s rule)
𝐴𝑔𝑒 (𝑦𝑒𝑎𝑟𝑠)
Dose for a Child = 𝐴𝑔𝑒 𝑦𝑒𝑎𝑟𝑠 :12
x Adult Dose
2 𝑦𝑒𝑎𝑟𝑠
Dose for a Child = 2 𝑦𝑒𝑎𝑟𝑠 :12
x 250 mg

Dose for a Child = 35.71 mg


Drug Dosage Based on Age

Cowling’s Rule:

𝐴𝑔𝑒 𝑛𝑒𝑥𝑡 𝑏𝑖𝑟𝑡𝑕𝑑𝑎𝑦 ( 𝑖𝑛 𝑦𝑒𝑎𝑟𝑠)


Dose for a Child = 24
x Adult Dose
Example:
Determine the dose for a 1-year old child if the
adult dose is 125 mg (use cowling’s rule).
𝐴𝑔𝑒 𝑛𝑒𝑥𝑡 𝑏𝑖𝑟𝑡𝑕𝑑𝑎𝑦 ( 𝑖𝑛 𝑦𝑒𝑎𝑟𝑠)
Dose for a Child = x Adult Dose
24

2
Dose for a Child = x 125 mg
24

Dose for a Child = 10.42 mg


Drug Dosage Based on Age

Fried’s Rule:

𝐴𝑔𝑒 𝑖𝑛 𝑚𝑜𝑛𝑡𝑕𝑠 𝑥 𝐴𝑑𝑢𝑙𝑡 𝐷𝑜𝑠𝑒


Dose for a Child =
150
Example:
Determine the dose for a 3-months old child if the
usual adult dose is 375mg (use fried’s rule).
𝐴𝑔𝑒 𝑖𝑛 𝑚𝑜𝑛𝑡𝑕𝑠 𝑥 𝐴𝑑𝑢𝑙𝑡 𝐷𝑜𝑠𝑒
Dose for a Child = 150
3 𝑚𝑜𝑛𝑡𝑕𝑠 𝑥 375 𝑚𝑔
Dose for a Child = 150

Dose for a Child = 7.5 𝑚𝑔


Drug Dosage Based on Weight

Clark’s Rule:

𝑊𝑒𝑖𝑔𝑕𝑡 (𝑙𝑏) 𝑥 𝐴𝑑𝑢𝑙𝑡 𝐷𝑜𝑠𝑒


Dose for a Child = 150
Example:
Determine the dose for a 2-year old child weighing
40 lb if the usual adult dose is 250 mg (use clark’s
rule).
𝑊𝑒𝑖𝑔𝑕𝑡 𝑙𝑏 𝑥 𝐴𝑑𝑢𝑙𝑡 𝐷𝑜𝑠𝑒
Dose for a Child = 150
40 𝑙𝑏 𝑥 250 𝑚𝑔
Dose for a Child = 150

Dose for a Child = 66.67 𝑚𝑔


Drug Dosage Based on Body Weight

𝐷𝑟𝑢𝑔 𝐷𝑜𝑠𝑒 (𝑚𝑔)


Patient’s Dose (mg) = Patient’s weight (kg) x 1 (𝐾𝑔)
Example:
The dose of gentamicin for premature and full-term neonates is 2.5 mg/kg
administered every 12 hours. What would be the daily dose for a newborn
weighing 5.6 lb?

𝐷𝑟𝑢𝑔 𝐷𝑜𝑠𝑒 (𝑚𝑔)


Patient’s Dose (mg) = Patient’s weight (kg) x
1 𝑘𝑔 1 (𝐾𝑔)
Patient’s Weight (kg) = 5.6 lb x 2.2 𝑙𝑏 = 2.55 kg
2.5 𝑚𝑔
Drug Dose = 1 𝐾𝑔
2.5𝑚𝑔
Patient’s Dose (mg) = 2.55 kg x 1 𝐾𝑔

24 𝑕𝑟𝑠
Patient’s Dose (mg) = 6.38 mg/12 hours x = 12.76 mg
1 𝑑𝑎𝑦
Based on Body Surface Area

The Square Meter Surface Area Method:


𝑃𝑎𝑡𝑖𝑒𝑛𝑡 ′ 𝑠 𝐵𝑆𝐴(𝑚2)
Patient’s Dose (mg) = x Drug Dose (mg)
1.73𝑚2

Determining Patient’s BSA:


Nomogram
BSA Equation (Mosteller’s Formula)
Nomogram
Mosteller’s Formula;

𝐻𝑡 𝑐𝑚 𝑥 𝑊𝑡 (𝑘𝑔)
BSA (m2) =
3600
Example:
If the adult dose of a drug is 75 mg, what would be the dose
for a child weighing 40lb and measuring 32 inches in height
using the BSA Nomogram?

From the nomogram, the BSA = 0.60 m2

0.60 𝑚2
Patient’s Dose (mg) = x 75 mg
1.73𝑚2

Patient’s Dose (mg) = 26. 01 mg or 26 mg


Example:
The daily dose for a child 4 years of age, 39 inches in
height, and weighing 32 lb for a drug with an adult dose
of 100 mg (Use the BSA equation)
𝑃𝑎𝑡𝑖𝑒𝑛𝑡 ′ 𝑠 𝐵𝑆𝐴(𝑚2)
Patient’s Dose (mg) = x Drug Dose (mg)
1.73𝑚2
99.06 𝑐𝑚 𝑥 14.5454 𝑘𝑔
𝐻𝑡 𝑐𝑚 𝑥 𝑊𝑡 (𝑘𝑔) BSA (m2) =
BSA (m2) = 3600
3600

2.54 𝑐𝑚 = 0.63 m2
Ht (cm) = 39 in x = 99.06 cm
1 𝑖𝑛 0.63𝑚2
Patient’s Dose (mg) = 1.73𝑚2
x 100 mg
1 𝑘𝑔
Wt (kg) = 32 lb x = 14.5454 kg
2.2 𝑙𝑏 Patient’s Dose (mg) = 36.42 mg
PERCENTAGE, RATIO STRENGTH
AND OTHER EXPRESSIONS OF
CONCENTRATIONS
PERCENT

• Corresponding (%) sign


• By the hundred
• Parts in a hundred
• An essential component of pharmaceutical calculations
• Uses:
a. To express the strength of a component in a pharmaceutical preparation
b. To determine the quantity of a component to use when a percent strength
is desired
Percent Preparations

• For solutions or suspensions of solids in liquids, percent weight in


volume
• For solutions of liquids in liquids, percent volume in volume
• For mixtures of solids or semisolids, percent weight in weight
• For solutions of gases in liquids, percent weight in volume
PERCENT CONCENTRATIONS

•Percent weight-in-volume (w/v)


•Percent volume-in-volume (v/v)
•Percent weight-in-weight (w/w)
Percent weight-in-volume (w/v)

• expresses the number of grams of a constituent in 100 mL of


solution or liquid preparation and is used regardless of whether
water or another liquid is the solvent or vehicle.
• Expressed as: % w/v.
• Equation
𝑎𝑚𝑜𝑢𝑛𝑡 𝑜𝑓 𝑠𝑜𝑙𝑢𝑡𝑒 (𝑔)
Percent weight-in-volume(%w/v) = 𝑥 100
𝑎𝑚𝑜𝑢𝑛𝑡 𝑜𝑓 𝑠𝑜𝑙𝑢𝑡𝑖𝑜𝑛 (𝑚𝐿)
Percent weight-in-volume (w/v)
Equation
𝑎𝑚𝑜𝑢𝑛𝑡 𝑜𝑓 𝑠𝑜𝑙𝑢𝑡𝑒 (𝑔)
Percent weight-in-volume(%w/v) = 𝑥 100
𝑎𝑚𝑜𝑢𝑛𝑡 𝑜𝑓 𝑠𝑜𝑙𝑢𝑡𝑖𝑜𝑛 (𝑚𝐿)

If the amount of solute (g) is unknown


Amount of solute (g) = %w/v expressed in decimal x amount of solution (mL)
%
%w/v expressed in decimal =
100
If the amount of solution is unknown
𝑎𝑚𝑜𝑢𝑛𝑡 𝑜𝑓 𝑠𝑜𝑙𝑢𝑡𝑒 (𝑔)
Amount of solution (mL) = 𝑤
% 𝑒𝑥𝑝𝑟𝑒𝑠𝑠𝑒𝑑 𝑖𝑛 𝑑𝑒𝑐𝑖𝑚𝑎𝑙
𝑣
Example:
How many grams of dextrose are required to prepare 4000 mL of 5%
solution?
Amount of solute (g) = %w/v expressed in decimal x amount of solution (mL)
%
%w/v expressed in decimal = 100

5%
%w/v expressed in decimal = 100 = 0.05 (g/mL)

𝑔
Amount of solute (g) = 0.05 (𝑚𝐿) x 4000 mL

Amount of solute (g) = 200 g


Example:
Rx Antipyrine 5%
Glycerin ad 60 mL
How many grams of antipyrine should be used in preparing the prescription?
Amount of solute (g) = %w/v expressed in decimal x amount of solution (mL)
%
%w/v expressed in decimal = 100

5%
%w/v expressed in decimal = 100 = 0.05 (g/mL)

𝑔
Amount of solute (g) = 0.05 (𝑚𝐿) x 60 mL

Amount of solute (g) = 3 g


POP QUIZ

1. Rx Ofloxacin ophthalmic solution 0.3%


Disp. 10 mL
How many milligrams of ofloxacin are contained in each milliliter of the dispensed prescription?

Answer: 3 mg of ofloxacin

2. A formula or an antifungal shampoo contains 2% w/v ketoconazole. How many grams of


ketoconazole would be needed to prepare 240 mL of the shampoo?

Answer: 4.8 g of ketoconazole


Percent volume-in-volume (v/v)

• expresses the number of milliliters of a constituent in 100 mL of


solution or liquid preparation.
• Expressed as: % v/v.
• Equation
𝑎𝑚𝑜𝑢𝑛𝑡 𝑜𝑓 𝑠𝑜𝑙𝑢𝑡𝑒 (𝑚𝐿)
Percent volume-in-volume(%v/v) = 𝑎𝑚𝑜𝑢𝑛𝑡 𝑜𝑓 𝑠𝑜𝑙𝑢𝑡𝑖𝑜𝑛 (𝑚𝐿)
𝑥 100
Percent volume-in-volume (v/v)
Equation
𝑎𝑚𝑜𝑢𝑛𝑡 𝑜𝑓 𝑠𝑜𝑙𝑢𝑡𝑒 (𝑚𝐿)
Percent volume-in-volume(%v/v) = 𝑥 100
𝑎𝑚𝑜𝑢𝑛𝑡 𝑜𝑓 𝑠𝑜𝑙𝑢𝑡𝑖𝑜𝑛 (𝑚𝐿)

If the amount of solute (mL) is unknown


Amount of solute (mL) = %v/v expressed in decimal x amount of solution
(mL)
%
%v/v expressed in decimal = 100
If the amount of solution is unknown
𝑎𝑚𝑜𝑢𝑛𝑡 𝑜𝑓 𝑠𝑜𝑙𝑢𝑡𝑒 (𝑚𝐿)
Amount of solution (mL) = 𝑣
%𝑣 𝑒𝑥𝑝𝑟𝑒𝑠𝑠𝑒𝑑 𝑖𝑛 𝑑𝑒𝑐𝑖𝑚𝑎𝑙
Example:
How many liters of a mouthwash can be prepared from 100 mL of cinnamon
flavor if its concentration is to be 0.5% (v/v)?
𝑣𝑜𝑙𝑢𝑚𝑒 𝑜𝑓 𝑠𝑜𝑙𝑢𝑡𝑒 (𝑚𝐿)
Amount of solution (mL) = 𝑣
% 𝑒𝑥𝑝𝑟𝑒𝑠𝑠𝑒𝑑 𝑖𝑛 𝑑𝑒𝑐𝑖𝑚𝑎𝑙
𝑣
%
%v/v expressed in decimal = 100
0.5 %
%v/v expressed in decimal = = 0.005 (mL/mL)
100

100 𝑚𝐿
Amount of solution (mL) = 𝑚𝐿
0.005
𝑚𝐿

1𝐿
Amount of solution (mL) = 20,000 mL x 1000 𝑚𝐿 = 20 L of mouthwash
Example:
The formula for 1 liter of an elixir contains 0.25mL of a flavoring oil.
What is the percentage (v/v) of the flavoring oil in the elixir?
𝑎𝑚𝑜𝑢𝑛𝑡 𝑜𝑓 𝑠𝑜𝑙𝑢𝑡𝑒 (𝑚𝐿)
Percent volume-in-volume(%v/v) = 𝑥 100
𝑎𝑚𝑜𝑢𝑛𝑡 𝑜𝑓 𝑠𝑜𝑙𝑢𝑡𝑖𝑜𝑛 (𝑚𝐿)

0.25 𝑚𝐿
Percent volume-in-volume(%v/v) = 𝑥 100
1000 𝑚𝐿

Percent volume-in-volume(%v/v) = 0.025 % v/v of flavoring oil


POP QUIZ

1. A dermatologic lotion contains 1.25 mL of liquefied phenol in 500


mL. Calculate the percent strength of liquefied phenol in the lotion.
Answer: 0.25% liquefied phenol
2. What is the percent strength (v/v) if 225 g of a liquid having a
specific gravity of 0.8 are added to enough water to make 1.5 L of
the solution?
Answer: 18.75% v/v
Percent weight-in-weight (w/w)

• expresses the number of grams of a constituent in 100 g of


solution or preparation.
• Expressed as: % w/w
• Equation
𝑎𝑚𝑜𝑢𝑛𝑡 𝑜𝑓 𝑠𝑜𝑙𝑢𝑡𝑒 (𝑔)
Percent weight-in-weight(%w/w) = 𝑎𝑚𝑜𝑢𝑛𝑡 𝑜𝑓 𝑠𝑜𝑙𝑢𝑡𝑖𝑜𝑛 (𝑔)
𝑥 100
Percent weight-in-weight (w/w)
Equation
𝑎𝑚𝑜𝑢𝑛𝑡 𝑜𝑓 𝑠𝑜𝑙𝑢𝑡𝑒 (𝑔)
Percent weight-in-weight(%w/w) = 𝑥 100
𝑎𝑚𝑜𝑢𝑛𝑡 𝑜𝑓 𝑠𝑜𝑙𝑢𝑡𝑖𝑜𝑛 (𝑔)

If the amount of solute (g) is unknown


Amount of solute (g) = %w/w expressed in decimal x amount of solution (g)
%
%w/w expressed in decimal = 100
If the amount of solution is unknown
𝑎𝑚𝑜𝑢𝑛𝑡 𝑜𝑓 𝑠𝑜𝑙𝑢𝑡𝑒 (𝑔)
Amount of solution (g) = 𝑔
% 𝑒𝑥𝑝𝑟𝑒𝑠𝑠𝑒𝑑 𝑖𝑛 𝑑𝑒𝑐𝑖𝑚𝑎𝑙
𝑔
Example:
Antibacterial gel contains 0.05% w/w of drug. Calculate the quantity of
this agent (in grams) in each 60 g tube of the product.
%
%w/w expressed in decimal = 100
0.05 %
%w/w expressed in decimal = = 0.0005 (g/g)
100

𝑔
Amount of solute (g) = 0.0005 (𝑔 ) x 60 g

Amount of solute (g) = 0.03 g


Example:
How many grams of azelaic acid are contained in 30 grams tubes of
the 15% w/w ointment?
%
%w/w expressed in decimal =
100
15 %
%w/w expressed in decimal = = 0.15 (g/g)
100

𝑔
Amount of solute (g) = 0.15 (𝑔 ) x 30 g

Amount of solute (g) = 4.5 g of azelaic acid


POP QUIZ

1. What is the percentage strength (w/w) of a solution made by


dissolving 62.5 g of potassium chloride in 187.5 mL of water?
Answer: 25% potassium chloride
2. How many grams of hydrocortisone should be used in preparing
120 suppositories, each weighing 2 g and containing 1% of
hydrocortisone?
Answer: 2.4 g of hydrocortisonw
RATIO STRENGTH

• Expresses the concentration of weak solution.


• For example,
5% means 5 parts per 100 or 5:100.
Although 5 parts per 100 designates a ratio strength, it is customary to
translate this designation into a ratio, the first figure of which is 1; thus,
5:100 = 1:20.
Ratio Strength

For solids in liquids =1 g of solute or constituent in 1000 mL of


solution or liquid preparation.
For liquids in liquids = 1 mL of constituent in 1000 mL of
solution or liquid preparation.
For solids in solids = 1 g of constituent in 1000 g of mixture.
Example:

Express 0.02% as a ratio strength.


0.02 (%) 1 (𝑝𝑎𝑟𝑡)
=
100 (%) 𝑥 (𝑝𝑎𝑟𝑡𝑠)

x (0.02%) = (1 part) (100%)


0.02% 0.02%

= 5000 parts

Ratio strength = 1:5000


Example:

Express 1:4000 as a percentage strength.


4000 𝑝𝑎𝑟𝑡𝑠 100 %
=
1 𝑝𝑎𝑟𝑡 𝑥 (%)

4000 parts (x) = (1 part) (100%)


4000 parts 4000 parts

= 0.025 %

Percent strength = 0.025 %


PARTS PER MILLION (ppm) AND PARTS PER
BILLION (ppb)

• Strengths of very dilute solutions


• Expressed in terms of ppm and ppb
• Example:
5 ppm = 5 parts in 1,000,000 parts
2 ppb = 2 parts in 1,000,000,000 parts
Example:

The concentration of a drug additive in animal feed is 12.5 ppm. How many
milligrams of the drug should be used in preparing 5.2 kg of feed?

12.5 ppm = 12.5 g (drug) in 1,000,000 g (feed)


1000 𝑔
5.2 kg x = 5,200 g (1,000,000 g) (x) = (12.5 g) (5,200 g)
1 𝑘𝑔
1,000,000 g 1,000,000 g
1,000,000 𝑔 5,200 𝑔
= 1000 𝑚𝑔
12.5 𝑔 𝑥 x = 0.065 g x
1𝑔

= 65 mg
Milligrams Percent

- Express the number of milligrams of substance in 100 mL of


liquid.
- Denotes the concentration of a drug or natural substance in
biologic fluid, as in blood.
- Example: non-protein nitrogen in blood is 30 mg% means that
each 100 mL of blood contains 30 mg of non-protein blood.
mg/mL

• Measurement of a solutions concentration.


Example:
Convert 4% (w/v) to mg/mL
1000 𝑚𝑔
4𝑔 4gx = 4000 mg
4% w/v = 1𝑔
100 𝑚𝐿

4000 𝑚𝑔
100 𝑚𝑙
= 40 𝑚𝑔/𝑚𝐿
PROOF STRENGTH

• Twice the percentage strength of alcohol


• Example: 50% is 100 proof
Proof Spirit

• Is an aqueous solution containing 50 % (v/v) of


alcohol.
Proof gallon

𝑊𝑖𝑛𝑒 𝑔𝑎𝑙𝑙𝑜𝑛 𝑥 𝑃𝑒𝑟𝑐𝑒𝑛𝑡𝑎𝑔𝑒 𝑠𝑡𝑟𝑒𝑛𝑔𝑡𝑕 𝑜𝑓 𝑠𝑜𝑙𝑢𝑡𝑖𝑜𝑛


Proof gallons = 50%

𝑊𝑖𝑛𝑒 𝑔𝑎𝑙𝑙𝑜𝑛 𝑥 𝑃𝑒𝑟𝑐𝑒𝑛𝑡𝑎𝑔𝑒 𝑠𝑡𝑟𝑒𝑛𝑔𝑡𝑕 𝑜𝑓 𝑠𝑜𝑙𝑢𝑡𝑖𝑜𝑛 (𝑝𝑟𝑜𝑜𝑓)


Proof gallons = 100 𝑝𝑟𝑜𝑜𝑓
Example:

How many proof gallons are contained in 5 wine gallons of


75% v/v alcohol?
𝑊𝑖𝑛𝑒 𝑔𝑎𝑙𝑙𝑜𝑛 𝑥 𝑃𝑒𝑟𝑐𝑒𝑛𝑡𝑎𝑔𝑒 𝑠𝑡𝑟𝑒𝑛𝑔𝑡𝑕 𝑜𝑓 𝑠𝑜𝑙𝑢𝑡𝑖𝑜𝑛
Proof gallons = 50%

5 𝑊𝑖𝑛𝑒 𝑔𝑎𝑙𝑙𝑜𝑛 𝑥 75%


Proof gallons = 50%

Proof gallons = 7.5 𝑝𝑟𝑜𝑜𝑓 𝑔𝑎𝑙𝑙𝑜𝑛𝑠


Altering product strength

• The percentage or ratio strength (concentration) of a


component in a pharmaceutical preparation is based on its
quantity relative to the total quantity of the preparation.
• If the quantity of the component remains constant, any
change in the total quantity of the preparation, through
dilution or concentration, changes the concentration of the
component in the preparation inversely.
In pharmacy practice

• The reduction in the strength of a commercially available


pharmaceutical product may be desired to treat a particular
patient, based on the patient’s age (e.g., pediatric or elderly)
or medical status, or to assess a patient’s initial response to a
new medication.
• The strengthening of a product may be desired to meet the
specific medication needs of an individual patient
Altering product strength

Problems may be solved by any of the following


methods:
1. Inverse proportion
2. The equation: (1st quantity) × (1st concentration) = (2nd quantity)
× (2nd concentration or Q1 x C1 = Q2 x C2
3. Traditional calculations, by determining the quantity of active
ingredient present and relating that amount to the quantity of the
total preparation
Example:
If 500 mL of a 15% v/v solution are diluted to
1500 mL, what is the percent strength (v/v) of the
dilution?
• By inverse proportion
1500 mL 15%
_____________ = _________
500 mL x

(1500 mL) (X) = (500 mL) (15%)


1500 mL 1500 mL

X = 5% v/v
Example:
If 500 mL of a 15% v/v solution are diluted to 1500
mL, what is the percent strength (v/v) of the
dilution?
• By traditional calculation
Amount of solution X Concentration expressed in decimal = Amount of solute
𝑚𝐿
Amount of solute (g) = 0.15 (𝑚𝐿) x 75 mL
𝑎𝑚𝑜𝑢𝑛𝑡 𝑜𝑓 𝑠𝑜𝑙𝑢𝑡𝑒 (𝑚𝐿)
Percent volume-in-volume(%v/v) = 𝑎𝑚𝑜𝑢𝑛𝑡 𝑜𝑓 𝑠𝑜𝑙𝑢𝑡𝑖𝑜𝑛 (𝑚𝐿)
𝑥 100
75 𝑚𝐿
Percent volume-in-volume(%v/v) = 1500 𝑚𝐿
𝑥 100
Percent volume-in-volume(%v/v) = 5% v/v
Example:
If 500 mL of a 15% v/v solution are diluted to
1500 mL, what is the percent strength (v/v) of the
dilution?
• By equation
Q1 x C1 = Q2 x C2
(500 mL) (15%) = (1500 mL) (x)
1500 mL 1500 mL

X = 5%
Stock Solutions

• Stock solutions are concentrated solutions of


active (e.g., drug) or inactive (e.g., colorant)
substances and are used by pharmacists as a
convenience to prepare solutions of lesser
concentration.
Q1 x C1 = Q2 x C2
Example
How many milliliters of a 1% w/v stock solution of a certified red dye
should be used in preparing 4000 mL of a mouthwash that is to contain
1:20,000 w/v of the certified red dye as a coloring agent?

1:20,000 = 0.005%w/v
Q1 x C1 = Q2 x C2
(4000 mL) (0.005%) = (x) (1%)
1% 1%

X = 20 mL
POP QUIZ

• If 250 mL of 1:800 (v/v) solution were diluted in 1000 mL,


what would be the ratio strength (v/v)?
• If a pharmacist added 12 grams of azelaic acid to 50 g of an
ointment containing 15% azelaic acid , what would be the
final concentration of the azelaic acid in the ointment?
Electrolyte Preparations

• are used in the treatment of disturbances of the electrolyte


and fluid balance in the body.
• They are provided by the pharmacy as oral solutions, syrups,
tablets, capsules, and, when necessary, intravenous
infusions.
Electrolyte Solutions

• Electrolyte ions in the blood plasma include the


cations Na+, K+, Ca2+, and Mg2+ and the anions Cl−,
HCO3−, HPO42−, SO42−, organic acids, and protein.
• Electrolytes in body fluids play an important role in
maintaining the acid–base balance.
• They also play a part in controlling body water
volumes and help regulate metabolism.
Milliequivalents (mEq)

• is used almost exclusively in the United States by clinicians,


physicians, pharmacists, and manufacturers to express the
concentration of electrolytes in solution.
• This unit of measure is related to the total number of ionic charges
in solution, and it takes note of the valence of the ions
• a unit of measurement of the amount of chemical activity of an
electrolyte.
Milliequivalent

____MW____
Valence
1 mEq = _________________
1000
Sample Problems
1. A physician prescribes 10 mEq of potassium chloride for a patient. How
many illigrams of KCl would provide the prescribed quantity?
2. A physician prescribes 3 mEq/kg of NaCl to be administered to a 165-lb
patient. How many milliliters of a half–normal saline solution (0.45% NaCl)
should be administered?
3. What is the concentration, in milligrams per milliliter, of a solution
containing 2 mEq of potassium chloride (KCl) per milliliter?
4. What is the concentration, in grams per milliliter, of a solution containing 4
mEq of calcium chloride (CaCl2 · 2H2O) per milliliter?
Millimoles

•A mole is the molecular weight of a substance in


grams.
•A millimole is one-thousandth of a mole and is,
therefore, the molecular weight of a substance in
milligrams.
Millimoles

MW
1 mmol = __________
1000
Sample problem

1. How many millimoles of monobasic sodium phosphate


monohydrate (m.w. 138) are present in 100 g of the substance?
2. What is the weight, in milligrams, of 5 mmol of potassium
phosphate dibasic (m.w. 174)?
3. If lactated Ringer’s injection contains 20 mg of calcium chloride
dihydrate (CaCl2 · 2H2O) m.w 147 in each 100 mL, calculate the
millimoles of calcium present in 1 L of lactated Ringer’s injection.
Osmolarity

•Osmotic pressure is proportional to the total number of


particles in solution. The unit used to measure osmotic
concentration is the milliosmole (mOsmol).
•For dextrose, a nonelectrolyte, 1 mmol (1 ormula
weight in milligrams) represents 1 mOsmol.
Milliosmoles

Wt. of substance(g/L)
mOsmol = ___________________ x No. of Species x 1000
Molecular weight (g)
1. A solution contains 10% of anhydrous dextrose in water for
injection. How many milliosmoles per liter are represented by this
concentration? Molecular weight of anhydrous dextrose = 180
2. Calculate the osmolarity, in milliosmoles per liter, of a parenteral
solution containing 2 mEq/mL of potassium acetate (KC
2H3O2—m.w. 98).
3. What is the osmolarity of an 8.4% w/v solution of sodium
bicarbonate( m.w. 84)?
ISOTONIC SOLUTION
What is Isotonicity

•Two solutions that have the same osmotic pressure are


termed isosmotic. Many solutions intended to be mixed with
body fluids are designed to have the same osmotic pressure
for greater patient comfort, efficacy, and safety.
• A solution having the same osmotic pressure as a specific
body fluid is termed isotonic(meaning of equal tone) with that
specific body fluid.
• When a solvent passes through a semi-
permeable membrane from a dilute solution into
a more concentrated one, the concentration
become equalized and this is called as
osmosis.

154
• The pressure responsible for this phenomenon
is termed as osmotic pressure and varies with
the nature of the solute.

155
• If the solute is non-electrolyte, its solution contains
only molecules and the osmotic pressure varies with
the concentration of the solute. If the solute is an
electrolyte, its solution contains ions and the osmotic
pressure varies with the both the concentration of the
solute and its degree of dissociation.

156
• Isotonic – a solution having the same osmotic pressure as the
specific body fluid.

• Hypotonic – solutions of lower osmotic pressure with the body


fluid.

• Hypertonic – solutions having higher osmotic pressure with the


body fluid.

157
Example: Calculation of the Factor

1. Ferrous Sulfate is a 2 ion electrolyte, dissociating 60% in a


certain concentration. Calculate its dissociation (i) factor.

158
2. Sodium Carbonate dissociates in a certain concentration at
70%. Calculate its dissociation (i) factor.

159
• Most medicinal salts approximate the dissociation of sodium chloride in weak
solutions. If the number of ions is known, we may use the following values:

Non-electrolytes and subs of slight dissociation: 1.0


Subs. that dissociate in to 2 ions: 1.8
3 ions: 2.6
4 ions: 3.4
5 ions: 4.2

160
Calculations of the NaCl equivalent:

Formula:

MW of NaCl X i factor of Subs = NaCl equivalent


i factor of NaCl MW of Subs

161
Examples:

1. Ephedrine hydrochloride (mw= 202) is a 2 ion electrolyte,


dissociating 75% in a given concentration. Calculate the
sodium chloride content.

162
2. Morphine sulfate (mw=759) is a 3 ion electrolyte,
dissociating 60% in a concentration. Calculate the sodium
chloride equivalent.

163
Procedure in the calculation of isotonic solutions with NaCl equivalents:

1. Calculate the amount in grams of sodium chloride represented by


the ingredients in the prescription. Multiply the amount in grams of
each substance by its sodium chloride equivalent.

2. Calculate the amount in grams of sodium chloride alone, that would


be contained in an isotonic solution of the volume specified in the
prescription, namely, the amount of sodium chloride in a 0.9%
solution of the specified volume.

164
3. Subtract the amount of sodium chloride represented by the
ingredients in the prescription (Step 1) from the amount of
sodium chloride, alone, that would be represented in the
specific volume of an isotonic solution (Step 2). The answer
represents the amount (in grams) of sodium chloride to be
added to make the isotonic solution.

165
4. If an agent other than sodium chloride, such as boric acid,
dextrose or potassium nitrate is to be used to make a solution
isotonic, divide the amount of sodium chloride (Step 3) by the
sodium chloride equivalent of the other substance.

166
Example Calculations of tonicic agent required:

1. Rx Cocaine hydrochloride 0.6


Eucatropine hydrochloride 0.6
Chlorobutanol 0.1
Sodium chloride q.s.
Purified water ad 30
Make isoton. Sol.
Sig. For the eye.
How many grams of sodium chloride should be used in compounding the
prescription?

167
2. Rx Tetracaine hydrochloride 0.1
Zinc sulfate 0.05
Boric acid q.s.
Purified water ad 30
Make isoton. Sol.
Sig. Drop in eye
How many grams of boric acid should be used in compounding the
prescription?

168
Using an isotonic Sodium chloride to prepare other isotonic solutions:

A 0.9% w/v sodium chloride solution may be used to compound


isotonic solutions of other substances as follows:

Step 1: Calculate the quantity of the drug substance needed to fill


the prescription or medication order.

Step 2: Use the following equation to calculate the volume of water


needed to render a solution of the drug substance isotonic.

169
g of drug x drug’s E value = mL of water needed to
0.009 to make an isoton.sol’n

Step 3: Calculate the volume 0.9% w/v sodium chloride


solution to complete the required volume of the prescription
or medication order.

170
Example:

1. Rx Phenylephrine hydrochloride 1%
Chlorobutanol 0.5%
Sodium chloride q.s.
Purified water ad 15
Make isoton. Sol.
Sig: Use as directed
How many mL of a 0.9% solution of sodium chloride should be used in
compounding the prescription?

171
Freezing Point Data in Isotonicity Calculations

Freezing point data (∆Tƒ) can be used in isotonicity


calculations when the agent has a tonicic effect and does not
penetrate the biologic membranes in question. The freezing
point of both blood and lacrimal fluid is -0.52°C. A
pharmaceutical solution that has a freezing pt same with the
blood is said to be isotonic.

172
1. How many milligrams of Naphazoline hydrochloride and
sodium chloride are needed to produce 75 mL of 1% isotonic
solution with lacrimal fluid?

173
2. How many mg of Atropine sulfate and boric acid are required
to prepare 15 mL of 1% of atropine sulfate isotonic with
tears?

174
3. How many milligrams of dextrose and sodium chloride are
needed to produce 1L of 1% dextrose isotonic with blood?

175
How is HLB Values computed?

• The systematic choice of emulsifying agents in the


formulation of many emulsion systems depends on their HLB
(hydrophile-Lipophile-Balance) values.
• These values form the basis of the so-called HLB system,
which was developed by Griffin.
The system presupposes a scale of HLB
numbers and is based on the facts

1. that every surfactant or emulsifier molecule in part


hydrophilic and in part lipophilic
2. that a certain balance between these two parts is necessary
for various types of surfactant functions. In this scheme,
each surfactant or emulsifying agent is assigned a number
that varies from 1 to 20.
• The lower values are assigned to substances that are
predominantly lipophilic (oil loving) and have a tendency to
form water-in-oil (w/o) emulsions.
• The higher values are given to those materials that show
hydrophilic (water-loving) characteristics and favor the
formation of oil-in-water (o/w) emulsions.
• Consequently, the HLB number of an emulsifying agent is an
index of the type of emulsion that has the greatest tendency
to form.
• When two or more emulsifiers are combined, the HLB of the
combination is determined arithmetically by adding the
contribution that each makes to the HLB total of the mixture
SAMPLE PROBLEM

• What is the HLB of a mixture of 40% of Span 60 and 60% of Tween 60?
HLB of Span 60 = 4.7
HLB of Tween 60 = 14.9
HLB % of mixture
Span 60 4.7 x 40% = 1.9
Tween 60 14.9 x 60% = 8.9
HLB of mixture = 10.8
SAMPLE PROBLEM

• In what proportion should Tween 80 and Span 80 be blended to obtain a required HLB of 12?
HLB of Tween 80 = 15
HLB of Span 80 = 4.3

15 7.7 parts of tween 80


12
4.3 3.0 of Span 80
What is pharmacoeconomics?

• The term pharmacoeconomics encompasses the economic


aspects of drug, from the costs associated with drug
discovery and development to the costs of drug therapy
analyzed against therapeutic outcomes.
• Drug therapy and other means of treatment are intended to
serve the health care interests the patient while being cost
effective. In prescribing drug therapy, clinical as well as
economic factors are important considerations in the
selection of the drug substance and drug product.
• For example, if an expensive drug reduces morbidity and
hospitalization time, it is considered both therapeutically
advantageous and cost effective. If, however, a less
expensive drug would provide therapeutic benefit comparable
to the more expensive drug, the less costly drug is likely to be
selected for use.
Cost differential between drugs

Example:
An antihypertensive drug is available from various
manufacturers at prices per 100 tablets ranging from P6.26 to
P25.50, with a mean price of P10.75. If a patient presents a
prescription for a 6 month supply of the drug calling for two
tablets daily, calculate the differentials in the cost of the drug to
the pharmacy between the highest, mean and lowest cost
products.
Solution

6 month supply = approximately 180 days


2 tablets a day x 180 days = 360 tablets
Lowest price:
P6.25/100 tablets = P0.0625/tablets x 360 tablets = P 22.5

Mean price
P10.75/100 tablets = P0.1075/tablets x 360 tablets = P 38.70

Highest price:
P25.50/100 tablets = P0.2550/tablets x 360 tablets = P 91.80
Differentials:
Highest price to lowest price:
P91.80 – P22.50 = P69.30
Highest price to mean price:
P91.80 – P38.70 = P53.10
Mean price to lowest price:
P38.70 – P22.50 = P16.20

Depending on which product is dispensed, these differentials would be reflected in the


prescription price charges to the patient.
Discounts

• Discounts provided by suppliers may be based on quantity buying


and/or payment of invoices within a specified time period.
• In addition, for nonprescription products, discounts may be
available for certain seasonal or other promotional products,
bonuses in terms of free merchandise, and advertising and display
allowances.
• These discounts provide the pharmacy with a means of increasing
the gross profit on selected merchandise.
Net cost given list price and allowable discount

• Example:
The list price of an antihistamine elixir is P6.50 per pint, less 40%.
What is the nest cost per pint of elixir?
List Price Discount Net Cost
100% - 40% = 60%
P6.50 x 0.60 = P3.90, answer
Markup

• The term mark up, sometimes used interchangeably with the


term margin of profit (gross profit), refers to the difference
between the cost of merchandise and its selling price.
• Calculating the selling price of merchandise to yield a
given percent of gross profit on the cost involves the
following.
Example

The cost of 100 antacid tablets is P2.10. What would be the selling price
per tablet to yield a 66 2/3% gross profit on cost?

Cost x % of gross profit = gross profit


P2.10 x 66 2/3% = P1.40
Cost + Gross profit = Selling price
P2.10 + P1.40 = P3.50
Example

The cost of 100 antacid tablets is $2.10. What should be the selling price per 100 tablets to yield a 40% gross profit on
the selling price?
Selling price = 100%
Selling price - Gross profit = Cost
100% - 40% = 60%

60 (%) ($) 2.10


----------- = -----------
100 (%) ($) x

x = $3.50, answer.
What are the Common Methods of Prescription
Pricing?

1. Percent Mark up. In this common method, the desired


percent markup is taken of the cost of the ingredients and
added to the cost of the ingredients to obtain the
prescription price.

Prescription Price = Cost of ingredients + (cost of ingredient x % markup)


Example

If the cost of the quantity of a drug product to be dispensed is


P4.00 and the pharmacist applies an 80% markup on cost,
what would be the prescription price?

P4.00 + (P4.00 x 80%) = P4.00 + P3.20 = P7.20


2. Percent Markup plus a minimum professional fee. In this
method, both a percent markup and a minimum professional
fee are added to the cost of the ingredient.

Prescription price= Cost of ingredients + (cost of ingredients x % markup) + minimum professional fee
Example

If the cost of a drug product to be dispensed in P4.00 and


pharmacist applies a 40% markup on cost plus a professional
fee of P2.25, what would be the prescription price?

P4.00 + (P4.00 x 80%) + P2.25 = P4.00 + P3.20 + P2.25 = P7.85


3. Professional Fee. This method involves addition of a
specified professional fee to the cost of ingredients used in
filling a prescription.

Cost of the ingredients + professional fee = prescription price


Example

If the cost of the quantity of a drug product to be dispensed is


P4.00 and the pharmacist applies a professional fee of P4.25,
what would be the prescription price?
P 4.00 + P 4.25 = P 8.25
END OF
DISCUSSION
REFERENCE

• Ansel, H. C., Pharmaceutical Calculations, 15th ed., Wolters Kluwer,


2017.
• Remington’s Pharmaceutical Sciences
Thanks!
PHARMACEUTICAL CALCULATIONS ✧ SYSTEMS OF MEASURE
A. Metric System
✧ FUNDAMENTALS Kilo—Hepto—Deka----deci—centi—milli—/--/--micro—nano
ORDER OF OPERATIONS
(P)- Parenthesis  BASE- gram, Liter & meter
E*- Exponents
M/D- Multiply/Divide (from left to right in the problem) CONVERT:
A/S- Add/Subtract (from left to right)  63000 mcg → 0.063 g
 24 mg → 0.024 g
EXAMPLE:  3 Dg → 3000 cg
[(3.5 + 3 x 4)2 + 12 ÷ 250] x 0.08205 =?
DIMENSIONAL ANALYSIS
= 3.5 + 12 • Convert 35mg/mL g/L
= (15.5)2 35 mg x 1 g x 1000 mL = 35 g/L
= [(240.25) + (12 ÷ 50)] 1 mL 100 mg 1L
= 240.25 + 0.048
= 240.298 x 0.08205 • If an intravenous fluid is adjusted to deliver 15 mg of
FINAL ANSWER: 19.72 medication to a patient per hour, how many milligrams of
medication are delivered per minute?
A. Fractions, Decimals and Percentage 15 mg x 1 hr x 1 min = 0.25 mg/min
FRACTION DECIMAL PERCENTAGE (per 100) 1 hr 60 mins x
½ 0.5 50%
2/3 0.67 67% TRY THIS!!!!
1 ½ (1 x 1 + 2) 1.5 150% A 36-lb child is to be given amoxicillin suspension. The dose is
= 3/2 13 mg/kg and the strength of the suspension is 250mg/5mL.
How many mL should be given the child?
[kapag ibabalik  Body weight: 36 lbs
sa mixed  Dose: 13 mg/kg
fraction, isa  Strength: 250 mg/5 mL
lang yung
makukuhang 2 COMPUTATION:
dun sa 3 so 1 36 lbs x 1 kg x 13 mg x 5 mL = 4.25 mL
(whole x (kg) 2.2 lbs 1 kg 250 mg
number), then
yung B. Apothecary System (weight)
remainder 20 grains = 1 scruple 60 minims = 1 fluidrachm
yung 3 scruple = 1 drachm 8 fluidrachm = 1 fl oz
numerator 8 drachm = 1 ounce 16 fl oz = 1 pint
which is 1 din
12 ounces = 1 lb 2 pints = 1 quart
tas 2 parin
4 quarts = 1 gallon
denominator]
=1½
C. Avoirdupois System
4/3 1.33 133%
1 ounce (oz) = 437.5 gr
3/8 0.375 37.5%
1 pound (lb) = 16 ounces
1/8 0.125 12.5%
D. Other important conversion factors
Reduce the fractions 2/3, 1/6 & ¾ to a common denominator
LENGTH VOLUME
2/3 x 4/4 8/12
1 ft = 12 in 1 mL = 20 drops or gtt
1/6 x 2/2 2/12
1 in = 2.54 cm 1 fl oz = 29.57 mL
¾ x 3/3 9/12
1 m = 39.37 in 1 pt = 473 mL
WEIGHT 1 gal = 3785 mL
In preparing batches of a lotion, a pharmacist used ¼ oz, 1/12
1 g = 15.432 gr
oz, 1/8 oz and 2/3 oz of phenol. What is the total amount of
phenol used? 1 kg = 2.2 lbs (Avoir)
1/4 x 6/6 6/24 1 gr = 65 mg
1/12 x 2/2 2/24
Grain (gr)
1/8 x 3/3 3/24
2/3 x 8/8 16/24 1 scruple 20 gr
TOTAL: 27/24 or 1 3/24 or 1.125 oz 1 drachm 3 scruple
8 oz 8 drachm
1 pound 12 oz
EXAMPLE PROBLEM:
Minims
A suspension is 250 mg/5 mL. Calculate the amount needed to
1 60
give a dose of 400 mg fluidram
1 8
250 mg x 400 mg = 8 mL fluidounce
5 mL x 1 pint 16
1 quart 2
1 gallon 4
 Fer-in-sol® drops contain 75 mg ferrous sulfate 0.60
PROBLEMS mL of the solution. If a pharmacist dispensed 60 mL
43 g → 663.58 gr COMMON HOUSEHOLD SYSTEM: of the solution, how many grams of ferrous sulfate
1 glassful = 5 mL were dispensed?
42 km → 42,000 m 1 tumblerful = 240 mL COMPUTATION:
176 lbs → 80 kg 1 teacupful = 120 mL 75 mg : 0.60 mL : : x (mg) : 60 mL
1 wineglassful = 60 mL = 7,500 mg → 7.5 g
14 fl. oz → 413.98 mL 1 tablespoonful= 15 mL
1 dessertspoonful = 8 mL  If a 10 mL vial of insulin contains 200 units of insulin
250 gr → 16,250 mg 1 teaspoonful = 5 mL per milliliter, and a patient is required to receive 20
½ tsp = 2.5 mL units twice daily, how many days will the product last
the patient?
TEMPERATURE (FORMULA) COMPUTATION:
 F= 9/5(C+32) 200 units : 1 mL : : 40 units : x (mL)
 C= 5/9(F-32) = 0.2 mL/day
 K= C+273.15
*9C=5F-160 0.2 mL : 1 day : : 10 mL : x (days)
= 50 days
-40⁰C to ⁰F 9C=5F-160
9C+160=5F  A patient receives 30 mL of heparin every hour. If
[9(-40)] +160=5F there are 40 units in each milliliter, how many units
-200 = 5F does the patient receive in 24 hours?
5 5 COMPUTATION:
= -40 ⁰F 40 units : 1 mL : : x (units) : 30 mL
98.6⁰F to ⁰C 9C=5F-160 = 1,200 units
9C=[5(98.6⁰F)]-160
9C=333 1,200 units : 1 hr : : x (units) : 24 hrs
9 9 = 28, 800 units
= 37 ⁰C
98.6⁰F to K K= 37⁰C+273.15 = 310.15 K IV FLUIDS AND RATE OF FLOW
FORMULA OF RATE OF FLOW
✧ RATIO AND PROPORTION R= V x D
1. If 3 tablets contain 975 mg of aspirin, how many T
milligrams should be contained in 12 tablets?
COMPUTATION:  A medication order for a patient weighing 154 lb calls
3 tabs : 975 mg : : 12 tabs : (x) mg for 0.25 mg of amphotericin B per kg of body weight
to be added to 500 mL of 5% dextrose injection. If the
= 3,900 mg amphotericin B is to be obtained from a constituted
injection that contains 50mg/10mL, how many mL
2. In a clinical study, a drug produced drowsiness in 30 should be added to the dextrose injection?
of the 1500 patients studied. How many patients of a
certain pharmacy could expect similar effects, based GIVEN:
on a patient count of 100? 154 lbs → 70 kg
COMPUTATION: Dose: 70 kg x 0.25 mg= 17.5 mg
30 : 1500 : : x : 100
= 2 patients COMPUTATION:
50 mg : 10 mL : : 17.5 mg : x (mL)
3. If a vitamin solution contains 0.5 mg of fluoride ion in = 3.5 mL
each mL, then how many grams of fluoride ion would
be provided by a dropper that delivers 0.6 mL  A medication order calls for 1000 mL of D5W to be
COMPUTATION: administered over an 8-hour period. Using an IV
0.5 mg : 1 mL : : x (mg) : 0.6 mL administration set that delivers 10 drops/mL, how
= 0.3 mg → 0.003 g many drops per minute should be delivered to the
patient?
PROBLEMS COMPUTATION:
 A formula for a cough syrup contains 1/8 gr of R= V x D
codeine phosphate per teaspoonful. How many T
grams of codeine phosphate should be used in R= 1000 mL x 10 gtts/mL
preparing 2 pints of the cough syrup? 480 mins
GIVEN: R= 20.83 gtts/mins or 21 ftts/mins
1/8 grains → 0.125 grains
0.125 gr x 1 g = 0.008 g *RATIO & PROPORTION
15.432 gr 10 gtts : 1 mL : : x : 1000 mL
1 teaspoonful = 5 mL = 10, 000 gtts
1 pint = 473 mL x 2 = 946 mL
10,000 gtts : 480 mins : : x : 1 min
COMPUTATION: =20.83 gtts/mins or 21 gtts/mins
0.008 g: 5 mL : : x (g) : 946 mL
= 1.51 g
✧ DENSITY AND SPECIFIC GRAVITY From the following formula for an estradiol vaginal gel,
DENSITY- mass per unit volume of a substance calculate the quantity of each ingredient required to prepare 1
D= Mass (g) lb of gel:
Volume (mL/cc) Estradiol 200 g x 1.53 = 306 g
Polysorbate-80 1 g x 1.53 = 1.53 g
SPECIFIC GRAVITY- weight of a substance to the weight of an Cellulose, 2% 95 g x 1.53 = 145.35 g
equal volume of a substance chosen as a standard (H2O)
Sp. Gr= weight of the substance (g) COMPUTATION:
weight of the equal volume of water (mL) 1 lb = 454 g

o <1= lighter than water CF= 454 g = 1.53


o >1= heavier than water 296 g

1. If 54.96 mL of an oil weighs 52.78 g, what is the The formula for a ciprofloxacin otic drop is given in the
specific gravity of the oil? literature as follows:
COMPUTATION: Ciprofloxacin 1 g x 30 = 30 g
Sp Gr= 52.78 g = 0.96 g/mL Propylene glycol 50 mL x 30 = 1500 mL
54.96 mL Glycerin qs ad 100 mL x 30 = 3000 mL

2. What is the weight in g of 750 mL glycerin having a How many grams of ciprofloxacin would be required to
density of 1.25 g/mL? prepare 200 15 mL bottles of the ear drop?
COMPUTATION:
D= M → M= D x V COMPUTATION:
V 200 x 15 mL = 3000 mL
M= 1.25 g/mL x 750 mL
M= 937.5 g CF= 3000 mL = 30
100 mL
3. What is the weight in g of 500mL glycerin having a
density of 1.25g/mL? What is the specific volume? “ALL DRUGS ARE POISON; IT IS JUST A MATTER OF DOSE”
COMPUTATION:
M= D x V
M= 1.25 g/mL x 500 mL
M= 625 g

Specific Volume= 500 mL = 0.8 g/mL


625 g

PROBLEM
 A glass plummet weighs 12.64 g in air, 8.57 g when
immersed in water, and 9.12 g when immersed in an
oil. Calculate the specific gravity of the oil
COMPUTATION:
12. 64 g – 8.57 g = 4.07 g (H2O) Plasma Drug Concentration Time Curve
12.64 g – 9.12 g = 3.52 g (oil)
MED & MTD
Sp Gr= Wt. (subs) = 3.52 g = 0.86
Wt (H2O) 4.07 g -- DOSE
--- SINGLE DOSE
✧ REDUCING AND ENLARGING FORMULAS ----DAILY DOSE
----- DIVIDED DOSES
From the following formula, calculate the quantity of each ------ DOSAGE REGIMEN
ingredient required to make 240 mL of calamine lotion:
Calamine 80 g x 0.24 = 19.2 g The schedule of dosing:
Zinc oxide 80 g x 0.24 = 19.2 g e.g. four times a day for ten days
Glycerin 20 g x 0.24 = 4.8 g
Bentonite magma 250 mL x 0.24 = 60 mL
Calcium hydroxide topical soln qs ad 1000 mL

CONVERSION FACTOR:
F= to be prepared (want)
std prep’n (have)

F= 240 mL = 0.24
1000 mL
✧ CALCULATION OF DOSES Child Dose using BSA
Monotherapy VS Adjunctive Therapy STEP 1:
Priming Dose VS Monitoring Dose BSA = √Px height (cm) x Px weight (kg)
Prophylactic Dose VS Therapeutic Dose 3600
= √120 cm x 35 kg = 1.08 m2
PATIENT PARAMETERS 3600
DRUG DOSAGE BASED ON AGE
STEP 2:
Young’s Rule= Age x Adult Dose CD= Px BSA (m2) x Drug Dose (mg)
Age + 12 1.73 m2
= 1.08 m2 x 350 = 218.50 mg
Cowling’s Rule= Age + 1 x AD 1.73 m2
24
PROBLEMS
Fried’s Rule= Age (months) x Adult Dose  If the dose of the drug is 150 mcg, how many doses
150 are contained in 0.120 g?
COMPUTATION:
DRUG DOSAGE BASED ON BODY WEIGHT 150 mcg → 0.000150 g
Clark’s Rule= Wt (lbs) x Adult Dose
150 0.000150 g : 1 dose : : 0.120 g : x (dose)
= 800 doses
BMI= Wt in kg
Ht in m2  A patient weighing 120 lbs was administered 2.1 g of
a drug supposed to be dosed at 30mg/kg. Was the
DRUG DOSAGE BASED ON BODY SURFACE AREA dose administered correct, or was it an overdose, or
BSA= √Px height (cm) x Px weight (kg) was it an underdose?
3600 COMPUTATION:
120 lbs x 1 kg = 54.55 kg
BSA= √Px height (in) x Px weight (lbs) 2.2 lbs
3131
30 mg : 1 kg : : x (mg) : 54.55 kg
2
CD= Px BSA (m ) x Drug Dose (mg) = 1,636.5 mg → 1.64 g (overdose)
1.73 m2
RENAL EXCRETION FORMULAS
A. JELLIFFE EQUATION (renal failure, unstable CrCl)
Weight Classifications BMI= kg/m2
MALE
Underweight <18.5
CrCl= 98-0.8 (Patient’s age in years-20)
Normal 18.5-25
Serum creatinine in mg/dL
Overweight 25-29.99
Obese 30 and above
FEMALE
CrCl= [98-0.8 (Patient’s age in years-20)] x 0.9
1. Calculate the dose (IN MILLIGRAMS) for a 5-yr old Serum creatinine in mg/dL
child, 1.2meter in height and weighing 35kg. for a
drug with an adult dose of 350 mg. Using the
following: B. COCKCROFT – GAULT EQUATION (common)
MALE
Young’s rule = Age x Adult Dose CrCl= (140-Patient’s age in years) x Body weight in kg
Age + 12 72 x Serum creatinine in mg/dL
= 5 x 350 mg = 102.94 mg
5 + 12 FEMALE
CrCl= (140-Patient’s age in years) x Body weight in kg x 0.85
Cowling's rule = Age + 1 x AD 72 x Serum creatinine in mg/dL
24
= 5 + 1 x 350 mg = 87.5 mg C. SANAKA EQUATION (older people, ↓muscle mass)
24
D. SCHWARTZ EQUATION (children, 1-18 y/o)
Fried’s rule = Age (months) x Adult Dose
150 - Determine the creatinine clearance rate for an 80-
= 60 months x 350 mg = 140 mg year-old male patient weighing 70 kg and having a
150 serum creatinine of 2mg/dl. Use both Jelliffe and
Cockroft – Gault equations.
Clark’s rule = Wt (lbs) x Adult Dose
150 JELLIFFE EQUATION
= 77 lbs x 350 = 179.67 mg CrCl= 98-0.8 (Patient’s age in years-20)
150 Serum creatinine in mg/dL
= (98-0.8)(80-20)
BMI= Wt in kg = 35 kg = 24.31 kg/m2 2
Ht in m2 1.2 m2 = 2,916 mL/min
COCKCROFT – GAULT EQUATION PERCENTAGE STRENGTH
CrCl= (140-Patient’s age in years) x Body weight in kg %W/V or g/mL
72 x Serum creatinine in mg/dL Clobex lotion contains 0.05% w/v lobetasol propionate in 118
= (140-80)(70) mL containers. Calculate the content of the drug, in
72 x 2 milligrams.
= 29.17 mL/min COMPUTATION:
0.05 g x x (g) = 0.059 g → 59 mg
- Calculate the creatinine clearance rate for an 80-yo 100 mL 118 mL
female patient weighing 70 kg and having a serum
creatinine of 2 mg/dL. Use both Cockroft and Gault %V/V or mL/mL
and Jelliffe Equations. In preparing 250 mL of a certain lotion, a pharmacist used 4
mL of liquefied phenol. What was the % v/v of liquefied phenol
2,916 mL/min x 0.90 = 2,624.44 mL/min in the lotion?
COMPUTATION:
29.17 x 0.85 = 24.79 mL/min 4 mL x 100 = 1.6% v/v
250 mL
✧ EXPRESSION OF CONCENTRATIONS
A. Percentage Strength → RS= 100 ÷ % %W/W or g/g
→ mg%= % x 1000 FINACEA gel contains 15% azelaic acid in 50 g tubes. Calculate
B. Ratio Strength → PS= divide RS x 100 the grams of azelaic acid in each tube of product.
C. Mg % → %= mg% ÷ 1000 COMPUTATION:
D. Ppm/Ppb 15 g x x (g) = 7.5 g
E. Proof Strength 100 g 100 g

1. CONVERT 5ppm to %= 5:1,000,000 Mg%


5 x 100 = 0.0005% If a patient is determined to have a serum cholesterol level of
1,000,000 200 mg/dL,
(a) what is the equivalent value expressed in terms of
2. CONVERT 5ppm to RATIO STRENGTH milligrams percent.
100 ÷ 0.0005= 200,000 200 mg x 1 g x 1 dL = 0.002 g/mL
= 1:200,000 1 dL 1000 mg 100 mL

3. CONVERT 0.03% to mg%. 0.002 x 100= 0.2% w/v → 0.2% x 1000= 200 mg%
0.03% x 100= 30 mg%
(b) How many milligrams of cholesterol would be present in a
4. CONVERT 0.0005% to PPM 10 mL sample of the patient’s serum?
0.0005 x 1,000,000= 5PPM 200 mg x x (mg) = 20 mg
100 100 mL 10 mL

5. CONVERT 2:3 to % PROOF STRENGTH


2 X 100= 66.67% Proof gallons = wine gallons x %v/v strength
3 50% v/v

6. CONVERT 25% to RATIO STRENGTH. Proof gallons = wine gallons x proof strength
100 ÷ 25%= 1:4 100 proof

TRY THIS! 1. How many proof galloons are there in 5 wine galloons of
1. 35: 100 → % 75% alcohol?
35 x 100= 35% COMPUTATION:
100 Proof gallons = wine gallons x %v/v strength
50% v/v
2. 45% → RS
100÷45%= 1:22.22 = 5 x 75% = 7.5 proofs
50% v/v
3. 8 PPM → RS
8 x 100= 0.0008% 2. How many wine galloons are there in 20 proof galloons with
1,000,000 proof strength of 40?
COMPUTATION:
100 ÷ 0.0008= 125,000 Proof gallons = wine gallons x proof strength
= 1:125,000 100 proof

4. 8 PPB → RS WG x PS = PG x 100 → WG= PG x 100


8 x 100= 0.0000008 PS PS PS
1,000,000,000 = 20 x 100 = 50 wine gallons
40
100 ÷ 0.0000008= 125,000,000
= 1:125,000,000
UNITS OF POTENCY STOCK SOLUTION
Measure of potency for some antibiotics, endocrine products, How much (in milliliters) of a 1:50 (2%) stock solution of
vitamins ephedrine sulphate should be used in compounding the
Examples: following prescription?
- Insulin Solution or Suspension
Rx
Most commonly used drug in units Ephedrine Sulphate 0.25%
- U-40, Rose water ad 30 mL
- U-100,
- U-500 COMPUTATION:
o mean 40, 100 and 500 USP Q1= Q2 x C2
C1
Insulin Units per mL – Ampicillin Sodium = 30 mL x 0.25% = 3.75 mL
 845 –988 ug of Ampicillin per mg 2%

U50 = 50 units / mL How many milliliters of a water should be added to 80 mL of a


U100 = 100 units / mL 20% w/v aqueous solution to prepare a 3% w/v solution?
COMPUTATION:
How many mL of U-100 insulin should be used to obtain 40 Q2= Q1 x C1 → 80 mL x 20% = 533.33 mL-80= 453.33 mL
units of insulin? C2 3%
COMPUTATION:
100 units x 40 units = 0.4 mL ALLIGATION
1 mL x (mL) - Method used to solve problems that involve mixing
two products of different strengths to form a product
If neomycin sulfate has a potency of 600mcg of neomycin per having a desired intermediate strength.
mg how many mg of neomycin sulfate would be equivalent in - Arithmetical method of solving problems that involve
potency to 1mg of neomycin? the mixing of solutions or mixtures of solids
COMPUTATION: possessing different percentage strengths
600 mcg → 0.6 mg
Two Methods
1 mg N SO4 x x (mg) = 1.67 mg  Alligation Medial- MAD
0.6 mg 1 mg N  Alligation Alternate- desired percentage

DILUTIONS AND CONCENTRATIONS ALLIGATION MEDIAL


Q1 x C1 = Q2 x C2 - Method by which the “weighted average” percentage
strength of a mixture of two or more substances of
Q1: Quantity 1 known quantity and concentration may be quickly
C1: Concentration 1 calculated
Q2: Quantity 2
C2: Concentration 2 1. A pharmacist mixes 200g of 10% mupirocin ointment, 450g
of 5% mupirocin ointment, and 1000g of petrolatum as the
If 500 mL of a 15% v/v solution are diluted to 1500 mL, what diluent. What is the % of mupirocin in the finished product?
will be the percentage strength (v/v)? COMPUTATION:
COMPUTATION: 200 g x 10%= 20
x= C2 450 g x 5%= 22.5
1000 g x 0%= 0
Q1 x C1 = Q2 x C2 1650 g 42.5
Q2 Q2
C2= Q1 x C1 42.5 g x 100= 2.58%
Q2 1650 g
= 15 % x 500 mL = 5%
1,500 mL 2. What is the percentage (v/v) of alcohol in a mixture of
3000mL of 40% (v/v) alcohol, 1000mL of 60% (v/v) alcohol and
DILUTIONS OF ALCOHOLS AND ACIDS 1000mL of 70% (v/v) alcohol?
How much water should be mixed with 5000 mL of 85% v/v COMPUTATION:
alcohol to make 50% v/v alcohol? 3000 mL x 40%= 1200 mL
COMPUTATION: 1000 mL x 60%= 600 mL
Q1 x C1 = Q2 x C2 1000 mL x 70%= 700 mL
5000 mL 2500 mL
Q2= Q1 x C1 → 5000 mL x 85% = 8,500 mL
C2 50% 2500 mL x 100= 50% v/v
C2-C1 → 8,500-5000= 3500 mL 5000 mL
ALLIGATION ALTERNATE ALLIGATION WITH 4 CONCENTRATIONS
1. Mixture of 95%(v/v) alcohol and 50%(v/v) alcohol will result In what proportions may a manufacturing pharmacist mix
to 70%(v/v) alcohol. Find the proportion of 95% and 50 % 20%, 15%, 5% and 3% zinc oxide ointment to produce a 10%
alcohol to make 150 mL of 70% alcohol? ointment
COMPUTATION: COMPUTATION:
95% 20% 20% 7%
70% 15% 5%
50% 25% 10%
20:25 → 4:5 5% 5%
FOR 95% 3% 10%
5 parts x x = 83.33 mL of 95% 7:5:5:10
9 parts 150 mL
A. Mole (n)
FOR 50% Mole = g
4 parts x x = 66.67 mL of 50% MW (g/mole)
9 parts 150 mL
What is the mole and millimole of a 50g of NaCl. (MW NaCl =
83.33 + 66.67= 150 mL 58.5 g/mol)
COMPUTATION:
2. A pharmacist has a 70% alcoholic elixir and a 20% alcoholic Mole = g = 50 g = 0.8547 moles
elixir. He needs a 30% alcoholic elixir to use as a vehicle for MW 58.5 g/mol
medications. In what proportion must the 70% elixir and the
20% elixir be combined to make a 250mL 30% elixir? Millimole= 0.8547 x 1000= 854.7 millimoles
COMPUTATION:
70% 10% Millimole
30% Millimole = mole x 1000
20% 40% - A mole is the molecular weight of a substance in
10:40 → 1:4 grams. A millimole is 1/1000 of the molecular weight
in grams.
ALLIGATION WITH 3 CONCENTRATIONS
1. A hospital pharmacist want to use three lots of zinc oxide A flavored Potassium chloride packet contains 1.25 g of
ointment containing, respectively, 50%, 20% and 5% zinc Potassium chloride. How many millimoles of Potassium
oxide. In what proportion and volume should they be mixed to chloride is represented in each packet?
prepare a 500mL of 10% zinc oxide ointment? COMPUTATION:
COMPUTATION: K= 35.5
50% 5% Cl= 39
20% 5% 74.5 g/mol
10%
5% 40%+10%= 50% Mole= 1.25 g = 0.0168 x 1000= 16.78 millimoles
5:5:50 → 1:1:10 74.5 g/ mol

1 part x x (mL) = 41.67 mL of 50% A solution contains 420 mg of Sodium bicarbonate per
12 parts 500 mL teaspoon. How many millimoles are contained in a 50 ml vial?
COMPUTATION:
1 part x x (mL) = 41.67 mL of 20% 420 mg NaHCO3 x x = 4,200 mg NaHCO3
12 parts 500 mL 5 mL 50 mL

10 parts x x (mL) = 416.67 mL of 5% Na= 23


12 parts 500 mL H= 1
C= 12
2. A hospital pharmacist want to use three lots of zinc oxide O= 16x3= 48
ointment containing, respectively, 90%, 50% and 5% zinc 84 g/mol
oxide. In what proportion and volume should they be mixed to
prepare a 500mL of 75% zinc oxide ointment? 4200 mg= 4.2 g
COMPUTATION: Mole= 4.2 g = 0.05 x 1000= 50 millimoles
90% 70%+25%= 95% 84 g/mol
75%
50% 70% B. Milliequivalent
f → factor → equivalents/mole
5% 25%
→ Valence → total number of positive charges (salts)
95:70:25 → 19:14:5
→ total number of replaceable hydrogens (acids/bases)
EXAMPLE:
19 parts x x (mL) = 250 mL of 90%
HNO3 (nitric acid)- 1 hydrogen= 1 factor
38 parts 500 mL
H2SO4 (sulfuric acid)- 2 hydrogen= 2 factor
CH3COOH (acetic acid)- 1 factor
14parts x x (mL) = 184.21 mL of 50%
38 parts 500 mL

5 parts x x (mL) = 65.79 mL of 5%


38 parts 500 mL
gEW = MW(g/mol) Ferrous FeSO4 152 76 76,000
sulfate V= 2
f (equivalents/mole)
Aluminum Al2(CO3)3 104 17.33 17,330
mEq = MW carbonate V= 3
f x 1000 F= 6

Formula of ions and anions, their atomic/molecular weights, - The quantities of electrolytes administered to
charge/valence patients are usually expressed by the term
VALENCE= 1 milliequivalents (mEq).
ION FORMULA ATOMIC EQUIVALENT - The reason that weight units (mg, g) are not used is
WEIGHT WEIGHT because the electrical activity of the ions, which in
Lithium Li+ 7 7 this instance is important, may be best expressed as
Sodium Na+ 23 23 mEq.
Potassium K+ 39 39
Ammonium NH4+ 18 18 mEq= mg x valence
AW
VALENCE= 1 mg= mEq x AW
ION FORMULA MOLECULAR GRAM valence
WEIGHT EQUIVALENT
WEIGHT 1. What is the concentration, in milligrams per milliliter, of a
Acetate C2H3O2- 59 59 solution containing 2 meq of potassium chloride (kcl) per
Bicarbonate HCO3- 61 61 milliliter?
Chloride Cl- 35.5 35.5 COMPUTATION:
x= mg/mL
Gluconate C6H11O7- 195 195
Dibasic H2PO4- 97 97
2 mEq KCl/mL
Phosphate
mEq= mg x v → mg= MW x mEq
VALENCE= 2
MW v
ION FORMULA MOLECULAR GRAM
= 74.5 g/mol x 2 mEq = 149 mg/mL
WEIGHT EQUIVALENT
1
WEIGHT
2. How many milliequivalents of potassium chloride are
Magnesium Mg++ 24 12 represented in a 15 ml dose of a 10% (w/v) potassium chloride
Calcium Ca++ 40 20 elixir?
Ferrous Fe++ 56 28 COMPUTATION:
x= mg/mL
VALENCE= 2 10 g x x (g) = 1.5 g x 1000 → 1,500 mg
ION FORMULA ATOMIC EQUIVALENT 100 mL 15 mL
WEIGHT WEIGHT
Carbonate CO3-- 60 30 mEq= mg x v
Sulfate SO4-- 96 48 MW
Hydrogen HPO4-- 96 48 = 1,500 mg x 1= 20.13 mEq
Phosphate 74.5

VALENCE= 3 C. Mole fraction (X)


ION FORMULA ATOMIC EQUIVALENT - It is the ratio of the number of moles of one
WEIGHT WEIGHT component to the number of moles of all
Aluminum AL+++ 27 9 components present
Ferric Fe+++ 56 18.7 - Determine the number of moles, n, of all
components
VALENCE= 3 - Divide n of the desired component with the total ntot
ION FORMULA ATOMIC EQUIVALENT of all components, multiply by 100%
WEIGHT WEIGHT
Citrate C6H5O7--- 189 63 What is the mole fraction of sodium chloride (MW NaCl = 58.5
g/mol) in a normal saline solution (NSS)? (MW H2O = 18
Chemical Molecular Gram Milliequivalent g/mol)
Formula Weight Equivalent Weight COMPUTATION:
Weight
NSS= 0.9% → 0.9 g NaCl & 100 mL H2O
Sodium NaCl 58.5 58.5 58,500
chloride V= 1 1 mol x x = 0.0154 moles
58.5 g 0.9 g
Magnesium MgSO4 119 59.5 59,500
sulfate V= 2 1 mol x x = 5.5556 moles
18 g 100 g
Calcium CaCO3 100 50 50,000
carbonate V= 2
TOTAL: 5.5710 moles
Aluminum Al(Cl)3 133.5 44.5 44,500
chloride V= 3 MF (NaCl) = 0.0154 = 0.0028 x 100= 0.28%
Ammonium NH4Cl 53.5 53.5 53,500 5.5710
chloride V= 1 MF (H2O) = 5.5556 = 0.9972 x 100= 99.72%
5.5710
MOLARITY (M) MOLALITY (m) NORMALITY (N) 2. Convert 0.5 N Boric Acid to M.
Mole/L Mole/Kg g of solute ÷ EW M= N = 0.5 N = 0.5 M
L F 1
Mole= g/MW EW= MW/f
G. Osmolarity
D. Molarity (M) - Osmotic pressure is proportional to the total number
- A molar solution contains a mole (one-gram of particles in solution
molecular weight) in a liter of solution or one - The unit used to measure osmotic concentration is
millimole per milliliter of solution. the milliosmol (mOsmol)
- A mole is the molecular weight expressed in grams.
- A millimole is one thousandth part of a mole. 1. How many mOsmols are represented in a L of 0.9% NaCl
solution? MW 58.5
M= no. of mole solute = g solute/molar mass = g solute COMPUTATION:
no. of L of sol’n L of sol’n (mol w)(L of sol’n) 0.9 g x 100 mL = 9 g/L
100 mL 1L
E. Molality (m)
- A molal solution contains a mole (one-gram mOsmol/L= wt. of substance (g/L) x no. of species x 1000
molecular weight) in one thousand grams of solution MW (g/mol)
= 9 g/L x 2000 = 307.69 mOsmols/L
m= no. of mol solute = g solute/mol wt = g solute 58.5 g/mol
kg of solvent kg of solvent (mol wt)(kg of solvent)
2. A solution contains 1% w/v Al2(SO4)3.14H20 (MW = 600).
F. Normality (N) Assuming there is a complete ionization, calculate the
- A normal solution contains one-gram equivalent osmolarity of the solution
weight of solute in a liter of solution or one-gram COMPUTATION:
milliequivalent weight in a milliliter of solution 1 g x 1000 mL= 10 g/L
100 mL 1L
N= no. of g-eq wt = g solute/eq wt = g solute
L of sol’n L of sol’n (eq wt)(L of sol’n) mOsmol/L= wt. of substance (g/L) x no. of species x 1000
MW (g/mol)
N= Mxf = 10 g/L x 5000 = 83.33 mOsmols/L
M= N/f 600 g/mol

PROBLEM: Dissolve 5.00 g of NiCl2•6 H2O (237.7 g/ mol) in 3. A solution contains 10% of anhydrous dextrose in water for
enough water to make 250 mL of solution. Calculate the injection. How many milliosmoles per liter are represented by
Molarity. this concentration? Molecular weight of anhydrous dextrose=
COMPUTATION: 180
M = g/MW x L or mole/L COMPUTATION:
10 g x 1000 mL= 100 g
5 g x 237.7 = 0.0210→ 0.0210 = 0.0840 M 100 mL 1L
x (mL) 1 mol 0.25 L
mOsmol/L= wt. of substance (g/L) x no. of species x 1000
USING MOLARITY MW (g/mol)
What mass of oxalic acid, H2C2O4, is required to make 250. = 100 g/L x 1000= 555.56 mOsmols/L
mL of a 0.0500 M solution? 180 g/mole

g= (M)(MW)(L)  Two solutions that have the same osmotic pressure


= 0.0500 M (90 x 0.25 L) = 1.13 g are termed isosmotic. Many solutions intended to be
mixed with body fluids are designed to have the same
TRY THIS MOLALITY PROBLEM osmotic pressure for greater patient comfort,
 25.0 g of NaCl is dissolved in 5000. mL of water. Find efficacy, and safety.
the molality (m) of the resulting solution.  A solution having the same osmotic pressure as a
COMPUTATION: specific body fluid is termed isotonic (meaning of
m= mole → 1 mole x x (mole) = 0.4274 mol equal tone) with that specific body fluid.
kg 58.5 g 25 g  Solutions of lower osmotic pressure than that of a
= 0.4274 = 0.0854 m body fluid are termed hypotonic (swell), whereas
5 kg those having a higher osmotic pressure are termed
hypertonic (crenation).
TRY THIS!!!
1. What is the Molarity and Normality of a solution prepared METHODS OF ADJUSTING ISOTONICITY
by dissolving 8.050g Ba(OH)2 in 1500ml of solution? Class I
COMPUTATION: 1. Sodium Chloride Equivalent Method
1 mole x x (mole) = 0.0471 mol 2. Freezing Point Depression Method/ Cryoscopic
171 g 8.050 g Method

M= mole = 0.0471 = 0.0314 M Class II


L 1.5 1. White-Vincent's Method
N= g = 8.050 g = 0.0628 N 2. Sprowl’s Method
EW x L 85.5 x 1.5 mL
In practice, a 0.90% w/v sodium chloride solution is considered How many grams of sodium chloride should be used in
isotonic with body fluids. compounding the following prescriptions?
Rx
gramssolute per 1000 g of water = 0.52 x molecular weight Pilocarpine nitrate (E=0.23) 0.3 g
1.86 x dissociation (i) Sodium chloride q.s.
Purified water ad 30.0 mL
SODIUM CHLORIDE EQUIVALENT METHOD Make isoton. sol.
- A sodium chloride equivalent, E value, is defined as Sig. For the eye.
the weight of sodium chloride that will produce the
same osmotic effect as 1 g of the drug. COMPUTATION:
Step 1: 0.23 x 0.3= 0.069 g
EXAMPLE CALCULATIONS OF THE SODIUM CHLORIDE Step 2:
EQUIVALENT 0.9 g x x = 0.27 g
100 mL 30 mL
The sodium chloride equivalent (E) of a substance may be Step 3: 0.27 g – 0.069 g = 0.20
calculated as follows:
How many grams of boric acid should be used in compounding
=MWNaCl x i factor of substance the following prescriptions?
i factor MW of substance
Rx
= 58.5 x i factor of substance Phenacaine HCl 0.2 1% (0.6)
1.8 MW of substance Chlorobutanol 0.24 ½% (0.3)
Boric acid q.s.
i factor Purified water ad 60.0
Nonelectrolytes and substances of slight dissociation 1 Make isoton. sol.
Substances that dissociate into 2 ions 1.8 Sig. One drop in each eye
Substances that dissociate into 3 ions 2.6
Substances that dissociate into 5 ions 3.4 COMPUTATION:
Substances that dissociate into 2 ions 4.2 1 g x x = 0.6
100 mL 60 mL
Computation for the i factor
 Zinc chloride is a 3-ion electrolyte, dissociating 80% in 0.5 g x x = 0.3
a certain concentration. Calculate its dissociation (i) 100 mL 60 mL
factor.
COMPUTATION: Step 1:
Zn= 80% P: 0.2 x 0.6= 0.12
Cl= 80% C: 0.24 x 0.3= 0.07
Cl= 80% TOTAL: 0.192 g
Undissociated= 20%
TOTAL= 260 ÷ 100= 2.6 Step 2:
0.9 g x x = 0.54 g
Computation for the E value 100 mL 60 mL
 Papaverine hydrochloride (MW 376) is a 2-ion
electrolyte, dissociating 80% in a given concentration. Step 3: 0.54 g - 0.192 g= 0.348
Calculate its sodium chloride equivalent.
Step 4: 0.348 ÷ 0.52= 0.67 g
E= 58,5 x 1.8 = 0.16
1.8 376 How much potassium nitrate could be used to make the
following prescription isotonic?
Calculate the sodium chloride equivalent for timolol maleate,
which dissociates into two ions and has a molecular weight of Rx
432.2 Silver nitrate (0.33) 1:500 w/v (0.12)
Make isoton. sol 60 mL
= 58.5 x i factor of substance Sig. For eye use
1.8 MW of substance
= 58.5 x 1.8 = 0.14 COMPUTATION:
1.8 432.2 1 g x x (g) = 0.12 g
500 mL 60 mL
E-VALUE METHOD
Steps to follow Step1: 0.33 x 0.12= 0.04 g
1. Determine the amount of NaCl present in the
preparation Step 2:
2. Amount of NaCl represented by the substance 0.9 g x x = 0.54 g
3. Subtract amount of NaCl present in the prep’n to 100 mL 60 mL
amount of NaCl rep by the substance
4. If an agent other than sodium chloride, (boric acid), Step 3: 0.54 g – 0.04 g= 0.50 g
divide the amount of sodium chloride (step 3) by the
sodium chloride equivalent of the other substance Step 4:
E= (58.5/1.8) x (1.8/101) = 0.58
0.50 g = 0.86 g If more than one ingredient is contained in an isotonic
0.58 g preparation, the volume of isotonic solution obtained by
mixing each drug with water are additive.
Freezing Point Depression Method
 Dependent only on the number of particles in the V= [(w1 x E1) + (w2 x E2) + . . . (wg x Eg)] x 111.1
solution.
Blood plasma has a freezing point of −0.52 {or freezing point Make the following prescription isotonic with tears
depression of 0.52, i.e., (− [−0.52])}.
Rx
Ephedric Sulfate 0.5
Boric Acid 0.2
Ammonium Chloride 0.25
Purified Water ad 60.0

The E values of ephedrine sulfate, boric acid, and ammonium


chloride are 0.23, 0.52, and 1.08, respectively.

COMPUTATION:
V= w x E x 111.1
= (0.5 g x 0.23 x 111.1) + (0.2 x 0.52 x 111.1) + (0.25 x 1.08 x
111.1)
= 54.33 mL of H2O

Make the following solution isotonic with tears

Procaine Hydrochloride (0.21) 1% (0.6 g)


Sodium Chloride q.s.
Purified Water ad 60
Compound the prescription: Freezing point depression (Tf) of
1% atropine solution is 0.07
1 g x x = 0.6 g
Rx 100 mL 60 mL
Atropine Sulfate 1%
Sodium Chloride q.s. COMPUTATION:
Purified Water ad 100 V= w x E x 111.1
Make isoton. sol. = 0.6 g x 0.21 x 111.1
Sig. One drop in each eye = 13.998 or 14 mL
COMPUTATION:
Step 1: 0.52-0.07= 0.45 SPROWLS METHOD
Step 2: V= 0.3 g x E x 111.1
0.9 x x = 0.78 g
0.52 0.45 DISCOUNTS
Provided by supplier may be based on the quantity on buying
How many milligrams each of NaCl and dibucaine HCl are and/or payment of invoice within a specific period.
required to prepare 30 mL of 1% solution of dibucaine HCl
- Maybe a means of increasing gross profit.
isotonic with tears?
 To make the solution isotonic, the freezing point 1. The list price of an antihistamine elixir is P65 per pint, less
must be lowered to -0.52⁰C 40%. What is the nest cost per pint of elixir?
 A 1% solution of dibucaine HCl has a freezing point COMPUTATION:
lowering of 0.08⁰C 65 x 0.40= 26
COMPUTATION: 65-26= 39
Step 1: 0.52-0.08= 0.44
Step 2: 2. Vitamin A capsule is sold at a price of Php 12, less 25%,
0.9 g x x = 0.76 g/100 mL what will be the cost per cap?
0.52 g 0.44 COMPUTATION:
12 x 0.25= 3
0.76 g x x (g) = 0.228 g → 228 mg 12-3= 9
100 mL 30 mL

White-Vincent Method
- provided a method for readily finding the correct
volume of water in which to dissolve a drug to
produce a solution iso- osmotic with tears.
- followed by the addition of an isotonic vehicle to
bring the solution to the final volume.

V= w x E x 111.1
where:
w= weight of the drug in grams
E= sodium chloride equivalent
SERIES DISCOUNT PHARMACOECONOMIC ASPECT OF PROOF STRENGTH
- A technique used in promotional deals ALCOHOL AND TAXATION
- Chain of deductions that can be converted to a single - Purchase of Alcohol for Pharmaceutical use can
discount equivalent. receive a refund on the alcoholic tax.

The list price of 12 bottles (100 caps) of analgesic tablet is If the the tax on alcohol is quoted at P135.0 per proof gallon.
P360, less trade discount of 33.33%. If purchased in quantity How much tax would be collected on 10 wine gallons of
of per dozen an additional discount of 10% is allowed by the alcohol marked at 190 proofs
trader, plus a 2% cash discount for payment within 10 days. COMPUTATION:
Calculate the net cost of 144 bottles when purchased under Proof gallons = wine gallons x proof strength
the terms of the offer. 100 proof
COMPUTATION: = 10 x 190 = 19 PG
12 bottles x 144 bottles = 4,320 100
360 x P135 x x = P2,565
1 PG 19 PG
1-0.3333= 0.6667
1-0.10= 0.9 CONTEMPORARY COMPOUNDING RECONSTITUTION:
1-0.02= 0.98 - Powders or crystals occupy a greater volume than
after reconstitution
0.6667 x 0.9 x 0.98= 0.59% - Once the powder or crystals are dissolved in the
solvent, the volume may be more than the amount of
4,320 x 0.59= 2,539.91 solvent added

A promotional deal provides a trade discount of 33.5%, an off- Label instruction for an ampicillin prodct calls for the addition
invoice allowance of 12% and a display allowance of 5%. of 78mL of water to make 100mL of constituted suspension
Calculate the single discount equivalent to these deductions. such that each 5 mL contains 125mg of ampicillin. Calculate
COMPUTATION: the volume of dry powder in the product and the total content
1-0.335= 0.665 of ampicillin?
1-0.12= 0.88 COMPUTATION:
1-0.05= 0.95 100-78 mL= 22 mL

0.665 x 0.88 x 0.95= 0.56 125 mg x x = 2,500 mg


1-0.56= 0.44 x 100= 44% 5 mL 100 mL

MARKUP Using the product from last problem. If the Physician desires
- Difference between the cost of merchandise and its an ampicillin concentration of 100mg/5mL. How many
selling price. milliliters should be added to the dry powder?
- Margin of profit or gross profit COMPUTATION:
- Sometimes used interchangeably with margin of 125 mg/5 mL= 2.5%
profit or gross profit 100 mg/5 mL= 2%
- Refers to the difference between the cost of
merchandise and its selling price. Q1C1 = Q2C2
Q2= Q1C1
MARKUP PERCENTAGE C2
- AKA percentage gross profit = (2.5%)(78)
- Refers to the mark up divided by the selling price. 2%
= 97.5 mL
Cost = Selling price – gross profit
The label of a dry powder for oral suspension states that when
GP on the SP = cost + gross profit 111mL of water are added to the powder, 150mL of a
GP on the cost suspension containing 250mg of ampicillin per 5mL are
SP = cost + (cost x gross profit) prepared. How many milliliters of purified water should be
used to prepare, in each 5mL of product, the correct dose of
1. The cost of 100 antacid tablets is P21. what should be the ampicillin for a 60lbs child based on the 8mg/kg dosing.
selling price per 100 tablets to yield a 66% gross profit on the COMPUTATION:
cost? 150 mL-111 mL= 39 mL
COMPUTATION:
SP = cost + (cost x gross profit) 250 mg x x = 7,500 mg
= 21 + (21 x 0.66) 5 mL 150 mL
= 34.86
60 lbs= 27.27 kg
2. The cost of 120 antacid tablets is Php 25. What would be
the selling price per tablet to yield a 65% gross profit on cost? 8 mg : 1 kgs : : x : 27.27 kg
COMPUTATION: = 218.16 mg
SP = cost + (cost x gross profit)
= 25 + (25 x 0.65) 218.16 mg x 7,500 mg = 171.89 mL-39 mL= 132.89 mL
= 41.25 5 mL x
Compounding, Dispensing, and Incompatibilities

PHARMACY →is the art or practice of preparing & preserving the drugs. Important role of RPh (ASIDE
DISPENSING): compounding of drugs prescribed by the physician.
COMPOUNDING MANUFACTURING
• Preparation, mixing, assembling, packaging, or labeling of a drug or • Production, preparation, propagation (can be more than
device thousands depending on the manufacturing order),
• Result of a practitioner’s prescription drug order or initiative based conversion or processing of a drug or device, either
on pharmacist/patient/prescriber relationship in the course of directly or indirectly, by extraction from substances of
professional practice or the purpose of, as in incident to research, natural origin or independently by means of chemical or
teaching or chemical analysis and not for sale dispensing biological synthesis, and includes any packaging or
• NOTES: repackaging of the substance(s) or labeling or relabeling of
-It was performed since the beginning of the pharmacy profession, its container, and the promotion and marketing of such
hence, professional prerogative drugs or devices
-Around 1990s → apothecaries performed this already → have • Includes the preparation and promotion of commercially
specific area for on-the-spot compounding (e.g. suspension, available products from the bulk compounds for resale by
ointment) pharmacies, practitioners, or other persons.
-IT INCLUDES THE PREPARATION OF DRUGS • NOTES:
-SMALL SCALE -End product of manuf: PRODUCTS (which have
-compounding: you just need to prepare, mix ingr., assembly, then undergone different processes, e.g. packaging, labeling)
place in proper packaging such as amber bottle, wide mouth, - LARGE SCALE
ointment jar, and lastly label with the name of the drug & patient) -more advance than compounding
-REMEMBER: NOT FOR SALE -PURPOSE: for SELLING the final product → that will be
-PURPOSE: in response with the Dr. prescription drug order (e.g. commercialize in the pharmacies
compounded Rx which includes the instructions of physician;
extemporaneous compounding, is the process of compounding
when the meds in Rx is not available → not available commercially
→ so personalized); on the other hand, STERILE COMPOUDING →
preparation of parenterals
-A TRUE PHARMACEUTICAL SERVICE INVOLVE NOT ONLY THE
REDISTRIBUTION OF A COMMERCIALLY AVAILABLE COMMODITY
BUT ALSO COMPOUNDING SERVICES (since most of the people only
thought that drugs are ALL readily made already)
-MOST COMMON on HOSPITAL (compounding area) & even
community pharmacy → mostly topical e.g. ointments, suspensions

EXTEMPORANEOUS COMPOUNDING → it is defined as the timely preparation of a drug product according to a


physician’s prescription, a drug formula, or a recipe in which amounts of the ingredients are calculated to meet
the needs of a particular patient or group of patients.
• prepare (based on physician’s prescription), mix (correct amount of ingredients by computation), & assembles
the drug product
• ONE OF THE PROFESSIONAL FUNCTIONS OR PRACTICES OF PHARMACISTS
• can be applied in research, chemical analysis, teaching or academe
• COMPOUNDED DRUGS ARE NOT AVAILABLE FOR RESALE (preparing again for large scale selling); personalized
esp. when not commercially available (different preparation, DF, dose, etc), but, rather, are personal and
responsive to the patient’s immediate needs.
• PURPOSE: prepare an individualized drug treatment for a patient based on an order from a duly licensed
prescriber
• FUNDAMENTAL DIF BETWEEN COMPOUNDING & MANUFACTURING: existence of
pharmacist/prescriber/patient relationship that controls the compounding of the drug preparation
(REMEMBER THE RELATIONSHIP: pharmacist-prescriber-patient relationship)
• HOWEVER, in the Ph not that popular, unlike on US → because of the availability of commercialized drugs.
Although the need for need for extemporaneous compounding has decreased due to the availability of
standardized commercial drug products, the need for extemporaneous preparations continues especially in
cases such as:
o Dispensing pediatric medications (unlike on adults → needed to be in specialized doses so need to adjust
the dose e.g. paper tabs)
o It may be necessary to dilute standard adult strengths of drugs to obtain measurable doses (adjusting of
dose again)
o Combining topical products for use in dermatology: for example, the use of certain antibiotic with a
specific anti-inflammatory agent (POPULAR EXAMPLE OF COMPOUNDING)

DIFFERENTIATE COMPOUNDING & MANUFACTURING BY GIVING 3 DEVICES/3 EQUIPMENT USED IN THE


PREPARATION OF SOLUTIONS:
• COMPOUNDING → In school → Tool box: beaker, mortar & pestle, stirring rod, graduated cylinder only for
SMALL SCALE PRODUCTION
• MANUFACTURING → LARGE SCALE → mixers, big containers, filtration systems

PREPARATION vs. PRODUCTS


• The USP uses the term PREPARATION to refer to compounded prescriptions and the term PRODUCTS to refer
to manufactured pharmaceuticals
• END PRODUCT OF COMPOUNDING: Preparation
• END PRODUCT OF MANUFACTURING: Product

COMPOUNDED PRESCRIPTION
• a prescription with two or more components, requiring compounding or mixing the components
• TYPES OF COMPOUNDED PRESCRIPTION:
o Isolated prescription → one that the pharmacist is not expecting to receive nor expecting to receive again
(ONE TIME PREPARATION ONLY)
o Routine prescription → one that the pharmacist may expect to receive in the future on a routine basis, and
there may be some benefit to products quality to standardize preparations like this (preparation protocols
on file) → REPEATING ORDER/HAVE PATTERN
o Batch prepared prescription → one of which multiple identical units are prepared as a single operation in
anticipation of a receipt of prescription → Pharmacist should be capable, have experience, & willing to do so
because this is PER BATCH COMPOUNDING

FACTORS TO CONSIDER IN COMPOUNDING:

• Stability
• Compounding support
• Training & experience of pharmacist → skill-related examination like in the States to know the
incompatibilities in mixing, & practices in compounding
• Environmental/compounding facility
• Equipment
• Formulas
• Chemical supplies

1. Stability
• Consider:
o Chemical
o Physical
o Microbiological stability
• Assign beyond-use-date → the time and date after which the compounded product should not be stored
nor transported (not usable anymore)
• For non-sterile compounded drug product packaged in tight, light, resistant containers and stored at the
controlled room temperature:
o For non-aqueous liquids and solids formulation e.g. tablets (manufactured drug product/commercially
drug product is the source of the active ingredients) → beyond-use date is not later than 25% of the
time remaining until the product expiration date or 6 months, whichever is earlier.
• For product with the a USP or NF substance (coming from scratch not from commercially produced
products) as the source of active ingredients → the beyond-use date is lesser than or equal to 6 months
• For ALL other formulations → the beyond-use date is not later than intended duration of therapy or 30
days, whichever is earlier unless there is a supporting valid scientific stability information applicable to the
specific preparation
• PRACTICE: beyond used date for the formulation of the ff: non-aqueous formation e.g. capsule without
water in it (6 months maximum BUD), oral formulation with water e.g. suspension (14 days under
refrigeration maximum BUD), topical containing water e.g. ointment (30 days maximum BUD) → IF WATER
CONTAINING PREPARATION THE BUD IS SHORTER BECAUSE OF MICRO GROWTH

• Commonly recommended STORAGE TEMPERATURES:


o Freezer between (-) 25 degrees C & (-) 10 degrees C
o Cold temperature: not exceeding 8 degrees C
o Refrigerator: thermostatically controlled between 2 degrees C & 8 degrees C
o Cool temperature: between 8 degrees C & 15 degrees C
o Controlled room temperature: thermostatically controlled between 20 degrees C & 25 degrees C with
allowed excursions between 15 to 30 degrees C

2. Equipment → based on the type and extent of the services one chooses to provide
• Equipment surfaces that come in contact with ingredients or compounded preparations should not be
reactive, additive, or sorptive so that the purity of the preparation is not compromised. (SHOULD NOT
INTERFER WITH THE INGREDIENTS/PRODUCT COMPOUNDING)
• EQUIPMENTS USED IN COMPOUNDING:
1. Laminar flow hood → for aseptic compounding of sterile solutions (low particulate matter → so there
are areas like clean room before entering to control/limit the no. of particulate matter)
2. Refrigerator → to maintain temperatures as specified in the USP; used in storage
3. BALANCES → an instrument for determining the relative weights of substances (weighing balances in
manufacturing are more enormous as to compare with compounding balances e.g. analytical balances
▪ should be selected correctly for the specific task at hand, used skillfully (should know what type of
balance to use, & how to read the measurement), protected from damage (sensitive → so should be
maintained), and checked periodically (for calibration → to get accurate results)
▪ ALL BALANCES MUST HAVE A CERTAIN DEGREE OF ERROR THAT CAN BE TOLERATED TO PRESCRIPTION,
COMPOUNDING, & in the PHARMACEUTICAL MANUF → the USP allow maximum error of 5% on a
single weighing operation since the SR (sensitivity requirement) of the balance represent the absolute
error in using that balance
▪ The percentage error will depend on the amount of the drug & it will increase as the amount of drug
DECREASES
▪ For the MWQ (minimum weighable quantity) → usually no more than 5% error
3.1 PRESCRIPTION BALANCES → uses the taut wire frame or torsion principle e.g. CLASS III balance →
have a maximum maintenance sensitivity of 6 mg with no load & with full load, used to weigh
quantities up to 60 g
o CLASSIFICATION OF PRESCRIPTION BALANCE: single-beam (equal-arm or unequal-arm), compound
lever, torsion, and electronic
o CLASS A or CLASS III (used to weigh up to 60 grams of ingredient depending on the stated capacity)
PRESCRIPTION BALANCES
o Other balances may be used, provided they give equivalent or better accuracy → like electronic
single-pan balance (balances that have sensitivity of less than 10 mg)
o THEREFORE, ELECTRONIC BALANCES are MORE ACCURATE than CLASS A/III
o All prescription departments like in HOSPITAL PHARMACY/COMPOUNDING AREAS must have at
least CLASS III BALANCE
o Pharmacists and technicians should be familiar with: capacity, sensitivity, readability, precision, &
accuracy
o CAPACITY → maximum weight that can be placed on the balance pan
o SENSITIVITY → the smallest weight that gives a perceptible change in the indicating element
o READABILITY → in the electronic balance (the smallest weight increment that can be read on the
digital displayed of the balance e.g. 0.001 g); on the double-pan balance (the smallest increment
determined by the value of a hash-mark on the graduated dial or weigh beam e.g. on metric scale of
a dial each mark = 0.01 g)
o PRECISION → reproducibility of the weighing measurement as expressed by a standard deviation
o ACCURACY → closeness of the displayed weight as measured by the balance, to the true weight

PRACTICES FOR SOLVING MINIMUM WEIGHABLE QUANTITY

100%
MWQ = 2.5 mg x = 83.33 mg
3%

3.2 WEIGHING DISH → used in weighing the ingredients; usually made of aluminum or polystyrene
plastic, with a capacity of 5-250 ml (comes with different sizes)
o Very handy and securely contain substances being weighed
o Useful for weighing liquids because they have rigid sides
o More expensive than weighing papers

3.3 Weighing papers → usually used for POWDERS


o Glassine paper → preferred because → glazed, & transparent paper with a limited moisture
resistant
▪ Has surface that does not absorb materials placed on them
▪ Drugs and chemicals are easily slipped off for complete transfer
o Parchment paper → more acceptable for most purposes
▪ More absorbent than glassine → should not be used weighing thick liquids like coal tar
o Wax paper → transparent & water proof type of weighing paper
o Simple bond paper
4. SPATULAS:
▪ Stainless spatula with wooden or plastic handles
▪ Small double bladed, nickel stainless steel spatulas → AKA micro spatula
▪ Hard rubber or Teflon-coated stainless steel spatulas
▪ Flexible rubber spatulas → AKA rubber scrapers or rubber policeman

5. VOLUMETRIC DEVICES → used to measure the solvent/liquids used; consider the capacity of the volumetric
devices (TC = to contain, or TD = to deliver)
▪ Graduates: cylindrical and conical
-Conical graduates are the easiest to use with wide mouths, and bases are easiest to clean. Liquids may be
stirred in them with the aid of a stirring rod
-As the diameter of the graduate increase, the accuracy of the measurement decreases hence with this,
the conical graduate have narrow structure → MUCH ACCURATE READING, so no doubt conical cylinders
are more accurate than BEAKERS since the latter have larger diameter
-Conical graduates → are combination of wide mouths & narrow portion = more accurate than other
devices
▪ Pipettes – more accurate than conical graduate especially with small amount of liquids needed to be
measure
▪ Medicine dropper

6. FUNNELS → plastic or glass, with different capacities and different stern lengths and diameters
▪ Funnels with narrow stern diameter → used for transferring solutions (e.g. liquid) from one container
to another and used for filtering solutions
▪ Funnels with short larger diameter sterns/ POWDER FUNNELS→ are used for transferring powder
from mortars and other mixing vessels, and may also be used for transferring emulsions or
suspensions

7. MORTAR & PESTLE


▪ Wedgwood mortars → MOST FAMOUS & IDEAL, ideal for particle size reduction or TRITURATION, &
making emulsions but cannot be used in preparation that causes STAIN, drugs present in small
quantity, & for VERY POTENT or HAZARDOUS DRUGS
▪ Porcelain mortars → more attractive white lace surface, they provide less shearing efficacy compare
to wedgwood mortar, and like ceramic mortars it is also LESS DURABLE
▪ Ceramic mortars → almost similar to Wedgwood mortar however, this type of mortar have smoother
surface but with same uses & precautions with wedgwood mortar (DISAD: LESS DURABLE compare to
wedgwood mortar)
▪ Glass mortars → have the most beautiful texture → because it has smooth non-porous interior or
surface, used for drugs producing stains, useful for making solutions, diluting creams & lotion, & ideal
for making VERY POTENT or triturating HAZARDOUS DRUGS, and like Wedgwood → also used for
reducing particle size of powders esp. HARD CRYSTALS. However, not efficient in making emulsions.

COMPOUNDING
COMPOUNDING INGREDIENTS:
o Component → any ingredient for use in compounding a drug product
o Active ingredient → chemicals, substances, or other components of articles intended for the use in the
diagnosis, cure, mitigation, treatment or prevention of diseases
o Adjuvants → inert ingredients

Grade of ingredients
GRADE DESCRIPTION
Primary standard Highest purity: required for accurate volumetric analysis e.g. titration
Spectroscopic Grade Very high purity
Analytical Reagent Very high purity
ACS High purity; conforms to minimum standards set by the American Chemical Society
USP/NF Meets minimum purity standards: conforms to tolerance set by the USP/NF for contaminants
→ used in pharmacy for active ingr → USP; for inactive ingr/adjuvants→ NF
CP Refined, but still of unknown quality
Technical or commercial Undetermined quality
Unofficial FCC Certified to meet or exceed the specifications that are prescribed for your food, & chemicals
codex
Food grade Used for food or these chemicals have the clearance for use in foods
Cosmetics grade For cosmetics purposes

▪ FLAVORANTS – imparts pleasant flavor and often odor to a preparation


▪ Flavoring methodology:
1. Blending → blending of flavors e.g. fruit flavors that are blended with sour taste
2. Overshadow → these are flavors with longer & stronger taste that are added to the obvious taste
3. Physical → formation of insoluble compounds of the offending drug e.g. suspension
4. Chemical → uses chemical reactions like absorption or complexation
5. Physiological → anesthetizing the taste buds

Taste Flavor
Bitter Chocolate, anise
Sweet Fruit, berry, vanilla
Sour Citrus, rootbeer, anise, strawberry
Salty Butterscotch, maple, peach, melon, rasp berry

PRACTICE:
What is the appropriate flavoring methodology can be used for the following:
• Medications with sour taste → BLENDING (refer to table)
• Medications with bitter taste → Blending, physiological, & physical (in the form of suspension)

▪ SOLVENT
1. Water → universal solvent, can dissolve both ionic and polar solutes, if organic compounds INSOLUBLE
IN WATER but the SALT FORM is soluble in water
2. Aromatic water → most commonly used as flavoring agents, easily driven off from the solution, easily
salted-out by electrolytes
3. Syrup → solvent, flavorant, medicinals
4. Glycerin → best solvent for phenols, iodine, boric acid, borates, tannic acid, and cresol
5. Alcohol → good solvent for organic compounds such as alkaloids, glycosides, camphor, phenol, tannins,
balsams resins & for some inorganic salts
6. Elixirs → not preferred for salts because it accentuate (can EMPHASIZE) the saline taste
▪ Aromatic elixir, NF contains approximately 22% alcohol
7. Oils
▪ PMSC (peanut, corn, sesame, and mineral oil)
▪ Oleic acid is used as a base for alkaloids
▪ Olive oil for sprays and drops

▪ STIFFENING AGENTS → increase the thickness or hardness of the preparation use in ointments & creams
e.g. cetyl alcohol, paraffin, white & yellow wax

▪ SUPPOSITORY BASE → a vehicle for drug substances formulated into suppositories e.g. cocoa butter,
witepsol (example of saturated fatty acid) , wecobee, PEG mixtures
▪ SUFACTANT → agents which reduces interfacial tension e.g. benzalkonium Cl, polysorbate 80, SLS,
TWEENS, & SPANS

▪ Suspending agent/Emulsifying agent → increases viscosity and reduces rate of sedimentation e.g. CMC,
MC, bentonite, acacia, agar, veegum, tragacanth

▪ Sweetening agent → imparts sweetness e.g. mannitol, saccharin, sorbitol, sucrose

▪ Tonicity agent → renders solutions similar in osmotic dextrose characteristics to physiologic fluids (with
similar tonicity with the product & the physiologic fluids)

▪ TABLET EXCIPIENT
▪ Essential components → ingredients that imparts satisfactory characteristics
to the formulation e.g. diluents, binders, disintegrants
▪ Compression aids → ingredients that imparts compression characteristics to
the formulation e.g. glidants, anti-adherence, lubricants
▪ Supplementary components → ingredients that imparts additional desirable
physical characteristics to the finish products e.g. colorants, flavorants,
sweetening agents, and adsorbent

▪ SOLUTIONS (solute x solvent) → are liquid preparations that contain one or more chemical substances
dissolved in a suitable solvent or mixture of mutually miscible solvents
▪ Most solutions are unsaturated with the solute → unsaturated solution (the conc. of the solute is below
the solubility limit)
▪ The strengths which are usually expressed in terms of % strength, although for very dilute preparation,
ratio strength are used.
▪ ADVANTAGES:
1. Completely homogenous dose → one preparation
2. Immediate availability of drug for absorption and distribution → will not undergo disintegration will
directly proceed with dissolution
3. Provides flexible dosage form → may be administered by any route of administration, can be taken
by or administered to patients who cannot swallow tablets, or capsules, & doses are easily adjusted
(because it is not a fixed dose unlike tablets & capsules)
▪ DISADVANTAGES:
1. Drugs and chemicals are less stable when in solution than when in dry, solid form
2. Some drugs are not soluble in solvents that are acceptable for pharmaceutical use
3. Drugs with undesirable taste require special additives or techniques to mask the taste when in
solution
▪ GENERAL RULES IN PREPARING SOLUTIONS
1. Each drug or chemical is dissolved in the solvent in which it is most soluble → solubility
characteristics of each drug should be known e.g. organic compounds → not soluble in water so find
suitable solvent
2. If an alcoholic solution of a poorly water soluble drug is used, the aqueous solution is added to the
alcoholic solution → maintain high alcohol solution as possible
3. The salt form of the drug is used not the acid or base forms which both have poor solubility
4. Flavoring and sweetening agents are prepared ahead
5. The proper vehicle should be selected

▪ FACTORS TO CONSIDER IN PREPARING SOLUTIONS


1. Stability →of the active drug and the preparation as a whole is considered.
▪ Instability such as: color fading, flavor loss, cloudiness, precipitation, & microbial growth
2. pH
3. Solubility of the chemicals → usually stated on the number of parts
(by weight or by volume of a liquid) of the substance; SOLUBILITY = g
of the solute/ml solvent
4. Taste (for oral solutions)
5. Packaging need
Terms of solubility Parts of Solvent
Very soluble <1
Freely soluble 1-10
Soluble 10-30
Sparingly soluble 30-100
Slightly soluble 100-1000
Very slightly soluble 1000-10000
insoluble >10000

▪ TECHNIQUES TO INCREASE DISSOLUTION RATE:


1. Apply heat
2. Reducing the particle size of the solute
3. Utilizing of a solubilizing agent
4. Subjecting the ingredients to rigorous agitation

▪ FACTORS THAT AFFECT SOLUBILITY


1. Particle size → the finer or the smaller the PS, the greater the surface area that comes contact with
the solvent that make the process rapid
2. pH → the solubility of the weak acid increase with increasing pH; while the solubility of weak bases
decreases with increasing pH → RATIONALE: more of the weak base is in a unproteinated form
making it less polar = less water soluble
3. Extent of agitation → the greater the agitation, the more saturated solvent passes over the drug =
the faster the dissolution
4. Temperature
o Positive heat → most of the drug absorb specific heat when they are dissolve resulting in an
increase in solubility & temperature
o Negative heat → do not absorb heat that can decrease the solubility within the rise of temperature

▪ METHODS OF PREPARATIONS OF SOLUTION:


1. Simple solution → prepared by dissolving solute in a suitable solvent then heat or stir; the solvent
here may contain other ingredients which solubilize/stabilize the active ingredient (e.g. of simple
soln: Iodine solution → SOLUBILITY: 1:2, 950 parts of water, however, if there is a presence of K
iodide = formation of soluble polyiodides like in the case of Lugol’s solution or Strong iodine solution)
2. Solution by chemical reaction → prepared by reacting 2 or more solutes with each other in suitable
solvent e.g. combination of calcium carbonate (base) + lactic acid (acid) = calcium lactate (salt form of
the reaction which is soluble)
3. Solution by extraction → solutions coming from plants or drug products, and need to choose correct
extraction process. Produces/FINAL PRODUCT: extractives

▪ Types of solutions:
1. Non-sterile solutions: Oral solutions, Topical solutions, & other solutions
2. Sterile solutions: Parenteral solutions, ophthalmic solutions, & nasal solutions

▪ By site of administration: 1. Oral solutions, 2. Otic solutions, 3. Ophthalmic solutions, 4. Topical solutions
▪ Based on composition: syrups (contain mainly of sugar; aqueous solution with sugar), elixirs (sweetened
hydroalcoholic solution, HYDROALCOHOLIC: combination of alcohol & water), spirits (solution of aromatic
materials if the solvent is alcoholic), aromatic waters (oils with water, water is the solvent in the solution)

▪ Based on method of preparation & concentration:


1. Tinctures or fluid extracts → solutions prepared by extracting active constituents from crude drugs
2. Syrups → these are aqueous solutions that contain sugar with or without flavoring agents & medicinal
substances
2.1 Non-medicated or flavored vehicles → contain flavoring agents but without medicinal substances
2.2 Medicated syrups → prepared commercially for therapeutic value
▪ PREPARATION OF SYRUPS:
o Solution with heat → increase the dissolution, however, when heat is used in preparation of solution
there is almost certain of inversion of a slight portion of sucrose (since the used sugar in syrups is
SUCROSE → adding heat would result to the invert sugar = separation of fructose & glucose, therefore,
the inverted sugar gathered here is more fermentable, darker in color, & retard other substances than
sucrose e.g. alcohol fermentation) & levulose/fructose is also formed (sweeter than sucrose, hence
inverted sugar is sweeter sucrose)
o By agitation or solution without heat
o By simple admixture
o Reconstitution
o Percolation (USP method) → in this procedure, purified water or an aqueous solution is permitted to
pass slowly through a bed of crystalline sucrose, thus, dissolving it and forming a syrup (overheated
syrup = caramelirization). REMEMBER: method used for the preparation of syrup according to USP.
3. ELIXIRS (contains 5-40% alcohol; 10-12% self-preserving)→ clear, sweetened hydroalcoholic solutions
intended for oral use and are usually flavored to enhance palatability. Examples are: Dexamethasone elixir
& Digoxin elixir
▪ Types of elixir:
3.1 Medicated
3.2 Non-medicated

4. TINCTURES (15-80% alcohol) → alcoholic or hydroalcoholic solutions prepared from vegetable materials
or from chemical substances. Example: Opium tincture

5. LINIMENTS → solutions or mixtures of various substances in oil, alcoholic solutions of soap, or emulsions
and may contain preservatives.
▪ USES: rubefacient (external application that will cause redness, dilation of the capillaries, and increase in
the blood circulation), counter irritant (used to produce superficial inflame in order to reduce the deeper
inflammation), & mildly astringent
6. ENEMA → (6.1 evacuation enemas → use to evacuate the bowel; 6.2 retention enemas → used to retain
the bowel to elicit its therapeutic effect); solutions intended to be administered rectally
7. DOUCHES
8. OTIC SOLUTIONS
9. GARGLES → can extend up until the throat to pharynx (deep throat yern); used to relieve sore throat →
SHOULD BE EXPECTORATED
10. MOUTHWASHES → used to mechanically clean the mouth

GARGLES & MOUTHWASHES → not to be swallowed, they may be concentrated solutions which may contain
antiseptics, analgesics, & weak astringents, & needed to be diluted first with warm water before use.

11. COLLODIONS (a special solution) → are liquid preparations containing pyroxylin (a nitrocellulose) in a
mixture of ethyl ether & ethanol.
▪ They are applied to the skin by means of a soft brush or other suitable applicator and, when the ether and
ethanol have been evaporated, leave a film of pyroxylin on the surface
▪ The official medicated collodion, salicylic acid collodion USP, contains 10% w/v salicylic acid in flexible
collodion USP and is used as a keratolytc agent in the treatment of warts.
▪ Collodion is made flexible by the addition of castor oil and camphor.

12. GLYCERINS or GLYCERITES → are solutions or mixtures of medicinal substances in not less than 50% by
weight of glycerin.

13. STERILE SOLUTIONS

13.1 Nasal solutions → administered as nose sprays or nose drops; isotonic to nasal secretions and
buffered to normal pH range as nasal fluids
13.2 Irrigation solutions (commonly used for wounds) → sterile, non-pyrogenic solutions use to wash or
bathe surgical incisions, wounds, or body tissues

Commonly used Parenterals Pharmaceutical Solutions

SUMMARY OF THE SOLUTIONS

DISPERSED SYSTEM (Pharmaceutical suspensions & emulsions)

• Consists of 2 phases: internal & external (like dispersed and continuous dispersion medium); consist of
particulate matter known as dispersed phase, dispersed throughout a continuous or dispersion medium.
• Dispersed systems are classified according to particulate size:
Molecular dispersion < 1 nm TRUE solutions
Colloidal dispersion 1nm – 0.5 micrometers COLLOID
Coarse dispersion > 0.5 micrometers Suspension & emulsion
• COARSE DISPERSIONS:
1. Suspension → solid drug in liquid vehicle (solid particles dispersed in liquid medium)
o Liquid preparations that consist of solid particles dispersed throughout a
liquid phase in which the particles dispersed throughout a liquid phase in
which the particles are not soluble
o Suspensions are liquid preparation of drugs containing finely divided drug
particles distributed uniformly throughout the vehicle
o IMPORTANT: resuspendability of the particles → because when left upon
standing will form sediments so there should be enough suspending
agent used in the formulation
If the solute dissolved = solution
o PROBLEMS IN FORMULATING SUSPENSION: THE SOLUTE HAS NOT BEEN DISSOVED = suspension
▪ Aggregation → small particles aggregate into clumps or floccules (similar with flocculation). It
increases the rate of sedimentation and may prevent pourability (since the particle size increases).
▪ Sedimentation → the velocity of fall (downward movement) of a suspended particle in a vehicle of a
given density is greater for larger particles than is for smaller particles. The greater the density the
greater the descent.
▪ Settling and aggregation → the suspension shall form loose networks of flocs (FLOCS → COLLECTION
OF PARTICLES) that settle rapidly, do not form cakes and are easy to resuspend. Settling and
aggregation may result in formation of cakes (suspension) that is difficult to resuspend or phase
separation (emulsion)

o CLASSIFICATION OF SUSPENSION:
1.1 Deflocculated → more
aesthetically pleasing than flocculated (does not
produce loose aggregates); sedimentation is
slow but once it forms sedimentation
IRREVERSIBLE/IMPOSSIBLE TO SUSPEND → will
form cakes
1.2 Flocculated → Sediment
rapidly but can be easily resuspend
(REVERSIBLE)

o Five broad categories of suspending agents:


1. Natural polysaccharide → only problem is the variability
every batches and their microbial contamination. These
materials should not be used externally as they leave a sticky
feel on the skin
2. Semi-synthetic polysaccharides → derived from naturally
occurring polysaccharide cellulose.
3. Clays → naturally occurring ORGANIC materials which are
mainly dehydrated silicates.
4. Synthetic thickeners → introduce to overcome the variable
property of the natural products
5. Miscellaneous compounds
o CONTAINERS FOR SUSPENSION: pack in WIDE MOUTH amber bottle (purpose → shaking) for internal &
external use. There should be an adequate air space above the liquid allow shaking and ease of pouring. A
medicine spoon or oral syringe should be given when the suspension is for oral use (esp. if for infants)
o COMPOUNDING SUSPENSION:
1. Triturate the solid into finest particle
2. For, indiffusible solids, choose the appropriate suspending agent
3. Flavoring agents and preservatives should be selected and added if the product is intended for oral use
4. The vehicle containing the suspending agent is added in divided portions while mixing
5. NEVER FILTER → because the solute will be filtered out
6. The product is brought o required volume using the vehicle
7. Dispense with a shake well label when necessary
o LABELING: A “shake well: label should be placed on the label of the final product. If external use → indicate
in the label
o GENERAL CLASSES OF SUSPENSIONS: oral suspension, externally applied suspension, & parenteral suspension
o OTHER TYPES OF SUSPENSION:
1. Gels → semisolid system made up of either small inorganic or large organic molecules interpenetrated by
a liquid
2. Magmas/milks → dispersion of insoluble inorganic drugs
3. Mixtures → liquid dispersions or preparations containing finely divided insoluble materials intended for
internal use.
4. Lotions→ are either liquid or semisolid preparations that contain one or more active ingredients in and
appropriate vehicle. They are usually suspension of solids in aqueous medium. Some lotions are
emulsions or solutions (Examples: Calamine lotion). Can contain other appropriate excipients like
antimicrobial or preservatives, & stabilizers. Intended to be applied to the unbroken skin without friction.

o RATIONALE BEHIND THE ADDITION OF SUSPENDING AGENTS: increase the viscosity, thereby slowing down
sedimentation (e.g. suspending agents suitable for oral preparation → tragacanth & acacia

2. Emulsions → liquid drug in liquid vehicle (liquid droplets dispersed in liquid medium)
o Two-phase systems in which one liquid is dispersed throughout another liquid in a form of small droplets
(solute). A dispersion in which the dispersed phase is composed of small globules of a liquid distributed
throughout a vehicle in which it is immiscible.
o A heterogeneous mixture that contain at least one immiscible liquid
o DIFFERENCE BETWEEN DISCONTINUOUS PHASE AND CONTINUOUS PHASE:
DISCONTINUOUS /INTERNAL /DISPERSED PHASE DISPERSION MEDIUM/EXTERNAL/CONTINUOUS PHASE
• Liquid droplets → serving as solute • SOLVENT

o TYPES OF EMULSIONS:
2.1 Oil-in-water emulsions (o/w) → when the oil phase is dispersed as globules throughout an aqueous
continuous phase
2.2 Water-in-oil (w/o) → when oil phase serves as the continuous phase
2.3 Multiple emulsions → the dispersed phase of these emulsions contains smaller droplets that are
miscible with the continuous phase. These could be o/w/o or w/o/w
2.4 Microemulsions → the dispensed phase of these emulsions are in nanometer size range, & the MOST
STABLE TYPE OF EMULSIONS
o PHYSICAL STABILITY CONCERNS:
▪ Aggregation – the fusion of the dispersed droplets; REVERSIBLE
▪ Coalescence – complete fusion of droplets; merging of small droplets forming larger droplets, which will
later on lead to complete separation of phases → IRREVERSIBLE
▪ Creaming – migration of the droplets of the internal phase to the top of the emulsions; opposite of
sedimentation
▪ Sedimentation – downward movement
▪ Cracking – breaking of emulsion; the irreversible coalescence of droplets of the internal phase and
separation of the dispersed phase as a separate layer
▪ Phase inversion – when an emulsion change from o/w to w/o (the most stable range of dispersed phase
concentration is 30% to 60%)
o SMALL SCALE EXTEMPORANEOUS PREPARATION/ METHODS COMPOUNDING EMULSIONS:
1. Dry gum method/Continental method → used for forming emulsions using natural emulsifying agents
(acacia) and requires a special order of mixing. It uses 4:2:1 method (4 parts of FIXED OILS like COD
LIVER OIL or LIQUID PETROLATUM + 2 parts of water + 1 part of gum/emulsifying agent (acacia). This is
added in the initial or primary emulsions. Water is added rapidly.
2. Wet gum method/English method → uses the small proportion of ingredient as the dry gum method in
preparing the primary emulsion but requires a different order of mixing (water 2 + emulsifying agent 1 =
fixed oils 4 parts). The oil is added slowly to provide droplets. FINAL PRODUCT: oil-in-water emulsion.
PRACTICE:
A. Acacia emulsions – o/w
▪ Primary emulsions (4:2:1) (o:w:a)
▪ Ration for volatile oils- 3:2:1 or 2:2:1

PREPARATION OF PRIMARY EMULSION USING DRY GUM METHOD → add the water and oil all at once then
triturate
PREPARATION OF PRIMARY EMULSION USING ENGLISH METHOD → add wetting agent to the acacia (to prevent
the lumps), then add the water then gradually add the oil & then triturate.

3. Bottle method or Forbes bottle method (3:2:1) → it is useful for extemporaneous preparations of
emulsions from volatile oils or oleaginous substances or low viscosity (e.g. turpentine oil)
4. Beaker method/In situ method → used in the preparing emulsions using synthetic emulsifying agents
(e.g. NaOH, KOH); satisfactory method regardless of the order of mixing
5. Nascent →means beginning to exist or to develop (preparation of emulsifier); uses any of the two soaps
– hard or soft soaps; PROTOTYPE: lime water emulsion

o USES OF EMULSIFYING AGENTS: act by lowering the interfacial tension of the droplets and the solvent,
provide a barrier around the droplet as they forms and prevent coalescence of droplets
o TYPES OF EMULSIONS ACCORDING TO FUNCTIONS:
1. True or primary emulsifiers → capable of stabilizing emulsions by themselves
2. Stabilizers or auxiliary agents → used in combination with the primary emulsifiers (cannot form
acceptable emulsions when they are alone)
o TYPES OF EMULSIONS ACCORDING TO FUNCTIONS:
1. Natural – may be derived from vegetable (e.g. tragacanth, acacia, align, pectins) and animal sources (egg
yolk, casein, wool fat, lanolin, cholesterol, lecithin). The surface activity is low but they achieved their
emulsifying property by increasing viscosity.
2. Synthetic – are anionic, cationic, or nonionic. Effective at lowering the interfacial tension between the oil
& water phase because the molecules possessed the lipophilic and hydrophilic properties.
▪ Anionics – monovalent, polyvalent, and organic soaps (carboxyl, sulfate, sulfonate)
▪ Cationics – quaternary and pyridinium nitrogen (benzalkonium chloride)
▪ Nonionic – polyhydroxyl groups (SPANS 1-9) & TWEENS (11-20)
▪ HLB values (1-4) – emulsifiers and surfactants are characterized by the hydrophilic-lipophilic balance
(HLB): a relative ration of non-polar groups in the surfactant. Sometimes determined experimentally by
the used of reverse phase chromatography.

HLB VALUES

CHOOSING EMULSIFYING AGENTS


Acacia Forms a good, stable emulsion of low viscosity
Tends to cream easily, is acidic
Stable at a pH range of 2-10
It is negatively charged, dehydrates easily, and usually requires a preservative
Tragacanth Forms a stable emulsion that is coarser than acacia emulsion
Used mainly for its effects on viscosity
It is difficult to hydrate
Agar An anionic gum that is primarily used to increase viscosity
Its stability is affected by heating, dehydration, and destruction of charge
Susceptible to microbial degradation
Pectin Used in the same proportion as tragacanth and favor o/w emulsion
Gelatin Provides good emulsion stabilization in a concentration of 0.5-1.0%
TYPES: Type A gelatin (+) → prepared from acid treated precursor, used in acidic media; Type B gelatin
(-) → prepared from alkali treated precursor, used in basic media
Methyl cellulose is nonionic and induces viscosity
It is used as a primary emulsifier with mineral oil and cod liver oil, and yields an o/w emulsion
It is usually used in 2% concentration
Carboxymethylcellulose anionic and is usually used to increase viscosity; forms o/w emulsion
It tolerates alcohol up to 40%, forms a basic solution, and precipitates in the presence of free acids

STABILITY
• Preservatives (to help the product last longer) – is added if the preparation is intended to last longer than a
few days. It must be soluble in the water phase to be effectuve. Generally, a combination of methylparaben
(0.2%) and propylparabn (0.02%) may be used.
SEMISOLID DOSAGE FORM

GENERAL CHARACTERISTICS
• Semisolid dosage forms may contain active drugs intended to:
1. Act solely on the surface of the skin to produce a local effect (e.g. antifungal agent) → surface only
2. Release the medication, which, in turn, penetrates into the skin (e.g. cortisol cream) → much deeper
rather than only on the surface of absorption
3. Release the medication for systemic absorption through the skin (e.g. nitroglycerin)

EMOLLIENT Soften the skin


PROTECTIVE Protection against harmful influences esp. if injured, expose skin surface
OCCLUSIVE Prevent moisture loss & promotes water retention → CREATE HYDROPHOBIC BARRIER:
preventing water or moisture to evaporate
HUMECTANT Retain/absorb moisture because of its hygroscopicity (e.g. silica gels)

OINTMENTS/UNGUENT/OCCULENTUM/CHRISMA
• Semi-solid preparations intended for application to skin or mucous membranes
• Classified as:
o Medicated ointments → with medications
o Nonmedicated ointments → used for physical effect they provide as protectants, emollients, or lubricants

OINTMENT BASES → affect the property of the ointment & dictates the possible combination of ingredients for
the purpose which that ointment will serve

1. Oleaginous Bases (Hydrocarbon Bases) → anhydrous and insoluble in water


• Fatty/oily/greasy, non-water washable, offer the best emollient & occlusive effects
• When combined with the powdered substances, mineral oil or liquid petrolatum, LEVIGATING AGENT, is
added
• EXAMPLES:

1.1 Petrolatum, USP → purified mixture of semisolid hydrocarbon obtained from petrolatum
o aka Yellow Petrolatum, Petrolatum Jelly e.g. Vaseline (Cheesebrough-Ponds)

1.2 White Petrolatum, USP (White Vaseline®) → purified mixture of semisolid hydrocarbon, obtained from
petroleum that has been nearly or wholly decolorized
o considered more aesthetically pleasing e.g. White Vaseline (Cheesebrough-Ponds)

1.3 Yellow Wax → purified wax obtained from the honeycomb of the bee Apis mellifera (European Honeybee)

1.4 Yellow Ointment (Simple Ointment) → yellow wax + Petrolatum USP


o purified wax obtained from the honeycomb of the bee Apis mellifera (European Honeybee) →
contributes to the stiffness of the ointment

1.5 White Ointment, USP → white wax (yellow wax that has been bleached) + White petrolatum, USP → more
aesthetically appealing

2. Absorption Bases → greasy and difficult to remove compared to HC bases e.g. Aquaphor, Polysorb, Nivea
*emollient & occlusive effect, greasy; non-water washable, may permit the incorporation of aqueous solutions in
small amount

• TYPES:
2.1 Anhydrous hydrophilic material → absorbs water to become W/O emulsion; can be used when the
presence of water would cause stability problems (e.g. antibiotics)
o Needs/requires to absorb water first in order to produce W/O emulsion
o EXAMPLES:
a. Hydrophilic Petrolatum, USP (Aquaphor®) → modification of the original formulation which contains
ANHYDROUS LANOLIN (because of allergy/hypersensitivity) stearyl alcohol + wax adds firmness and
heat stability
o Aquaphor → can absorb 3x its weight of waterb.
b. Eucerin – a 50% W/O emulsion
c. Anhydrous Lanolin (Wool fat) → wax-like substance obtained from the from the wool of the sheep
(Ovis aries); purified wax like substance, cleaned, deodorized, and decolorized → contains NMT 0.25%
moisture

2.2 Hydrous material (emulsion bases) → are W/O emulsion already that have ability to absorb water; impart
excellent emollience and occlusiveness
o DO NOT NEED TO ABSORB WATER → already W/O but still have the ability to absorb water
o EXAMPLES:
a. Hydrous Lanolin (Wool fat) → NMT 25-30% moisture
b. Cold cream/Galen’s cerate → white wax, spermaceti, almond oil, sodium borate; one of the oldest
preparation used up to now

3. Water Removable Bases (Water-washable Base) → resembles o/w emulsion & creams
• may dilutes with water or aqueous solutions
• have the ability to absorb water, non-greasy, non-occlusive, can absorb serous discharges
• Example:

3.1 Hydrophilic Ointment, USP → methyl and propyl parabens (have the addition of preservatives since the
preparation contain water & to prevent mold formation or deterioration bcs of microorg), SLS, propylene
glycol, stearyl alcohol, white petrolatum, purified water, CAN ABSORB 30-50% w/w

3.2 Vanishing cream (Dermabase) → once applied it would only leave a transparent thin film (STEARIC ACID)
on the surface making it appear that it vanished

4. Water Soluble Base (Greaseless Base) → do not add large amount of H2O into the base because they soften
easily
• complete water washable “greaseless” → absorb water to the extent of solubility
• do not contain oleaginous components used for the incorporation of solid substances
• Example:

4.1 PEG Ointment, NF → polymer of ethylene oxide and water;


• Average MW (below 600) = clear colorless liquid
• MW >1000 = wax-like materials
• MW 600-1000 = semisolids
4.2 Olive base

RECALL!
ANHYDROUS LANOLIN LANOLIN
• Wool fat • Hydrous wool fat
• NMT 0.25% • 25-30%
• Classified as Absorption Bases: fall under → • Classified as Absorption Bases: fall under →
Anhydrous hydrophilic material emulsion bases
OINTMENT CREAM
• Oil-based semisolid preparation → 80% oil & 20% • Water-based semisolid prep (contains 50% oil and
water 50% water)
• Greasy, smooth and soften (soft semisolid prep) • Non-greasy, rich and heavy
• Usually translucent, thicker consistency, less • White to off-white in color, thinner consistency,
spreadable, slower absorbed, slow onset of action easily spreadable, rapid onset of action
• Stays longer, not easily washed off, retain more • Easily washed off

CREAMS
• These are semi-solid preparations containing one or more medicinal agents dissolved or dispersed in either
o/w or w/o emulsions
• They are easier to spread and removed than ointments
• A dosage form comprising a viscous semisolid emulsion

COMPOUNDING PROCEDURE:
1. Trituration to reduce the PS → the PS of the sulfur and salicylic acid are
reduced separately in a wegdwood mortar and blended together
2. Using a pill tile, the powder mixture is levigated with the base
3. Using geometric dilution (by portion addition of active ingredient), the base
and powders are blended to the final weight
4. An ointment jar or plastic tube or collapsible tube is used for dispendsing, and
an ”external use only” label is placed on the container

GELS/JELLIES
• Many, but not all gels can be classified as semi-solids, other are magmas
• Are semi-solid systems consisting of dispersions of small or large molecues in an aqeuous liquid vehicle
rendered jellylike by the addition of a gelling agent
• They are sometimes called JELLIES
• Example: Hydrocortisone Gel USP
PASTES
• Characterized by their high solid content (about 25%) and are therefore stiffer than ointments
• Remain in place after application and are effectively employed to absorb serous secretions
• Not suitable to be applied to HAIRY PARTS
• Example: Zinc Oxide Paste USP
SUPPOSITORIES
• Classified under what: solid bodies intended to be inserted into a body orifice (rectal, vaginal, urethral)
• Types depend upon of various weights and shapes
• Melts at body temperature (ideally → RECTAL supp) or dissolve into the aqueous secretions of the body cavity
(ideally → VAGINAL supp)
• Used to deliver drugs for their local and systematic effects

-RECTAL supp → on children: MORE PENCIL LIKE IN SHAPE & about 32 mm


long; IDEAL SUPP BASE FOR RECTAL: Cocoa butter (bcs it melts at body
temp.)
-VAGINAL (also referred as VAGINAL INSERTS) → IDEAL SUPP BASE: PEG
base/glycerinated base (bcs it dissolves into the aqueous secretions)
-URETHRAL → MALE: usually 3-5 mm in diameter & 140 mm long; FEMALE
half the length (60-70 mm) & same diameter; used for local & anti-
infective effects; IDEAL SUPP BASE: Cocoa butter
SUPPOSITORY BASES:

1. Oleaginous Base (Fatty Base) → the most frequently employed suppository


a. Cocoa Butter, NF → roasted seeds of Theobroma Cocoa (fam. Sterculiaceae); also known as Theobroma
Oil, a fat-soluble mixture of triglycerides ((a) Oleopalmistostearin (b) Oleostearin) that is most often for
rectal suppositories. It melts at body temperature.
• melts at 30-36 C (86-97 F) → ideal suppository base
• Incompatibility: lowering of its melting point with EUTECTIC SUBSTANCES e.g. Phenol, Chloral
Hydrate, & thymol → can decrease its melting point when incorporated w/ it.
• REMEDY: add 4-6% white wax of 18-28% cetyl esters wax
• ADVANTAGES: can be used alone, non-irritating, used for hand molding sup, solidification temp; 12-
13 degrees below its MP
• BE CAUTIOUS: exhibits marked polymorphism because of its triglyceride content (so it cannot be
OVERHEATED beyond 35 degrees C)
• Polymorphs: (a) Alpha (melts at 18-20 degrees C – the temperature that it is supposedly for
SOLIDIFICATION) (b) Beta → more stable solid until 32 C (melt between 34-35 C)

BECAUSE OF THE POLYMORPHISM OF COCOA BUTTER → Cocoa butter substitutes (SYNTHETIC TRIGLYCERIDES) –
these substitutes are rich in TRIGLYCERIDES, possess almost the same properties as to what cocoa butter has e.g.
melts at body temperature and do not exhibit POLMORPHISM (unlike CB):
b. Wecobee → source from coconut oil (TG of coconut oil)
c. Witepsol → saturated fatty acids C12-C18; main saturated FA is lauric acid
d. Fattibase → made from palm oil, palm kernel oil, & coconut oil

2. Water-Soluble or Water-miscible Base → *Precaution: Moisten the Tip of the Suppository to prevent mucosal
irritation & to facilitate the smooth passage of suppository in the body-orifices.

a. Glycerinated Gelatin → is a water-miscible base often used in VAGINAL


• COMPONENTS: 70% glycerin, 20% gelatin, & 10% water (hence, formulated along a preservative)
• Has soft, rubbery consistency suitable for vaginal admi but not firm enough for rectal use
• Do not melt but dissolve slowly in the mucous secretions of the vagina
• Should contain preservative (methylparaben 0.18%, proplparaben 0.02%)
*most frequently used in preparation of vaginal suppositories; slower to soften & mix w/ the physiologic fluids

b. Polethylene Glycols → are water-soluble bases suitable for vaginal & rectal suppositories e.g. POLYBASE
*polymers of ethylene oxide & water; do not melt at body temperature but rather dissolve slowly in the
body’s fluid; for vaginal administration

b. Miscellaneous Bases → generally combination of lipophilic & hydrophilic substances (miscellaneous


bases/polyoxymethyl derivatives like TABLETED SUPPOSITORIES & INSERTS should also have
preservatives)
*mixtures of oleaginous & water-soluble or water-miscible materials
*Example: Polyoxyl 40 stearate → a surface-active agent that is employed in a number of commercial
suppository

GLYCEROGELATINS
• Plastic masses intended for topical application and containing gelatin (15%), glycerin (40%), and water (35%) in
addition to the medicinal substance (10%)
• They are used by first melting and applying on the skin surface with a brush, after which, it will harden and
usually covered with a bandage
• are applied to the skin for the long term. Example: Zinc Gelatin (Zinc gelatin boot) → treatment of varicose
ulcers

SOLID DOSAGE FORMS

POWDERS
• Are intimate mixtures of dry, finely, divided drugs and/or chemicals that may be intended for internal or
external use (small individualized drug particles)
• One of the primary solid dosage form that is commonly used → simplest DF & basis for other DF e.g. tablets,
granules, capsules, suspension, parenterals, etc.
• TYPES OF POWDERS:
1. BULK POWDERS → for internal or topical use, offer a convenient method of dispensing non-potent
powder drugs that have doses not requiring moderate to large volumes of powder

1.1 Dusting powders – these are locally applied nontoxic preparations that are intended to have non
systemic action. Always dispensed in a very fine state for effectiveness of application and also to
minimize irritation. Applied to various parts of the body either as lubricant, protective, absorbent,
antiseptic, antipruritic, antihidrosis, astringent, antiperspirant, and usually contained in a sifter top
containers.
1.2 Dentifrices- powder generally containing a soap or detergent, mild abrasive and an anticariogenic
agent → for oral cavity/oral admi
1.3 Douche powders → are intended to be dissolved in water prior to use as antiseptics, or cleansing
agents for a body cavity (e.g. vaginal douche – usually vaginal application/cavity)
1.4 Insufflations → these are finely divided powders introduced into body cavities such as the ears, nose,
throat, tooth socket and vagina. finely divided powders intended to body cavity by a device known as
“Insufflator”

2. DIVIDED POWDERS → have individual doses of powder packaged in folded papers or plastic bags; AKA
“chartulae”; paper tablets; “papelitos”

USP Standards for Vegetable & animal Drugs


Descriptive Term Seive Size Limit
Very Coarse No. 8 NMT 20% pass through No. 60
Coarse No. 20 NMT 40% pass through No. 60
Moderately Coarse No. 40 NMT 40% pass through No. 60
Fine No. 60 NMT 40% pass through No. 100
Very Fine No. 80 No limit Greater fineness

USP Standard Powder of Chemical Drugs


Descriptive Term Seive Size Limit
Coarse No. 20 NMT 60% pass through No. 40
Moderately Coarse No. 40 NMT 60% pass through No. 40
Fine No. 80 NO LIMIT GREATER FINENESS
Very Fine No. 120

• GENERAL PRINCIPLES IN COMPOUNDING POWDERS → Some solids have unique properties and requires
special handling:
o Efflorescent powders → crystallize powder containing water hydration hence, give off or releases
moisture (EFFERENT → moving away = releasing MOISTURE; requires special handling to avoid
liquefaction prior giving to px)
o Hygroscopic powders → absorb moisture but does not LIQUIFY e.g. silica gels/desicants
o Deliquescent powders → absorb moisture and LIQUIFY
o Pharmaceutical Eutectic mixture → lowers MP of the preparations upon comminution of phenol,
camphor, menthol, thymol, drugs e.g. phenacitin, acetanilide, ASA even at normal temp. LIQUEFY → bcs
of the MP lowering but EUTECTIC MIXTURES can still be used in powders upon the addition of INERT
substances e.g. magnesium carbonate, magnesium oxide, substitute ABSORBENTS e.g. kaolin, starch,
bentonite

• BASIC TYPES OF POWDER PAPERS:


o White bond paper → an opaque aesthetically appealing paper but has no moisture-resistant properties
o Vegetable parchment → a thin, semi-opaque, moisture-resistant paper
o Glassine → a glazed transparent, moisture resistant paper
o Waxed paper → a transparent waterproof paper (Hygroscopic or volatile drugs – preferably protected
with waxed paper then double wrapped with bond papers)

TABLETS
• Are solid pharmaceutical dosage forms containing drug substances with or without suitable diluents and have
been traditionally prepared by either compression or molding methods

CAPSULES
• Are solid dosage forms in which one or more medicinal or inert substances (as powders, compact, beads or
granulation) are enclosed within a small gelatin shell
• More preferred oral DF in extemporaneous preparation
• CAPSULE SIZES:
o A list of capsule sizes and the approximate amount of powder that may be contained in the capsule appear
on the side of the capsule box
o Capsule sizes are for oral administration in humans range from no. 5 the smallest, to no. 000, the largest.
o No. 00 is usually the largest oral size suitable for the patients
o Size 000 are sometimes used to encapsulate medication for rectal or vaginal use – moistened before
insertion
o Capsules for veterinarian nos. 10, 11, 12 containing approx. 30, 15, and 75 g respectively
o Capsule sizes available for Human use:

Capsule Size Volume Capacity (mL) Powder Capacity (mg)


000 1.40 650-2000
00 0.95 390-1300
0 0.68 325-910
1 0.50 225-650
2 0.37 195-520
3 0.30 130-390
4 0.21 95-260
5 0.13 60-130

• PREPARATION OF HARD AND SOFT CAPSULES


1. As with the bulk powders, all ingredients are triturated and blended, using geometric dilution.
2. The correct capsule size must be determined by trying different capsule sizes, weighing them, and then
choosing the appropriate size.
3. Before filling capsules with the medication, the body and cap of the capsule are separated. Filling is
accomplished by using the punch method.
a. The powder formulation is compressed with a spatula on a pill tile or paper sheet with uniform depth
of approximately half the length of the capsule body.
b. The empty capsule body is repeatedly pressed into the powder until full.
c. The capsule is then weighed to ensure an accurate dose. An empty tare capsule of the same size is
placed on the pan containing the weights
d. For a large number of capsules, capsule-filling machines can be used for small-scale use to save time.
4. The capsule is wiped clean of any powder or oil and dispensed in a suitable prescription vial.

• TYPES OF CAPSULES:
1. Soft Gelatin Capsules → prepared from gelatin shells.
o Glycerin or a polyhydric alcohol (e.g. sorbitol) is added to these shells to make them elastic or plastic
like. These shells contain preservatives (e.g. methyl and propylparabens, sorbic acid) to prevent the
growth of fungi
o Are oblong, elliptical or spherical e.g. VITAMIN A,E, Digoxin (Lanoxi capsules), chloral hydrate
o 3 PRODUCTION PROCESSES OF SGC:
1. Plate process → the oldest
2. Reciprocating die process
3. Rotary die process

LOZENGES
• Usually intended for treatment of local irritation or infection of the mouth or throat. May also contain active
ingredients intended for systemic absorption after swallowing.

PASTILLES
• Subclass of lozenges – that is molded lozenges. Traditionally, pastilles were soft lozenges containing
medicaments in a transparent glycerinated gelatin base or a base of acacia, sucrose and water. Usually are
flavored and colored to match the flavor.

LOZENGES
• Often used for a subcategory of compressed lozenges. Discoid shaped solid containing the medicinal agent in a
suitable flavored base.
Chemical stability → determined with the use of chemical assays and tests
• Second check → That one way or another will lead to change of structure in the drug molecule would
contribute to the stability as a whole or in general of a particular drug product

Physical → tested by organoleptic (5 senses)

Micro → sterility or resistance test; antimicro drugs should be retained under specific limits

Therapeutic → should remain unchanged even though expose to factors that may affect the stability of the
product

Incompatibilities → even the active ingredients reacting with the inactive ingredient. Usually occurs after cross
contamination, mixed ups, compounding, & even dispensing. It can also arise before, during & after administration
of drugs.

Things should be considered:


-Temperature: it should be stored properly

1. Physical – recognizable, visible, observable changes


A. Incomplete solution → Insolubility of the solute in the solvent or the immiscibility of the liquid solute in a
solvent
• REMEDIES:
▪ Whenever compounding any liquid DF → very important to determine the solubility, polarity, &
miscibility of the components (e.g. if the sample is insoluble with the solvent instead of solution
switch to suspension; or if the sample will form droplets around the solvent → EMULSION)
▪ Appropriate solvent → if slightly soluble only then add more solvent or create other dispersed
system instead
▪ WHEN A SOLID SUBSTANCE FAIL TO DISSOLVE IN A LIQUID → refer to other DF or select the correct
solvent system → PROBLEM ENCOUNTERED MOSTLY WITH THE DF OF SOLUTION: prepare another
DF → emulsion
A.1. IMMISCIBLITY → problem encountered to EMULSION; Remedy → add an emulsifier

B. Precipitation from solution


B.1. For IV solutions → after the definition because of an emboli or thrombus cause by a ppt
• Precipitation → very important to understand the solvent effects of particular substance with a drug
(e.g. when water is added to an alcoholic solution of a drug is poorly soluble in water; alcohol added to
a water solution of a drug is poorly soluble in alcohol) because it will affect the solubility
1. CHANGE OF SOLVENT SYSTEM: when water is added to an alcoholic solution of a drug is poorly soluble
in water → then add water = ppt
▪ e.g. salicylic acid → soluble in alcohol but only slightly soluble in water → so if salicylic acid is to be
dissolve in an alcoholic solution → avoid adding more water because it will lead to the ppt of
salicylic aicd
▪ codeine phosphate → very soluble in water but not in alcohol → so if alcohol is added a codeine
PO4 dissolve in water = ppt
• SOLVENT EFFECTS MAY RESULT BY ADJUSTING COMPOUNDING STRATEGIES → consult proper
references with regards of the solubility of the preparation or decrease the conc. of the drug to a
certain level that the drug will still soluble in the solvent system
• PPT PROBLEM ENCOUNTERED IN MOSTLY IN THE PREPARATION OF SOLUTION
2. CHANGE OF TEMP → not all drugs are soluble in warm water or soluble in temp that are high bcs there
are drugs more soluble in cooler temp
3. pH → employment of buffers; for some drugs the pH is needed to be lowered, neutral or
increase/alkaline so to avoid ppt (e.g. potassium salts of penicillin → more soluble in alkaline
preparation; phenobarbital Na or the salt form of phenobarbital →if pH has to be lowered some of the
salt form will be converted to the neutral free acid form and thereby ppt phenobarbital → freely
soluble in water but the pure drug states of phenobarbital is slightly soluble in water); affects or alters
the degree of ionization; problem happens when drug solutions with dif pH are combined or when a
drug that generates another pH is added to the original solution
4. Temperatures → dec. (MOST only NOT ALL drugs); PN drugs also like Cisplatin, Cotrimoxazole,
Metronidazole are examples of drugs that ppt when REFRIGERATED

C. POLYMORPHISM – inert property of some drugs or substances like ampicillin, barbiturates,


hydrocortisone, & sulfa drugs
• Cocoa butter → alpha (18 degrees C), gamma (22), beta prime (28) & beta stable (34) polymorphs

D. LIQUEFACTION OF SOLID INGR → give off water and when they give off water the given weight of the
resultant powder no longer contain the same amount of drugs
• REMEDY: Addition of lactose → lactose will absorb the water instead of the hygroscopic or deliquescent
drug

F. Leaching → Remedy: substitute PLASTIC to the glass etc.


G. Vaporization → common problem or incompatibility among highly volatile drugs
H. Loss of water → e.g. emulsion → PHASE INVERSIONS in o/w to w/o bcs of the loss of water

Solidification → problem encountered in DF like SUSPENSION (e.g. acacia in the presence of bismuth salts can
form a cake at the bottom of the container: CEMENTATION)

General remedies for physical incompatibilities:


1. Modify the order of mixing → which one is added what first
2. Change the conc. of the solvent/solute
3. Change the form of ingr → instead of hydrated → use anhydrous or vice versa
4. Add therapeutically inactive subs. → e.g. inert powders like lactose to absorb the moisture instead
5. Omit an inactive ingr→ esp. if it is the one causing the incompatibility
6. Change the DF → e.g. if not soluble in water instead of soln switch to emulsion
7. Separate dispensing → e.g. solid DF → instead formulating mixed product → divided powders/chartulaes

2. CHEMICAL INSTABILITIES → are further enhance by adverse temperature, light, humidity, oxygen, & even
presence of CO2

2.1. OXIDATION – primary cause of chemical instabilities (e.g. Epinephrine can undergo color changes and
would become inactive if oxidized; color changes is an indication of loss of therapeutic activity; Eph when
oxidized/expose to air/light/heat/alkali → will become somewhat pinkish)
2.3 EVOLUTION OF GAS/EFFERVESCENCE → instability common to NaHCO3 containing drugs; common to citric
and tartaric acid once it comes contact with NaHCO3 and will liberate CO2 → bubbles
2.4 COMPLEXATION/CHELATION → e.g. avoid taking tetracycline with antacids, milk, calcium/iron supp.,

OTHER EXAMPLES: strong RA + strong OA = explosive

3. THERAPEUTIC → e.g. Pen G (bacteriocidal) + Tetracycline (bacteriostatic) = antagonizes the effects of Pen G
• Precipitant drug – causes the interaction either drug, food, or chemical
• Object drug – the one that is being affected
Adverse drug reaction (ADR)—The World Health Organization defines an ADR as “any response to a drug which is
noxious (harmful) and unintended, and which occurs at doses normally used in man for prophylaxis, diagnosis, or
therapy of disease, or for the modification of physiological function.”
• Result of the intrinsic properties of the drug and it cannot be prevented
• Examples: 1. Tetracycline (CHON synthesis inhibitor; DOC for leptospirosis particularly Doxycycline) – ADR:
enamel dysplasia; 2. Phenytoin (Dilantin); anti-seizure drug but have teratogenic effect (ADR: fetal hydantoin
syndrome → have cleft palate & mental retardation) & gingival hyperplasia (overgrowth of the gum tissue
around the teeth)

Side effects → expected; notified by the manufacturer → written in the drug’s label or literature

ADE → anything that happens after the administration ADR → caused by the intrinsic property of the drug →
of the drug that’s why it is UNPREVENTABLE
• Commonly caused by human error (e.g.
medication errors → wrong dose, wrong route,
wrong time, wrong drug, wrong DF, omission
error, extra dose error)

RISK FACTORS OF ADR:

1. Age (pediatrics, geriatrics, infants) → 1. Children → viral disease like influenza or chicken pox + ASA = Reye’s
syndrome (very rare; can lead to inflammation, swelling of the brain, loss of function & impairment of the liver)
7. Ethnicity → fast acetylators (Asians & Eskimos) and slow acetylators (Caucasians & Egyptians)

TYPES OF ADR:

1. Type A - Augmented → depending on the DOSE (overdosing/underdosing) & the MOST COMMON TYPE of
ADRs
• SUBTYPES:
1.1. Extension effects → related to the pharmacologic action of the drug; common when → OVERDOSING
of the drug occurs; e.g. BZD (sedative hypnotics)→ SEDATION → related to indication & caused by
overdosing
1.2. Side effects → unrelated to the pharmacologic action of the drug; e.g. Morphine (analgesic) →
CONSTIPATION → unrelated;
1.2.1 effects of MORPHINE TO THE BODY: Histamine release, Euphoria, CV collapse, Decrease of GI
motility, Decrease of cough reflex, Resp. dep, Emesis, Analgesia, Miosis;
1.2.2 ACE inhibitor → cough → because of the suppression of the ACE resulting in the accumulation of
the substances that are normally metabolize by ACE like bradykinin or tachykinin → BRADYKININ
production or accumulation causes the cough – that’s why patients taking ACEi that experienced
angioedema & cough = alternative is ARBs (angiotensin receptor blocker -SARTAN);
1.2.3. Minoxidil → Hypertrichosis → excessive hair growth anywhere on the body in either males or
females → hence, Minoxidil is also promoted in the market as a hair growth product (Regrow)
1.2.4. Antineoplastics → bone marrow suppression; RBC life cycle → 120 days, platelets → 5 to 7 days,
WBC → 6 to 12 hrs

2. TYPE B → Bizarre → RAREST & most fatal or serious


• SUBTYPES:
2.1. Idiosyncratic → genetically determined reaction; unusual effects
2.1.1 Malignant hyperthermia → disease that causes fast rise in the body temperature and therefore
causing severe muscle contraction → can lead to muscle breakdown (Succinylcholine, Halothane,
Sevoflurane)
2.1.2 Hemolytic anemia → a form of anemia cause by hemolysis → abnormal breakdown of RBC (G6PD
+ antimalarials/sulfonamides)
2.1.3 SJS (Steven Johnsons Syndrome) → immune complex mediated hypersensitivity that would
typically involve the skin & the mucous membrane → losing 10% of the outer layer of the skin or
necrolysis); typical symptoms of SJS: productive cough with thick purulent sputum, headache, malaise,
arthralgia (joint pain) → (CBZ, Phenytoin, Sulfonamides)

2.2. Hypersensitivity reax → immune response to environmental antigens resulting to symptomatic


reactions & upon secondary exposure to the same antigen = allergens; exposure to antigens/allergen
(ACID)
2.2.1 TYPE 1 hypersensitivity reaction: Anaphylactic/Immediate hypersensitivity response →most
common; an allergic reaction that is provoke by the re-exposure to a specific type of antigen/allergen e.g.
pollen; after the reexposure to the same allergen → cross linking of the cell bound IgE happens; IgE-
mediated → after cross linking → DEGRANULATION (releasing histamine, leukotrienes, prostaglandin);
commonly characterized by anaphylaxis, hay fever, asthma, urticarial/hives, angioedema
▪ Penicillins → anaphylaxis (do not administer other antibiotics that has cross sensitivity with Pen
e.g. CEPHALOSPORINS → because they have the same structure → BETA LACTAM RING

2.2.2 TYPE 2: Cytotoxic → IgG or IgM mediated → the antibodies produced here will bind to the antigen
on the patient’s own cell surfaces; TARGET: cells like leukocytes (WBC), erythrocytes (RBC) that will
produce complemented mediated lysis
▪ Examples of type 2: Acute rheumatic fever, autoimmune diseases, blood transfusion reaction;
METHYLDOPA (DOC for eclampsia), Chloramphenicol (DOC for typhoid fever) → Aplastic anemia
(deficiency of all blood cells caused by the failure of the bone marrow development)

2.2.3 TYPE 3: Immune complex → occurs when there is little antibody and excess antigens = often leads
to small immune complexes instead of the normal large complexes in the body; small complexes → not
effectively cleared and thus will cause problems; PROBLEMS that the patients will experience → tissue
damage, & inflammation
▪ Blood dyscrasia → dyscrasia is a non-specific term that refers to disease or disorder especially of
the blood
▪ Arthus reaction → a type of local type 3 hypersensitivity reaction which involves the deposition of
the antigen or antibody complexes mainly in our vascular walls like pericardium, synovium, &
glomerular
▪ SLE (Systemic lupus erythematosus) → autoimmune disease in which the own immune body
system mistakenly attacks the healthy tissues; so the commonly affected parts are: skin, joints,
kidneys, brain and other organs; drugs causing SLE: Hydralazine, Quinidine, Procainamide,
Isoniazid, Phenytoin

2.2.4 TYPE 4: Delayed or Cell mediated → takes 2-3 days to develop; not antibody mediated but rather
CELL MEDIATED
▪ Morbiliform → refers to the rash that looks like measles; these rashes have macular lesions that
are red in color and usually 2-10 mm in diameters

PRACTICE QUESTIONS:
Hypersensitivity reactions are examples of TYPE B (bizarre) of ADR
Anaphylactic reactions → acute generalized reactions that occur when a previously sensitized person re-exposed
to a particular antigen
Cytotoxic reactions → clinical manifestations of this type of reaction includes hemolytic anemia,
thrombocytopenia, and granulocytopenia
Cell mediated reactions → contact dermatitis e.g. urushiol
Immune complex mediated reaction → these reactions result from the formation of drug antibody complexes in
serum, which often deposit in blood vessels walls, resulting in activation of complement and endothelial cell injury.
3. TYPE C → Continuous
3.1. Addiction → characterized by behaviors that include one or more of the ff: Overdrug use/impaired drug
control use/compulsive use despite the harm
3.2. Dependence
3.3. Tolerance → repeated use of the drug will lead to reduced effect/tolerant/dec. on the physiological &
psychological response with the product with the continuous use of the same dose (e.g. pain killers)

4. TYPE D → Delayed; have specific time that the symptoms will appear
4.1. Carcinogenicity – administration of carcinogenics/anticarcinogenics → can lead to cancer/chronic
toxicity;
▪ antineoplastics like Busulfan, Melphalan, Cyclophosphamide, -platins, & mustine or nitrosoureas;
▪ overcooked spots/sunog in food → meats (carcinogenic)
4.2. Teratogenicity – can affect the baby/fetus (e.g. when the mother take these meds during pregnancy →
leading to birth defects)
▪ CBZ – can lead to neural tube defects
▪ DES (diethylstilbestrol) – increase risk of development of vaginal and adenocarcinoma after
puberty
▪ Phenytoin – FHS (fetal hydantoin syndrome)
▪ Streptomycin – 8th nerve damage
▪ Tetracycline – enamel dysplasia or the discoloration of the teeth
▪ Thalidomide – mainly used before for morning sickness of the pregnant women → later on
discovered to cause phocomelia
▪ Isotretinoin – powerful teratogen
▪ LITHIUM CARBONATE → EBSTEIN ANOMALY (a right ventricular outflow tract obstruction defect;
tricuspid valve of the baby is not functioning normally → the return of the blood from atrium to
ventricle)

PRACTICE QUESTIONS:
• Category C - If animal studies have shown an adverse effect but there is no adequate and well-controlled
studies in pregnant women
• Category A – adequate, well controlled studies in pregnant women have not shown an increased risk of fetal
abnormalities
• Category B – animal studies have revealed no evidence of harm to the fetus; however there are no adequate
and well controlled studies in pregnant women; animal studies have shown an adverse effect, but adequate
and well controlled studies in pregnant women have failed to demonstrate a risk to the fetus (e.g. used of
Lansoprazole)
• Category D – adequate, well-controlled or observational, in animals or pregnant
women have demonstrated a risk to the fetus; however the benefits of the therapy
may outweigh the potential risk
• Category X – use of the product is contraindicated in women who are or may become
pregnant (e.g. use of Leflunomide)
• Cause birth defects: Vit A (ISOTRETINOIN), Finasteride, Nicotine, Alcohol, tretinoin,
warfarin, & ACEi (EXCEPT: Vit B9/FOLIC ACID → supplement for pregnant women;
should be BEFORE & DURING pregnancy so as to avoid neural tube defects/any
abnomalities)

TERATOGENIC DRUGS DISEASE


Lithium Ebstein anomaly
VPA (Valproic acid) Spina bifida → spinal cord of the baby is not completely developed
Anabolic steroids Masculinization of female fetus
Thalidomide → prescribed Phocomelia/Flipper babies (bcs of limb deformities) → manifestations: polyneuritis, limb
before to pregnant women for deformities, mental retardation → happened in late 1950s up to early 1960s, up to 10,000
morning sickness birth defects & thousands of fetal deaths worldwide
CBZ & VPA Neural birth defects
DES Risk of developing vaginal adenocarcinoma after puberty
Phenytoin FHS (fetal hydantoin syndrome)
Streptomycin 8th cranial nerve damage
Tetracycline Discoloration or defects of the teeth; altered bone growth

5. TYPE E → End of Use → has something to do with WITHDRAWAL SYMPTOMS: manifestations → agitation,
anxiety, muscle aches, increase tearing, insomnia, runny nose, sweating, yawning
▪ Taking BZD → then D/C = rebound insomnia
▪ Taking Clonidine→ then D/C = rebound HTN (happens when BP rises when you stop taking the drug)
▪ Rhinitis medicamentosa → also known as rebound rhinitis or chemical rhinitis – a condition characterized
by a nasal congestion without rhinorrhea or sneezing that is triggered by the use of topical
vasoconstrictive medication for more than 4-6 days

6. TYPE F → Failure of Therapy (not effective therapy)


▪ Drug-drug interactions → pharmacodynamic effect, antagonistic effect → NO EFFECT
▪ Drug instability → drug is not stored properly → not efficient
▪ Wrong route of administration → take note the routes e.g. supp. rectally not orally, chewing tabs → to be
chewed first before swallowing, buccal tabs → in the buccal pouch
▪ Drug resistance → mostly antibiotics
▪ Expired drug → the nearer the drug’s expiration date the lesser its efficacy (according to the shelf life)

PRACTICE:

DRUG INTERACTION (can be drug-food, drug-lab, drug-drug, drug-herb, drug-disease) → taking 2 or more drugs
that will produce ADR; commonly happens to polypharmacy type of patient; resolve with proper time interval;
results can be positive (beneficial in the effect of the drug) or negative (hinder the effect/action of the drug)

EXAMPLES OF INTERACTION:
Bisacodyl (object drug) + milk (precipitant drug) = alteration in pH; increase pH of the GI content causes
disintegration of the enteric coated Bisacodyl
Barbiturate (object drug) + alcohol (precipitant drug) = increase the effect of Barbiturate → increase in sedation

A. DRUG-FOOD INTERACTION
▪ MAOI + Tyramine-rich food → dec NE absorption → causing hypertensive crisis
▪ INH + Histamine-rich food → flushing (vasodilation)
▪ ASA + Caffeine → both acidic → so alteration in pH → caffeine aids in the inc. abs of ASA

Increased by food → INCREASE THE BA OF THE DRUGS


-Acarbose → used to tx T2DM → alter the cycle of bone formation and the
breakdown in the body
-Alendronate → slowdown the bone loss
-Griseofulvin & Theophylline → increased abs by FATTY FOOD

B. DRUG-LAB TEST

1. Penicillin, Chlorampenicol, Vit. C, Isoniazid, Streptomycin + GLUCOSE URINE TEST (Benedict’s test) = false
(+) result
2. Chlordiazepoxide = false (-) result
3. Rifampicin(red-orange colored urine), Vit B12 (intense yellow), Chloroquine (brown) → affect urinalysis
because of the changes in color of the urine

C. DRUG-DRUG
▪ CLASSIFICATION: Pharmacokinetics, pharmacodynamics, pharmaceutical

1. PK → ADME (Distribution → albumin, alpha 1 glycoproteins)


▪ ABSORPTION:
1.1 Altered pH:
Bisacodyl + antacid = antacid will diminish the therapeutic effect of Bisacodyl,
Ketoconazole + Bisacodyl = dec. KETOCONAZOLE/ SALICYLATES abs → why? Ketoconazole/Salicylate
is best abs on acidic medium, hence, the addition of antacids like H2 blockers, PPI will lower the
acidity level of the stomach thereby increasing the pH level (basic) = affecting the abs and dissolution
of Ketoconazole/Salicylate → REMEDY: take the Ketoconazole 2 hours or 1 hour before taking the
antacid → do not use simultaneously

1.2 Altered bacterial flora: Antibiotics + Digoxin (for CHF) → Digoxin is metabolized by the intestinal flora →
so if the antibiotics is also used the antibiotics will affect the normal flora therefore → the Digoxin will
not be metabolized or excreted = increase levels of Digoxin in the body/ enhance oral BA of Digoxin→
can be therapeutically use when high level of Digoxin is needed for the therapy or DIGOXIN
INTOXICATION

1.3 Formation of chelates or complexes → Tetracycline, Fluoroquinolones, Penicillamine + precipitant


metal/drugs e.g. Ca, Fe salts, Mg, Al = complexes/ chelates

1.4 Gastric emptying time and rate and effect of GI motility to the drugs
o GET → the process by which the contents of the stomach are move to the SI esp. in the
duodenum → the longer the GET the longer the transportation of the food content in the SI; the
faster the GET the faster the transportation of the contents to the SI; when the rate is decrease =
increase in the absorption of the drug when it is most absorb in acidic envi e.g. STOMACH
(desirable); while if the drug is supposedly abs in the SI then the GET is slow/longer = low rate of
abs
-Anticholinergics like Atropine, Propantheline = dec. GER→ dec. GET → so Atropine + antacid = increase
antacid activity because Atropine will make the GET of the antacid longer → therefore increasing the
activity of the antacid on the stomach where it is mostly absorbed because of the acidic environment
-Atropine + Amphetamine = decrease Amphe activity → Amphe is most absorb in the SI and when
Atropine makes longer the GET of the Amphetamine in the stomach before it can reach the SI → the Amp
abs will be decrease
-NICOTINE = increase the GET therefore increasing GER → fast movement of drug from the stomach to
the SI = more absorption
-INCREASE MOTILITY → less absorption more excretion → upon administration the drug is more likely to
be excreted already so dec absorption
-ADSORPTION → e.g. activated charcoal = dec. the abs of the drug because instead of the body absorbing
the drug → it will be adsorb in the agent instead (hindering/lessen the drug absorb in the systemic
circulation)
-INTERRUPTION OF ENTEROHEPATIC CIRCULATION → antibiotics + oral contraceptive agents = lead in the
increase of unwanted pregnancy because bacteria in the intestines will cause hydrolysis in the conjugated
form of estrogen = decreasing the amount of estrogen concentration → inc. chance of unwanted
pregnancy bcs OCP will not take effect

PRACTICE:
Epinephrine will decrease the percutaneous absorption of transdermal Lidocaine or transdermal Fentanyl due to
vasoconstriction.
This drug is inactivated by intestinal flora → DIGOXIN → because it is metab by intestinal flora/ so if with
concurrent use of antibiotics = affect the DIGOXIN abs
Erythromycin (Macrolide → example of enzyme inhibitors) → GIT metabolism; ERYTHROMYCIN STEARATE SHOULD
NOT BE GIVEN WITH FOOD; estolate form and ethylsuccinate form are the only possible given varieties of Eryth
with FOOD
Lidocaine → alteration of the gastric pH

▪ DISTRIBUTION → displaced protein binding; protein binding depends on the affinity of the drugs to the
plasma protein (common plasma protein in protein binding: albumin, alpha 1 glycoproteins) so most likely
bound drugs/protein bound drugs are capable to displace others; free drug → will take effect; FREE DRUG
INCREASE BY DISPLACEMENT BY ANOTHER DRUG WITH HIGHER AFFINITY; so the distribution of the drug
may be affected by plasma protein binding & the displacement tissue and cellular interaction

▪ METABOLISM (MAJOR ORGAN: liver; but can also occur in skin, GIT, lungs) → enzyme inducers (MOSTLY
MACROLIDES): stimulate metabolism therefore, when the drug is metab = lesser absorption in the body;
enzyme inhibitors: inhibit the metabolism leading to increase BA of drugs and inc. of conc. of the drug in
the body; the effect of one drug to the metabolism of another that is well documented; MOST
COMMONLY INVOLVED ENZYME: CYP450 family → major metabolizing enzyme in the phase 1 metabolism
or OXIDATION PROCESS
-CBZ & Phenytoin induce the enzyme → CYP 3A4; these two drugs also cause AUTOINDUCTION: they
induce their own metabolism
-TOBACCO → induces CYP1A2
1. ENZYME INDUCTION →
-oral contraceptive + barbiturates (enzyme inducer) = reduced contraceptive effects
-acetaminophen (hepatotoxic) + phenobarbital (enzyme inducer → inc. clearance of APAP in the liver →
inc. the risk of hepatotoxicity)

2. ENZYME INHIBITION → dec. metab = accumulation of drug or conc. = toxicity/intoxication (e.g. grape
juice + CCB’s, HMG-CoA reductase inhibitors, CBZ, Cyclosporine = increase absorption of these drugs)

-ALTERATION OF GIT METAB → e.g. Phenelzine (MAOi) + cheddar or pickled fish (tyramine-rich food) =
hypertensive crisis
▪ EXCRETION → commonly affected: tubular secretion, GFR, & alteration of urine pH
Methylxanthines (e.g. caffeine) → increase renal blood flow & GFR → therefore
decreases the time for reabsorption of various drugs leading to more rapid urinary
drug excretion
Probenecid → block the active tubular secretion of the drug penicillin and other
antibiotics
ASA (acid) + NaHCO3 (basic) – alteration in the urine pH = inc. excretion/clearance
(render the drug ionized & water soluble drug ready for excretion)
-salicylates undergo alkalinization decreasing its reabsorption thereby increasing
its clearance → increasing excretion

INTERACTIONS DUE TO CHANGES IN EXCRETION → polar (ideal for excretion) – lipophilic or non-polar (ideal for
absorption)

PRACTICE:
Effects of increased theophylline metabolism → increased elimination/excretion & decreased blood level
Theophylline + Thiabendazole = decreases metabolism of theophylline & increased serum levels of theophylline
Captopril (Capoten) + Potassium sparing diuretics = may cause hyperkalemia

2. PD → Additive, potentiation, synergistic, antagonism

2.1 Additive → 1 + 1 = 2; common → sedative-hypnotics and ethanol; Aminoglycoside + FUROSEMIDE =


ototoxicity
2.2 Synergistic → 1 + 1 = 3; common in antibiotics
2.3 Potentiation → 1 + 0 = 2 ; Amoxicillin + Clavulanic acid (beta-lactamase inhibitor → no antibiotic
property but when added to Amoxicillin it will prevent the degradation of it); Levodopa + Carbidopa =
Sinemet (anti-parkinsonism)
2.4 Antagonism = 1 + 1 = 0; no effect; common on antidotes e.g. Atropine (opioids) + Naloxone, Procaine +
Sulfonamide = antagonism in the bacterial activity of Sulfonamide

3. PHARMACEUTICALS → more on the dosage form and physical properties of the drugs
▪ AMINOGLYCOSIDES:
Vestibulotoxic → Streptomycin, Gentamicin
Ototoxic → Neomycin, Amikacin, Kanamycin
Nephrotoxic → Neomycin, Tobramycin, Gentamicin
▪ THIAZIDES (excrete the electrolytes → therefore causing hypokalemia) + DIGITALIS (inc abs in
hypokalemia envi; if high K DIGITALIS will not take effect = should be on normal K) = increase toxicity of
digitalis (because of low K levels)

DRUG-HERBAL INTERACTIONS: NOTE → just because herbal meds are natural it doesn’t necessarily conclude that
these are already SAFE; 1. Do not take herbal meds if the condition of the patient is serious 2. Multiple medications
→ therefore only take herbal meds when the condition of the patient is manageable & does not take multiple
meds; it should only be taken as SUPPLEMENTARY

DRUG-CHEMICAL → SMOKING INCREASES THE METABOLISM OF THE DRUG

PRACTICE
PHARMACOGENETIC VARIATIONS → ethnic populations e.g. Chloroquine C/I with G6PD patients because it will
lead to hemolytic anemia

Well studied drug interaction → established


OUR LADY OF FATIMA UNIVERSITY
College of Pharmacy
STABILITY
- the capability of a particular formulation to remain within its physical, chemical, microbiological,

therapeutic, and toxicological specifications.


Types of Stability
Types of Stability
 CHEMICAL
Those manifested by the drug's
molecular structure
Chemical change results to
different drug molecule
Each active ingredient should
retain its chemical integrity and
labeled potency within specified
limits
Types of Stability
 PHYSICAL
The properties of drugs and
dosage forms that can be seen or
tested by physical means:
• Appearance
• Palatability
• Uniformity
• Dissolution
• Suspendability
 MICROBIOLOGICAL
Sterility or resistanceto microbial growth is retained
according to the specified requirements.

 THERAPEUTIC
The therapeutic effect remainsunchanged

 TOXICOLOGICAL

No significant increase in toxicity occurs


INCOMPATIBILITIES
- problem which could occur as a result of interaction
between two or more drugs or chemicals.

- includes physical, chemical and therapeutic


incompatibilities
THINGS TO CONSIDER IN INTERPRETING
COMPATIBILITY REPORTS:

o Manufacturer of the drug


o Drug concentration
o The base solution or diluent and their manufacturer
o Order of mixing
o Time frame
o Temperature
o Test methods
PHYSICAL INCOMPATIBILITIES

@ Physical or chemical
interaction that leads to
visible recognizable change
o Incomplete Solution
o Precipitation from solution
o Polymorphism
o Liquefaction of Solid
Ingredients
o Sorption and leaching
o Evaporation
o Loss of water
A. Incomplete Solution
 Insolubility of components
 Immiscibility of components
 Remedies:
o Use an appropriate solvent system
o For oral or topical product, make suspension or emulsion,
if possible
o Consult prescriber for any changes in the prescription
A. Incomplete Solution

1. lnsolubiIity - inability of the material to dissolve in a


particular solvent system.
Problem encountered in
what dosage form? Solution
@ Remedy when a solid substance fail to dissolve
ina liquid? prepare a suspension
A. Incomplete Solution

2. lmmiscibility - condition wherein two or more


liquids fail to dissolve or mix with one another

Problem encountered in
Emulsion
what dosage form?

Remedy? Add emulsifier


B. Precipitation from Solution
 For oral or topical solution
- will cause therapeutic failure or toxicity.
 For IV solutions
- insoluble particles can lodge in capillaries and block them causing
severe consequences or even death.
B. Precipitation from Solution

3. Precipitation - from physical causes such as:


- Change of solvent system
- Change of temp
- Change in pH
- Salting - out

Problem encountered
in what dosage form? Solution
Precipitation
Change of solvent system

@ when two solutions are mixed, one aqueous and


another an alcoholic, there is a great possibility
of forming solids that come out due to the change
in the solvent
Example:

Gums (Agar, Tragacanth, acacia) in aqueous soln +


alcohol
Example:

@ Alkaline salts like potassium penicillin when placed


on an acidic solution
o precipitation of penicillin
B. Precipitation from Solution
 Change in temperature
o The solubility of most drugs decreases as the temperature of the
solution decreases
o Opposite effect
 When calcium gluconate and potassium phosphate are
added in PN solution, calcium phosphate and dibasic
calcium phosphate will be formed in equilibrium
 At warm temperature, insoluble dibasic calcium phosphate
may precipitate
Precipitation- Change of temp

@ absorption of heat (endothermic) or


giving up heat (exothermic)

endothermic or exothermic?

ORAL RECITATION
B. Precipitation from Solution
 Change in temperature
o Strategies in handling solutions sensitive to change in
temperature
 For parenteral products – check the product insert and
consult references
 For oral or topical solution – be aware of the possible
problems if product will be used or administered at a
different temperature
B. Precipitation
from Solution Salling out agent ir1. water
(Polassium phosphate )

@ Salting out - a
competitive process
wherein the molecule
of a substance
compete for the water - - ,,- '
;-,,... ./

•"
I V

in solution forming 11 l

precipitate
Aqueous SF solution SF 11anopartid es
@ Remedy:
o Use another
solvent
@ an electrolyte is added in solid form to a solution of an
organic non-electrolyte. The non-electrolyte will be
precipitated

@ The precipitation of an organic substance from a


saturated solution when highly soluble salts are added
Precipitation

3. Camphor alcoholic solution + Chloral


hydrate

4. Pectin + excess alcohol


o Pectin is stable at cold acid solution

5. HCI causes precipitation of theophylline


from aminophylline solution
.t

B. Polymorphism \;

@ Change in crystalline form


@ Different polymorphs will exhibit • :,
different physical properties, ·
... \.
such as melting point and
dissolution rates = affects
bioavailability
@ Some drugs exist in different
crystalline structures in the solid
states
@ Example: Cocoa butter -With
several polymorphic forms with •
melting points : 18°,22°, 28° 1111!1SRU BIIIER

and 34°
o When used as base for I
UNREFINED
suppositories is overheated, it COCOA BUTTER
1 ()(1% PUtE • V fC /\1' • U UEl f' H d f

may melt at room


::=::II I.IM
,.,':i'. ..··'1:.-1-, :.:=..:
-I ,Cl //'OII..=..""
WSl >I:- =-
Sl(JNM01$1U9;11U J.."0011' SICIN (t.U11o(;II Ot lll

temperature or liquefy •u
•- =
............
. .....---..n,,e;-....0(1)).H' -·
..,.-":... " :::,-
when handled during ----•
• wo,,.,.,.coou.,.;,w••-- ·- rdl"'
- - IUlWT.Jl'
insertion ·---..·. -· 111111
o Should be melted slowly
and carefully at
temperature not greater
than 34°C
D. Liquefaction of
Solid Ingredients
@ Efflorescent powders (e.g.
morphine acetate, ferrous
sulfate, codeine, caffeine
atropine sulfate, etc.)
o Handling Strategies
■ Store & dispense the
powder In tight container
■ Use Anhydrous substitute
for hydrate
INCOMPATIBILITIES

PHYSICAL INCOMPATIBIL TY
Liquefaction - due to ability to absorb or
give off water upon exposure
- Eutexia
- Hygroscopy
- Deliquescence
Liquefaction - Eutexia
@ lowering of melting point of the substances
@ Examples include aspirin, antipyrine, salicylates,
salol, menthol, thymol
@ Mixtures of phenolic aldehydes, ketonic
compounds and alcohols
@ triturating camphor+ menthol
Liquefaction - Hygroscopy

@ Substances which absorb moisture from the air


but do not dissolve

Hygroscopic substances
@ Sorbitol
@ CMC
@ Dextran
Liquefaction - Deliquescence

@ absorbs moisture from the atmosphere forming water of


hydration and dissolves in the absorbed moisture.

Deliquescent substances:
@ Sodium permanganate
@ Chlorides - Magnesium, Aluminum, Zinc, Calcium,
metacholine Cl
C. Liquefaction of Solid Ingredients

@ Hygroscopic and deliquescent drugs (e.g. Ephedrine


sulfate, potassium citrate, Phenobarbital Na, Sodium
iodide, etc)
@ Remedy for these drugs:
o Store and dispense drug in a tight container
o For solid compounded product - use inert
powdered ingredient (e.g. lactose) that will
absorb water or add insoluble powder
E. Sorption
 Adsorption – molecules are concentrating at the interface
 Absorption – the molecules being absorbed are penetrating into the capillary
spaces of the absorbing surface

+ + + + + + + +
+ + + + + + + + + + + +
+ + + + + + + + + +
+ +
+ + + +
+ + + + +
+ + + + + +
+
+ + + + + + + + + + + +
+ + + + + + + + + + + +
+ + + + ++ + +
adsorption absorption sorption
@ Commonly occur in polyvinyl chloride (PVC)
containers - due to plasticizer
o Drugs that are poor water-soluble or
lipophilic (e.g. Lorazepam, diazepam,
nitroglycerin, nicardipine, ISDN, etc) have
greater tendency to sorb to PVC to
dissolve in its plasticizer
@ Handling drugs that sorb to surfaces:
o Check product inserts and other
references
o Be suspicious of new drugs form an
existing class with sorption problem
o Decrease contact time
o Control ternperature
I
@ Leaching plasticizer from
plastics - for drugs that
contain surfactants or
cosolvents = carcinogenic
based on study in animals
@ Strategy includes:
o Use container substitute
(glass, polyvinyl,
polyethylene,
polyurethrane)
NDC 30 I 1191

Nitro-Dur·
(nitroglycerin)
Trarudctmcll fn!\1m'ISy!,1J'IT\
G. Vaporization 0.4 m g / hr
(20 cm2)
lldl""'1COMalll,01110 CICM
, ... • WM>O' nl01'1
Rx only
@ A.K.A. Volatilization
@ It is the process of liberation of
the active ingredients
@ Example: Nitroglycerin (a
drug used for angina pectoris)

• • ••• •
••
• • •
• • • • • • ••

•• ••
I
@ Common in liquid
dosage forms
o Emulsions (phase
inversions in O/W
emulsions)
o Suspensions and
0 I \.\
solutions
(inc. potency)
INCOMPATIBILITIES

PHYSICAL INCOMPATIBILTY
Solidification - formation of gel, cement, sediment, or
aggregates
Problem
encountered in Suspension
what dosage form?
• Acacia in the presence of bismuth salts can form a "cake"
at the bottom of the container.
CEMENTATION
General remedies for physical incompatibilities
Modify Modify the order of mixing
Change Change the kind of concentration of the solvent
Change Change in the form of ingredient
Add Add therapeutically inactive substance
Omit Omit an inactive ingredients
Change Change the dosage form
Dispense Separate dispensing
Likely physical Effects Steps to prevent
Formulation instability instability
problems

1.Lossof flavour Change in smell Use of proper


Oral solutions or feel or taste excipients and
2.Change in taste suitable packing
materials.
3.Presenceof off
flavours due to
interaction with
plastic bottle

4.Lossof dye

5.Precipitation

6.Discoloration
Formulation Likely Physical.... Effect Steps to prevent
Parenteral 1. Discoloration due to photo Change in Use of
solutions chemical reaction or oxidation. appearance antioxidants
Ex: thiamine hydrochloride
physical and in bio- (0.5%)
instability occurs 2. Presence of precipitate due availability. Acetylcystane or
to interaction with container or
due to: Change in 0.02 - 1%Ascorbic
stopper.
(1) Interaction of appearance acid) or Chelating
the contents with 3. Presence of "whiskers". If and in agents (0.01- 0.075
some small pinholes are present in bioavailability sodium edetale) to
the container. the ampule due to improper prevent
(2) Changes in sealing the solution wicks out, the
liquid evaporates and the solid discoloration.
Chemical Change in stopper or
settles onthe outside. It further
composition. helps in wicking out more material of the
solution and long lines of crystals container will
form on the outside of the vial
eliminate the
which are called whiskers. This
may happendue too small hole problem.
(<0.5 µm) going undetected or the
crack developing during storage.
Formulation Likely physical... Effect Steps to prevent
Parenteral Solutions 4. Clouds: A cloud Checking of the
Cont.. will appear in the manufacturing
product due to: (i) process Increasing
Chemical changes solubility by the use
(an ester eg.: of cosolvents (eg:
polysorbate may polyethylene glycol)
hydrolyse producing
or by other methods
an acid which is
such as micellar
poorly soluble). (ii)
Solubility product approach or
maybe exceeded. complexation will
(iii) The original reduce clouding.
preparation of a
supersaturated
solution or the use of
a metastable form
(ex: calcium
gluceptate).
Formulation Likely physical... Effect Steps to prevent

Suspensions Settling, caking, crystal Loss of Design of product


druggrowth content based on proper
uniformity in pre-formulation
different doses studies.
from the bottle
and loss of
elegance.

Emulsions Changes in:


1. . Loss of drug Design of product
a) Particle size, content based on proper
b) Polymorphic uniformity, loss of pre-formulation
state,or hydration or elegance and studies.
solvation state change in drug
c) Consistency release rate.
d) drug release rate
2. Caking or
coalescence.
3. Bleeding
Formulation Likely physical... Effect Steps to prevent

Tablets Change in Change in Design of product


a) Disintegration drug release based on proper
Time pre-formulation
b) Dissolution Profile studies
c) Hardness
d) Appearance

Capsules Change in Change in drug Design of product


a) Appearance release based on proper
b) Dissolution pre-formulation
c) Strength studies.
The conversion of an optically active form
to an optically inactive form is called:

ORAL RECITATION
the ability of a drug to
exist in more than
one crystalline form.

Example:
Precipitation is a common
problem of what dosage form?

Precipitation can be
Physical or Chemical Incompatibility?
The degradation of aspirin involves:
A. Hydrolysis
B. Racemization
C. Oxidation
D. Photolysis
E. None of the above
2. Chemical Incompatibility
 Occurs as a result of chemical interaction among
the ingredients of a given prescription
 Visible changes in the reaction is not necessarily
observed but can be determined by analytical
method.
"chemical
incompatibiIities"

1. Oxidation 7. Evolution of Gas


2. Reduction 8. Cementation
3. Hydrolysis 9. Gelatinization
4. Solvolysis 10. Polymerization

5. Racemization 11. Photooxidation

6. Explosion
A. Oxidation
 Occurs when one drug losses electron to the other
 Triggered by light, heavy metals, oxygen, oxidizing agents
A. OXIDATION

o Drugs susceptible to oxidation includes:


■ Catecholamine e.g. epinephrine
■ Phenolics e.g phenylephrine, morphine
■ Phenothiazines, chlorpromazine
■ Thiols e.g captopril
■ Others: Amphothericin 8, tetracycline,
furosemide, etc.
A. OXIDATION

@ Strategies
o Protect from oxygen
o Protect from light
o Add metal-chelating agent e.g. edetate disodium
o Add an antioxidant
o Control storage ternperature
o Control pH
o Separate drugs that are easily oxidized from those
easily reduced
)Ji
+

@ Most common type of incompatibility and drug


B. HYDROLYSIS degradation
@ Triggered by presence of water, acids, bases,
catalyst e.g. dextrose
@ Drugs susceptible to hydrolysis:
o Amides esp with beta-lactam rings -
penicillins and cephalosporins
o Esters- procaine, tetracaine, aspirin,
belladona
B. HYDROLYSIS

@ Strategies include:
o For solid - control exposure to moisture by
using light containers and desiccants
o Control the pH
o Check appropriate references
o Consider the drug's concentration
o Control storage temperatures
C. Evolution of Gas
@ Commonly caused by NaHC03
and carbonate buffers =
effervescence
@ Desired use in some powders
and tablets e.g. Alka seltzer
@ Strategy:
o Do not combine drug
products that generate acid
pH with sodium bicarbonate
or drug products that
contain carbonate buffers
o For vulnerable products,
store in tight container
D. Complexation

@ Forming inactive complex


e.g. tetracycline with
multivalent ions
@ Strategy:
o Avoid combining
tetracycline with drug or
food containing _.,
multivalent ions •J shutterstock.com · 435483340
E. Racemization
@ The conversion of one enantiomer
to a racemate
@ It is the action or process of
changing from an optically active
compound into a racemic
compound or an optically inactive
mixture
HO
@ Example: -0 -0
o Epinephrine = the I-enantiomer (1R,2R)-tramadol (1S,2S)-tramadol

is approximately 15-20x more


active than d-enantiomer
o Other drugs: bupivacaine,
albuterol, levalbuterol,
omeprazole, and esomeprazole
,I
F. Epimerization
.
@ Formation the pair of
diasteriomers that differ
, ...
-.. .... o <,o,o c"
T•tt.1eycl-
only in the configuration A<1lvo •ntlblotlc
about one carbon atom
(epimers)
@ Example: Tetracycline ► s«i«tsfor1nubloloc

undergoes reversible .... , i,s t,vtty

[polttr•cyc on• M,ydrotctr.xycl


epimerization to
epitetracycline (of Iittle
antibacterial activity) ,n
Epl.lnhydrotctr.xycl.no
a solution In.active bru kdow n
products
- Rate is dependent on pH
and presence of citrates and
phosphate ions

Remedy
- Prepare suspension
of tetracyline
- Add buffer to
maintain the pH
OTHER EXAMPLES
• Reducing agent (RA) + oxidizing agent (OA)
Explosive
• Sugar + KMn04,
mixtures
• Glycerin + KMn04
• A chemical reaction in which two or more molecules combine to
Polymerization form larger molecules that contain repeating structural units
• Formation of 5-0H-methylfurfural from dextrose
• Also known as photodecomposition
• Degradation by light
Photolysis
• Manifests as change in color
• Example: nifedipine, nitroprusside, riboflavin
Cementation • Cake formation (Acacia+ Bismuth salts)
• Gel formation (Acacia+ Iron salts)
Gelatinization
• Acacia is used as emulsifying agent

130
Physical or Chemical Incompatibility
Physical or Chemical Incompatibility
IDENTIFY
Acacia in the presence of bismuth
salts can form a "cake" at the
bottom of the container

The conversion of an optically


active form to an optically
inactive form
The degradation of Aspirin to
salicylic acid and acetic acid in
the presence of water
The precipitation of an
organic substance from a
saturated solution when
highly soluble salts are added
3. THERAPEUTIC INCOMPATIBILITY

@ A problem that occurs when two or more


drugs are administered to a patient
@ Also called drug interaction - one drug
alters the effect of another drug
o Pharmacokinetic
o Pharmacodynamic
3. THERAPEUTIC INCOMPATIBILITY

 Undesirable pharmacological interactions between two or more


ingredients that leads to:
o Additive or potentiated therapeutic effects - may cause toxicity
or death
o Antagonistic or destruction of the effect - sub- therapeutic or
lack of effect
o Ex: Dec. therapeutic activity of pen G after giving tetracycline
(bacteriostatic)
Dispensing and Incompatibilities
Part Ill
zoom
UNIT OUTCOMES

At the end of this unit, the students are


expected to:
□ Classify medication related problems
such as Drug interactions and ARD
□ Identify ways to minimize medication
related problems
UNIT OUTLINE:

Adverse Drug Reactions


Drug Interactions
ADRVSADE


an injury resulting from medical response to a drug that IS
intervention noxious and
Outward medical occurrence
unintended, and that occurs at
doses normally used in humans

PREVENTABLE UNPREVENTABLE

INTENTIONAL UNINTENTIONAL
RISK FACTORS FOR ADR
1. Age
2. Concurrent medications
3. Duration of Therapy
4. Gender
5. Co-morbidities
6. Narrow Therapeutic Index Drugs
7. Ethnicity

Your footer here 7


TYPE A: AUGMENTED
• KEY NOTES!!
Dose Dependent
Predictable
common
Reproducible

• SUBTYPES
A. EXTENSION EFFECTS
B. SIDE EFFECTS
TYPE A: AUGM ENTE D
EXTENSION
SIDE EFFECTS
EFFECTS

• Benzodiazepine- • Morphine-
SEDATION CONSTIPATION
• ACE lnh- COUGH
• Glibenclamide-
HYPOGLYCEMIA • Minoxidil-
HYPERTRICHOSIS
• Warfarin- BLEEDING
• Furosemide-
• Antibiotics- DIARRHEA HYPOKALEMIA
• Albuterol- • Thiazide-
--,.-
TACHYCARDIA HYPERGLYCEMIA and
HYPERURICEMIA
TYPE B: BIZZARE
• KEY NOTES!!
NOT dose related
Unpredictable
RARE
Unknown Mechanism
SERIOUS and FATAL
• SUBTYPES
A. IDIOSYNCRATIC
B. HYPERSENSITIVITY RXN
• Type 1 : Anaphylactic/lmmediate
• Type 2 : Cytotoxic
• Type 3 : Immune complex
• Type 4 : Delayed
TYPE B: BIZZARE
A. IDIOSYNCRATIC
- Malignant Hyperthermia
• Succinylch oline
• Halothane
• Sevoflurane
- Hemolytic Anemia
• G6PD+ Antimalarials
• G6PD + Sulfonamides
- Steven Johnsons Syndrome
• CArbamazepine
• Phenytoin
• Sulfonamides
TYPE B: BIZZARE
A. HYPERSENSITIVITY
- Type 1: Anaphylactic/lmmediate
• Penicillins- Anaphylaxis
• Clindamycin- Pseudomembranous
colitis

- Type 2: Cytotoxic
• Methyldopa- Hemolytic Anemia
• Chloramphenicol- Aplastic Anemia
TYPE B: BIZZARE
A. HYPERSENSITIVITY
- Type 3: Immune Complex
• Blood Dyscrasia
• Arthus Reaction
• SLE
• Steven Johnsons Syndrome
- Type 4: Delayed or Cell mediated
• Tuberculin Skin Test
• Morbiliform Rash
• Contact Dermatitis
Hypersensitivity Reactions
TYPE OTHER NAMES EXAMPLES MEDIATORS
•Atopy
I Allergy (Immediate) •Anaphylaxis lgE
•Asthma

•Autoimmune Hemolytic
11 Cytotoxic, Antibody Anemia
lgM or lgG
dependent • Thrombocytopenia
•Erythroblastosisfetalis

•Serum sickness
111 Immune Complex •Systemic Lupus
lgG
Disease Erythematosus(SLE)

•Contact Dermatitis
IV Delayed-Type • Mantoux Test
T-cells
Hypersensitivity •Chronic Transplant
Re jection
• Multiple Sclerosis

•Grave's Disease
V Autoimmune • Myasthenia Gravis
lgM or lgG
-1 A
Disease
• •
Hypersensitivity reactions are examples of what
type of ADR?
A. Type A
B. Type C
C. Type B
D. Type D
HYPERSENSITIVITY REACTIONS
acute generalized reactions
that occur when a previously
sensitized person re exposed to
a particle antigen.

Clinical manifestations of
this type reaction include
hemolytic anemia,
thrombocytopenia and
granulocytopenia.
HYPERSENSITIVITY REACTIONS
Contact dermatitis - URUSHIOL

These reactions result from the


formation of drug antibody
complexes in serum, which often
deposit in Blood vessels walls,
resulting in activation of
complement and endothelial cell
• •
injury.
TYPE C: CONTINUOUS
• KEY NOTES!!
DOSE related
TIME dependent
Inc. Duration =
Inc.Risk
 Addiction-PERSON TAKES THE DRUG COMPULSIVELY
EX: MARIJUANA, OPIATES
• Dependence
- Physical Dependence- CRAVING AND COMPULSIVE DRUG-
SEEKINGBEHAVIOR (BODY)
- Psychological Dependence-ABSTINENCE FROM DRUG
PRODUCESPHYSIOLOGICAL SYMPTOMS (MIND)
EX: BZD , CAFFEINE , COCAINE
• Tolerance- REDUCED EFFECT WITH REPEATED USED OF THEDRUG
• EX: NICOTINE
TYPED: DELAYED
• KEY NOTES!!
Tl ME dependent
• Carcinogenicity (malignant neoplasms)
- Antineoplastics
- Heterocyclic Amines
- Aromatic Hydrocarbons
- Nitrosamines
- Aflatoxin

• Teratogenicity
- CBZ
- DES
- PHENYTOIN
- STREPTOMYCIN
- TETRACYCLINE
- THALIDOMIDE
- ISOTRETINOIN
Teratogenicity
TYPE D: DELAYED
Category Animals Humans Descripti Example
on
A y y SAFE Folic Acid, Ferrous
S04
B y N SAFE PARACETAMOL
Amoxicillin
C y N UNSAFE Rifampicin
Theophyline
D y y UNSAFE Tetracycline, Valproic a
Phenytoin, Strepto
(B>R) Carbamazepine

X y y TERATOG lsotretinoin
Thalidomide
ENIC DiethylstiIbestrol
If animal studies have shown an adverse effect
but there is no adequate and well-controlled
studies in pregnant women:
A. Category A
B. Category B
C. Category C
D. Category D
E. Category X
Learning check.................

adequate, well- controlled


studies in pregnant women
have not shown an increased
risk of fetal abnormalities.

animals studies have revealed


no evidence of harm to the
fetus ; however there are no
adequate and well controlled
studies in pregnant women
Learning check.................
adequate, well- controlled or
observational, in animals or pregnant
women have demonstrated a risk to
the fetus; however the benefits of
therapy may outweigh the potential
risk.
animal studies have shown an
adverse effect, but adequate and
well controlled studies in pregnant
women have failed to demonstrate a
risk to the fetus
It! t
Learning check.................

use of the product is contraindicated


in women who are or may become
pregnant.

Use of Lansoprazole

Use of Leflunomide
Can cause birth defects, EXCEPT:
A. Vitamin B9
B. Vitamin A
C. Finasteride
D. Nicotine
E. Alcohol
Teratogenic drugs

Ebstein anomaly Lithium

Spina bifida VPA

Masculinization of Anabolic Steroids


female fetus
Phocomelia
DRUG: Thalidomide
Manifestations Polyneuritis
Limb deformities
Mental retardation
neural birth defects Carbamazepine
& Valproic acid

risk of developing
vaginal adenocarcinoma DES
after puberty
fetal hydantoin Phenytoin
syndrome

discoloration & defects of


teeth & altered bone Tetracycline
growth
8th cranial nerve Streptomycin
FOLIC ACID A. Category A
Paracetamol B. Category B
Thalidomide C. Category C
lsotretinoin D. Category D
Amoxcillin E. Category E
F. Category X
A. Category A
B. Category B
C. Category C
D. Category D
E. Category E
Theophylline F. Category X
Vitamin A
Tetracycline
Rifampicin
Ampicillin
TYPE E: END OF USE

• KEY NOTES!!
UNCOMMON
WITHDRAWAL

► Opiate Withdrawal -> Opioids


► Rebound lnsomia -> BZD
► Rebound Hypertension Clonidine
->

► Adrenal Crisis Corticosteroids


->

► Rhinitis Medicamentosa Nasal decongestant


->
• KEY NOTES!!
unexpected failure of efficacy
Common
dose-related
► Drug-drug Interactions
► Use Of Counterfeit Drugs
► Drug Instability
► Patient's Non-compliance
► Wrong Route Of Administration
► Drug Resistance
► Expired Drugs
Carcinogenicity

Idiosyncratic rxn

dose dependent
Extensive sedation
caused by decreased
clearance of a usual dose
of a benzodiazepine
Anaphylaxis
Cardiotoxicity caused by
doxorubicin
Mantoux test
predictable
Alcohol dependence

Counterfeit drugs

Clonidine- rebound
hypertension

Extension of
pharmacologic effect
Teratogenicity

Ethambutol- optic
neuropathy

alcohol- delirium tremens

Carcinogenicity
DRUG INTERACTION
- it refers to any ADR produced by the admin of drug or co-
exposed of the drug with another substance, which modifies the
patient’s response to the drug.
 Drug-food
 Drug-laboratory
 Drug-drug
Precipitant drug

• one that causes the


interaction
OBJECT DRUG
• one that is affected in the
interaction
EXAMPLES:
Bisacodyl + Milk -> alteration of
pHBarbiturates +Alcohol : inc.
sedation
A. DRUG-FOOD INTERACTION

• CNS Depressants + Caffeine


Antagonism
• Warfarin + Green Leafy Vegetables
Antagonism
• Tetracycline + Dairy Products
Dec. absorption
• MAOI + Tyramine-Rich Foods
Dec. metabolism of NE
• INH + Histamine-Rich Foods
Flushing
• BISACODYL + MILK
Alteration of pH
• ASA + Caffeine
alteration in Ph -> inc absorption
A. DRUG-FOOD INTERACTION

Increased by food Decreased by food


Griseofulvin* Captopril
Acarbose lsoniazid
M etoprolol Penicillamine
ltraconazole Penicillins
TheophylIine* Erythromycin stearate
Tetracycline
Alendronate
Quinolones
B. DRUG-LABORATORY TEST
INTERACTION
1. Penicillin, Chloramphenicol, Vit. C, INH,
Streptomycin
- glucose in Urine (Benedict's test)
- false (+) result

2. Chlordiazepoxide
- thyroid function test {1131 )

- false (-) result

3. Rifampicin (red-orange), Vit.B1 2 (intense yellow),


Chloroquine (brown)
- urinalysis
- change in color
C. DRUG-DRUG INTERACTION
 Pharmacokinetic
1. ALTERED GIT ABSORPTION
a. Altered pH
b. Altered bacterial flora
c. Formation of drug chelates or complexes
d. Drug induced mucosal damage and altered GIT motility
2. DISPLACED PROTEIN BINDING (Distribution)
3. ALTERED METABOLISM
a. Enzyme Induction
b. Enzyme Inhibition
4. ALTERED RENAL EXCRETION
a. Alteration of Urinary pH
b. Alteration of Active Transport
 Pharmacokinetic
1. ALTERED GIT ABSORPTION
a. Altered pH
ANTACID + BISACODYL
ANTACID + KETOCONAZOLE (DEC. K)
ANTACID + SALICYLATES (DEC. S)
b. Altered bacterial flora
ANTIBIOTICS+ DIGOXIN
c. Formation of drug chelates or complexes
TETRACYCLINE/ FLUOROQUINOLONES/
PENICILLAMINE + METAL
CHOLESTYRAMINE + DIGOXIN/ WARFARIN
SUCRALFATE + LEVOTHYROXINE (DEC TA)
1. ALTERED GIT ABSORPTION
d. Drug induced mucosa/ damage and altered GIT
motility
• Gastric Emptying Rate & GE Time
ATROPINE+ ANTACID (INC ANT A)
ATROPINE + AMPHETAMINE (DEC AMP A)
NICOTINE+ ANTACID (DEC ANT A)
• Increased GI Motility
CATHARTIC + ANY DRUG (DEC DRUG A)
• Adsorption of the Drug
ADSORBENT + ANY DRUG (DEC DRUG A)
• Interruption of Enterohepatic Circulation
ANTIBIOTICS + OCP
Changes in gastrointestinal motility
Increase GIT motility
Metoclopramide
Cathartics
Decrease GIT motility
Anticholinergics

Malabsorption caused by drugs


e.g. Penicillin and Neomycin
- neomycin-induced malabsorption state
MULTIPLE CHOICES:
Epinephrine will the
percutaneous absorption of transdermal
lidocaine or transdermal fentanyl.
A. Increase
B. Decrease
C. Not affected DUE TO
D. Hasten VASOCONSTRICTION

E. Slow
This drug is inactivated by intestinal flora:
A. Ketoconazole
B. Lidocaine Alteration of gastric pH
C. Digoxin
D. Erythromycin GIT metabolism
E. Fentanyl
A. Ketoconazole
Alteration of gastric pH
Ketoconazole + antacids

• Ketoconazole requires acidic medium for


Absorption
B. Lidocaine

• Not given orally because -ineffective

• with phenytoin = excessive cardiac


depression
• with procainamide =
Additive neurological effects

ADRs: pain, itching, burning and discomfort


MGT: Pramoxine
C. Erythromycin
• Should not be given with food=
erythromycin stearate

• Can be given with food =


estolate, ethylsuccinate

• Enzyme inhibitor
Alteration in GI Flora

Ampicillin + OCP
ResuIt:
increased chance of unwanted

pregnanci es

Anticoagulants + Antibiotics
Digoxin + Antibiotics
2. DISPLACED PROTEIN BINDING (Distribution)

Phenytoin is a highly bound to plas111a protein (90%),


Tolbuta1nide (96%), and warfarin (99%

Drugs that dis place these agent are Aspirin


Sulfonamides
phenylbutazone

WARFARIN + PHENYLBUTAZONE (HEMORRHAGE)


GLIBENCLAMIDE + PHENYLBUTAZONE (HYPOGLYCEMIA)
OHA + ASA (HYPOGLYCEMIA)
BILIRUBIN + SALICYLATES(KERNICTERUS)
B. Alteration of Distribution
Drug displacement interactions
e.g. Aspirin and Phenytoin
Warfarin + Phenylbutazone
Methotrexate + salicylates
Phenytoin +VPA

Reduced albumin concentration


- renal, hepatic and GI diseases
If valproic acid displaces phenytoin from
albumin and aspirin displaces valproic
acid, the most highly protein-bound is:
A. Aspirin
B. Phenytoin
C. Valproic acid
D. None of these
E. All of these
• Highly protein bound drugs
• Phenylbutazone
• Aspirin
• Warfarin

• Methotrexate
• Valproic acid
3. ALTERED METABOLISM

P henobarbital Metronidazole
Phenytoin Allopurinol
Grape fruit Juice
Rifampicin
lsonizaid
Carbamazepine
Cimetidine
Chronic Alcoholism Disulfiram
Charcoal boiled Food Acute alcoholism
St John's Wort Chloramphenicol
Ketoconazole
Erythromycin
Valproic acid
C. Drug metabolism interactions
1. Enzyme induction
oral contraceptives and barbiturates
- reduced contraceptive effects

acetaminophen and phenobarbital


- Increased acetaminophen clearance
- Increased production of hepatotoxic
metabolites of paracetamol
2. Enzyme inhibition
e.g. phenytoin and chloramphenicol
- increased phenytoin effects,
intoxication possible
alcohol and disulfiram
Benzodiazepine and Cimetidine
Alteration in GIT metabolism

phenelzine with cheddar or pickled fish

ResuIt:
Hypertensive crisis
Alteration in GIT metabolism

Grapefruit juice + CCB's

+ HMG - coA reductase


inhibitors

+ CBZ

+ Cyclosporine
Matching Type:
A - Enzyme inducer
B - Enzyme inhibitor
Matching Type:
A - Enzyme inducer
B - Enzyme inhibitor
4. ALTERED RENAL EXCRETION

Interaction Example I Effect


1. Glomerular M ETHYLXANTH INES Rapid renal
Filtration Rate excretion
2. Active Tubular PROBENECID (-) ACTIVE
Secretion TUBULAR
SECRETION
3. Alteration of ASA+ NaHC03 Increase
Urine pH excretion
D. Interactions due to changes in excretion
1. Changes in urinary pH
- drugs in the polar form are excreted more
favorably as compared to its non-polar or
lipophilic form.
D. Interactions due to changes in excretion
2. Changes in active kidney tubule excretion
e.g. Probenecid and penicillin
- affected drug serum level
raised, possibility of toxicity with
some drugs.

3. Changes in kidney blood flow


• Effects of Increased theophylline
metabolism:
A. Decreased excretion Increase
B. Increased blood level Decrease
C. Increased elimination
D. A and B
E. Band C
THEOPHYLLINE
• Theophylline + Thiabendazole
• Decreases metabolism of theophylline
• increased serum levels of theophylline
Learning check.................
Drug interaction Mechanisms

Fluoroquinolones + Metals Complexation

Ketoconazole + Antacids Alteration of


gastric pH

MAOl's + Tyramine Alteration of


Metabolism in
the GI tract
Learning check.................
PHARMACOKINETIC INTERACTION
Alteration of Absorption
Distribution
Metabolism
Excretion

Complexation and Adsorption Absorption

Alteration of urinary pH Excretion


Alteration of GI flora Absorption
Displacement Distribution
Learning check.................
Which statement is incorrect regarding interaction involving
tyramine containing substance and MAO inhibitor?
A. Inhibition of MAO results in the
accumulation of large amounts of
norepinephrine
B. Decrease ion the rate of intracellular
metabolism
C. Large amount of norepinephrine can
cause severe headache and hypotension
D. Large amount of stored epinephrine can
cause hypotensive crisis
E. None of the above
Learning check.................
Consumption of grapefruit juice can INCREASE or
DECREASE concentrations of the following drugs
Amlodipine
Lovastatin
Cyclosporine

Answer: INCREASE
Alteration of Metabolism in the GI
tract
Learning check.................

Effects of concurrent use of alcohol and chloral


hydrate

additive CNS depressant effects


Answer:
Trichloroethanol competitively
depresses the oxidation of
alcohol to aldehyde
The metabolic pathways for their
elimination are mutually
inhibited.
Capoten® + Potassium Sparing
Diuretics:

May cause hyperkalemia


C. DRUG-DRUG INTERACTION
•••
♦ Pharmacodynamic
1. ADDITIVE
INTERACTION EFFECT

ROH + Barbiturates SEDATION


ROH+ Antihistamines
ROH+ CNS
Depressants
ROH+ Chloral Hydrate
Aminoglycoside + Loop OTOTOXICITY
Diuretic
Flecainide + Verapamil negative inotropic &
chronotropic effects
C. DRUG-DRUG INTERACTION
•••
♦ Pharmacodynamic
2. SYNERGISTIC

INTERACTION EFFECT
Sufamethoxazole + INC. Bact ericid aI
Trimetoprim Activity
C. DRUG-DRUG INTERACTION
•••
♦ Pharmacodynamic
3. POTENTIATION
INTERACTION
Amoxicillin + Clavulanic AUGMENTIN
Acid
Ampicillin + Sulbactam UNASYN

PiperaciIIin + PIPTAZ
Tazobactam
Levodopa + Carbidopa
C. DRUG-DRUG INTERACTION
•••
♦ Pharmacodynamic
4. ANTAGONISM
INTERACTION
Phenoxybenzamine + Catecholamines:

Warfarin + Vit K

BZD + Flumazenil

Atropine+ Neostigmine
Occurs when two or more drugs, with or
without the same overt effect, are used
together to yield a combined effect that
has an outcome greater than the sum of
the single drugs active components alone.

SYNERGISM
Describes a particular type of
synergistic effect-a drug interaction in
which only one of two drugs exerts
the action that is made greater by the
presence of the second drug.

POTENTIATION
In this type of chemical incompatibility,
chemical reaction can be reduced by mixing
the solution in dilute forms or by changing the
order of mixing.

TOLERATED
This refers to the potential of effect
when 2 drugs with the same active
ingredient or with the same action are
taken at the same time.

ADDITIVE
ADDITIVE

Flecainide + Verapamil ResuIt:


severe cardiogenic shock

aspirin and quinidine ResuIt:


generalized petechiae
and blood in feces.
Increase bleeding time
C. DRUG-DRUG INTERACTION

•!• Pharmaceutical

 caused by a chemical or physical


incompatibility when two or more drugs are
mixed together
 occurs when drugs are mixed
inappropriately in syringes or infusion
fluids prior to administration
C. DRUG-DRUG INTERACTION
•!• Pharmaceutical
FLUID INCOMPATIBILITY
AMINO ACID Do not add any drug to amino
acid fluids. These fluids may
degrade acidifiable drugs.
BLOOD Do not add any drug to
blood. Its capacity prevents
recognition of drug
incompatibility.
SODIUM CHLORIDE Do not add amphotericin.

SODIUM BICARBONATE CaCl2, Ca gluconate,


Vanco mycin, Insuli n,
OTHER ESSENTIAL INTRXN
SUBX A SUBX B EFFECT
► Nephrotoxicity
► Ototoxicity
AMINO GLYCOSIDE LOOP DIURETIC

BETA BLOCKERS INSULIN ► Masking of


Hypoglycemic
effect
PRAZOSIN ► Inc " First Dose"
Phenomenon
THIAZIDES DIGITALIS ► Inc toxicity of
Digitalis
because of low
K levels
DRUG-HERBAL INTERACTIONS

@ Can occur as pharmacokinetic or pharmacodynamic interactions


@ Drug with narrow therapeutic windows are at greater risk for
drug-herbal interactions
@ Co enzyme Q10 has a chemical structure related to vitamin K, the
combination of warfarin and coenzyme Q10 results in antagonized
warfarin activity and inadequate anticoagulant
@ Ginger, garlic, and fever few given to patients on warfarin -> inc
risk of bleeding due to direct inhibition of platelets
@ Wheat grass will interact with Vit K and Warfarin

86
DRUG-CHEMICAL INTERACTIONS

@ Aromatic polycyclic HC in cigarettes can induce the


metabolism of the ff:
o Diazepam, Theophylline, Tricyclic Antidepressants
@ Ethanol can increase or decrease metabolism
o Chronic alcoholism can induce the metabolism of
tolbutamide, warfarin, and phenytoin
o Acute alcohol intoxication inhibits hepatic enzyme

87
POTENTIAL FOR DRUG INTERACTIONS
@ Multiple Drug Therapy
o The more drugs used by a patient, the greater potential for a drug interaction
@ Multiple Pharmacologic Effects
o Drugs are used for their primary effect but may also possess secondary
effects that may cause the interaction
@ Multiple Prescribers
o Patient are be seen by different physicians who may prescribe interacting
medications
@ Patient Noncompliance
o Patients need to follow proper instructions for taking medications
o Ex: Taking a drug with meal rather than on an empty stomach
@ Patient Risk Factors
o Older patients have more risk for DI than younger patients
o Patients with predisposing illness (Diabetes, Asthma, AIDS, and Alcoholism)
and patients who are clinically hypersensitive (atopic) are more at risk for DI
88
than non-topic patients
A - if effect is reduced or delayed by food
B - if effect is increased by food
C - if not affected by food

Aspirin
Phenytoin
Theophylline
Morphine
Penicillin
Food that should not be taken with food

• Tetracycline Decrease Absorption

• Penicillamine
• Alendronate Reduce bioavailability

• Captopril Reduce Absorption (30-40%)

• lsoniazid Decrease Absorption


With food

• Griseofulvin
• Acarbose
• Metoprolol
• ltraconazole increased therapeutic effect

Not affected or delayed by food

• Omeprazole
PHARMACOGENETIC INTERACTIONS

@ Pharmacogenetic variations in enzyme function can occur at


different rates in different ethnic population and, at a lesser
extent, in different geographical areas. Such pharmacogenetic
factors can lead to a wide variation in the potential for a drug
interaction.
o Genetic polymorphism may affect the toxicity and
clinical efficacy of certain drugs in certain
populations

92
Prevention of ADRs and Adverse Drug Events

Known ADRs and Side Medication and Product


Effects Device Error Defects

Unavoidable Avoidable
'
Preventable
- Adverse -
Events
Remaining
Uncertainties:
'
D Unexpected
ADRs and side
-- Injury or Death - effects
D Unstudied uses
□ Unstudied
populations

Figure 1. Schematic of preventable and unavoidable adverse events.


Clinical significance
ESTABLISHED – supported by well proven clinical studies
PROBABLE – very likely but might not be proven clinically
SUSPECTED – might occur and some data might be available
POSSIBLE – could occur and limited data are available
UNLIKELY – doubtful ; no good evidence of an altered clinical
effect is available
Drug interaction that is supported by well-
proven studies:
A. Established
B. Probable
C. Suspected
D. Possible
E. Unlikely
Drug interaction COULD OCCUR.

Drug interaction MIGHT OCCUR.

Drug interaction is VERY LIKELY to occur.


Well studied drug interaction.

Doubtful interaction
HOSPITAL PHARMACY

• Hospital pharmacy practice → a service in a hospital which is under the direction of a professionally
competent, legally qualified pharmacist; but now here in Philippines we are adapting → Clinical
pharmacy program; A service in a hospital which is under the direction of a professionally
competent, legally qualified pharmacist.

• Clinical pharmacy program → adaptation of patient-centered pharmacy services; integration of


hospital pharmacy but this time much involve in regards to preparation of patient profiles, recording
patient drug history, monitoring patient’s medication and patient counseling; pharmacists are not
often interacting with the patients → but through this program pharmacists are expected to participate
in the patient care like patient counseling, knowledge about drug information, services of pharmacy
such as monitoring so as to avoid ADEs or other undesirable effects; clinical pharmacy program is the
state and appropriate use of drugs in patients, rational selection of drugs, monitoring, dosing, &
controlling patient’s overall drug therapy; AIM: optimal use of medication or best outcome through
provision of drug formulation, monitoring of drug safety, & efficacy; through this program we can
predict possible drug interaction and prevent ADRs, & ADEs of meds; Preparation of patient
profiles, recording patient drug history, monitoring patient’s medication and patient counseling

• Institutional pharmacy → permitted by the board and is devoted exclusively to provide professional
services to a patient in an institutional other than the hospital setting; institution → social structure
where people cooperate, influences the role of people in the way they live; early institution → is
usually maintained and established through social and regulatory agencies; institutional pharmacy →
particular facility or a part of an institutional facility where drugs, devices, and other materials used in
the diagnosis and treatment of injury, illness and disease are dispensed, compounded, and distributed
and pharmacy primary care is provided e.g. supreme court → cater pharmacy services and
medications to the employees of the court; A part of an institutional facility where drugs, devices,
and other materials used in the diagnosis and treatment of injury, illness, and disease are
dispensed, compounded, and distributed and pharmacy primary care is provided.

• ASHP Guidelines on the Competencies Required in Institutional Pharmacy Practice (ASHP


guidelines and hospital guidelines = almost same)
o Effective administration and management of a pharmacy department in an institution
o Assimilation and provision of comprehensive information on drugs and their action.
o Development and conduct of a product formulation and packaging program.
o Conduct and participation in research
o Development and conduct patient-oriented services
o Conduct and participation in educational activities
o Development and conduct of a QAP for pharmaceutical service.

• Hospital Pharmacy Practice → where all medications are supplied to the nursing units and other
services for the procurement of medications; where Rx are filled for ambulatory patients and
outpatients → a division of hospital for the → manufacturing, compounding, and distribution or
dispensing of prescription of the hospitalized or ambulatory patients
o Department or service in a hospital which is under the direction of a professionally competent,
legally qualified pharmacist.
o Where all medications are supplied to the nursing units and other services.
o Where prescriptions are filled for ambulatory patients and outpatients.
o Where pharmaceuticals are manufactured in bulk.
Uniqueness of hospital pharmacy from the community pharmacy:

1. Internal forces affecting the practice of pharmacy on a hospital →


1.1. Organizational structure of hospital or institution → much broader than a community
pharmacy; BOARD OF TRUSTEES, ADMINISTRATION/AOH (that would implementing
the policies and philosophies of the boards), DEPARTMENT HEAD of each of the services,
• Business and accounting department → responsible for financial affair
• Building services department → maintenances, housekeeping, security
• Personnel department → for personnel policies
• Clinical Laboratory department → patient laboratory tests & services
• DEPENDING ON THE ORG STRUCTURE OF THE HOSP
1.2. Physician-pharmacist-nurse-patient relationship → involves a triad starting from a physician
prescribing a medication → then the nurse being instructed by that medication for
administration → the nurse will proceed to the pharmacist for the procurement of the
medication → once procured the RPh will inform the nurse about the dose and will prepare
→ nurse will receive and be the one administering the drug → the patient as the end receiver
of the medication & will be subjected to patient counseling

2. EXTERNAL FORCES → JCAHO: Joint Commission on Accreditation of Healthcare


Organizations
• Accreditation agencies → influence professionals standard of practice as these practice affect
patient care → standardized the how patient care should be done or the system and process in
which it is provided e.g. → ISO (International Organization for Standardization) depending
on what accreditation clause applying for, ASHP (American Society of Health-System
Pharmacists)
• Licensing agencies → exerting legal influences on the hospital operations e.g. DOH, & PRC
• Federal government → impose standards and regulations in the hospitals like for example →
conditions for participation of hospitals under Medcare
• Third-party (Hospital insurance agencies) → influence on the methods by which the hospital
reimbursed on the services rendered to the patient
• Social agencies and government welfare agencies → influence the services provided
especially to medically indigent or totally indigent patient e.g. DSWD, PCSO, non-
governmental organizations that provide financial support for indigent patients
• Governing board and public opinion → influences over qualities, objectives, and
philosophies. Because the hospital is an institution and for the community → so heavy
influence by the needs, expectation and demands of the members of the community →
demands can have direct & indirect impact as to the practice of pharmacy → all in all they
should be able to support the mission and goal of the hospital in providing patient care

ASHP GUIDELINES: Minimum Standard for Pharmacies in Hospitals

• Standard I: Practice management → is important because it would affect how patient care is
provided by the hospital pharmacy department. The leader for the pharmacy → should go through the
organizational structure that would support the mission of the department and the hospital in general.
The organizational structural development will require communication and collaboration with not
only internally or the pharmacy department itself but also with other departments and services
throughout the hospital, which every member of the pharmacy team should cultivate at every
opportunity.

*Effective leadership and practice management skills are necessary for the delivery of pharmacy
services in a manner consistent with the hospital’s and patients’ needs.

• Standard II: Medication-Use Policy Development → All committees that make decisions
concerning medication management and use shall have at least one pharmacist as a member. This
includes the P&T, infection-control, patient care, medication-use evaluation, medication safety,
nutrition, pain management, and information technology committees, as well as the institutional
review board (or their equivalents)

* Pharmacists shall be involved in the development, implementation, and assessment of care plans
(protocols, critical pathways, disease statement management programs, or clinical practice
guidelines), standing orders, and order sets that involve medication therapy.

• Standard III: Optimizing Medication Therapy → Creating relationship with the patient →
clinical pharmacy program, acquiring essential patient data, consulting with other health
professional about the medication therapy → to optimize medication use. Collaboration with
medical and nursing staffs, shall develop policies and procedures based on demonstrated best
practices for ensuring the quality of medication therapy. Clinical imperatives should be the primary
determinants of medication-use decisions.
• Standard IV: Drug Product Procurement and Inventory Management →
o Selecting sources of pharmaceutical products → The pharmacy shall be responsible for the
procurement, distribution, and control of all drug products used in the hospital for inpatient and
ambulatory patients. Policies and procedures governing these functions shall be developed by the
pharmacy with input from other appropriate hospital staff and committees. There shall be policies
and procedures for managing medication acquisition. These policies and procedures should
address such issues as formulary development (including initial evaluation for formulary
consideration, medication-utilization review programs, and therapeutic interchange), competitive
bidding, group purchasing, best practices, medication shortages, outsourcing, and cost-effective
patient services.
o Managing inventory → Particular system that should avoid and detect minimum inventory level
that would alert the pharmacist about the potential drug shortages. The pharmacy should develop
strategies for identifying alternative therapies, working with suppliers, collaborating with
physicians and other health care providers, and conducting an awareness campaign in the event of
a drug product shortage.
o Inspecting Storage Areas and Inventory Items → All stocks of medications shall be inspected
routinely to ensure the absence of outdated, unusable, recalled, or mislabeled products. Storage
conditions that would foster medication deterioration, storage arrangements that might contribute
to medication errors, and other safety issues shall be assessed, documented, and corrected.
o Returning Recalled, Expired, and Other Unusable Items → There shall be a written
procedure for the timely handling and documentation of a drug product recall. These procedures
should include an established process for removing from use any drugs or devices subjected to a
recall, notifying appropriate health care professionals, identifying patients who may have been
exposed to the recalled medication, and, if necessary, communicating available alternative
therapies to prescribers. The pharmacy shall be notified of any defective drug products or related
supplies and equipment encountered by the nursing or medical staffs. All drug product defects
should be reported to the FDA’s MedWatch reporting program.

• Standard V: Preparing, Packaging, and Labeling Medications →


o Preparing medications → Compounding. Drug formulations, dosage forms, strengths, and
packaging that are not available commercially but are needed for patient care shall be prepared by
appropriately trained personnel in accordance with applicable practice standards and regulations.
The pharmacy shall provide adequate quality-assurance procedures for these operations. Written
master formulas and batch records (including product test results, as appropriate) shall be
maintained, and a lot number or other method to identify each finished product with its
production and control history shall be assigned to each batch.
o Packaging Medications → Unit Dose Packaging. Whenever possible, medications shall be
available for inpatient use in single-unit packages and in a ready-to-administer form.
• Standard VI: Medication Dispensing and Delivery →
o Medication dispensing → Review of Medication Orders. All medication orders shall be
prospectively reviewed by a pharmacist and assessed in relation to pertinent patient and clinical
information before the first dose is administered or made available in an automated dispensing
device, except in emergent situations in which the treatment of the patient would be significantly
compromised by the delay that would result from pharmacist review of the order. There shall be a
procedure for retrospective review of these orders.
o Medication delivery and administration → Medication Delivery and Administration Drug
Delivery Systems, Administration Devices, and Automated Distribution Devices. The pharmacy
shall have responsibility for developing policies, procedures, and quality- assurance programs
regarding drug delivery systems, administration devices, and automated distribution devices that
ensure safety, accuracy, security, and patient confidentiality. The potential for medication errors
associated with such systems and devices should be thoroughly evaluated.
o Medication Administration: Only personnel who are authorized by the hospital in accordance
with applicable laws and regulations and appropriately trained shall be permitted to administer
medications to a patient. All administered, refused, or omitted medication doses should be
recorded in the patient’s medical record according to an established procedure, and all
medications that have not been administered should be returned to the pharmacy. No medication
should be administered to a patient unless medical and nursing personnel have been provided
with adequate information about, and are familiar with, its therapeutic use, method of
administration, potential adverse effects, and dosage.
• Standard VII: Monitoring Medication Use →
o Educating and counseling patients → Pharmacists shall be available to participate in patient
education. Pharmacists should help to ensure that all patients are given adequate information
about the medications they receive in order to help patients participate in their own health care
decisions and encourage adherence to medication regimens.
o Reviewing Patient Responses to Medication Therapy → Medication therapy monitoring shall be
conducted by pharmacists. Medication therapy monitoring includes a proactive assessment of
patient problems and an assessment of:

a. The therapeutic appropriateness of the patient’s medication regimen.


b. Therapeutic duplication or omissions in the patient’s medication regimen.
c. The appropriateness of the dose of the medication, as well as the route, method, and frequency
of administration of the medication.
d. Patient adherence to the prescribed medication regimen.
e. Medication–medication, medication–food, medication– dietary supplement, medication–
laboratory test, and medication–disease interactions.
f. Adverse drug reactions and other undesired effects.40
g. Patient medication allergies and sensitivities.
h. Clinical and pharmacokinetic laboratory data to evaluate the efficacy and safety of medication
therapy and to anticipate toxicity and adverse effects.
i. Physical signs and clinical symptoms relevant to the patient’s medication therapy.
j. Assessment of the effectiveness of the patient’s medication therapy.

• Standard VIII: Evaluating the Effectiveness of the Medication-Use System → There shall be an
ongoing, systematic program for quality assessment and improvement of pharmacy services and the
medication-use system. The program should include routinely evaluating the literature for new
technologies or successful practices that have been demonstrated to enhance safety in other
organizations to determine if such technologies or practices can improve the hospital’s medication-
use system. This program should be integrated with the hospital’s or health system’s quality
assessment and quality improvement activities.
o Assessing Pharmacy Services and Practices: Documentation of Pharmacist-Provided Patient
Care Services and Medication Therapy Outcomes → The pharmacy shall have an ongoing
process for consistent documentation of the patient care services provided by pharmacists and
patient outcomes from medication therapy.
o Improving the Medication-Use Process: Medication-Use Evaluation, Medication safety, and
Antimicrobial Stewardship and Infection Prevention and Control.
• Standard IX: Research → The pharmacist should initiate, participate in, and support clinical and
practice-related research appropriate to the goals, objectives, and resources of the specific hospital.

DEVELOPMENT AND EXPANSION OF HOSPITAL

• Indian and Egyptian culture during the 6th century BC


• Temples of the gods in early Greek and Roman civilization → were used as hospitals or healing were
associated with divine powers; continued ailments and deaths will be associated lack of purity
• Greek temples were the forerunners of modern hospitals to provide refugee and treatment for the sick
or to even teach young medical students. Greek temples such as Temple of Aesculapus (the Greek
god of Medicine) in 1134 BC and Temple of Kos (where Hippocrates had practice) in 460 BC
• First hospital in American continent was built by the Spaniards led by Cortez in Mexico. → named as
the Hospital of Immaculate Concepcion → renamed on 1663 to Hospital of Jesus of Nazareth
• In America, a hospital was built in 1663 on Manhattan Island → particularly for 6 soldiers
• First incorporated hospital in the US was the Pennsylvania Hospital established on 1751 which was
established through the effort of Dr. Thomas Bond.
• CERTAIN FACTORS THAT INFLUENCE THE DEVELOPMENT & EXPANSION OF
HOSPITALS (private, public, community):
1. Religious influences → mystical, magical and religious ceremonies have been conducted,
2. Doctrine of Christ → furthered intensifies the emotion and virtues of love, pity, and charity that
have led to the development of hospitals,
3. Military influence → due to the immediate need of care of those injured in the battle field, even
president Lincoln requested catholic sisters to care wounded army personnel,
4. FLEXNER report (Abraham Flexner) on medical educational → have caused revolutionary
development in medical education, medical internship training and help develop minimum standards
for patient care throughout the globe.
5. Works of Florence Nightingale → she revolutionized the quality of nursing care in hospitals, and
paved way in the development of schools of nursing.
6. Public interest in hospitals → caused the public to become more dependent and more confident in
hospital care

• HOSPITAL: structure which pools together all the health professionals, the diagnostic & therapeutic
facilities, equipment & supplies & the physical facilities into a coordinated system for delivering
health care to the public.
o PRIMARY FUNCTION OF HOSPITAL (according with AHA → American Hospital
Association): the provision of patient services whether diagnostic or therapeutic for any
medical condition. Additionally, the term hospitals refers to the → physical make-up of the
institution
• CLINIC: a facility or area where ambulatory patients are seen for special study and treatment by a
group of physicians practicing together, and where the patient is not confined in a hospital.

• REQUIREMENTS FOR ACCEPTING GENERAL HOSPITALS FOR REGISTRATION:


1. Maintain at least six inpatient beds, which shall be continuously available for the care of patients
who are nonrelated and who stay on the average in excess of 24 hours per admission.
2. Constructed, equipped, and maintained to ensure the health and safety of patients and to provide
uncrowded, sanitary facilities for the treatment of patients.
3. There shall be an identifiable governing authority legally and morally responsible for the conduct
of the hospital.
4. There shall be a CEO to whom the governing authority delegates the continuous responsibility for
the operation of the hospital in accordance with established policy.
5. There shall be an organized medical staff of physicians that may include but shall not be limited to
dentists.The medical staffs shall be accountable to the governing authorities for maintaining proper
quality of medical care, and shall be govern by laws.
6. Each patient shall be admitted on the authority of a staff member who will be responsible for the
patient’s diagnosis and treatment. Any graduate of a foreign medical school who will assume
responsibilities for patient care shall have any of the following:
o a valid license to practice medicine
o certificate from the Educational Council for Foreign Medical Graduates
o completed an academic year of supervised clinical training approved by the Committee on
Medical Education
7. Registered nurse supervision and other nursing services are continuous.
8. A current and complete medical record shall be maintained by the hospital for each patient and
shall be available for reference.
9. Pharmacy service shall be maintained and supervised by a registered pharmacist
10. Provide patients with food service that meets nutritional and therapeutic requirements, special diet
shall also be available.
11. Maintain diagnostic X-ray service with facilities and staff for a variety of procedures.
12. Maintain a clinical laboratory service with facilities and staff for a variety of procedures.
Anatomical pathology shall be available.
13. Maintain operating room service with facilities and staff.

• FOR SPECIAL HOSPITALS → no. 1-10 and addition of nos. 11-12


11. Diagnostic and treatment services shall be determined by the Board of Approval to be appropriate
for the specified medical conditions for which medical services are provided with suitable facilities
and staff.
12. When the institution provides pregnancy termination services, clinical laboratory services shall
include to provide tissue diagnosis.

CLASSIFICATION OF HOSPITALS:
1. TYPE OF SERVICES
1.1. General hospital → can provide services to any type of patients or illnesses
1.2. Special hospital → specific services e.g. →
o National Orthopedics Hospital of the Philippines (car incidents, injuries, bone disorder), National
Center for Mental Health, San Lazaro Hospital → special type according to disease
o Lungs Center of the Philippines, Philippine Heart Center, National Kidney Transplant Institute →
specific organs
o Philippine Children Center, National Children Hospital, Jose Fabella Hospital (maternal patient)
→ specific patients

2. LENGTH OF STAY:
o Short-term - < 30 days → for acute conditions or emergency cases
o Long-term - ≥ 30 days → long-term usually with patients with chronic disease
o Custodial → for patients with long-term ailments
3. OWNERSHIP
3.1 Governmental
o Federal hospital → armed forces, veterans, public health services; owned and operated by
members of the federal government
o State hospital → owned by the state but still regulated by the boards of control or division of the
state government
o County hospital → owned by the country; finance and controlled similarly to state hospitals;
usually for indigents
o City hospital → control by city government; also for indigents
o City-Country hospital
o District hospital
3.2 Non-governmental
o Non-profit oriented → financially supported by patients and contributors from religious
organizations or churches, community, fraternity, company, labor union
o Profit oriented → usually private and proprietary hospitals

BED CAPACITY → LESS THAN 100: very small hospitals, 100-500: small, 501-1,000: medium size,
more than 1,000: large hospital. Usual bed size: 36’’ by 80’’
• Under 50 beds
• 50-99 beds
• 100-199 beds
• 200-299 beds
• 300-399 beds
• 400-499 beds
• 500 beds & over

4 FUNDAMENTAL FUNCTIONS OF THE HOSPITALS


1. Patient care → THOUGH USUALLY HOSPITALS ARE CLASSIFIED TO TYPES OF
ACCOMODATION: on how the patient have the ability to pay or not → the full paid or the
private patient, partially or totally medical indigent patient or charity patient
2. Teaching or education → whether the hospital have university
3. Research → for the advancement of medical knowledge against the disease, and improvement of
hospital services
4. Public health → assist the community in reducing the incidence of sickness, increase the general
health of the population

STANDARDS OF PRACTICE
• 1918 - Accreditation of hospitals began
o The American College of Surgeons initiated its hospital standardization programme
• 1951- JCAH assumed the hospital standardization programme of The American College of Surgeon
• 1988- JCAH transitioned to a broader scope of accreditation and changed its name to JCAHO
• JCAHO → Establishes standards and provides accreditation services for other components of
healthcare delivery including home care, ambulatory care, behavioral healthcare organizations &
hospitals.
o Independent, voluntary agency and its actions are not subject to ratification by the organizations
represented by its component members
o The objective is to improve health care for the public, by evaluating health care organizations and
inspiring them to excel in providing safe and effective care of the highest quality and value

NOTES: ONE OF THE FACTORS THAT AFFECT THE STANDARDS OF PRACTICE OF


HOSPITAL IS THE ACCREDITING BODIES → JCAHO (The Joint Commission on Accreditation of
Healthcare Organizations) → ACS on 1918 → JCAH (The Joint Commission on Accreditation of
Hospital) on 1951 → 1988 JCAHO

*JCAHO → the net effect of this accreditation and program by JCAHO → enable the public to
disseminate between hospital services with those that are accredited and are not. The evaluation done by
this accreditation bodies will → help the public to define which hospital they want to be admitted
ORGANIZATION AND ADMINISTRATION

1. BOARD OF TRUSTEES → have the total accountability for the organization’s structure of the
hospital. Also referred as Board of Directors, Board of Regents; accountable for providing the
direction to the organization and oversight of the organization. Ultimately responsible for
governing the hospital
o INTERNAL ORGANIZATION:
▪ Board of Trustees
▪ CEO/President/Chairman → to lead the organization and make recommendations for the
boards. Depending on the type of hospital: if federal hospital there is a local hospitals that
organize the report for federal; if state, country, & city → same with federal which have the
governing boards or broads of trustees. But if non-profit or non-governmental → also have
broad of trustees. But the duties to implement the policies and strategic plan of the board
would be the responsibility of the CEO. CEO/President/Chairman → secretary of the
board; serve as the 2-way channel of communication between the boards and the medical
staff personnel. He will receive the guidelines, protocols & policies created by the boards to
have it relayed to the hospital staffs and personnel. The requests and recommendations of
the hospital staffs and personnel, the CEO is the one to speak with Boards regarding it.
▪ Vice President
▪ Secretary
▪ Treasurer
▪ Lawyer/Legal Counsel

2. DEPARTMENTS
• CLASSIFICATION:
o Services involve primarily in the professional care of the patient.
▪ Blood bank
▪ Central sterile supply
▪ Clinical laboratory
▪ Dental service
▪ Dietary and Nutrition service, etc.
▪ Nursing service, Dental service, Occupational therapy service, Pharmacy service → any
department involved in professional care
o Services which involve primarily in the business management or administrative work.
▪ Accounting
▪ Admitting
▪ Business office
▪ Credit and Collections
▪ Computer services → Computer service or Information system department
▪ Engineering and Maintenance
▪ Personnel and payroll
o Clinical division
▪ Department of medicine → Internal medicine, infectious diseases, dermatology,
endocrinology, geriatrics, pediatrics, psychiatry, pulmonary, rheumatology
▪ Department of surgery → General surgery, ophthalmology, OB-gyne, dental, orthopedic,
plastic surgery

• MEDICAL STAFF DUTIES:

1. Providing professional care


2. Maintaining its own efficiency
3. Self-government
4. Participating in the educational program of the hospital
5. Auditing its own professional work
6. Advising and assisting the administrator and governing board regarding medical policies

• TWO TYPES OF HOSPITAL STAFFS:


1. Open staff → One in which certain physicians, other than those on the attending or active medical
staff, are allowed to use the private room facilities, providing they comply with all rules & regulations
of the institution.
o These physicians are termed members of the courtesy medical staff
o not residents not solely employees of the hospital but given permission to use the facilities as
long as they conform with rules and regulations of the institution

2. Closed staff → One in which all professional services, private & charity are provided and
controlled by the attending or active medical staff.
o have major drawbacks but it is more desirable in an average hospital e.g. teaching hospital
(because they can select their own specialist that have the excellent reputation for the
institution)

• CATEGORIES OF MEDICAL STAFF:


o Honorary → retired former members/staff; emeritus → listed in the directory or honored
because of their outstanding contribution in the institution
o Consult → specialist recognized by passing specialty boards or belonging to a rational
organization under specialty
o Attending → most active involved in the hospital; primarily concerned with the patient care
o Associate → juniors or the least experienced member of the staff → eventually when earned
experience or cases → promoted to attending or active staff
o Courtesy → physicians who desired the privilege of attending private patients but have no
desired to become active staff members
o Resident → full-time employees and they provide specific services

HOSPITAL PHARMACY DEPARTMENT


The hospital shall maintained a pharmaceutical service that conforms with ethical and professional
standards and legal requirements → The development of hospital pharmacy department in many
countries was widely affected by educational standards and the caliber of different practitioners. Hospital
pharmacist as important professional specialty was virtually neglected in America for almost 169 years.
From the time that Jonathan Roberts became the 1st hospital pharmacist on Pennsylvania hospital in 1752,
Charles Rise of Bellevue hospital of New York City, Martin Wilbert of the German Hospital in
Philadelphia; The organization of the pharmacy must satisfy the needs for pharmacist performing their
role as a vital link in chain of health professionals dedicated to patient care. The pharmacist’s role is at all
time to provide an adequate supply of safe, effective, and good quality drugs in appropriate DF

• The Pharmacy – Its Organization and Personnel → the pharmacist is headed by the Director of
Pharmacy or the chief pharmacist (who is responsible in 1. reporting to the administrator of the
hospital the proper operation management of the pharmacy; 2. implements or formulates the
departmental administrative & professional policies of the pharmacy as per the approval of the AOH;
the professional & clinical policies relating the hospital pharmacy practice that have the direct
relationship with the medical staff or formulated and develop to the PTC or subject by approval
o Hospital pharmacy should be properly organized, meeting the minimum requirements prescribed
by the FDA, BLR, DOH, and HOMS
▪ FDA – Food and Drug Administration
▪ BLR – Bureau of Licensing and Regulations
▪ HOMS – Hospital Operations and Management Service

Staffing pattern → As the capacity of the hospital increases = demands in terms of staff required for the
pharmacy also increases

FUNCTIONS OF THE DEPARTMENT:

• Dispensing
o In-patient services → provide medication only to patients of the hospital on 24 hours per day
basis, inspect & control drugs in all treatment areas and operate with medical health research
o Out-patient services → compound and dispensing outpatient prescriptions also inspect and
control all clinic & emergency medication stations and maintain prescription records
• Compounding → 1. compound wide variety of items which are commonly used in the hospital; 2.
operate an overall drug packaging or compounding and precompounding or prepackaging programs;
3. undertake the programs in drug development and maintain unit dose program
• Purchasing and Inventory Control → maintain drug inventory control, purchase all drugs that are
in the formulary, receive and store drugs and interview medical service representatives
• Education, & Training → coordinate programs of the undergraduate and graduate pharmacy
students and participate in wide hospital education programs involving other healthcare professionals
such as the nurses and doctors; 2. Train newly employed department personnel; and Research →
develop new formulation of drugs esp. DF that are not commercially available, improve formulations
of existing products and cooperate with the medical research staff on projects involving drugs
• Records and Reports
• Quality Assurance → quality assay and control → perform analysis on products manufactured and
purchased and developed, develop assay procedures, and assist research division on special
formulations
• Drug Information → provide drug info, and drug therapy not only to the patient but even to other
healthcare professionals such as the physician, nurses and intern students, maintain the drug info
center, and prepare the hospital pharmacy newsletter, and maintain literature files
• Drug consultation to staffs and medical students
GENERAL QUALIFICATIONS (HOSPITAL PHARMACIST)
-Abreast with the updated list of the dangerous drugs, regularly informed with any matters relating to
drugs, uses, dose and other drug-related matters, communicate with other health professionals,
enthusiastic, resourceful, and alert in order to be dependable, available, and progressive. Dynamic,
conscientious, industrious, and helpful.

STORAGE
Proper storage is necessary to prevent the drugs from deterioration, and maintain its efficacy. The storage
must also protect the patients from potential hazards because the desired therapeutic response will not be
achieved and an ADR may be produce by toxic decomposed product

PTC (Pharmacy and Therapeutic Committees) → serves as advisory group of the medical staffs; which
serve as a line of communication between the medical staff and the pharmacy department

PRIMARY PURPOSES: 1. Advisory, 2. Education, 3. Policy development (would also be a necessary


role of the PTC regards the policies, regulations, and selection of therapeutic use of drugs and devices)
Chief pharmacist → secretary: will prepare the minutes of the meeting and maintains the permanent
records of the hospital
The committee should meet regularly at least 6 times per year and necessary especially for larger
hospitals → at least 12 times a year
FORMULARY SYSTEM → there will be a proposal for the inclusion of drug in the hospital formulary
once the formulary drug request form is filled or made available. Hospital NON-FORMULARY drugs
proposed for inclusion should also have the approval of the DOH; The formulary system’s purpose: 1.
Educate the physicians, 2. Teaching aid to the interns and medical staffs, 3. Promote safe, intelligent and
effective therapy in the hospital.

PART III: Special Information → based on age (Young’s rule, Cowling rule, Fried’s rule) and weight
(Clark’s rule), list of dialyzable poisons: water soluble, small volume of distribution, less than 0.5 L per
kg, low MW, less than 500 Dalton, and does not significantly binds to plasma proteins

DISPENSING → the procedure whereby the drugs are prescribed by the physician and serve upon proper
request whether made by the ward or out-patient department

MOBILE DISPENSING UNIT → 60 inches in height and 48 inches wide and 25 inches deep

IN-PATIENT CARE AREAS → patients who required hospitalization and get themselves admitted to a
hospital as needed for treatment until they are ready for discharge

OUT-PATIENT → 1. Emergency – involves diagnosing and treating life threatening illness and injuries
that needs immediate attention and may take place in ambulances or transportation vehicles, ER and ICU
(e.g. for emergency cases → chest pain, difficulty in breathing, heart attacks, serious injury, bleeding that
would not stop, and mental crisis), 2. Tertiary care → highly specialized medical care over an extended
period of time that involves advance and complex procedures and treatments performed by medical
specialists in state of art facilities (e.g. coronary artery bypass surgery, renal or hemodialysis, plastic
surgeries, neurosurgeries, severe burn treatments, and other very complex treatment procedures), 3.
Primary care → first place the patients go for medical care; patients may get primary care on doctor’s
office or community health center; focus of primary care: prevent disease through regular physical exams,
health screen, under focus of the patient general health by diagnosing and treating wide variety of
conditions. Any problem that requires special knowledge and skills → the primary doctor may refer the
patient to a specialist. The primary doctor follows the patient’s care while they are seeing a specialist.

TYPES OF OUT-PATIENT:
1. General → patients other than emergencies or who report directly to the out-patient department
2. Special
3. Referred → patient referred on an OPD his physician for specific diagnostic and treatment or
procedure
4. Emergency → the person given emergency care

FLOOR STOCK SYSTEM → under this system the supply of each drug stored in the nursing station in
advance and the nurse is responsible in all aspects of unit dose preparations as well as the administration.
o FREE → all medications available in the nursing station for the use of all patients in the ward and
not charge to the patient. Uses drug basket method and mobile dispensary
o CHARGE → the medications in the nursing station at all times and the patient is charge to every
dose administered to him. Drugs that will include in this system are the ones decided by the PTC.

Pilferage or stealing

INDIVIDUAL PRESCRIPTION ORDER SYSTEM → the physicians write a prescription for individual
patient → drugs are retained in the pharmacy in accordance with the physician’s initial order. This system
facilitates a convenient method for instituting patient drug charges and provides individualized patient’s
service.

UDDS → sufficient only for single dose administration and was first introduced in hospitals in 1960 to
prevent medication errors and reduce medication waste
o CENTRALIZED → the dose will only be dispense if it is due to be given to the patient. Utilizes
medication cart fill
o Decentralized → uses automated dispensing cabinets or satellite services

PART 2
• Clinical pharmacists are working inside the hospital → some of the hospitals required their staff
pharmacist to become a clinical pharmacist

• INVENTORY MANAGEMENT → as long as you have the products that are offered this is important
to check the drugs stored inside the facilities like in the hospital or even the community pharmacy →
essential to monitor esp. the regulated and prohibited dangerous drugs so it can be check properly. If
we don’t have this system we won’t know where are the products or if the products exited the
facilities matched with the amount and budget we budgeted for them → in hospital: have monthly or
annual (depends on the institution)
• INVENTORY → required most the effort and laborious part of community pharmacy → hence,
arranged depending on the pharmacy such as based on alphabetical, & therapeutic use arrangement.
There are assigned pharmacist or pharmacists assistance for faster transaction of the inventory

• Responsibilities of Pharmacist in Inventory Management:


1. Checking the drugs to be stocked in the hospital
a. Responsible and accountable for all drugs purchased. → esp. for prescription drugs
2. Developing an effective system in the control of purchase, inventory and adequate maintenance
of raw materials used for compounding and pharmaceuticals. → e.g. in the community pharmacy:
have the computerized system for inventory for easier and efficient workloads while others →
hand-to-hand

• Basic characteristics for the budget (BUDGET → formal quantitative expression of the plans
intended for the management) to be useful and effective: 1. SHORT RANGE PLAN for future
corporation, expressed in terms of money or units of work to be performed and even product activity
• BUDGET must begin within the hospital short-term and long-term plans and goals → budget will
depend on the institution
• BUDGET should not be release for nothing it should be allocated → Plan the necessities to be
organized esp. when it comes to money
• Since budget is based in the short & long-term goals → ADDRESS THE GOALS AND
OBJECTIVES CLEARLY → will help the institution to be successful. Should be monitored and
controlled properly in order to not be wasted.
1. Oriented toward achieving goals and objectives →
2. Realistic
3. Implemented by the one who prepared it
4. Contain certain internal mechanisms for review and analysis
5. Use consistent measurement tools and reporting periods

Inventory Management → Sum total of those activities or necessary for acquisition, storage, sale, disposal
or use of materials; inventory is the itemized list of the goods with their estimated work

• Functions of inventory management


1. Organizational → pertains to the management and the storage of the drugs which are actually
intended for the use of the hospital.
2. Financial → through inventory the owner of the establishment can determine the required budget
for the next acquisition

• The general objective of all inventory policies: ....must be to keep investment at the lowest level with
the needs of the hospital → to not pay for products which are unnecessary (fast & slow moving drugs;
fast moving drugs → always being purchased and dispensed e.g. emergency drugs)

• In requisitioning the drug needs, a pharmacist must think in terms of:


o the information on the balance of the revolving fund → revolving fund/cash is a specific amount of
money that is used to purchase items; when the money is expended it is constantly replaced →
hence, revolving
o the conditions such as seasonal, etc
o transportation and delivery time problems
o availability of alternate drugs
o the ratio of supply and demand → since it is business economics is considered → DRUGS ARE
ALWAYS IN DEMAND (sanaols)

• The problem of overstocking and understocking of drugs can be attributed to the following:
1. Ineffective purchasing administration without any control system.
2. Lack of technical requirements attached to documents.
3. Poor storage facilities.
4. Problem of availability.
5. Transportation and delivery problems, etc.
6. Geographical and climatic conditions time lag between requisition and purchase.

• Objectives and requirements of inventory management:


o To have the stock available when and where needed.
o To reduce undue strain on the government’s scarce resources. → consider scarcity
o To maximize utilization of available storage facilities.
o To provide area for cost reduction on drugs without compounding quality.
o To ensure minimal waste/expired drugs through proper storage management.

• Types of purchasing
1. Purchase thru public bidding
2. Emergency purchase
3. Negotiated purchase
4. Procurement from duly licensed manufacturers and exclusive distributors.
5. Procurement thru the procurement service.
6. Procurement from other Philippine General agencies or foreign government.
7. Purchase thru repeat orders.

• INVENTORY CONTROL TURN-OVER RATE (Turn-over rate → Rate at which the goods and the
products are sold and replace by new goods):
• Annual purchase/Annual inventory → if the turn-over rate decrease there are duplication of stocks,
large purchases of slow-moving products, & dead inventory; on the other hand, if the turn-over rate is
high it causes small volume purchasing

COMPOUNDING OF NONSTERILE PREPARATION AND PREPACKING

• Non–Sterile Compounding: History


o Non-sterile compounding are products that do not need aseptic technique and equipment.
Pharmacists are the only healthcare provider that is formally trained in the art and science of
compounding medications. Therefore, pharmacists are expected by the medical community and the
public to possess knowledge and skills that are necessary to compound extemporaneous
preparations (compounding the unavailable commercialized products or the personalized DF; if
not available → compound/prepare on the spot). The pharmacist have a responsibility to provide
compounding services for our patients
o Compounding → the practice wherein the pharmacist puts together a prescription formulation with
use of proper ingredients to produce a preparation (final product of compounding →
preparation) and not available for resale.
o Medicinal mixtures using plants, animals, and minerals dates back 4000 years → apothecary
started with plants, animals, and minerals
o In 1820, 80% of prescriptions in the first U.S.
o Pharmacopoeia were compounds
o Premade dosages do not necessarily treat everyone, are not always commercially available → e.g.
Pediatric doses, hospice patients (in other countries compounding role of the pharmacist is
common or highly in demand aside from dispensing medications esp. in hospital settings→ if the
technician pharmacist is the one that going to compound the meds supervision from the pharmacist
is needed)

• Use of Non–Sterile Compounding


• Non–sterile compounding → are medications that are made in a clean environment but not
necessarily free from all organisms not like sterile compound where the particulate matters are
being minimized
• Used for drugs that are taken orally or topically (not like parenterals → needed to be prepared
through sterile compounding → sterile compounded medications are intended for to be used as
injections, infusions and ophthalmic preparations)
• Non–sterile compounding: Done in vertical hood or on clean work surface as stated in USP
<795>
• Common items: Creams, ointments, oral suspensions
• Less common: Capsules, suppositories, syringe

• Reasons for Compounding


o No longer manufactured → not available commercially
o Patients may be allergic to something in a drug → the allergy causing ingredient will be replace by
another ingredient that the patient is not allergic to
o Specialized dosage/strength for specific patients → e.g. pediatrics
o Increased patient compliance
o Patient unable to ingest normal dosage form
o Medication requires flavorings

• Equipment Used in Compounding


o Personal protective equipment: Gloves, goggles, gown, hair cover, lab coat, mask, shoe covers
(proper gowning or PPE; not just any outfit just because it is a non-sterile compounding
procedure)
o Measuring devices: Graduated cylinders, syringes, pipettes, electronic filling machines → small
scale
▪ Measuring liquids requires reading a meniscus
o Mixing equipment: Mortar and pestle (glass mortar → solutions and suspensions or preparation
that are mixed together and stains or oily; Wedgwood mortar → ideal for PS reduction esp. in
crystals; Porcelain mortar → have smooth surface compared to Wedgwood mortar and is ideal for
mixing powders and pulverizing soft aggregates of crystals)
o Weighing equipment: Scales, electronic balances
• Preparing Solutions
o Solutions comprise: Solvent (larger part) and Solute (ingredient used in solvent)
o Measure carefully and mix thoroughly → volume should be properly measured
o Solubility will dictate the type of dosage form that needs to be prepared
o Reconstitution of premade oral suspensions may be done away from the compounding area →
common in the community pharmacy e.g. graduated cylinder and distilled water that are prepared
for reconstitution → PA’s role

• We have different factors to consider for selecting proper DF → 1. Physical and chemical
characteristics of the active ingredient; 2. The possible route of administration that will produce the
desired therapeutic effect e.g. oral, parenteral or topical; 3. Patient’s characteristics → age level of
consciousness, ability to swallow a DF like solid DF; 4. Specific characteristics of the patients → e.g.
disease; 5. Comfort of the patient; 6. Ease and convenience of administration

• Solids: Tablets, Capsules, and Lozenges


o Molds are used for forming these types of oral dosage forms
o Molded tablets disintegrate quickly when they come into contact with moisture
o Tablets or lozenges can be made one at a time or in multiple doses
o Lozenges: Normally made with flavors to enhance their taste and sugar
▪ Hard → troches
▪ Soft → pastilles
▪ Chewable

• Semisolids: Ointments, Sticks and Suppositories
o Medication sticks → easily transportable and convenient DF for administering topical
medications; may be compounded in different sizes and shapes for application of the different
parts of the body
▪ Applied directly to a site on the body that needs treatment
o Ointment: Hydrophobic base
▪ Petroleum jelly mixed with drug
▪ Jars or tubes
o Hard sticks and soft sticks
o Suppositories: Oleaginous bases, water-soluble bases, glycerinated gelatins

• Packaging
o Containers must:
▪ Be appropriate size → Must find a proper container for the drug in a sense that it would not
chemically or physically affect the contents e.g. to minimize the administration error → oral
liquids should not be packaged in syringes that are intended for injection
▪ Protect contents → Containers → must maintain the drug’s strength, quality and purity of the
compounded products. For glass → most common containers are the amber glasses (purpose:
to protect the contents of the container from light esp. the products that are light sensitive to
prevent photooxidation); also have blue, green and clear bottles (glasses that will transmit UV
light rays → green, blue and clear or flint glasses; should not be used in light sensitive drugs )
▪ Have childproof caps (not for jars and syringes)
▪ Have appropriate labels → label properly by including the generic and brand name with the
corresponding strength and quantity, lot number and batch number
▪ Common auxiliary labels placed on medication containers:
▪ Suspensions: Shake well
▪ Ophthalmic: For the eye

• Storage and Stability of Compounded Drugs → STABILITY, according to USPNF, is the extent to
which the DF will retain within the specified limits throughout its period the same properties and
characteristics that is possessed at the time of its preparation
o Consider stability of any additives
o Factors affecting stability:
▪ Amount of light and air
▪ Temperature
▪ pH alters longevity
▪ Particle size
▪ Property of water
o Solid forms have longer shelf life than liquid forms
▪ It is easier for a liquid product to degrade or for its components to separate
o All compounded products should be observed for signs of instabilities → QUALITY CONTROL

• Documentation → collaboration of FDA


o Documentation of records under quality assurance of FDA guidelines
o Compounding record (CR): Log
o Formulation record (FR): Recipe
o Each step of the compounding process should be documented. The pharmacist should maintain at
least 4 sets of records in the compounding area: 1. Compounding formula and procedure, 2. Log
of all the compounded items including the batch records and their sample batch labels, 3.
Equipment maintenance record → including the documentation of the checks of balances,
calibration of the equipment, 4. Records of ingredients that were purchased → include the
certificate of the chemicals

• Documentation:
1. Date prepared; name of ingredients; manufacturer of each ingredient; lot number and expiration
date of each ingredient [including sterile water]; amount or weight of each ingredient; dosage
form of each ingredient; pharmacy lot number assigned; pharmacy expiration date assigned; date
dispensed; patient’s name and medical record number
2. In addition, a step-by-step recipe is required, and both the pharmacist and the technician must
initial the records

• Documentation: Safety Data Sheets (SDSs)


o All chemicals should be stored inside cabinets or behind shelf brackets to avoid spillage
o Method of cleaning and disposing of agents or any equipment used depends on the type of agents
used
o SDSs contain emergency contact information in case of spillage or contact.
• Professionalism and Quality Control
o Appearance is important
o Great care must be taken when topping off jars of creams and ointments.
o Packaging affects medication inside and compliance
o Many medications can degrade with ultraviolet (UV) light exposure; therefore, they must be
placed in amber-colored containers to protect the medication.
o Follow all storage and labeling guidelines

COMPOUNDING OF STERILE PREPARATION AND PREPACKING


• STERILE COMPOUNDING → differ from non-sterile compounding: have many tests or
assessments for the sterility of the environment and the equipment used; have clean rooms and the
authorized persons should be on full gear to reduce particulate matter in air

• PARENTERAL NUTRITION via the IV administration → would include proteins, carbohydrates,


fats, minerals, electrolytes and vitamins given to comatosed patients and patients that cannot eat or
absorb food in order for them to maintain a good nutrition status
o Intravenous hyperalimentation
o Feeding of a patient by the intravenous infusion of fluids and basic nutrients
▪ Central Parenteral Nutrition: often called Total Parenteral Nutrition (TPN); delivered into a
central vein
▪ Peripheral Parenteral Nutrition (PPN): delivered into smaller or peripheral vein

• Total Parenteral Nutrition → for patient that are comatosed
o IV administration of nutrients precisely formulated in such a manner that the totality of the
patient’s nutritional needs were satisfied.
o IV administration of basic nutrients that are primarily for the purpose of promoting tissue
synthesis and growth
o Hypertonic solution → the needed nutrients of the body are provided in an amount of fluid that
does not exceed daily fluid capacity of the body; contains concentration of >300mOsm/L of
dextrose and amino acids
o Indicated for patients with alimentary tract obstruction, major burns, and massive trauma →
impossible; improbable → patients given TPN because of their anorexia nervosa and geriatric
patient; inadvisable → patients with inflammatory disease or malabsorption symptoms
o 1960, University of Pennsylvania – Dr. Stanley J. Dudrick, Dr. Jonathan E. Rhoads, Dr. Douglas
Wilmire
o Requires a catheter or port that empties into the superior or inferior vena cava → direct to blood
stream
o To successfully accomplish TPN all the required nutrients must be provided like CHON, CHO,
fats, electrolytes, vitamins, and trace elements
o Total Nutrient Admixture or 3-in-1 or All-in-One Parenteral Nutrition
▪ if lipid is added directly to the solution
▪ Pre mixed: Vitrimix, Kabiven, Nutripack

• Peripheral Parenteral Nutrition


o Nutritional support that supplements oral intake and provide only part of the daily nutritional
requirements. → usually given as a supplement to enteral feeding (enteral feeding → refers to the
intake of food via the GIT → GIT composed of: mouth, esophagus, and intestine; nutrition taken
through the mouth or the tube that goes directly to the stomach and intestine)
o Patient needs IV nutritional support but does not have a central line
o Therapy is expected to be short term (10-14 days)
o Energy and protein needs are moderate
o Formulation osmolarity is <600-900 mOsm/L
o Fluid restriction is not necessary

TPN → the only nutrition that the patient is PPN → meant to act as a supplement and used
receiving when the patient has other source of nutrition;
administered in smaller veins and the solution is
lower in nutrients and calorie content than TPN

• Goal of Therapy of TPN → To achieve adequate nutritional status and positive nitrogen balance until
exercise and the utilization of nutrients via GIT is possible. Other goals: maintain the weight,
metabolic integrity of the patient, and replete or supply malnourished patients which deficits in lean
body mass accompanied by deficits in visceral and serum proteins, restore hematologic and immune
function integrity

• Indication
o Oral feeding is not feasible → give IV preparations esp. to those unable to assimilate nutrients via
GIT; should only be used when the gut is not available
o Oral feeding is not enough
o Oral feeding is not recommended
o Oral feeding is dangerous

• PN is no value or little value:


o Minimal stress, GIT will not be available w/in 5 days
o Patient is taking oral diet
o Good nutritional support
o Risk exceeds benefit
o End stage, terminally ill

IV ADMIXTURE → when one or more sterile products are added to an IV fluid for administration; it is
prepared with aseptic technique or environment provided by laminar flowhood, in which the air is filtered
through HEPA (high efficiency particulate air) filter (collecting dust in the environment); full gear PPE
and have proper techniques required to follow and obey → hence, people performing IV admixture
preparations are highly trained professionals or pharmacist to minimize possible contamination

• Aseptic Technique → used in sterile compounding; when we say “aseptic” → free from
contamination
o Aseptic Preparation
▪ Involves procedures designed to preclude contamination by microorganisms during processing.
▪ Ability of the personnel to handle the sterile components of these IV solutions in the clean
environment of a Laminar flow hood without introducing viable microorganisms in the product.

• Aseptic preparation
o Thorough hand-washing with appropriate antimicrobial.
o Wearing of protective clothing, mask, head cover, shoes and gloves. (From head to toe; Gowning
area: when it comes to woman → tie hair before head cover, no make-up, no jewelries; different
gear in non-sterile and sterile compounding room)
o Disinfection of non-sterile materials to be used.
o In the clean room: minimize movements, no talking → e.g. sick people → continues talking so
they might contaminate the aseptic preparation, NO TOUCHING RULE: do not touch your hair,
gown, or other people

• Sources of product contamination


o People (most common)
▪ Touch contamination
▪ Generation of particulates from shedding cells or hair (so beard for men is not advisable →
have
o Air supply
▪ Heating Ventilation and Air Conditioning (HVAC)
o Infiltration
▪ Particles from adjacent spaces (e.g. anteroom); anteroom – referring to a small that is entered
before a larger room
o Internal generation
▪ Walls, floors, ceilings, packaging, equipment

• Importance of Aseptic Technique


o Parenteral administration bypasses the skin and gastrointestinal tract, the bodies natural barriers to
infection
o Giving a patient a contaminated product can cause serious adverse effects including DEATH →
direct BV or blood stream → unlike solid DF that will go through the esophagus, stomach, and
intestione etc., hence, should be monitored properly the no. of microorganisms
o Parenteral medications account for >40% of all medications administered in institutional practice

• Laminar Flow Workbenches


o LAMINAR FLOWHOOD → designed to reduce the airborne contamination during the
preparation of IV solution or IV admixtures
o PARTS of LAMINAR FLOWHOOD: blower, prefilter (to remove the gross contaminants e.g.
dirt, dust, lint from the clothing), HEPA filter (absorb dust etc.), plenum, workbenches (where all
the equipment and materials for sterile compounding is put under)
o Principle:
▪ Constant flow of HEPA filtered air
▪ Continuously sweeps the work area
▪ Prevents the entry of contaminated air
o HEPA filter - High Efficiency Particulate Air filter removes 99.97% of all air particles 0.3mm or
larger; vital of any air units so every after compounding → should be cleaned; the space between
the HEPA filters and the products being prepared is referred to us as the CRITICAL WORK
SURFACE

• HEPA filters remove 99.97% of all particles larger than 0.3 um


o the flow of air may be in either a horizontal or vertical pattern
o the best way to determine the proper functioning of a HEPA filter is to use the dioctylphthalate
(DOP) test using the vapor at room temperature → if DOP not available: poly-alpha olefin or PAO
→ a non-carcinogenic liquid and most common replacement for DOP
o usually made up of fiber glass that are randomly arranged

• Laminar Flow Workbenches


o All aseptic manipulations should be performed at least 6 inches within the hood. → not to close to
you should find the best spot that have no air contaminants
o Must run for 15 -30 minutes if turned off & back on
o All interior working surfaces should be cleaned → if going to withdraw from the vials wipe the
rubber portion of vial first to avoid contamination and to maintain sterility
▪ 70% isopropyl alcohol/other disinfecting agent or gauze pad with alcohol
▪ Clean, lint-free cloth
o Laminar flow hoods should be clean from back to front every compounding process. Before and
after compounding → cleaning

• Aseptic preparation
o Demands meticulous “NO – TOUCH” technique,
o EVERY TIME!
o Aseptic technique requires specific manipulations for:
▪ Syringes
▪ Needles
▪ Vials
▪ Ampules
▪ Removal of packaging
▪ Assembling of sterile products
▪ Hand placement

• Aseptic preparation: 1. Syringes


o – never touch the luer-lok tip or the plunger
o tip – provide the point of attachment with the needles,
o barrel – storage cylinder the liquid to be dispense/ hold the medication in place → pressure in the
seal reduces the volume of the barrel = releases contents
o plunge or the top collar – area for grasping during the injection procedure, plunger – pistol type
rod with a slightly cone shaped top → inside the plunger have latex free stopper which prevents
the leakage of medications around the plunger
• Aseptic preparation: 2. Vials and Ampules
o To prevent contamination
▪ Swab rubber closure with 70% alcohol using firm strokes in the SAME direction
o To prevent core formation
▪ Insert needle to penetrate the rubber closure at same point with both tip and heel of bevel
o To prevent vacuum formation
▪ Inject an equal amount of air for the volume of fluid to be removed
o Reconstituting drug powder
▪ Remove an equal amount of air for the volume of diluent added
o To break ampule
▪ Clean ampule neck with alcohol swab
▪ Leave swab in place
▪ Grasp ampule neck with thumb and index finger
▪ Use quick, firm, snapping motion away from body towards side wall of hood
▪ DO NOT BREAK TOWARD HEPA FILTER

Clean where the area/point by alcohol where the drug


contents will be expose

• Aseptic preparation: Removing Packaging


o Any sterile component or supply (e.g., syringes, needles, and ampules) should only be opened
and/or removed from their packaging within the laminar-flow workspace.
o When opening the wrapper on a needle, it should be peeled open.
o Tearing paper introduces paper particles into the hood which could lead to product contamination.

EMERGENCY MEDICINE PHARMACISTS – existing for already 30 years


• A large number of healthcare systems have either dedicated EMP or clinical RPh in the emergency
department or the ED is near or close in the pharmacy

• Emergency Medicine Pharmacy Practices:


o Pharmacists have played a key role in ED since 1970s
o Initial services focused on:
▪ inventory control
▪ development of 24-hour pharmacy satellites esp. in larger hospitals
o Clinical pharmacy services have evolved:
▪ Identification of drug-related problems e.g. ADRs, DI, incompatibilities
▪ ADR surveillance
▪ Pharmacokinetic and toxicology consultation
▪ On-call pharmacy services
▪ Provision of CPR or cardiopulmonary resuscitation

• Patient Care Roles of EMPs


o Direct patient care rounds → role: prevent medication errors by actively participating in the
medication selection and prescribing process
o Medication order review → the pharmacist will discuss drug related matters to the patient; assess
if the patient is compliance and determine the problems faced by the patient or patient counsel;
incorporate triage system → to help the pharmacist prioritize the high risk medications or
emergency medications; TRIAGE SYSTEM – refers to the methods that are used to assess the
severity of the patient’s condition within a short period of time after their arrival e.g. emergency
case, non-prioritized patient, prioritized patient
o Medication therapy monitoring → after the administration of the medication of the nurse → the
pharmacist should evaluate the efficacy and safety of the medications taken by the patient based
on the goals of the therapy, allergies, ADRs
o Patient care involving high-risk medications & procedures
o Resuscitation → pharmacists should prepare medications for immediate administration, suggest
different route of administration, answer drug information question from the patient or allied
health, and recognized & treat patient with presenting a toxicologic emergency
o Medication procurement and preparation → RPh: liaison in the pharmacy and emergency
department regarding the revision of the processes that are associated with medication
procurement, active role in the drug procurement and preparation based on operational work
flows; EMPs should be an integral part of the medication procurement and preparation process
for the medications in emergency dept as well as the dispensing of medications in one of the five
stages of medication use process
o Medication information → COMMON CAUSE OF ME: lack of information related to
medication therapy
o Medication histories → first step of the medication reconciliation process → collection of best
possible medication histories (all medications taken by the patient even the herbal products,
OTC)
o Medication reconciliation → done in every phases of patient care – before admission (best
possible medication histories), upon admission (new sets of medications prescribed), during,
discharge (compare the medications taken in the hospital and medications upon discharge)→
comparing of the drugs every stage to avoid ADRs); a process of comparing the patients
medication orders to all of medications that the patient has been taking to avoid ME like
omission, duplication, dosing errors, DI

• Administrative Roles of EMPs


o Medication and patient safety → collaborate with other healthcare provider to identify possible
sources of error; perform RCA (root cause analysis) – to identify the potential error trends or
system failures that can contribute to the development of safe medication practices, and processes
for the prevention of future events
o Quality-improvement initiatives
o Leadership duties and professional service → encourage to take on leadership roles in
institutional committees thereby advancing the roles of pharmacists in care
o Emergency preparedness → e.g. with the expertise in pharmacology and toxicology EMPs are
well suited to respond to emergency situations like natural disaster, disease outbreak, biological
and radiological or chemical exposures
o Education → play active roles in education of the pharmacy staffs e.g. pharmacy students and
student residents
o Research and scholarly activity → involvement in researches that could later on improve
processes of pharmacy department

• Central Role of the EMPs → must evaluate the appropriateness of medication therapy quickly and
accurately to optimize the patient care
o Improving patient outcomes
o Providing optimized pharmacotherapy regimens and therapeutic outcomes

CRASH CART AND ITS CONTENT

• Crash Cart/Emergency cart/Emergency response cart/Code cart/Emergency trolley


o A trolley for storing lifesaving drugs and equipment in a hospital ER, ICU, clinics and other
areas.
o Characterized by being easily movable and readily accessible
o
• Crash Cart Contents
o This vary from hospital to hospital. These include but are not limited to:
▪ Monitor/defibrillators & suction devices
▪ Advanced Cardiac Life Support (ACLS) drugs such as:
1. Epinephrine (injection used to treat life-threatening allergic reactions or anaphylaxis
caused by animal bites, food, medications, latex, hives; beta-alpha agonist or
sympathomimetic agent),
2. Atropine (prescription medicine used to treat the symptoms of low heart rate or
bradycardia, can reduce the salivation, and bronchial secretion for surgery, antidote to
cholinergic drugs, and mushroom poisoning),
3. Amiodarone (treat to treat and prevent irregular heartbeat because it can slow down the
nerve activity in the heart and relaxes an overactive heart),
4. Lidocaine (local anesthetic – have numbing action once administered; works by blocking
the working signals in our body and mostly in topical preparations that is used to reduce the
pain and discomfort; localized anesthesia for sunburn, skin irritation, insect bite, poison ivy,
oak, & sumac, minor cuts, and scratches),
5. Sodium bicarbonate (antacid for patients with heart burn and indigestion),
6. Dopamine (treat symptoms of low blood pressure, low blood output, and improve BF of
the kidneys; can be used alone or together with other meds; inotropic agent),
7. Dobutamine (treat cariogenic shock and sever heart failure and can be used in cardiac
stress test; IV), and
8. Vasopressin (also known as antidiuretic hormone– a small peptide hormone which
regulates the body’s retention of water)
▪ First line drugs for treatment of common problems such as:
→ Adenosine (prescription only drug for heart conditions e.g. rapid heart rate, involuntary
weight loss, and people who are very ill like patients with weak blood circulation), Dextrose
(source of energy), Diazepam or Midazolam (treat anxiety, alcohol withdrawal and seizures
because it will relieve muscle spasms), Epinephrine, Naloxone (reverse or antidote for opioid
overdose), Nitroglycerin (vasodilator), Diphenhydramine (antihistamine), Furosemide (loop
diuretic) and others
▪ Drugs for rapid sequence intubation: Succinylcholine (skeletal muscle relaxant for IV
administration; indicated as an adjunct to general anesthesia) or other paralytic agents, a
sedative drugs such as Midazolam
▪ Endotracheal tubes and other intubating equipments
▪ Vascular access devices –IV Cannula, Drip sets
▪ IV fluids like Normal Saline(NS), Lactated Ringer(LR), Hydroxyethyl starch
▪ Other drugs and equipment as chosen by the facility.

• TYPES OF CRASH CARTS:


o Based on developmental variations:
▪ Adult E-cart
▪ Pediatric E-cart
▪ Newborn Intensive Care E-cart

• General Information about Crash Cart


o A licensed staff member as designated by department head is responsible for checking the crash
cart.
o Each emergency cart is equipped with a lock and kept locked unless in use.
o If the lock is not intact, the cart is to be checked and unit personnel will replace any missing
supplies.
o Crash cart is recorded and checked every shift.
o Defibrillator load or calibration checks will be performed every shift. → Defibrillator: used to
restore patient’s heartbeat
o All carts will be opened and checked for contents once monthly and following each use.
o Laryngoscopes will be checked prior to placement on the cart and monthly.
o Oxygen cylinder is replaced when the tank has emptied.
o Drawers of crash carts are to be clearly labeled to identify contents.

• Hospital Code → Quick way to tell hospital workers who needs to attend to an emergency situation,
what they need to bring, and what they should expect. Used code names to alert their staffs in
emergencies or other situations communicated through an intercom or directly to staffs using their
own pager. This will also allow the hospital personnel to act quickly and appropriately to various
events
o Code Blue - the most universally recognized emergency code. This indicates a medical
emergency such as cardiac or respiratory arrest.
o Code Red - this typically means there is a fire or smoke within the hospital.
o Code Pink: if there is an infant or child abduction within the hospital (LabanLeni)
o Code Orange: if there is an external disaster
o Code Green: This indicates that the hospital is activating an emergency operations plan.
o Code Brown: if there is an hazardous spill
o Code Black: This indicates a bomb threat.
o Code tangerine: presence or fire; brown: presence of earthquake (FUMC in Valenzuela)

Hospital Practice Internship Program → internship: given to students in order to attain the needed skills
and experience once become RPh or professional
The internship program is designed to assist students in exploring career options through practical
training. The program provides students opportunity to acquire knowledge and develop their skills in the
hospital pharmacy. This is also a requirement for licensure examination. Through internship, students are
able to apply theories obtained from classroom lectures, to real challenges in the workplace.

• OBJECTIVES:
o Develop skills in communication and hospital pharmacy services, and
o technical know-how in manufacture of drugs;
o Exercise critical thinking in giving sound decision-making skills;
o Demonstrate competencies in the performance of tasks as required by establishments;
o Use a variety of communication technique in all health care settings;
o Undertake the process of scientific inquiry and utilize research findings in advancing the
pharmacy profession;
o Apply leadership, management, and collaborative skills within the health care delivery system in
developing, implementing, and evaluating the pharmaceutical care provided to patients;
o Demonstrate integration of ethics and legal issues in pharmacy practice, especially in applying
independent judgement and ethical decision making; and
o Integrate value, commitment and service to God in the practice of the profession.

CLINICAL PHARMACY

• A practice in which the pharmacist utilizes his professional judgment in the application of
pharmaceutical sciences to foster the safe and appropriate use of drugs, in or by patients, while
working with members of the health care team (Francke 1969)
• Current in the profession nowadays to advance the knowledge to practice → additional 1 year or
separate course offered by other schools e.g. UPM (5 years) – edge in clerkship
• GOAL: achieving pharmaceutical care or drug provision

• CLINICAL PHARMACIST:
o Interact with the health care team
o Interview and assess patients
o Make therapeutic recommendations
o Monitor patient response to drug therapy
o Provide drug information

• WHAT ARE REQUIRED IN CLINICAL PHARMACY:


o Knowledge: diseases, drug therapy, non-drug therapy, laboratory and diagnostic testing → patient
monitoring: reviewing the outcomes of drug therapy; if achieved desired outcome = successful
while if not what are the factors affecting it up for readjustment)
o Skills: Communication, patient monitoring, physical assessment, drug information provision,
therapeutic planning. COMMUNICATION THE MOST IMPORTANT → every now and then →
communicate to patient and healthcare professionals

• GENERAL CLINICAL PHARMACIST FUNCTIONS


o Providing drug information to physicians & other health professionals
o Medication history taking
o Medication profile preparation
o Drug therapy monitoring

• FUNCTIONS
o Patient education and medication
o Disease screening, monitoring, and maintenance care for patients with chronic diseases
o Counseling
o Participation in the management of emergency medical care
o Health information source for the public
o Drug use review and patient care audits
o In-service Education for Physicians, Nurses and Other health Professionals
o Specialized functions and services (ASHP 1983) → providers from the acute and ambulatory
settings

• Clinical pharmacists as COMMUNICATOR:


o Patient Counseling - provision of oral or written information about drugs & other health-related
information to a patient or his / her representative during the dispensing process or stay in
hospital (advocate for oneself).
o EXPLAIN IN LAYMAN TERMS → build rapport, avoid asking CLOSE ended question should
ask → OPEN ENDED QUESTIONS
o Scope of counseling:
▪ Generic name, trade name
▪ Techniques for self-monitoring
▪ Use, action & onset
▪ Potential drug interaction
▪ Route, dosage form & storage
▪ Contraindications
▪ Directions for use
▪ Relationship with lab or Xray procedure
▪ Action in case of missed dose
▪ Disposal of drugs & devices
▪ Precaution
▪ Any other relevant health information unique to an individual patient
▪ Side & adverse effects
o To achieve the desired therapeutic goals…
o The clinical pharmacist applies:
▪ Evidence-based therapeutic guidelines
▪ Evolving sciences
▪ Emerging technologies
▪ Relevant legal, ethical, social and cultural economic and professional principles

PRACTICE!
1. The following are the composition of TPN, EXCEPT?
A. lipids B. proteins C. enzymes D. CHO
2. A patient oriented pharmaceutical practice:
A. Community pharmacy
B. Hospital pharmacy
C. Clinical pharmacy
D. Institutional pharmacy

3. An organized structure which pools together all the health professions, the diagnostic and
therapeutic facilities, equipment and supplies into a coordinated system for delivering healthcare
to the public.
A. drugstore B. clinic C. health center D. NONE

1. C.
2. C.
3. D- hospital

IDENTIFY THE MEANING OF THE FOLLOWING CODES:


Code green - evacuation
Code black – bomb threat
Code red - fire
Code blue – hospital emergencies
Code yellow – missing patient
OUR LADY OF FATIMA
UNIVERSITY
College of Pharmacy

Clinical Pharmacy and


Pharmacotherapeutics 1
(PSMA 411)
CHECKLIST
• Concepts of Pharmacotherapy Care Planning
• Documentation of Pharmacotherapy Interventions
• Pharmacotherapy of Hematologic Disorders
• Pharmacotherapy of Coagulation Disorders
• Pharmacotherapy of Nervous System Disorders
• Pharmacotherapy of Cardiovascular Disorders
• Pharmacotherapy of Renal Disorders
• Pharmacotherapy of Autonomic System Disorders
UNIT OUTCOMES
• Demonstrate Understanding on the Concepts
of Pharmacotherapy Care Planning
• Demonstrate understanding on the
documentation of pharmacotherapy
interventions
• Demonstrate Competence on Disease State
Management of Hematologic disease,
Coagulative disorders, Neurologic Nervous
system disorders, Psychiatric Nervous system
disorders, Cardiovascular disorders, Renal
disorders and Autonomic system disorders
TOPIC OUTLINE

• Concepts of Pharmacotherapy Care Planning


• Documentation of Pharmacotherapy Interventions
• Pharmacotherapy of Hematologic Disorders
• Pharmacotherapy of Coagulation Disorders
• Pharmacotherapy of Nervous System Disorders
• Pharmacotherapy of Cardiovascular Disorders
• Pharmacotherapy of Renal Disorders
• Pharmacotherapy of Autonomic System Disorders
OUR LADY OF FATIMA
UNIVERSITY
College of Pharmacy

Concepts of
Pharmacotherapy Care
Planning

CLINICAL PHARMACY &


PHARMACOTHERAPEUTICS 1
(PHMA 411)
• A practice in which the pharmacist utilizes his
professional judgment in the application of
pharmaceutical sciences to foster the safe and
CLINICAL PHARMACY appropriate use of drugs, in or by patients, while
working with members of the health care team
(Francke 1969)
GENERAL CLINICAL
PHARMACIST
FUNCTIONS
• Providing drug
information to physicians
& other health
professionals
• Medication history taking
• Medication profile
preparation
• Drug therapy monitoring

7
PATIENT COUNSELING
• Patient Counseling - provision of oral
or written information about drugs &
other health-related information to a
patient or his / her representative
during the dispensing process or stay
in hospital

8
COMMUNICATION SKILLS:

Attending and active listening Empathic Responding Skills


skills 1. Reflecting - concentrating
1. Stop talking on the emotional meaning
2. Get rid of distractions 2. Paraphrasing - conveying
3. React to the ideas, NOT to the essence of what was
the person said
4. Read non-verbal messages 3. Focusing - getting back to
5. Listen to how something is the topic of conversation
said
6. Provide feedback to clarify
any message

9
PATIENT CARE
PROCESS
Why is
therapeutic
planning
necessary?
Facilitates the selection of
appropriate drug and nondrug
interventions for specific
patient problems
Provides a framework for
monitoring a patient’s
response to the drug and
nondrug interventions
It incorporates a well-thought-
out alternative regimens
1. Problem
Identification
STEP 1: Obtain patient data
STEP 2: Group related data
STEP 3: Determine each problem
STEP 4: Assess each problem
2.) Problem
prioritization
Step 1—Identify the active problems
Step 2—Identify the inactive problems
Step 3—Rank the problems
3.) Selection of patient-
specific drug and nondrug
interventions
Step 1—Determine short-term and long-term goals of
therapy
Step 2—Create a list of options
Step 3—Eliminate options based on patient-specific and
external factors
Step 4—Select appropriate drug and nondrug
interventions
Step 5—Identify alternative interventions
4.) Develop a
monitoring
plan
Step 1—Determine specific monitoring
parameters
• Select specific target outcomes
• Select monitoring intervals for each
parameter
Step 2—Integrate the monitoring plan
Step 3—Obtain data
Step 4—Assess the response to therapy
MEDICAL ABBREVIATIONS
• AB Apex Beat • AOG Age of Gestation
• ABG Arterial Blood Gas • AP Adynamic Precordium
• AFB Acid-fast Bacilli • APTT Activated Partial Thromboplastin Time
• AF Atrial Flutter/Fibrillation • ARDS Acute/Adult Respiratory Distress
Syndrome
• AHF Anti-hemophilic Factor
• ARF Acute Renal/Acute Respiratory Failure;
• AKA Above Knee Amputation Acute Rheumatic Fever
• ALD Alcohol Liver Disease • ASHD Arteriosclerotic Heart Disease
• ALL Acute Lymphocytic Leukemia • AST Aspartate Aminotransferase (formerly
• ALT Alanine Aminotransferase(formerly SGPT) SGOT)
• AMI Acute Myocardial Infarction • ATN Acute Tubular Necrosis
• ANS Autonomic Nervous System • AVM Arteriovenous Malformation
• ANST After Negative Skin Test • BA Bronchial Asthma
• BID Twice daily
MEDICAL ABBREVIATIONS
• BKA Below Knee Amputation • CA Carcinoma; Community-acquired
• BLE Both Lower Extremities • CABG Coronary Artery Bypass Graft
• BLF Both Lungs Field • CAD Coronary Artery Disease
• BM Bowel Movement • CAP Community-acquired Pneumonia
• BOV Blurring of Vision • CAT Computed Axial Tomography
• BPH Benign Prostatic Hypertrophy • CBC Complete Blood Count
• BPM Beats per minute • CBG Capillaty Blood Glucose
• BPP Biophysical Profile • CBS Clear Breath Sounds
• BSA Body Surface Area • CC/cc Chief Complaint
• BSRTL Both sluggish reactive to light • CHF Congestive Heart Failure
• BT Blood Transfusion • CHO Carbohydrate
• BUE Both Upper Extremeties • CHVD Chronic Hypertensive Vascular Disease
• BUN Blood Urea Nitrogen
• BUR Back-Up Rate
MEDICAL ABBREVIATIONS
• CK Creatinine kinase • CVD Cerebrovascular Disease
• CLAD Cervical Lymph Adenopathy
• CVP Central Venous Pressure
• CNIL Cephalic not in labor
• CVS Cardiovascular System
• COPD Chronic Obstructive Pulmonary
Disease • CXR Chest X-ray
• CPAP Central Pulmonary Airway Pressure • D/C Discharge; discontinue
• CrCL Creatinine clearance • DCMP Dilated Cardiomyopathy
• CRF Chronic Renal Failure • DHN Dehydration
• CRT Capillary Refill Time • DHS Distinct Heart Sound
• CSF Cerebrospinal Fluid • DIC Disseminated Intravascular Coagulopathy
• CT Computed Tomography • DKA Diabetic Ketoacidosis
• CTT Chest Tube Thoracostomy • DM Diabetis Mellitus
• CVA Cardiovascular/ Cerebrovascular Accident • DOB Difficultly of Breathing
MEDICAL ABBREVIATIONS
• DOE Dyspnea on Exertion • ESRD End-stage Renal Disease
• DOI Date of Injury
• ESR Erythrocyte Sedimentation Rate
• DTG Diffuse Toxic Goiter
• FBS Fasting Blood Sugar
• DTR Deep Tendon Reflex
• F/D Fairly developed
• Dx/Dxic Diagnosis
• FC Functional Class, Foleycath
• E/N Essentially normal
• FEP Full Equal Pulses
• ECE Equal Chest Expansion
• FHT Fetal Heart Tone
• EBRTL Equal briskly reactive to light
• FHx Family History
• EEG Electroencephalogram
• F/N Fairly nourished
• EF Ejection Fraction
• FT Full Term
• EOM Extra Ocular Movement
• FTSVD Full Term Spontaneous Vaginal
Delivery
MEDICAL ABBREVIATIONS
• FWB Fresh Whole Blood • HBP High Blood Pressure
• Fxn Function
• HDL High Density Lipoprotein
• GOPO Gravida para
• Hct Hematocrit
• GBS Guillain Barre Syndrome
• HEENT Head, Ear, Eyes, Nose, Throat
• GFR Glomerular Filtration Rate
• HF Heart Failure
• GI/GIT Gastrointestinal Tract
• Hgb Hemoglobin
• GIN Gastrointestinal Nutrition
• HHD Hypertensive Heart Disease
• GN Glomerulonephritis
• HPI History of Present Illness
• GS/CS Gram Stain/Culture Sensitivity
• HPN/HTN Hypertension
• HA Headache
• HR Heart Rate
• HAA Home Against Advice
• HSV Herpes Simplex Virus
• HAP Hospital-acquired Pneumonia
• Hx History
• I&O Intake & Output
• ICM Intracostal Margin
MEDICAL ABBREVIATIONS
• ICS Intercostal Space • LDH Lactate Dehydrogenase
• IDDM Insulin Dependent Diabetes Mellitus
• LDL Low Density Lipoprotein
• IE Infective Endocarditis
• LE Lower Extremities
• IHD Ischemic Heart Disease
• LFT Liver Function Tests
• INR International Normalized Ratio
• LLE Left Lower Extremity
• ITP Idiopathic Thrombocytopenic Purpura
• LLL Left Lower Quadrant
• IVP Intravenous Pyelogram
• LLQ Left Lower Quadrant
• KUB Kidney, Ureter, Bladder
• LMCA Left Mid Carotid Artery
• KVO Keep vein open
• LMCL Left Mid Clavicular Line
• LA Left Atrium
• LMP Last Menstrual Period
• LAAL Left Anterior Axillary Line
• LOC Loss of Consciousness/Level Of
consciousness
MEDICAL ABBREVIATIONS
• LPF Lower power field • MOI Mechanism of Injury
• LPM Liter per minute • MRI MagneticResonance Imaging
• LPSB Left parasternal border • MRS Methicillin-resistant Strain
• LT Lumbar tap • MV Mechanical Ventrilator
• LUE Left Upper Extremity • MVP Mitral Valve Prolapse
• LUQ Left Upper Quadrant • NABS Normoactive Bowel Sounds
• LV Left Ventricle • NBS Normal Bowel Sound
• LWI Latest Working Impression • NGT Nasogastric Tube
• MG Myasthenia Gravis • NIAE Not in acute exacerbation
• MGH May go home • NICRD Not in cardiorespiratory distress
• MOB Medications on Board • NIDDM Non-insulin Dependent Diabetes
Mellitus
MEDICAL ABBREVIATIONS
• NIL Not in labor • NSNI Non-septic non-induced
• NIR Non-insulin requiring
• NSTEMI Non-ST wave elevationMyocardial
• NIRD Not in respiratory distress Infarction
• NNO No new orders • NSSTWC Non-specific ST-wave changes
• NNS No new Suggestions • NV Neurovascular
• NO Non-obese • NVE Neck Vein Engorgement
• NORF No other remarkable findings • OB Occult Blood
• NPO Nothing per orem • OCP Oral Contraceptive Pill
• NRRR Normal Rate Regular Rhythm • OD Once daily/ Right eye
• NRTL Non-reactive to light • OF Osterized Feeding
• NSAID Non-steroidal AntiinflammatoryDrug • OS Left eye
• OU Both eye
MEDICAL ABBREVIATIONS
• P/SHx Personal/Social History • PT Prothrombin Time
• PE Physical Examination • PTA Prior to Admission
• PEEP Positive end expiratory Pressure • PTB Pulmonary Tuberculosis
• PGO Partial Gut Obstruction • PTC Prior to Consult
• PHx Personal History • PTH Parathyroid Hormone
• Plt Platelet • PTO/PTOR Prior to Operation
• PMP Post Menstrual Period • PTT Partial Thromboplastin Time
• PNB Pink Nail Beds • PU Pregnancy Uterine
• PND Paroxysmal Nocturnal Dyspnea • PUD Peptic Ulcer Disease
• POI Place of Injury • PVC Premature Ventricular Contraction
• PRBC Packed Red Blood Cells • PVT Paroxysmal VentricularTachycardia
• PS Pressure Support • PWI Present working Impression
• R/O Rule out
MEDICAL ABBREVIATIONS
• RA Right atrium • RRR Regular Rhythm and Rate
• RBC Red Blood Cells
• RTC Round the Clock
• RBS Random Blood Sugar
• RUE Right Upper Extremity
• RCM Right costal margin
• RUQ Right Upper Quadrant
• RF Renal Failure, Respiratory Failure
• RV Right Ventricle
• RHD Rheumatic Heart Disease
• RVR Rapid Ventricular Rate
• RLE Right Lower Extremity
• S/P Status Post
• RMCA Right mid-carotid artery
• SABE Sub-acute Bacterial Endocarditis
• RMI Regular Menstrual Interval
• SAH Sub-arachoid Hemorrhage
• ROR Red orange reflect
• SBP Systolic Blood Pressure
• ROS Review of Systems
• SC Spinal Cord
• RR Respiratory Rate
• SC/SQ Subcutaneous
• RRE Round, regular and equal
MEDICAL ABBREVIATIONS
• SCI Spinal Cord Injury • T/C To consider
• SCN Sternocleidomastoid • TCVS Thoracic Carrdiovascular Surgery
• SGOT Serum Glutamic Ovalo-Acetic • TFI Total Fluid Intake
Transaminase (also AST)
• SGPT Serum Glutamic PyruvicTransaminase • TIA Transient Ischemic Attack
(also ALT) • TKVO To keep vein open
• SIADH Syndrome of Inappropriate Antidiuretic • TOH Time of Injury
Hormone
• TPC Tonsilopharyngeal Congestion
• SLE Systemic Lupus Erythematosus
• TPR Temperature, pulse, Respiration
• SOB Shortness of Breath
• TSB Tepid sponge bathe
• SrCr Serum Creatinine
• TTHD Thyrotoxic Heart Disease
• SSS Sick Sinus Syndrome
• TVP Transvenous pacemaker
• SVT Supra-ventricular Tachycardia
• Tx/Txic Treatment
• U/A Urinalysis
MEDICAL ABBREVIATIONS
• UBP Usual Blood Pressure • WNL Within normal limits
• UE Upper Extremities
• WOF Watch out for
• UGIB Upper Gastrointestinal Bleeding
• 1,25-DHCC 1,25-dihydrocholecalciferol
• UO Urine Output
• 25-HCC 25-hydrocholecalciferol
• URTI Upper Respiratory Tract Infection
• 3P’s Polyphagia, Polydipsia, Polyuria
• UTI Urinary Tract Infection
• VA Vehicular Accident
• VF Visual field
• VS Vital signs
Pop Quiz
1. Skill/ability to communicate clearly and effectively with patients,
significant others (SO/relative or family), physicians, nurses,
pharmacists, and other health care team?
2. It is the provision of verbal or written information about drugs and
other health related information by a pharmacist to a patient
representative of the patient?
3. This is a systematic and comprehensive method that is employed to
identify, solve, and prevent drug therapy problems?
OUR LADY OF FATIMA
UNIVERSITY
College of Pharmacy

Documentation of
Pharmacotherapy
Interventions

CLINICAL PHARMACY &


PHARMACOTHERAPEUTICS 1
(PHMA 411)
PRINCIPLES OF
DOCUMENTATION

• Documentation is required to
record pertinent facts,
findings, and observations
about a patient’s health
history, including past and
present illnesses,
examinations, tests,
treatments, and outcomes.
• A complete and legible record;
Components of documentation: • Documentation for each
encounter with a rationale for the
encounter, physical findings, prior
test results, assessment, clinical
impression/ diagnosis and plan for
care;
• Identified health risk factors, and
an easily inferred rationale for
ordering diagnostic tests or
ancillary services; and
• The patient’s progress, response
to and changes in treatment, and
revision of the original
diagnosis/assessment.
PHARMACEUTICAL CARE

• Responsible provision of drug therapy


for the purpose of achieving definite
outcomes that improves a patient’s
quality of life (Hepler and Strand 1990).
• A patient-centered practice in which the
practitioner assumes responsibility for a
patient’s drug-related needs and is held
accountable for this commitment
(Cipolle 1998)

46
PHARMACEUTICAL CARE PLAN
• Assessment: a review of the medical conditions and symptoms to
determine the need for drug therapy

• Plan: a decision of an appropriate drug therapy based on the


assessment of the patient

• Monitoring: a review of the outcomes of drug therapy (goals and


endpoints) to determine if the patient is obtaining the desired
outcomes

47
• a distinct service or group of
services that optimize
therapeutic outcomes for
individual patients.
• patient centered assessment
and evaluation of the patient’s
full medication regimen and
not only individual
medications.

What is Medication
Therapy Management?
48
• Medication therapy review (MTR)
• Personal medication record (PMR)
• Medication-related action plan
Core (MAP)
• Intervention and/or referral
Elements
• Documentation and follow-up
of an MTM
Service

49
1. Medication Therapy Review:

• A systematic process of
collecting patient-specific
information, assessing
medication therapies to
identify medication-related
problems, developing a
prioritized list of
medication-related
problems, and creating a
plan to resolve them.

50
Personal Medication Record:

• A comprehensive record
of the patient’s
medications
(prescription and non-
prescription
medications, herbal
products, and other
dietary supplements).

51
Medication-Related Action Plan:

• The medication-related action plan


(MAP) is a patient-centric
document containing a list of
actions for the patient to use in
tracking progress for self-
management.

52
• MTM services are documented in a consistent manner, and a follow-up
MTM visit is scheduled based on the patient’s medication-related needs, or
Documentation the patient is transitioned from one care setting to another.
• Documentation is an essential element of the MTM service model. The
and Follow-up: pharmacist documents services and intervention(s) performed in a manner
appropriate for evaluating patient progress and sufficient for billing
purposes.

53
Documentation elements for the
patient record may include, but are not
limited to, the following:

Documentation Examples
category
Patient demographics Basic information: address, phone, e-mail, gender, age, ethnicity,
education status, patient’s special needs, health plan benefit/insurance
coverage
Subjective observations Pertinent patient-reported information: previous medical history, family
history,
social history, chief complaints, allergies, previous adverse drug
reactions
Objective observations Laboratory results, vital signs, diagnostic
signs, physical exam results, review of systems

54
Documentation Examples
category
Assessment Problem list, assessment of medication-related problems
Plan A care plan is the healthcare professional’s course of action for helping a
patient
achieve specific health goals
Education Goal setting and instruction provided to the patient with verification of
understanding
Collaboration Communication with other healthcare professionals: recommendations,
referrals, and
correspondence with other professionals (cover letter, SOAP note)

55
Documentation Examples
category
PMR A record of all medications, including prescription and nonprescription
medications,
herbal products, and other dietary supplements
MAP Patient-centric document containing a list of actions to use in tracking
progress
for self-management
Follow-up Transition plan or scheduling of next follow-up visit
Billing Amount of time spent on patient care, level of complexity, amount
charged

56
MTM
flowchart:

57
58
S = Subjective

O = Objective

SOAP NOTE
A = Assessment

P = Plan
EXAMPLE
• Pharmaceutical care including pharmaceutical care
plan process (CORE, PRIME and FARM/SOAP) is a
systematic method for recording the pharmacist’s
examination of patient pharmacotherapy and
subsequent modification of medication related
problems
FARM note is an alternative
approach to documenting drug-
related problems and plan which is
a pharmacist equivalent of a
physician progress note in a
systematized approach.
• It actually includes provisions for
identification and assessment of
actual or potential medication
related problems, description of
a therapeutic plan and
appropriate follow-up monitoring
of problems.
• F – Findings – patient specific information
• A – Assessment – evaluation of findings
(e.g) severity duration
• R – Resolution (including prevention) -
rationale for the intervention
• should be mentioned
• M – Monitoring (and follow up) – assess
efficacy, safety and outcome of the
intervention
FINDINGS
ASSESSMENT
• It contains pharmacist evaluation of the present
situation.
• It should delineate the thought process that led
to the conclusion on problem whether it did or did
not exist and active intervention either was or
was not necessary.
• The severity or urgency of the problem should be
indicated by stating whether the interventions
should be made immediately or within one day,
one week or longer.
• Therapeutic endpoint or outcome stated may be
short term goals or long term goals.
• It highlights the actions proposed to resolve drug
related problem based upon preceding analysis.
RESOLUTION • Common recommendations include non-
pharmacologic therapy like dietary modifications
or assisting devices.
• Rationale for selecting a particular therapy
should be clearly depicted. Alternative therapy
in case of emergency switching-off of drugs must
be enlightened.
• Patient counseling provided must be elaborated.
Monitoring
P - Pharmaceutical based problems

PRIME R - Risks to the patient

I - Interactions

M - Mismatch between medication


and condition or patient needs

E - Efficacy issues
Pop Quiz
1. A systematic process of collecting patient-specific information,
assessing medication therapies to identify medication-related
problems, developing a prioritized list of medication-related
problems, and creating a plan to resolve them.
2. A comprehensive record of the patient’s medications (prescription
and non-prescription medications, herbal products, and other
dietary supplements).
3. A highly structured format for documenting the progress of a
patient during treatment and is only one of many possible formats
that could be used by a health professional.
OUR LADY OF FATIMA
UNIVERSITY
College of Pharmacy

Pharmacotherapy of
Hematologic Disorders

CLINICAL PHARMACY &


PHARMACOTHERAPEUTICS 1
(PHMA 411)
ANEMIA
• It is a condition characterized by a decrease in hemoglobin or RBC
mass which results to decreased oxygen-carrying capacity of the
blood.
• It may decrease due to blood loss, increased destruction of RBCs or
decreased production of RBCs
GENETIC ETIOLOGY
•Hemoglobinopathies
•Thalassemias
•Enzyme abnormalities of the glycolytic pathways
•Defects of the RBC cytoskeleton
•Congenital dyserythropoietic anemia
•Rh null disease
•Hereditary xerocytosis
•Abetalipoproteinemia
•Fanconi anemia
NUTRITIONAL and CHRONIC DISEASE ETIOLOGIES

• Iron deficiency
• Vitamin B-12 deficiency
• Stravation and Generalized Malnutrition
• Renal disease
• Hepatic disease
• Chromic infections
• Neoplasia
• Collagen Vascular disease
PHYSICAL ETIOLOGY
• Trauma
• Burns
• Frostbite
• Prosthetic valve and surface
INFECTIOUS ETIOLOGIES
• Viral
• Hepa, Infectious mononucleosis, Cytomegalovirus
• Bacterial
• Clostridia, GM(-) sepsis
• Protozoal
• Malaria, Leishmaniasis, Toxoplamosis
RBC life cycle
RBCs CHARACTERISTIC

• RBC is highly deformable


and increase size from 7um
to 13 um when they
transverse capillaries with
3um diameter.
• They have negative charge
on their surface which
deflects the phagocytosis.
• They don’t have nucleus
that’s why they didn’t
undergo Kreb’s cycle.
• RBC relies on glycolysis via
Embden-Meyerhof and
Pentose Phosphate Pathway
FACTORS THAT DECREASES RBC PRODUCTION

• Inadequate erythropoietin
• Inadequate dietary intake
• Hypothyrodism
FACTORS THAT INCREASE RBC DESTRUCTION

• Hemorrhaging • Hemolysis
• Endometriosis • Disorder of spleen and liver
• Accidents • Genetic disorders (G6PD,
• GI lesions Thalassemia, Sickle cell anemia)
• Menstruation
• Chilbirth
• Excessive Uterine Bleeding
• Surgery
• Cirrhosis
• Fibrosis
DAILY NUTRITIONAL REQUIREMENTS
IRON DAILY INTAKE FOR ADULTS (19-50 Y/O) DAILY INTAKE FOR AGE OVER 14 Y/O
MEN 8 mg MEN 400 mcg
WOMEN 18mg WOMEN 400 mcg
DURING PREGNANCY 27 mg DURING PREGNANCY 600 mcg
WHILE 9 mg WHILE 500 mcg
BREASTFEEDING BREASTFEEDING

DAILY INTAKE FOR ADULT


ADULT (MEN AND WOMEN) 2.4 mcg
DURING PREGNANCY 2.6 mcg
WHILE BREASTFEEDING 2.8 mcg
CLINICAL MANIFESTATION
• Palor
• Feeling of coldness
• Lightheadedness or dizziness
• Unusual cravings
• Trouble concentrating
• Constipation

• Severe form:
• Brittle nails
• Shortness of Breath
• Chest pains
• Fainting
COMPLICATIONS
• The most serious complications of severe anemia arise from tissue
hypoxia.
• Shock, hypotension or coronary and pulmonary insufficiency can
occur.
• This is more common in older individuals with underlying pulmonary
and cardiovascular disease.
Diagnosis
• Personal Health history
• Family health history
• Physical Exam
• Laboratory Assessment
PHYSICAL DIAGNOSIS
• Optic Fundi evaluation
• Blood pressure
• Heart rate
• Heart murmur
• Enlarged lymph nodes, spleen and liver
• Atrophic glossitis of tongue
• Color of the skin
• Pale
• Jaundice
LABORATORY ASSESSMENT
• CBC (Complete Blood Count)
• Serum Iron Levels
• Ferritin test
• Vitamin B-12 test
• Folic acid test
• Stool test for occult
• Measuring of RBC mass
• These are time consuming and expensive and usually require transfusion of
radiolabeled erythrocytes.
ADDITIONAL ASSESSMENT
• Upper GI
• Barium enema
• Chest X-rays
• CT scan of abdomen
CLINICAL INTERVENTION
• Nutritional supplements
• IV of Vitamin B12
• Erythropoietin
• Blood transfusion
SICKLE CELL ANEMIA
• SCD is a genetic disease of RBC.
• Crescent shaped like RBCs which gives flexibility to travel through
even the smallest blood vessel.
• However, this shape makes them sticky and rigid and prone to getting
trapped in small vessels.
• It blocks the blood which causes pain and tissue damage
SICKLE CELL ANEMIA
• SCD is an autosomal recessive condition.
• Hemoglobin in RBC is easily displace.
• Hb has 2 alpha chains and 2 beta chains.
•SCD has 4 main types:
•Hemoglobin SS
•Hemoglobin SC
•Hemoglobin SB+ (beta) thalassemia
•Hemoglobin SB 0 (beta-zero) thalassemia
•Sickle cell trait
SICKLE CELL SUBTYPES
• Hb SS
- Most common type of SCD. Severe anemia.
• Hb SC
-2nd most common SCD. Less severe anemia.
• Hb SB+ thalassemia
-defected beta globin gene production resulted to reduced cell size. Not sever
anemia
• Hb SB 0 thalassemia
-defected beta globin gene. Same with Hb SS, severe anemia
CLINICAL MANIFESTATION
• It appear in babies as early as 4 months old but generally occurs
around 6 month mark.
• They usually experience:
• Excessive fatigue or irritability
• Fussiness ( in babies)
• Bed wetting (due to kidney problems)
• Jaundice (yellow eyes and skin)
• Swelling and pain in hands and feet
• Frequent infections
• Chest pain, backpain, pain in arm and legs
AT RISK FOR SCD
• Children with both parent carrier.
• Regions that have endemic malaria:
• Africa
• India
• Mediterranean
• Saudi Arabia
CIRCUMTANCES THAT CAN INDUCE SCD

• Illness
• Changes in temperature
• Stress
• Poor hydration
• altitude
COMPLICATION THAT INDUCE SCD
• Severe anemia • Lung disease
• Hand-Foot Syndrome • Priampism
• Splenic sequestration • Gallstones
• Delayed growth • Sickle cell chest Syndrome
• Neurological complication
• Eye Problems
• Skin Ulcers
• Heart Disease and Chest syndrome
DIAGNOSIS
• Hemoglobin electrophoresis
-measures different Hb level
• Sickle cell gene from amniotic fluid
• Detailed patient history
• Blood test
CLINICAL INTERVENTION
• Rehydration
• Symptomatic management
• Blood transfusion
• Supplemental Oxygen
• Pain Medication
• Bone marrow transplant
NON PHARMACOLOGIC
• Heating pads for pain
• Folic acid-rich food
• Diet regulation
• More fluids
• Exercise
• Less stress
• Contact your family doctor
Pop Quiz
1. An iron protein complex which combines with oxygen and carbon
dioxide is?
2. A condition characterized by decreased production or increased
destruction of RBCs
3. The term thrombocytopenia indicates a/ an:
4. T/F: Vitamin deficiency can cause anemia.
5. The chief function of platelet is to?
OUR LADY OF FATIMA
UNIVERSITY
College of Pharmacy

Pharmacotherapy of
Coagulation Disorders

CLINICAL PHARMACY &


PHARMACOTHERAPEUTICS 1
(PHMA 411)
FACTOR
• Factor I- Fibrinogen • Factor IX- Christmas factor
• Factor II- Prothrombin • Factor X- Stuart-Prowen factor
• Factor III- Thromboplastin • Factor XI
• Factor IV- Calcium • Factor XII
• Factor V- Labile F • Factor XIII
• Factor VI- (no longer used)
• Factor VII- Stable factor
• Factor VIII- Anti-hemophilic factor
HEMOPHILIA
• ROYAL DISEASE
• It affect royal families of England, Germany, Russia and Spain.
• Queen Victoria, Carrier of Hemophilia B. Passed the trait to her 3 out of 9 children.
• Leopold
• Alice and Beatrice

• A medical condition in which the ability of the blood clot is severely


reduced, causing the sufferer to bleed severely from even a slight injury
• The condition is typically caused by a hereditary lack of coagulation factor
of the FACTOR VIII or FACTOR IX.
• The severity of hemophilia that a person has is determined by the amount
in the blood.
• Lowering the factor in the blood, the more likely the bleed will occur.
HEMOPHILIA
• Majority of the population experiencing hemophilia are
• middle aged
• elderly people
• Young women who recently give birth
• It usually occurs in every 1 of 5,00 male births.
• Hemophilia A is 4x more common than hemophilia B.
GENETICS
• Hemophilia is caused by one of the genes that undergo changes or
mutation.
• These genes are located on the X chromosome
• Male have 1 X chromosome and 1 Y chromosome. X chromosome will be
inherited from their mothers.
• Female have 2 X chromosome and inherited from each parent.
• A female with 1 affected X Chromosome is a “ CARRIER” will
sometimes have hemophilia.
• She also can pass the affected X chromosome with a mutated clotting
factor on her children
Inheritance pattern for Hemophilia
PATHOPHYSIOLOGY
• FVIII mRNA is
detected in liver,
LIVER
spleen and tissue.
• Transfected cell lines
shows synthesis
which will move to
FVIII SPLEEN the lumen of ER.
• It will affect the
regulation of secretio
TISSUES proteins like the Ig
binding protein.
PATHOPHYSIOLOGY
• Part of FVIII in
endoplasmic LIVER
reticulum will be
degraded and the
others will be SPLEEN
transported to golgi
apparatus.
TISSUES
• Modifications will
occur.
CLOTTING CASCADE
• The main role of
coagulation system is to
produce stable fibrin clot at
sites of injury.
• There are 2 pathway
mechanism:
• Intrinsic
• extrinsic
CLOTTING CASCADE
Endothelial cell

Blood • Intrinsic pathway:


XII → XIIa
XI → Xia
IX→IXa
• Extrinsic pathway:
Platelet + Fibrin VII → VIIa

Fibrin Form into


Fibrin
Fibrinogen strand
CLASSIFICATION
• Hemophilia A (Classic Hemophilia)
• Lack or deficient of clotting factor VIII
• Hemophilia B (Christmas Disease)
• Lack or deficient of clotting factor IX
•Hemophilia C (Rosenthal Syndrome)
•Lack or deficient of clotting factor XI
CLINICAL MANIFESTATION
• Hemophilia can result in:
• Bleeding within joints that can lead to chronic joint disease and pain or tightness in
the joints.
• Bleeding into skin—which is bruising– or muscle and soft tissue causing building up
of blood in the area.
• Bleeding of the mouth and gums, and bleeding that is hard to stop.
• Bleeding after circumcision.
• Bleeding after having shots or vaccination.
• Bleeding in the head of an infant after difficult delivery
• Blood in urine or stool
• Frequent hard to stop nose bleeds.
• Bleeding in the head and sometimes in the brain which can cause long term
problems, such as seizures and paralysis.
• Death can occur if the bleeding cannot be stopped or if it occurs in a vital organ
LABORATORY EVALUATION
• Family history
• Usually right after birth, a blood from umbilical cord will be withdrawn and tested to determine the levels of clotting
factor.
• Factor VIII is readily available in the umbilical cord while the factor IX will fully developed until the baby is at least 6
months old.
• CBC (complete Blood count)
• Hemoglobin level
• Size and number of RBC and different WBC
• Platelets
• APTT (Activated Partial Thromboplastin time) test
• Measure the clotting time of factors VIII, IX, XI and XII.(Intrinsic pathway)
• PT (Prothrombin) Test
• Measure the clotting time of factors I, II, V, VII, and X. (Extrinsic pathway)
• Fibrinogen test
• AKA Clotting factor I
• Assessment on the ability of blood to form clot.
• It is alongside with PT or APTT or both.
SEVERITY IN HEMOPHILIA A
Level of Severity Levels of Manifestation
factor viii or ix
in blood
Normal (no hemophilia) 50-100% --------
Mild Hemophilia 5% - 50% Generally experience bleeding after serious
trauma
Moderate Hemophilia 1 - 5% Experience bleeding episodes after injury
Severe Hemophilia < 1% Spontaneous bleeding following an injury
often into their joints and muscles.
SEVERITY IN HEMOPHILIA A
Level of Severity Levels of Manifestation
Factor VIII in
the blood
Mild Hemophilia A 6-49%
Moderate Hemophilia A 1-5%
Severe Hemophilia A <1%
TREATMENT INTERVENTION
• Infusion of commercially prepared factor concentrates.
• Performing regular basis self-infusion or prophylaxis
•Inhibitors
• Commonly used in VWD (Von Willebrand’s Disease)
• Antibody that act as inhibitor to the factor concentrates.
• DDVAP (Desmopressin acetate)
• Synthetic version of vasopressin.
• Available in nasal spray and injectables.
• For ptx with mild hemophilia.
• Used for joint and muscle bleeding
• Aminocaproic acid
• Prevents the breakdown of blood clots.
• Used before dental procedures.
• Its available in tablet or oral liquid preparation.
Pop Quiz
1. Also known as Labile F.
2. What factors are being measured in an APTT or Activated
Partial Thromboplastin Time test?
3. Hemophilia B is a condition typically caused by a hereditary
lack of coagulation factor
4. Fibrinogen test also known as ________ test.
5. Where does the gene mutation take place?
REFERENCES
• Walker, Roger & Whittlesea, Cate. (2012). Clinical Pharmacy & Therapeutics.
5th ed. Elsevier Ltd.
• American Pharmacists Association, National Association of Chain DrugStores
Foundation. Medication therapy management in community pharmacy practice:
core elements of an MTM service (version 1.0).J Am Pharm Assoc.
2005;45:573-9.
• Cipolle RJ. Strand LM, Morley PC. Pharmaceutical Care Practice: The
Clinician’s Guide. New York: McGraw Hill; 2004.
• The American Pharmacists Association and the National Association of Chain
Drug Stores Medication Therapy Management in Pharmacy practice Core
elements of an MTM service model.
• https://www.globalpremeds.com/blog/2015/01/02/understanding-soap-format-
for-clinical-rounds/

REFERENCES

• Walker, Roger & Whittlesea, Cate. (2012). Clinical Pharmacy & Therapeutics. 5th ed.
Elsevier Ltd.
• Market House Book, LTD., 2009. The Bantam Medical Dictionary, 6th edition
• https://emedicine.medscape.com/article/198475-overview#a4
• https://www.healthline.com/health/anemia#outlook
• https://www.healthline.com/health/sickle-cell-anemia#outlook
• www. Hemophilia.org/Bleeding-Disorders/Types-of-Bleeding-Disorders/Hemophilia-A
OUR LADY OF FATIMA
UNIVERSITY
College of Pharmacy

Clinical Pharmacy and


Pharmacotherapeutics 1
(PSMA 411)
CHECKLIST
• Concepts of Pharmacotherapy Care Planning
• Documentation of Pharmacotherapy Interventions
• Pharmacotherapy of Hematologic Disorders
• Pharmacotherapy of Coagulation Disorders
• Pharmacotherapy of Nervous System Disorders
• Pharmacotherapy of Cardiovascular Disorders
• Pharmacotherapy of Renal Disorders
• Pharmacotherapy of Autonomic System Disorders
UNIT
OUTCOMES
1. Demonstrate appreciation of pharmacy as a health profession and the scope of
practice.

2. Demonstrate understanding of chemical, physical, and biological processes relevant


to pharmaceutical sciences.

3. Recognize the importance of the healthcare system and its components,


including the importance of pharmacovigilance.

4. Demonstrate understanding of morphology, anatomy, physiology, taxonomy, and


distinct features of plant species of pharmaceutical importance.

5. Relates ethical practices to administration, management and leadership in


pharmacy
6. Identify the different elements or components in a pharmacy operation while
practicing good managementskills.

7. Demonstrate understanding of the relationship among different levels of body


functions from molecular to systemic level in normal conditions and diseased
states, including the impact of immunization in vaccine- preventable diseases
TOPIC
OUTLINE
• Concepts of Pharmacotherapy Care Planning
• Documentation of Pharmacotherapy Interventions
• Pharmacotherapy of Hematologic Disorders
• Pharmacotherapy of Coagulation Disorders
• Pharmacotherapy of Nervous System Disorders
• Pharmacotherapy of Cardiovascular Disorders
• Pharmacotherapy of Renal Disorders
• Pharmacotherapy of Autonomic System Disorders
OUR LADY OF FATIMA
UNIVERSITY
College of Pharmacy

Pharmacotherapy of
Nervous System Disorders
(Epilepsy, Status epilepticus and Acute
Management of the Brain Injury)

CLINICAL PHARMACY &


PHARMACOTHERAPEUTICS 1
(PHMA 411)
EPILEPSY
• It is a chronic disorder that causes unprovoked,
recurrent unprovoked seizures.
• Seizures is a sudden rush of electrical activity.
•There are 2 main types of seizures:
•General seizures- whole brain
•Focal or partial seizures- one part of the brain

• It affects 65 million worldwide.


• It can affect everyone but it is common in young children
and older adults. It occurs slightly more in male than
females.
ETIOLOGY
• High fever
• Head trauma
• Very low blood sugar
• Alcohol withdrawal
• Genetic or Development disorders or neurological
disease
CLINICAL MANIFESTATION
• Symptoms may vary according to type of seizure.
• Focal (partial) Seizure
-Simple seizure that didn’t loss the consciousness of the patient. It
includes:
- Changes in the sense of taste, smell, sight, hearing or touch.
- Dizziness with the tingling and twitching sensation of the limbs

• Focal (partial) Seizure


- Complex partial seizure which involves the loss of
awareness. It includes the:
- Staring blankly
- Unresponsiveness
- Performing repetitive movements
CLINICAL MANIFESTATION
• Generalized seizures
• Absence seizures
- Aka petit mal seizure which involves stare blank, lip smacking,
repetitive movements and usually short loss of awareness.
• Tonic seizures
-muscle stiffness.
• Atonic seizures
- Loss of muscle control and cause sudden falling down.
• Clonic seizures
-repeated, jerky muscle movement of face, neck and arms.
• Myoclonic seizures
-Spontaneous quick twitch of arms and legs.
CLINICAL MANIFESTATION
• Generalized seizures
• Tonic-clonic seizures
- Aka Grand mal seizures.
- Causes stiffening and shaking of body
- Loss of bladder or bowel control
- Biting of tongue
- Loss of consciousness
- Can’t remember what happened or you feel slightly ill for few
hours.
BEHAVIORAL CONNECTION
• Children with epilepsy tend to have more learning
and behavioral problems.
• About 15-35% has intellectual disabilities
•Some people experience a change in behavior in the minutes or hours
before the seizure.
•They are show inattentiveness, irritability, hyperactivity and
aggressiveness
TRIGGERS OF EPILEPSY
• Lack of sleep
• Illness or fever
• Stress
• Bright lights, flashing lights or patterns
• Caffeine, alcohol, medications or drugs.
• Skipping meals, overeating or eat a specific food
CLINICAL ASSESSMENT
• Medical history
• Neurological examination
• Examine the motor abilities and mental functioning
• EEG (Electroencephalogram)
• Most common used test for epilepsy
• The changes in normal brain wave patterns are common in
epileptic patients, with or without seizures.
• Imaging test to check the presence of tumor and other
abnormalities
• CT scan
• MRI
• PET
• Single-photon emission computerized tomography
LABORATORY ASSESSMENT
• Blood Chemistry
• Check the signs of infectious disease
• Monitor the liver and kidney function
• Monitor the glucose level
CLINICAL MANAGEMENT
•Vagus nerve stimulator
•Invasive procedure.
•It gives electric stimulation that runs through your neck to prevent seizures.

•Ketogenic Diet
•Provides high fat and low carbohydrate diet
CLINICAL MANAGEMENT
• Keeping a journal

Date Activitie Happening Unusual Unusual Food Level of Hours


and s around to stressor eaten fatigue of sleep
time senses s and (0-10)
time

Apri Class Car crash Nothing Milktea Tapsilog 9 3 hours


l 06 reportin
g
CLINICAL MANAGEMENT
• Brain Surgery (Resection)
• The area of the brain that causes seizure activity can be
removed or altered
•Anti-epileptic (anti-convulsant) drugs
• To reduce the episode of seizure
• Levetiracetam, Lamotrigine, Topimirate, VPA,
Carbamazepine and Ethosuximide
DIETARY RECCOMMENDATION

• Ketogenic Diet
• The goal is to force the body to use fat for energy
instead of glucose.
•Modified Atkins Diet
•This diet is also high fat and involves a controlled carb intake.
•The common side effect is constipation due to low in fiber and high
fat diet.
•Medical alert bracelet
CLINICAL MANAGEMENT
• No cure for epilepsy only management with
medications and other non pharmacologic
strategies.
• Yoga and other stress relieving therapies.
• Several complementary therapies might work for
them.
• Be cautious about reducing or stop taking your
medication. Consult your family doctor with regards to
that matter.
STATUS EPILEPTICUS
• Status Epilepticus means continuous state of seizure.
• It is a medical emergency when a seizure hits the 5 min
mark.
• There are 2 main forms of Status epilepticus, the
convulsive and non-convulsive type

• Convulsive type:
-more common and dangerous.
-involves tonic-clonic and sometime referred as grand mal
• Non-convulsive type:
- “Epileptic twilight” state
- Doesn’t lose the consciousness
CAUSES of SE
• Low blood sugar
• HIV
• Head trauma
• Heavy alcohol or drug use
• Kidney or liver failure
CLINICAL ASSESSMENT
• EEG
• Blood chemistry
• Urinalysis
• CT scan
• Chest X-ray
CLINICAL MANAGEMENT
• First line treatment at home:
• Protect the head of the patient
• Move away from the danger
• Resuscitate when needed
• Give emergency medication:
• Midazolam- buccal administration
• Diazepam- rectal administration
• Call an ambulance
• First line treatment at hospital:
• High Oxygen concentration followed by intubation
• Cardiac and Respiratory assessment
• Diazepam IV or Lorazepam IV to suppress seizure activity
• Individual care plan:
• States when the medication will use.
• States how much should be given
• States the necessary steps should be taken afterward
ACUTE MANAGEMENT OF TRAUMATIC BRAIN INJURY

• Head injury can be defined as any alteration in


mental or physical functioning related to blow to
the head.
• Loss consciousness does not need to occur
• Traumatic brain injury is a leading cause of death
and disability in trauma patients
GLASGOW COMA SCALE SCORE
• Generate numerical summed score for eye, motor
and verbal abilities.

13-15 Mild Injury


9-12 Moderate Injury
>8 Severe Injury
CLINICAL MANIFESTATION
• Bruising or bleeding on the head and scalp
• Blood in the ear canal
• Anosmia – due to shearing of the olfactory nerves at
the cribriform plate.
• Abnormal Postresuscitation pupillary reactivity
• Isolated internuclear opthalmoplegia secondary to
traumatic brain stem injuries
• Cranial nerve VI and VII palsy
• Hearing loss
• Dysphagia
• Focal motor findings
PATHOPHYSIOLOGY
• Gross structural changes in head injury.
• Skull fracture (Simple linear fashion or complicated
depressed manner)
• Bone fragments
• Neuronal loss
• Loss of neuron from dorsal thalamus
• Neurochemical changes
• Catecholamine surges in plasma and CSF
• Monro-kellie hypothesis
• The total intercranial volume is a sum of brain tissue,
cerebral spinal fluid, venous blood and arterial blood.
CLINICAL ASSESSMENT
• Bedside cognitive testing- is to measures the
patient’s level of consciousness, attention and
orientation.
• Imaging test
• EEG
LABORATORY ASSESSMENT
• Sodium levels
• Magnesium levels
• Coagulation studies
• Blood alcohol
• Renal function test and Creatinine kinase levels
• Neuron-specific enolase and protein S-100B
CLINICAL INTERVENTION
• Management of intercranial pressure
• Hypertonicity
• Surgical evacuation of Subdural Hematomas
Pop Quiz
1. It is a continuous state of seizure.
2. An imaging test used to check the presence of
tumors and abnormalities in the brain based on
the brain wave patterns.
3. What cranial nerve is responsible for the attacks
like epilepsy?
OUR LADY OF FATIMA
UNIVERSITY
College of Pharmacy

Pharmacotherapy of
Nervous System Disorders
(Headache, Migraine and Tension, Pain
Management and Parkinson’s Disease)

CLINICAL PHARMACY &


PHARMACOTHERAPEUTICS 1
(PHMA 411)
MIGRAINE

• Migraine is common, recurrent, primary headache


of moderate to severe intensity that interferes with
normal functioning and is associated with GI,
neurologic and autonomic symptoms.
PATHOPHYSIOLOGY
• Defect in the activity of neuronal calcium channels
mediating neurotransmitter release in the brainstem areas
that modulate cerebral vascular tone and nociception.
• It will lead to vasodilation of intercranial extracerebral blood
vessels with activation of the trigeminovascular system.
• Heritability of migraine.
• Migraine triggers may be modulators of the genetic set point that
predisposes to migraine headache.
• Serotonin receptors
• Acute antimigraine drugs such as ergot alkaloids and triptan
derivative are agonist of vascular and neuronal 5HT1 receptor
subtypes.
• It will result in vasoconstriction and inhibition of vasoactive neuropeptide
release and pain signal transmission.
CLINICAL MANIFESTATION
• Recurring unilateral episodes of throbbing head pain.
• Associated with nausea, vomiting and sensitivity to
light, sound and/ or movement.
• Migraine aura (classic migraine)
• A recurring headache that strikes after or at the same time as
sensory disturbances.
• Premonitory symptoms:
• Neurological (Phonophobia, photophobia, hypersomnia
difficulty in concentrating)
• Psychological (anxiety, depression, euphoria, irritability,
drowsiness, hyperactivity, restlessness)
• Autonomic (polyuria, diarrhea, constipation)
• Constitutional (stiff neck, yawning, thirst, food cravings,
anorexia
CLINICAL ASSESSMENT
• Comprehensive headache history
• Headache evaluation
• Stable pattern of headache
• Absences of daily headache
• Positive family history for migraines
• General medical and neurologic examination
• Diagnostic and laboratory assessment
• Neuroimaging
NONPHARMACOLOGIC

• Application of ice in head


• Period of rest or sleep, usually in darkquiet
environment.
• Behavioral intervention
PHARMACOLOGIC INTERVENTION

• Pretreatment with antiemetics.


• First line treatment for mild to moderate
migraine attacks is simple analgesics and
NSAIDs.
• Use of ergot alkaloids and derivatives
• Serotonin receptor agonist
• Prophylactic therapy for recurring migraine
attacks.
• Preventive therapy for headache that
occurs in a predictable pattern.
TENSION-TYPE HEADACHE
• It is the most common type
of primary headache and
more common in women
than men.
• Pain is usually mild to
moderate, bilateral, non
pulsatile.
• Episodic headaches may
become chronic.
PATHOPHYSIOLOGY
• It can originate from myofascial factors and
peripheral sensitization of nociceptors
• Involvement of central mechanism.
• Activation of supraspinal pain perception structures
CLINICAL MANIFESTATION
• Premonitory symptoms and aura are absent
• Pain is usually mild to moderate, bilateral, non
pulsatile and in the frontal and temporal areas, but
occipital and parietal areas can also be affected
• Mild photophobia or phonophobia may occur.
• Pericranial or cervical muscles may have tender
spots or localized nodules in some patients.
PAIN MANAGEMENT
• Pain is an unpleasant, subjective, sensory and
emotional experience associated with actual or
potential tissue damage or described in terms of
such damage.
PATHOPHYSIOLOGY
• Nociceptors are pain sensing nerve cells. It can be
located in either somatic or visceral
• It will propagate the information to the spinal cord
nearby.
• The action potential will trigger the release pain
neuro transmitter (substance P).
• It will ascends to higher centers.
• The thalamus acts as relay station and passes the
impulses to the central structure.
• Body will modulate the pain.
• Endogenous opiate system will be activated.
• Neurotransmitter such as enkephalins, dynorphins and
B-endorphins will bind to the receptors mu, delta and
kappa
NEUROPATHIC PAIN
• It is resulted from nerve damage.
E.g.
• Postherpetic neuralgia
• Diabetic neuropathy
•Acute pain is usually a nociceptive but can be neuropathic
•Chronic pain can be both neuropathic and functional pain, for
example, a pain that persist after the healing of acute injury.
FUNCTIONAL PAIN
• It refers to the abnormal operation of the nervous
system
E.g.
• Fibromyalgia
• Irritable bowel syndrome
• Tension-type headache
CLINICAL MANIFESTATION
• Patient can have a distress from trauma or appear to
have no noticeable suffering
• Acute pain
• Sharp or dull, burning, shock-like tingling, shooting radiating,
fluctuating intensity, varying location and occur in a timely
relationship with an obvious noxious stimulus
• Acute pain can cause HTN, tachycardia, diaphoresis,
mydriasis and palor.
• Acute pain there is no relation with comorbid
conditions unlike chronic pain, there is.
• Chronic pain occurs without a timely relationship with a
noxious stimulus.
CLINICAL ASSESSMENT
• Patient medical history
• Pain Scale
• Blood Chemistry
• PQRST characteristic
• Medications
CLINICAL INTERVENTION
• Non-opioid agents
• Given for mild to moderate pain
• Fewer side effects (GI problems)
•Opioid agents
• Given for severe pain at high dose
• Partial agonist and antagonist- selectivity with the pain receptor site.
• Might exhibit allergic reaction.
• Intrathecal and epidural opioids are often
administered with patient controlled analgesia.
• Safe and effective (lesser side effect)
•Morphine is the first line agent for moderate to severe pain.
• Cause respiratory depression that why it’s given with naxolone
PARKINSON’S DISEASE
• Parkinson’s disease has highly characteristic
neuropathologic findings and clinical presentation.
This includes:
• Motor deficits
• Mental deterioration
PATHOPHYSIOLOGY
• 2 known hallmarks in the substantia nigra pars
compacta:
1. Loss of neurons
2. Presence of lewy bodies
- Alpha-synuclein
• The degree of nigrostriatal dopamine loss has good
correlation with loss and severity of motor function.
• The inhibition of thalmus is resulted from inactivation
of dopamine-1 and dopamine-2
• The tremor exhibited by the patients with Parkinson’s
disease is due to degeneration of nigrostriatal
dopamine neurons
COMPLICATION
• Thinking difficulties
• Cognitive problems such as dementia occurs in the later stage of
Parkinson’s disease.
•Depression and emotional changes
• In the very early stage, it is usually developed.
• Receiving treatment for depression can make it easier to handle the challenges of Parkinson’s
disease.
•Swallowing problems
• Saliva may accumulate in your mouth due to slowed swallowing, leading to drooling.
• Chewing and eating problems
• Late-stage of PD affects the muscle in the mouth which makes
chewing difficult.
• It can lead to choking and poor nutrition.
•Sleep problems and sleep disorders
• People with PD often frequently waking up throughout the night, waking early or falling asleep
during the day
•Bladder problems and constipation
CLINICAL MANIFESTATION
• Initial symptom is sensory but as the disease
progresses, there are one or more classic symptoms
presents.
• Resting tremor
• Rigidity
• Bradykinesia
• Postural instability
• Resting tremor
• Tremor is present mostly in the hands and sometimes
characterize as pin- rolling
•Muscle rigidity
• Increase muscle resistance to passive range of motion.
•Intellectual deterioration
CLINICAL ASSESSMENT
• Medical history
• Medication that maybe a drug-induced parkinsonism
• Neuroimaging
• SPECT scan aka Dopamine transporter (DAT Scan)
• Deep Brain Stimulation (DBS)
• Placing electrodes that are surgically implanted in the brain.
PHARMACOLOGIC INTERVENTION

• Carbidopa-Levodopa
• Levodapa protects the conversion of dopamine outside the brain.
•Carbidopa-Levodopa infusion
•It is administered through the feeding tube.
•Dopamine agonists
•Mimics the dopamine in the brain.
•MAO-B inhibitors
•Help prevents the breakdown of brain dopamine by inhibiting the
enzyme MAO B.
PHARMACOLOGIC INTERVENTION
• COMT inhibitors
• It prolongs the effect of levodopa therapy by blocking an
enzyme that breaks down dopamine
•Anticholinergics
• This medication help control the tremor associated with Parkinson’s disease.
•Amantadine
• Provide short-term relief of symptoms of mild, early-stage Parkinson’s disease.
SURGICAL PROCEDURES

• DBS
• Deep Brain stimulation-
implant electrodes into
specific part of the brain.
• The electric pulses from the
heart is send to the brain to
reduce Parkinson’s disease
symptoms.
NONPHARMACOLOGIC INTERVENTION

• Speech-language pathologist
• Diet
• Food high in fiber and drinking an adequate amount of
fluids can help prevent constipation
•Exercise
• Increase muscle strength, flexibility and balance
• Improve well-being and reduce depression or anxiety.
ALTERNATIVE INTERVENTION
• Massage
• Reduce muscle tension and promote relaxation
• Tai chi
• It is an ancient form of Chinese exercise, tai chi employs slow, flowing
motions that may improve flexibility, balance and muscle strength
• Yoga
• It can increase flexibility and balance.
• Alexander technique
• It focuses on the muscle posture, balance and thinking about how you use
muscles and reduce muscle tension.
• Meditation
• It can reflect and focus your mind on an idea or image.
• It may reduce stress and pain and improve your sense of well-being
• Pet therapy
• Increase the flexibility and movement and improve your emotional health.
Pop Quiz
1. It is the pain neurotransmitter?
2. Type of primary headache is bilateral and non-
pulsatile.
3. It focuses on muscle posture, balance and thinking
about how you use muscles and reduce muscle
tension.
4. T/F: Episodic headaches are acute conditions
5. Treatment Management for Parkinson’s Disease.
OUR LADY OF FATIMA
UNIVERSITY
College of Pharmacy

Pharmacotherapy of
Nervous System Disorders
(Alzheimer’s disease and Anxiety
Disorders)

CLINICAL PHARMACY &


PHARMACOTHERAPEUTICS 1
(PHMA 411)
ALZHEIMER'S DISEASE

• It is a progressive disorder that causes brain cells to


waste away or degenerate.
• It is the common cause of dementia.
• The early signs of the disease will develop severe
memory impairment and lose ability to carry out
everyday task
• Current medications may temporarily improve
symptoms or slow down the rate of decline.
• There is no treatment that cures Alzheimer’s
disease
SYMPTOMS

• Making Judgements and decisions


• Uncharacteristic choices in social interactions
• Ineffective response to everyday problem.
• Changes in personality and behavior

• Depression Apathy
• Social Withdrawal Mood swings
• Distrusts in other Irritability
• Changes in sleeping habits Wandering
• Loss of inhibitions Delusions
• Aggressiveness
CAUSES
• Genetic
• Appears in middle ages.
• Lifestyle
• Drug misuse
• Environmental factors
• Stress
CAUSES
PROTEINS:
1. Plaque – Beta amyloid, a large protein leftover
fragment which disrupts the cell to cell
communications
2. Tangles – Tau proteins, that initiates the neuron’s
transport system of nutrients and essential
materials are being deformed and cause
“Neurofibrillary tangles” that causes disrupts the
transport system.
RISK FACTORS
• Age
• Family history and
genetics
• Down Syndrome
• Sex
• Mild cognitive
impairment
• Past head trauma
• Poor sleep patterns
• Lifestyle and Heart
health
COMPLICATIONS
• Can’t communicate the pain experiencing
• Can’t report symptoms of another illness
• Can’t follow prescribed treatment plan
• Can’t notice side effects
DISEASE PROGRESSION SYMPTOMS

• Swallowing
• Balance Control
• Bowel Control
• Bladder Control
• Bedsores
• malnutrition
PREVENTION
• Exercise regularly
• Eat Healthy
• Follow prescribed guidelines for disease
management
• Quit smoking
DIAGNOSIS
• Self –reporting
• Laboratory and imaging test
• Thyroid hormone levels and vitamin deficiencies
• MRI – may show possible brain shrinkage
• CT – rule out strokes, tumors and head injuries
• Physical and Neurological exam
• Motor skills and cognitive function
TREATMENT
• PHARMACOLOGICAL INTERVENTION
• Cholinesterase inhibitors
• Boost Cell to cell communication
• Memantine (Namenda)
• Slows the progression
• Anti depressants
• To control behavioral symptoms
SUPPORT AND MANAGEMENT
• Established routines
• Minimizing memory-demanding task
• Use daily checklist
• Regular doctor appointments
• Remove excess furniture
• Install sturdy handrails
• Reduce number of mirrors
• Keep photographs and other meaningful objects.
ALTERNATIVE MEDICINE
• Rich in Omega 3 fatty acid food
• Curcumin
• Gingko
• Vitamin E
ANXIETY DISORDER
• Experiencing occasional anxiety is
a normal part of life.
• However, people with anxiety
disorders frequently have intense,
excessive and persistent worry
and fear about everyday
situations.
• You may avoid places or situations
to prevent theses feelings.
• Symptoms may start during
childhood or the teen years and
continue into adulthood.
TYPES OF ANXIETY

• AGORAPHOBIA
• Fear and often avoid places or situations that feel
trapped, helpless or embarrassed.
• ANXIETY DISORDER DUE TO MEDICAL CONDITION
• Caused by physical health problem.
• GENERALIZED ANXIETY DISORDER
• Persistent and excessive anxiety and worry about
activities or events – event ordinary or routine issues.
TYPES OF ANXIETY

• PANIC DISORDER
• Repeated episodes of sudden feelings of intense anxiety
and fear or terror that reach a peak within minutes.
• SELECTIVE MUTISM
• Failure of children to speak in certain situation.
• SEPARATION ANXIETY DISORDER
• Related to separation from parents or others who have
parental roles.
TYPES OF ANXIETY
• SOCIAL ANXIETY DISORDER
• Due to feelings of embarrassment, self-consciousness and
concern about being judged or viewed negatively by others.
• SPECIFIC PHOBIAS
• Exposed to a specific object or situation and desire to avoid it.
Phobias provoke panic attacks in some people.
• SUBSTANCE-INDUCED ANXIETY DISORDER
• Intense anxiety or panic that are a direct result of misusing
drugs.
• Other specified anxiety disorder and unspecified
anxiety disorder
• Terms for anxiety or phobias
When to see a Doctor
• You feel like you’re worrying too much
• Your fear, worry or anxiety is upsetting to you and
difficult to control
• You feel depressed, have trouble with alcohol or
drug use, or have other mental health concerns
along with anxiety
• You have suicidal thought or behavior

NOTE: Worries may not go away on their own and


they may get worse over time if you don’t seek help
CAUSES
• UNDERLYING MEDICAL ISSUES:
• Heart Disease
• Diabetes
• Thyroid problems (Hyperthyroidism)
• Respiratory Disorders (COPD and asthma)
• Drug misuse or withdrawal
• Alcohol withdrawal or medication withdrawal
• Chronic pain or IBS
• Rare tumors that produce certain fight or flight
hormones
RISK FACTORS
• Trauma
• Stress due to illness
• Stressful situation
• Personality
• Other mental health disorders
• Relatives with an anxiety disorder
• Drugs or alcohol
COMPLICATION
• Depression
• Substance misuse
• Trouble sleeping
• Digestive or bowel problems
• Social isolation
• Problems functioning at school or work
• Poor quality of life
• Suicide
PREVENTION
It has no way to predict the development of anxiety, but these
are the steps to reduce the impact of symptoms
• Get help early
• Stay active
• Avoid alcohol or use of drugs
CLINICAL DIAGNOSIS

• Give you a psychological evaluation


• Discussion of thoughts, feelings and behavior to help
pinpoint a diagnosis and check for related complications.
• Compare your symptoms to the criteria in the DSM-5
• Diagnostic and Statistical Manual of Mental Disorder
(DSM-5) published by American Psychiatric Association
PHARMACOLOGIC TREATMENT
•Psychotherapy
•Cognitive behavioral therapy (CBT)
•Medication
•Anti depressants
•Anti anxiety medication
NON PHARMACOLOGIC TREATMENT

• Keep physically active


• Avoid alcohol and recreational drugs
• Quit smoking and caffeinated beverages
• Stress management and relaxation techniques.
• Sleep a priority
• Eat healthy
NON PHARMACOLOGIC INTERVENTION

• Be informed to your disorder


• Stick to your treatment plan
• Take action and practice strategies
• Keep a journal
• Support group
• Time management techniques
• Socialize
• Break the cycle
Pop Quiz
1. Fear and often avoid places or situations that feel
trapped, helpless or embarrassed.

2. A large protein leftover fragment which disrupts


the cell to cell communications.

3. Intense anxiety or panic that are a direct result of


misusing drugs.
OUR LADY OF FATIMA
UNIVERSITY
College of Pharmacy

Pharmacotherapy of
Nervous System Disorders
(Bipolar Disorder, Major Depressive
Disorder and Schizophrenia)

CLINICAL PHARMACY &


PHARMACOTHERAPEUTICS 1
(PHMA 411)
BIPOLAR DISORDER
• Also known as Manic depression
• Severe high and low moods and changes in sleep,
energy, thinking and behavior
• These feelings involves irritability and impulsive or
reckless decision making
TREATMENT
• Medication
• Mood Stabilizers
• Antipsychotics
• Antidepressants
• Anti anxiety medications
• Continued therapy sessions
• Substance abuse treatment
• Hospitalization
• Finding right medication
• This requires patience, your doctor will help you to identify which medication
perfectly works for you.
• Side effects
• Stop taking it when you experiencing symptoms that worsens the condition.
• Medications and pregnancy
• Psychotherapy
• Family, group or individual settings
HOME REMEDIES
• Quit drinking or using recreational drugs
• Form healthy relationships
• Create a healthy routine
• Check first before taking other medication
• Don’t stop taking medication and therapy sessions
• Be honest
MAJOR DEPRESSIVE DISORDER
• A constant sense of hopelessness.
• May feel difficulty to work, study, eat and enjoy
friends and activities
• Depression is ISN’T WEAKNESS.
SYMPTOMS
• People typically have multiple episodes
• Feelings of sadness, tearfulness, emptiness or
hopelessness
• Angry outbursts, irritability or frustration, even over
small matters
• Loss of interest or pleasure in most or all normal
activities, such as sex, hobbies or sports
• Sleep disturbances, including insomnia or sleeping too
much
• Tiredness and lack of energy, so even small tasks take
extra effort
SYMPTOMS
• People typically have multiple episodes
• Reduced appetite and weight loss or increased cravings for
food and weight gain
• Anxiety, agitation or restlessness
• Slowed thinking, speaking or body movements
• Feelings of worthlessness or guilt, fixating on past failures or
self-blame
• Trouble thinking, concentrating, making decisions and
remembering things
• Frequent or recurrent thoughts of death, suicidal thoughts,
suicide attempts or suicide
• Unexplained physical problems, such as back pain or
headaches
SYMPTOMS
• People typically have multiple episodes
• In younger children, symptoms of depression may
include sadness, irritability, clinginess, worry, aches and
pains, refusing to go to school, or being underweight.

• In teens, symptoms may include sadness, irritability,


feeling negative and worthless, anger, poor performance
or poor attendance at school, feeling misunderstood and
extremely sensitive, using recreational drugs or alcohol,
eating or sleeping too much, self-harm, loss of interest
in normal activities, and avoidance of social interaction.
RISK FACTORS
• Points that triggers depressions.
• Certain personality traits, such as low self-esteem and being
too dependent, self-critical or pessimistic
• Traumatic or stressful events, such as physical or sexual
abuse, the death or loss of a loved one, a difficult
relationship, or financial problems
• Blood relatives with a history of depression, bipolar disorder,
alcoholism or suicide
• Being lesbian, gay, bisexual or transgender, or having
variations in the development of genital organs that aren't
clearly male or female (intersex) in an unsupportive situation
• History of other mental health disorders, such as anxiety
disorder, eating disorders or post-traumatic stress disorder
RISK FACTORS
• Points that triggers depressions.
• Abuse of alcohol or recreational drugs
• Serious or chronic illness, including cancer, stroke,
chronic pain or heart disease
• Certain medications, such as some high blood pressure
medications or sleeping pills (talk to your doctor before
stopping any medication)
COMPLICATIONS
• Excess weight or obesity, which can lead to heart
disease and diabetes
• Pain or physical illness
• Alcohol or drug misuse
• Anxiety, panic disorder or social phobia
• Family conflicts, relationship difficulties, and work or
school problems
• Social isolation
• Suicidal feelings, suicide attempts or suicide
• Self-mutilation, such as cutting
• Premature death from medical conditions
PREVENTION
• Take steps to control stress
• Reach out to family and friends
• Get treatment at the earliest sign of a problem
• Consider getting long term maintenance treatment.
SCHIZOPHRENIA
• Schizophrenia may result in some combination of
hallucinations, delusions, and extremely disordered
thinking and behavior that impairs daily functioning
and can be disabling.
SYMPTOMS
• Delusions
• Hallucinations
• Disorganized thinking (speech)
• Extremely disorganized or abnormal motor
behavior
• Negative symptoms
WHEN TO SEE A DOCTOR
• People with schizophrenia often lack awareness
that their difficulties stem from a mental disorder
that requires medical attention. So it often falls to
family or friends to get them help.
SUICIDAL THOUGHTS AND BEHAVIOR

• Make sure someone stays with that person


• Call local emergency number
• Take the person to the nearest hospital
CAUSES
• Genetics
• Brain chemistry
• Dopamine
• Glutamate
• Environment
RISK FACTORS
• Family history
• Some pregnancy and birth complications
• Taking mind-altering drugs
COMPLICATIONS
• Suicide, suicide attempts and thoughts of suicide
• Anxiety disorders and obsessive-compulsive disorder
(OCD)
• Depression
• Abuse of alcohol or other drugs, including nicotine
• Inability to work or attend school
• Financial problems and homelessness
• Social isolation
• Health and medical problems
• Being victimized
• Aggressive behavior, although it's uncommon
DIAGNOSIS
• Physical exam
• Tests and screenings
• Psychiatric evaluation
• Diagnostic criteria for schizophrenia
TREATMENT
• Medications:
• 2nd generation Antipsychotic
• Long-acting injectable antipsychotics
• Psychosocial interventions
• Individual therapy
• Social skills training
• Family therapy
• Rehabilitation and support employment
COPING AND SUPPORT
• Learn about schizophrenia
• Stay focused on goals
• Avoid alcohol and drug use
• Ask about social services assistance
• Learn relaxation and stress management
• Join a support group
OUR LADY OF FATIMA
UNIVERSITY
College of Pharmacy

Pharmacotherapy of
Nervous System Disorders
(Sleep Disorders, Substance-related
disorders and Eating Disorders)

CLINICAL PHARMACY &


PHARMACOTHERAPEUTICS 1
(PHMA 411)
SLEEP DISORDER
• Sleep disorders are a group of conditions that
affect the ability to sleep well on a regular
basis.
• Some of the signs and symptoms of sleep
disorders include excessive daytime
sleepiness, irregular breathing or increased
movement during sleep. Other signs and
symptoms include an irregular sleep and
wake cycle and difficulty falling asleep.

Our Lady of Fatima University


College of Pharmacy
TYPES OF SLEEP
DISORDER

TYPES OF INSOMIA
• Chronic, which is when insomnia happens on a
regular basis for at least one month
• Intermittent, which is when insomnia occurs
periodically
• Transient, which is when insomnia lasts for just a few
nights at a time

Our Lady of Fatima University


College of Pharmacy
TYPES OF SLEEP
DISORDER

SLEEP APNEA
• Pausesin breathing during sleep.
• Causes less oxygen causing to wake up at night.

• TYPES OF SLEEP APNEA


1. Parasomnias – abnormal movements and
behaviors during sleep.
Ex: Sleepwalk, sleeptalk, g roaning,
nightmares,bedwetting, teeth grinding

Our Lady of Fatima University


College of Pharmacy
TYPES OF SLEEP
DISORDER
RESTLESS LEG SYNDROME
• Overwhelming need to move the legs.
• This accompanied by tingling sensation in the
legs.
NARCOLEPSY
• Sleep attacks
• Feel suddenly extremely tired and fall asleep
without warning
• It causes sleep paralysis

Our Lady of Fatima University


College of Pharmacy
SYMPTOMS

• Difficulty falling or staying asleep


• Daytime fatigue
• Strong urge to take naps during the day
• Irritability or anxiety
• Lack of concentration
• Depression

Our Lady of Fatima University


College of Pharmacy
DIAGNOSIS
• Polysomnography–evaluates oxygen levels,
body movements and brain waves.

• Electroencephalogram– assesses
electrical activity in the brain.

• Genetic Blood testing– other underlying


health conditions that might be causing
sleeping Our
problems
Lady of Fatima University
College of Pharmacy
TREATMENT

Medical treatments:
• Sleeping pills
• Melatonin supplements
• Allergy or cold medication
• Medications for any underlying health issues
• Breathing device or surgery (usually for sleep
apnea)
• A dental guard (usually for teeth grinding)

Our Lady of Fatima University


College of Pharmacy
TREATMENT
Lifestyle Changes
• Incorporating more vegetables and fish into your diet,
and reducing sugar intake reducing stress and anxiety
by exercising
• Creating and sticking to a regular sleeping schedule
• Drinking less water before bedtime
• Limiting your caffeine intake, especially in the late
afternoon or evening
• Decreasing tobacco and alcohol use
• Eating smaller low carbohydrate meals before
bedtime
Our Lady of Fatima University
College of Pharmacy
SUBSTANCE-
RELATED ABUSE
• A direct activate the brain’s reward system causes feeling of
pleasure
• 10 different classes of drugs causes addiction
• Alcohol morphine, oxycodone)
• Caffeine • Sedatives, hypnotics, and
• Cannabis anxiolytics (eg, lorazepam,
• Hallucinogens (eg, LSD, secobarbital)
phencyclidine, psilocybin) • Stimulants (eg, amphetamines,
• Inhalants (volatile cocaine)
hydrocarbons [eg, paint • Tobacco
thinner, certain glues]) • Other (eg, anabolic steroids)
• Opioids (eg, fentanyl,
SYPMTOMS
• Feelingthat you have to use the drug regularly — daily or even several
times a day
• Having intense urges for the drug that block out any other thoughts

• Over time, needing more of the drug to get the same effect

• Taking larger amounts of the drug over a longer period of time than you
intended
• Making certain that you maintain a supply of the drug
• Spending money on the drug, even though you can't afford it
• Not meeting obligations and work responsibilities, or cutting back on social
or recreational activities because of drug use
• Continuing to use the drug, even though you know it's causing problems in
your life or causing you physical or psychological harm
• Doing things to get the drug that you normally wouldn't do, such as stealing
Our Lady of Fatima University
• Driving or doing other risky activities when you're under the influence of
the drug College of Pharmacy
SYPMTOMS

• Spending a good deal of time getting the


drug, using the drug or recovering from the
effects of the drug
• Failing in your attempts to stop using the
drug
• Experiencing withdrawal symptoms when
you attempt to stop taking the drug

Our Lady of Fatima University


College of Pharmacy
SIGNS OF DRUG
INTOXICATION

MARIJUANA
• Euphoria
• Heightened senses
• Increased BP and HR
• Red eyes and dry mouth
• Decreased coordination and slow reaction time
• Exaggerated cravings for certain foods at
unusual times.

Our Lady of Fatima University


College of Pharmacy
SIGNS OF DRUG
INTOXICATION

K2, SPICE AND BATH SALTS


• Euphoria
• Elevated mood
• Altered senses
• Paranoia and hallucination
• Increased heart rate, BP or HR
• Confusion

Our Lady of Fatima University


College of Pharmacy
SIGNS OF DRUG
INTOXICATION

BARBITURATES, BZD ABD HYPNOTICS


• Drowsiness and slurred speech
• Lack of coordination
• Changes in mood
• Problems in concentration
• Lack of inhibition
• Slowed breathing
• Dizziness

Our Lady of Fatima University


College of Pharmacy
SIGNS OF DRUG
INTOXICATION
METH, COCAINE AND OTHER STIMULANTS
• Excess confidence
• Increased alertness and restlessness
• Behavioral changes, Irritability
• Rambling speech, dilated pupils
• Nasal congestion, mouth sores and tooth decay
• Insomnia

Our Lady of Fatima University


College of Pharmacy
SIGNS OF DRUG
INTOXICATION

CLUB DRUGS
• Hallucinations, memory problems
• Paranoia, reduced inhibitions, poor judgements
• Dilated pupils
• Chills and sweating
• Reduced consciousness
• Heightened sense

Our Lady of Fatima University


College of Pharmacy
SIGNS OF DRUG
INTOXICATION

HALLUCINOGENS (LSD and PCP)


• Hallucinations
• Impulsive behavior
• Reduced perception of reality
• Rapid shifts of emotion
• Rapid HR and HBP
• Tremors and Flashbacks
• Feeling of separation body to surroundings
• Lack of pain sensation

Our Lady of Fatima University


College of Pharmacy
SIGNS OF DRUG
INTOXICATION

HALLUCINOGENS (LSD and PCP)


• Aggresiveness
• Problems with speaking, thinking and memory
• Impaired judgement
• Intolerance to loud noise
• Sometimes seizures or coma.

Our Lady of Fatima University


College of Pharmacy
SIGNS OF DRUG
INTOXICATION

INHALANTS
• Euphoria or intoxication
• Decreased Inhibition
• Combativeness
• Dizziness, nausea and vomiting
• Involuntary eye movements and slurred speech
• Irregular heartbeats
• Treamors
• Rash around nose and mouth

Our Lady of Fatima University


College of Pharmacy
SIGNS OF DRUG
INTOXICATION

OPIOID PAINKILLERS
• Reduced sense of pain
• Agitation, drowsiness or sedation
• Slurred speech
• Constricted pupils
• Problems with coordination
• Constipation
• Runny nose or nose sores
• Needle marks

Our Lady of Fatima University


College of Pharmacy
CAUSES

• Environment
• Family and friend beliefs
• Exposure to peer group

• Genetics
• Inherited genetic traits

Our Lady of Fatima University


College of Pharmacy
RISK FACTORS

• Family history of addiction


• Mental health disorder
• Peer pressure
• Lack of family involvement
• Early use
• Taking a highly addictive drug

Our Lady of Fatima University


College of Pharmacy
COMPLICATION

• Can cause psychotic behavior, seizures or


death
• Coma or death when taken with alcohol
• MDMA can damage the brain
• May develop brain damage of different
levels of severity

Our Lady of Fatima University


College of Pharmacy
PREVENTION
• Communicate
• Listen
• Set a good example
• Strengthen the bond
• Stick with the treatment plan
• Avoid high-risk situations
• Get help

Our Lady of Fatima University


College of Pharmacy
EATING DISORDER

• Persistent eating behavior that negatively


impact your health, your emotion and
ability to function in important areas of life

Our Lady of Fatima University


College of Pharmacy
ANOREXIA
NERVOSA
• Characterized by abnormally low body weight,
intense fear of gaining weight and distorted
perception of weight or shape.
• They excessively limit calories or use other
methods to lose weight.

Our Lady of Fatima University


College of Pharmacy
SYMPTOMS

PHYSICAL SYMPTOM
• Extreme weighth loss
• Thin appearance
• Abnormal Blood counts
• Fatigue
• Insomnia
• Dizziness
• Bluish discoloration

Our Lady of Fatima University


College of Pharmacy
SYMPTOMS

PHYSICAL SYMPTOM
• Hairbreaks or fall out
• Absence of menstruation
• Constipation and abdominal pain
• Dry or yellowish skin
• Low Blood pressure
• Swelling arms or legs

Our Lady of Fatima University


College of Pharmacy
SYMPTOMS

EMOTIONAL AND BEHAVIORAL


SYMPTOMS
• Severe diet restrictions
• Exercise excessively
• Self-induced vomiting
• Denial of hunger
• Flat mood
• Social withdrawal

Our Lady of Fatima University


College of Pharmacy
CAUSES

Biological
• Genetic tendency toward perfectionism,
sensitivity and perseverance
Psychological
•O C personality that make stick to strict diet.
Environmental
• Peer pressure

Our Lady of Fatima University


College of Pharmacy
RISK FACTORS

Genetics
• First degree relative who had same disorder
Dieting and starvation
• Starvation affects the brain and mood
Transitions
• New school, home or job, a relationship breakup
or death or illness

Our Lady of Fatima University


College of Pharmacy
COMPLICATIONS

• Anemia
• Heart, Bone and Muscle problems
• GI and kidney problems
• Electrolyte deficiencies
• In Female, absence of period
• In Male, decreased testosterone

Our Lady of Fatima University


College of Pharmacy
PREVENTION

STEPS:
• Identify the early signs
• Ask questions
• Observe
• Habit and satisfaction about eating habits

• Routine medical appointments


• Listen and care for them

Our Lady of Fatima University


College of Pharmacy
DIAGNOSIS
• Physical exam
• BMI
• Vital signs
• Lab test
•CBC and other more specialized blood tests
• Psychological evaluation
• Thoughts, feeling and eating habits
• Other studies
• Bone density, heart and lung problems

Our Lady of Fatima University


College of Pharmacy
TREATMENT

• Hospitalization
• For medical complications
• Medical care
• Feeding tubes
• Restoring a healthy weight
• Medical doctors, dietitian and other healthcare
professional

Our Lady of Fatima University


College of Pharmacy
TREATMENT

• Psychotherapy
• Family-based therapy
• Individual therapy

• CHALLENGES:
• Thinking you don't need treatment
• Fearing weight gain
• Not seeing anorexia as an illness but rather a
lifestyle choice

Our Lady of Fatima University


College of Pharmacy
REFERENCES
• http://www.healthline.com/health/epilepsy
• https://www.nhs.uk/conditions/epilepsy/diagnosis/
• https://www.webmd.com/epilepsy/status-epilepticus
• https://www.epilepsy.com/learn/challenges-epilepsy/seizure-emergencies/status-epilepticus
• https://www.healthline.com/health/status-epilepticus#takeaway
• https://www.ncbi.nlm.nih.gov/pmc/articles/PMC5747306/
• https://emedicine.medscape.com/article/1163653-overview#a1
• https://www.mayoclinic.org/diseases-conditions/sleep- disorders/symptoms-causes/syc-20354018
• https://www.healthline.com/health/sleep/disorders
• https://www.mayoclinic.org/diseases-conditions/drug- addiction/symptoms-causes/syc-20365112
• https://www.msdmanuals.com/professional/psychiatric-disorders/substance-related-disorders/overview-of-
substance-related-disorders
• https://www.mayoclinic.org/diseases-conditions/anorexia-
• nervosa/diagnosis-treatment/drc-20353597
OUR LADY OF FATIMA
UNIVERSITY
College of Pharmacy

Clinical Pharmacy and


Pharmacotherapeutics 1
(PSMA 411)
CHECKLIST
• Concepts of Pharmacotherapy Care Planning
• Documentation of Pharmacotherapy Interventions
• Pharmacotherapy of Hematologic Disorders
• Pharmacotherapy of Coagulation Disorders
• Pharmacotherapy of Nervous System Disorders
• Pharmacotherapy of Cardiovascular Disorders
• Pharmacotherapy of Renal Disorders
• Pharmacotherapy of Autonomic System Disorders
UNIT OUTCOMES
1. Demonstrate appreciation of pharmacy as a health profession and the scope of
practice.

2. Demonstrate understanding of chemical, physical, and biological processes relevant


to pharmaceutical sciences.

3. Recognize the importance of the healthcare system and its components,


including the importance of pharmacovigilance.

4. Demonstrate understanding of morphology, anatomy, physiology, taxonomy, and


distinct features of plant species of pharmaceutical importance.

5. Relates ethical practices to administration, management and leadership in


pharmacy
6. Identify the different elements or components in a pharmacy operation while
practicing good management skills.

7. Demonstrate understanding of the relationship among different levels of body


functions from molecular to systemic level in normal conditions and diseased
states, including the impact of immunization in vaccine- preventable diseases
TOPIC OUTLINE

• Concepts of Pharmacotherapy Care Planning


• Documentation of Pharmacotherapy Interventions
• Pharmacotherapy of Hematologic Disorders
• Pharmacotherapy of Coagulation Disorders
• Pharmacotherapy of Nervous System Disorders
• Pharmacotherapy of Cardiovascular Disorders
• Pharmacotherapy of Renal Disorders
• Pharmacotherapy of Autonomic System Disorders
OUR LADY OF FATIMA
UNIVERSITY
College of Pharmacy

PHARMACOTHERAPY OF
CARDIOVASCULAR
DISORDERS
CLINICAL PHARMACY &
PHARMACOTHERAPEUTICS 1
(PSMA411)
UNIT OUTLINE:
I. ACUTE CORONARY SYNDROME
II. ARRYTHMIA
III. CARDIOPULMONARY ARREST
IV. HEART FAILURE
V. HYPERTENSION
VI. ISCHEMIC HEART DISEASE
VII. SHOCK
VIII. STROKE
IX. VENOUS THROMBOEMBOLISM
A name given to three types of coronary
artery disease that are associated with
sudden rupture of plaque inside the
coronary artery.
Acute
Coronary
The location of the blockage, the length of
Syndrome time that blood flow is blocked and the
amount of damage that occurs determines
the type of acute coronary syndrome.
These life-threatening conditions require
emergency medical care.
Unstable angina

Types of Acute Non-ST segment elevation


Coronary myocardial infarction or heart
attack (NSTEMI)
Syndrome
ST segment elevation
myocardial infarction or heart
attack (STEMI).
• Aging
• High blood pressure
• High blood cholesterol
Risk factors • Cigarette smoking
• Lack of physical activity
• Unhealthy diet
• Obesity or overweight
• Diabetes
• Family history of chest pain, heart disease or stroke
• History of high blood pressure, preeclampsia or
diabetes during pregnancy
Electrocardiogram (ECG)

Blood tests
Diagnosis
Coronary angiogram

Echocardiogram

Myocardial perfusion imaging

Computerized tomography (CT) angiogram

Stress test
ANGINA

• A chest pain or
discomfort caused
when the heart muscle
doesn't get enough
oxygen-rich blood.
1.Stable (classic)
angina
•most common form, has a
Types of more predictable pattern,
angina which is brought on by
exertion, emotional stress,
or a heavy meal , which is
usually relieved by rest ,
nitroglycerin.
2. Unstable angina

• Angina is considered unstable and requires


further evaluation if patients experience
• (1) Rest angina, which usually is prolonged > 20
Types of min occurring within a week of presentation
• (2) Severe new-onset angina refers to angina
angina with 2 months onset of initial
presentation
• (3) Increasing angina refers to previously
diagnosed angina that is distinctly more
frequent, longer in duration, or lower in
threshold
• (4) Decreased response to rest or nitroglycerin
3. Angina decubitus
(nocturnal angina)
• This angina occurs in the
Types of recumbent position and is not
specifically related to either rest
angina or exertion.
• Nitrates such as nitroglycerin
may relieve the paroxysmal
nocturnal dyspnea associated
with angina decubitus
Types of angina

4. Prinzmetal's angina (vasospastic or variant angina)


• Coronary artery spasm that reduces blood flow precipitates .
• artery already has a fixed obstruction owing to thrombi or plaque
format ion.
• It usually occurs at rest (i .e., pain may disrupt sleep) rather than with
exertion or emotional stress
• Calcium-channel blockers, rather than β-blockers, are most effective for this
form of angina. Nitroglycerin may not provide relief , depending on the
cause of vasospasm.
Mechanism of action. These agents inhibit the
enzyme HMG-CoA and reduce cholesterol product
ion.

Statins or Indications. These agents are effective in lowering


LDL levels, while increasing HDL levels and lowering
HMG-CoA triglyceride levels. They are primarily used to lower
LDL cholesterol levels.
reductase • Currently available agents include the following:
inhibitors • Atorvastatin (Lipitor) : 10-80 mg by mouth daily
• Fluvastatin (Lescol ) : 20-80 mg by mouth at bedt ime
• Lovastatin (Mevacor , various) : 10-80 mg by mouth in the
evening
• Pravastatin (Pravachol ) : 10-80 mg by mouth daily
• Rosuvastatin (Crestor ) : 5-40 mg by mouth daily
• Simvastatin (Zocor , var ious): 5-80 mg by mouth in the
evening
Statins or HMG-CoA reductase inhibitors

Adverse effects are:


• Headache and dyspepsia
• These agents can elevate liver function tests alanine amino transferase
(ALT) and aspartate aminotransferase (AST)
• incidence of myopathy
Intravenous nitroglycerine is
used in the immediate
used in treatment of stable
treatment of unstable
angina
angina and is used for long-
term therapeutic relief .

Nitrates used in
more effective than nitrates
Nitrates combination with β-
adrenergic blockers have
or β-adrenergic blockers
used alone
been shown to be

Precautions and monitoring effects


• Blood pressure and hear t rate should be monitored because all
nitrates can increase heart rate while lowering blood pressure
• Nitrate- induced headaches are the most common side effect
MOA: β-Blockers reduce oxygen demand, both at rest and
during exertion, by decreasing the heart rate and
myocardial contractility, which also decreases arterial
blood pressure

β-Blockers should be avoided in Prinzmetal angina (caused


Beta by coronary vasospasm) because they increase coronary
resistance and may induce vasospasm.

adrenergic
blockers (3) Asthma is a relative contraindication because all β-
blockers increase airway resistance and have the potential
to induce bronchospasm in susceptible patients

Patients with diabetes and others predisposed to


hypoglycemia should be warned that β-blockers mask
tachycardia, which is a key sign of developing hypoglycemia
MOA: Two actions are most pertinent in the
treatment of angina.

• These agents prevent and reverse coronary spasm by inhibiting


calcium influx into vascular smooth muscle and myocardial muscle.

Calcium- This results in increased blood flow, which enhances myocardial


oxygen supply.

channel Calcium-channel blockers decrease coronary vascular


blockers resistance and increase coronary blood flow, resulting
in increased oxygen supply.

Calcium-channel blockers decrease systemic vascular


resistance and arterial pressure; in addition, they
decrease inotropic effects, resulting in decreased
myocardial oxygen demand.
(1) Calcium-channel blockers are used in
stable (exertional ) angina that is not cont
rolled by nitrates and β-blockers and in patients
for whom β-blocker therapy is inadvisable.

Combination therapy—with nitrates, β-


Indications blockers, or both—most effective.

(2) These agents, alone or with a nitrate, are


particularly valuable in the treatment of
Prinzmetal angina. They are considered the
drug of choice in treatment of angina at rest .
INTERVENTIONS
• administer oxygen
• monitor vital signs, status of cardiopulmonary
• monitor ECG
• place client in semi- to high-Fowler’s position
• minimize precipitating events: reduce stress and anxiety; avoid
overexertion and smoking; avoid extremes of temperature; and
maintain low cholesterol, low saturated fat diet and eat small, frequent
meals
• gradual increase in activities and exercise: regular exercise
MYOCARDIAL INFARCTION
The death of myocardial cells from
inadequate oxygenation, often caused by a
sudden complete blockage of a coronary
artery; characterized by localized formation of
necrosis (tissue destruction) with subsequent
healing by scar formation and fibrosis
MYOCARDIAL
INFARCTION
Risk factors: atherosclerosis, CAD, thrombus
formation, hypertension, diabetes mellitus
Signs & Symptoms
• Pain usually substernal with possible radiation to the neck, jaw, back,
and arms; severe, crushing, viselike with sudden onset; unrelieved by
rest or nitrates
• Nausea and vomiting
• Dyspnea
• Skin: cool, clammy, ashen
• Elevated temperature
• Initial increased in blood pressure and pulse, with gradual drop in
blood pressure
• Restlessness
MANAGEMENT
• administer oxygen as ordered to relieve dyspnea and prevent
arrhythmia
• monitor ECG and hemodynamic procedure
• assess cardiovascular/lung
• monitor urinary output
• drug therapy: pain relief (morphine), anticoagulants,
thrombolytic, blood thinners (ASA), antiplatelet drugs
(clopidogrel), NTG
Non-pharmacological management of ACS
• Eat a heart-healthy diet
• Don’t smoke
• Be active
• Check your cholesterol intake
• Control blood pressure
• Maintain a healthy weight
• Manage stress
• Monitor alcohol intake
CARDIAC
ARRYTHMIA

CLINICAL PHARMACY &


PHARMACOTHERAPEUTICS 1
(PSMA411)
60-100 beats per minute

Origin of the heartbeat should be


NORMAL from the SA node
HEART
All cardiac impulses should propagate
RHYTHM through normal conduction pathway

Impulses should have a normal


velocity
• Cardiac arryhthmias are deviations from the normal heart beat pattern
• These include:
• Abnormalities of impulse formation
• Heart rate
CARDIAC • Rhythm
• Site of impulse origin
ARRHYTHMIA • Conduction disturbances

• These disrupts the normal sequence of atrial and ventricular


activation
BRADYARRYTHMIA

• This refers to a slow heartbeat —


a resting heart rate less than 60
beats a minute.
CLASSIFICATION
TACHYARRYTHMIA

• This refers to a fast heartbeat — a


resting heart rate greater than
100 beats a minute.
Tachycardias Atrial fibrillation Atrial flutter

originating in
the atria
include:
Supraventricular Wolff-Parkinson-
tachycardia White syndrome
• A rapid heart rate caused by
chaotic electrical impulses in
the atria. These signals result
in rapid, uncoordinated, weak
contractions of the atria.
• The chaotic electrical signals
bombard the AV node, usually
resulting in an irregular, rapid
rhythm of the ventricles. Atrial
fibrillation may be temporary,
but some episodes won't end
unless treated.
• This is associated with serious
complications such as stroke.
Atrial Fibrillation
• Similar to atrial
fibrillation.
• The heartbeats in atrial
flutter are more-organized
and more-rhythmic
electrical impulses than in
atrial fibrillation.
• This may also lead to
serious complications
such as stroke.

Atrial flutter
• A broad term that includes
many forms of arrhythmia
originating above the
ventricles (supraventricular)
in the atria or AV node.
• This type of arrhythmia
seem to cause sudden
episodes of palpitations that
begin and end abruptly

Supraventricular tachycardia
• A type of supraventricular
tachycardia, there is an extra
electrical pathway between the
atria and the ventricles, which is
present at birth.
• This pathway may allow
electrical signals to pass
between the atria and the
ventricles without passing
through the AV node, leading to
short circuits and rapid
heartbeats.

Wolff-Parkinson-White syndrome
Ventricular tachycardia

Tachycardias
occurring in
the ventricles
include: A rapid, regular heart rate that originates with
abnormal electrical signals in the ventricles. The
rapid heart rate doesn't allow the ventricles to
fill and contract efficiently to pump enough
blood to the body.
Tachycardias occurring in the ventricles
include:
Ventricular fibrillation

This occurs when rapid, chaotic electrical impulses cause the ventricles to quiver
ineffectively instead of pumping necessary blood to the body. This serious
problem is fatal if the heart isn't restored to a normal rhythm within minutes.

Most people who experience ventricular fibrillation have an underlying heart


disease or have experienced serious trauma.
Long QT syndrome
Tachycardias • A heart disorder that carries an increased
risk of fast, chaotic heartbeats.
occurring in the • The rapid heartbeats, caused by changes
ventricles in the electrical system of your heart, may
lead to fainting, and can be life-
include: threatening.
• In some cases, heart's rhythm may be so
erratic that it can cause sudden death.
• Chest pain (angina)
Signs and symptoms • Nausea or vomiting
• Indigestion
• Shortness of breath (dyspnea)
• Sudden, heavy sweating
(diaphoresis)
• Lightheadedness, dizziness or
fainting
• Unusual or unexplained
fatigue
• Feeling restless or
apprehensive
• Bradycardia/ tachycardia
CAD

High blood pressure

Congenital heart disease

Risk Factors Thyroid problems

Diabetes

Obstructive sleep apnea

Electrolyte imbalance
a. Electrocardiogram (EKG or ECG)
b. Echocardiogram
c. Electrophysiology test
DIAGNOSTIC d. Stress test
TESTS e. Cardiac catheterization
f. Head-up tilt table test
DRUGS

• DISOPYRAMIDE, PROCAINAMIDE,
QUINIDINE

CLASS 1A ACTION:

• Binds to open and inactivated sodium


channels and prevents sodium influx,
thus slowing the upstroke of Phase 0,
also decreases the slope of Phase 4
depolarization
DRUGS

• LIDOCAINE, MEXILETINE,
PHENYTOIN

CLASS 1B ACTION

• Modest depression of conduction,


shortening of repolarization,
which decreases the action
potential
DRUGS

• FLECAINIDE, MORICIZINE,
PROPAFENONE

ACTION
CLASS 1C
• Strong depression of conduction,
with mild or no effect on
repolarization.
• Approved only for refractory
ventricular arrythmias
DRUGS

• PROPRANOLOL, ESMOLOL, ACEBUTALOL

ACTION
CLASS II • Diminish Phase 4 depolarization,
prolonging AV conduction and
decreasing heart rate and contractility
• Useful in treating tachyarrythmias
caused by increased sympathetic
activity
DRUGS
• AMIODARONE, SOTALOL,
IBUTILIDE, DOFETILIDE

CLASS III ACTION


• Prolongs the refractory period
and action potential, they have
no effect on myocardial
contractility and conduction
time
DRUGS
• VERAPAMIL AND DILTIAZEM

ACTION
CLASS IV • Inhibit AV node conduction by
depressing AV and SA nodes where
calcium channels predominate
• They decrease inward current
carried by calcium, resulting in a
decreased rate of Phase 4
Spontaneous depolarization.
UNCLASSIFIED ANTIARRYTHMICS

DRUGS ACTION
ATROPINE, ADENOSINE ATROPINE- anticholonergic blocks vagal
effects on the SA node, promoting
conduction through the AV node.
ADENOSINE- Slow conduction through
the AV node, interrupt reentry pathways
through the AV node.
Eating a heart-healthy diet

Staying physically active and keeping a healthy


weight

Non-
pharmacological Avoiding smoking

management
Limiting or avoiding caffeine and alcohol

Reducing stress, as intense stress and anger can


cause heart rhythm problems
Cardiopulmonary
Arrest

CLINICAL PHARMACY &


PHARMACOTHERAPEUTICS 1
(PSMA411)
Sudden cessation of heartbeat
and cardiac function, resulting in
the loss of effective circulation.

Absence of systole; failure of the ventricles of


Cardiopulmonary the heart to contract (usually caused by
ventricular fibrillation) with consequent
Arrest absence of the heart beat leading to oxygen
lack and eventually to death.

Cardiac arrest
Causes of Cardiopulmonary
• Coronary artery disease
arrest • Heart attack
• Enlarged heart
(cardiomyopathy)
• Valvular heart disease
• Congenital heart disease
• Electrical problems in
the heart
Pathophysiology
Family history of coronary artery disease

Smoking

High blood pressure

Risk factors High blood cholesterol

Obesity

Diabetes

A sedentary lifestyle
• A previous episode of cardiac arrest or a family
history of cardiac arrest
Other factors that • A previous heart attack
might increase • Age — the incidence of sudden cardiac arrest
your risk of increases with age
sudden cardiac • Being male
arrest include:
• Using illegal drugs, such as cocaine or
amphetamines
• Nutritional imbalance, such as low potassium or
magnesium levels
• Obstructive sleep apnea
• Chronic kidney disease
Complications

• When sudden cardiac arrest


occurs, reduced blood flow to
your brain causes
unconsciousness.
• If your heart rhythm doesn't
rapidly return to normal, brain
damage occurs and death
results.
• Survivors of cardiac arrest
might show signs of brain
damage
Sudden cardiac arrest Sometimes other signs
Signs & signs and symptoms are and symptoms occur
symptoms immediate and drastic before sudden cardiac
and include: arrest. These might
include:
• Sudden collapse
• No pulse • Chest discomfort
• No breathing • Shortness of breath
• Loss of consciousness • Weakness
• Palpitations
Clinical evaluation

Cardiac monitor and electrocardiography (ECG)

Echocardiography, chest x-ray or chest


ultrasonography
Diagnosis
Diagnosis of cardiac arrest is by clinical findings of:

• apnea
• pulselessness,
• Unconsciousness
• Arterial pressure is not measurable.
• Pupils dilate and become unreactive to light after several
minutes.
• A cardiac monitor should be applied; it may
indicate ventricular fibrillation (VF), ventricular
tachycardia (VT), or asystole.
Diagnosis • The patient is evaluated for potentially treatable
causes; a useful memory aid is "Hs and Ts":
• H:Hypoxia, hypovolemia, acidosis (hydrogen ion),
hyperkalemia
or hypokalemia, hypothermia, hypoglycemia.
• T:Tablet or toxin ingestion,
cardiac tamponade, tension
pneumothorax, thrombosis (pulmonary embolus
or myocardial infarction) or trauma
Treatment

• CPR
• Defibrillation
• Drugs to treat the
underlying causes of the
cardiac arrest
Drugs used during
resuscitation

Epinephrine (Adrenaline)
• First line cardiac arrest drug, given after every 3
minutes of CPR
• Dose 1mg (10ml of 1 in 10,000) IV
• Causes vasoconstriction, increased systemic
vascular resistance increasing cerebral and
coronary perfusion
• Increases myocardial excitability, when the
myocardium is hypoxic or ischaemic
Drugs used during
resuscitation

Atropine
• Given for asystole or pulseless electrical activity
with a rate less than 60 beats per minute
• A 3mg is given as a single intravenous dose
• It blocks the activity of the vagus nerve on the SA
and AV nodes, increasing sinus automaticity and
facilitating AV node conduction
Drugs used during
resuscitation

Amiodarone
• For Refractory VF/VT; haemodynamically stable VT
and other resistant tachyarrhythmias
• If VF or pulseless VT persists after the first 3
shocks then Amiodarone 300mg is considered.
• If not pre-diluted, must be diluted in 5% dextrose
to 20ml.
• Should be given centrally but in an emergency can
be given peripherally
• Increases the duration of the action potential in
the atrial and ventricular myocardium
Drugs used during
resuscitation

Magnesium Sulphate
• In the event of cardiac arrest, where the rhythm is
a pulseless ventricular
tachycardia, magnesium may be considered.
• The effects of magnesium may be due to several
mechanisms, including improved potassium
transport through myocardial potassium channels
and shortening of the action potential duration.
Drugs used during
resuscitation

Lidocaine
• One of the most important class-1b
antiarrhythmic drug.
• It is used intravenously for the treatment of
ventricular arrhythmias (for acute myocardial
infarction, digoxin poisoning, cardioversion,
or cardiac catheterization) if amiodarone is not
available or contraindicated.
Drugs used during
resuscitation

Sodium Bicarbonate
• Given for severe metabolic acidosis and
Hyperkalaemia

Calcium
• Administered when pulseless electrical activity
caused by: Hyperkalaemia Hypocalcaemia or
overdose of Calcium channel blocking drugs
HEART FAILURE
CLINICAL PHARMACY & PHARMACOTHERAPEUTICS 1
(PSMA411)
Occurs when the heart’s
delivery of blood is
inadequate for the needs
of the tissues.

HEART Occurs when the heart is


FAILURE ETIOLOGY: CAD and
hypertension
unable to pump sufficiently
to maintain blood flow to
meet the needs of the body.

A complex condition associated


with several symptoms arising
from defects in left ventricular
filling and/or emptying, including
shortness of breath and exertional
fatigue.
Mechanism of Myocardial
Muscle Contraction

Minoxidil
Diazoxide

Excess calcium is restored Calcium

CONTRACTION
Ejection Fraction Numbers
50-75% —Normal
36-49% —Below Normal
35% & Below —Low
Ejection Fraction
What it Means
Measurement
55-70% Normal
40-55% Below Normal
May confirm diagnosis of
Less than 40% heart failure
Patient may be at risk of
<35% life-threatening irregular
heartbeats
Primary Signs
and Symptoms
Associated with
all types of Heart
Failure
TYPES OF HEART FAILURE
Right-sided heart failure Left-sided heart failure Congestive heart failure

• Back-ups in the area that • Failure to properly • Fluid backs up into the
collects "used" blood) pump out blood to the lungs and tissues
body
Right sided heart failure
• The heart's pumping action moves
"used" blood that returns to the
heart through the veins through
the right atrium into the right
ventricle.
• The right ventricle then pumps the
blood back out of the heart into
the lungs to be replenished with
oxygen.
• This occurs as a result of left-sided
failure.
Left sided heart failure
• The heart's pumping action moves
oxygen-rich blood as it travels from
the lungs to the left atrium, then on
to the left ventricle, which pumps it
to the rest of the body.
• The left ventricle supplies most of the
heart's pumping power, so it's larger
than the other chambers and
essential for normal function.
• In left-sided or left ventricular (LV)
heart failure, the left side of the
heart must work harder to pump the
same amount of blood.
Two Types of Left sided Heart Failure

Occurs when the heart


Systolic left ventricular
contracts normally, but
dysfunction - the left
the ventricles do not
ventricle heart muscle
relax properly or are stiff
doesn't contract with
and less blood enters the
enough force
heart during normal filling
Left sided heart failure
Systolic Failure Diastolic Failure
• Reduced • Hypertrophy
mechanical • Stiffening
pumping • Loss of adequate
relaxation necessary
(contractility) in reducing filling
• Reduced ejection and CO
fraction (<45%) • Normal ejection
fraction
• Significantly reduced
stroke volume
Congestive heart failure
• As blood flow out of the heart
slows, blood returning to the heart
through the veins backs up, causing
congestion in the body's tissues.
Often swelling in the legs and
ankles, but it can happen in other
parts of the body, too.
• Sometimes fluid collects in the
lungs and interferes with breathing,
causing shortness of breath,
especially when a person is lying
down.
• This is called pulmonary edema and
if left untreated can cause
respiratory distress.
NYHA classification of functional status of the patientwith
heart failure
I No symptoms with ordinary physical activity (suchas walking or
climbing stairs)
II Slight limitation with dyspnea on moderate tosevere
exertion (climbing stairs or walkinguphill)
III Marked limitation of activity, less than ordinary activity causes dyspnea
(restricting walking distance and limiting climbing to one flight ofstairs)

IV Severe disability, dyspnea at rest (unable to carryon


physical activity without discomfort)
Common Tests for Heart Failure
• Physical examination
• Blood tests
• Chest X-ray
• Electrocardiogram (EKG or ECG)
• Echocardiography (abbreviated as
"echo")
• Exercise stress test
• Radionuclide ventriculography or
multiple-gated acquisition scanning
( MUGA)
• Cardiac catheterization
• Magnetic resonance imaging (MRI)
TREATMENT OF HEART FAILURE
• ACE inhibitors
• First line treatment for all types of heart failure
• Exert effects by reducing both the preload and
afterload on the heart, thereby increasing
cardiac output
• Diuretics
• Relieve pulmonary and peripheral edema by
increasing sodium reabsorption and chloride
excretion though blockade of sodium
reabsorption in the renal tubule
• ARB
• Beta blockers
• Digoxin
• Nitrates/ hydralazine
Common drug-drug interactions with prescribed heart failuremedication

DRUG INTERACTS WITH RESULT OF INTERACTION

Diuretic NSAIDS Decreased effect of diuretic Increased risk of


Carbamazepine Lithium hyponatremia Excretion of lithium impaired

ACE inhibitor or NSAIDS Antagonism of hypotensive effect; increased risk or renal


ARB impairment
Ciclosporin Lithium Increased risk of hyperkalemia
Diuretics Excretion of lithium impaired
Enhanced hypotensive effect; increased risk of
hyperkalemia with potassium-sparing drugs

Digoxin Amiodarone Propafenone Increased digoxin level (need to halve dose) Increased
Quinidine digoxin level (need to halve dose) Increased digoxin level
Verapamil (need to halve dose) Increased risk of AV block
Increased risk of hypokalemia and toxicity Increased
Diuretics
cardiac toxicity if hypokalemia present
Amphothericin
Common drug-drug interactions with prescribed heart failuremedication

Drug Interactswith Result of interaction

Nitrates Sildenafil Increased hypotensive effect Increased excretion of


Heparin heparin

Spironolactone Digoxin Spironolactone may interfere with measurement of digoxin


plasmalevels, resulting in inaccurate interpretation

Beta blocker Amiodarone Increased risk of bradycardia


Diltiazem Increased risk of AV block and bradycardia Increased risk of
Verapamil hypotension, heartfailure and asystole
Common drug-disease interactions with prescribed heart failure medication

Drug Concurrentdisease Potentialoutcome

Diuretic Prostatism Hyperuricemia Liver Urinary retention/ incontinence Exacerbation of gout


cirrhosis Encephalopathy

ACEinhibitor Renal artery stenosis Severe aortic Renal failure


stenosis Renal impairment hypotension Exacerbation of heart failure
Renal failure
Hypotension and cardiogenic shock

Beta blocker Asthma Bradyarrythmias Bronchoconstriction / resp. arrest Exacerbation of HF


Hypotension Further hypotension and cardiogenic
shock

Digoxin Bradyarrythmias Renalimpairment Exacerbation of heart failure Exacerbation of heart failure


and digoxin toxicity leading to cardiac arrythmias
Patient education and selfmonitoring

Drug Advice Comment


ACEinhibitors Improve symptoms Avoid standing Monitor for hypotension, dizziness, cough,
rapidly taste disturbance, sore throat, rashes,
tingling in hands, joint pain
Diuretics Will casuediuresis Timing of dose Monitor for incontinence,muscle weakness,
Flexible dosing confusion, dizziness, gout, unusual gain in
weight within a short time
Beta blockers Symptoms worsen Monitor for hypotension, dizziness,
initially headache, fatigue, GI disturbances,
Gradual increase in dose bradycardia
Cardiac glycosides Report toxic symptoms Monitor for signs orsymptoms of toxicity
Patient education and selfmonitoring

Drug Advice Comment


Nitrates Timing of dose Monitor for HA,hypotension,
Postural hypotension dizziness, flushing, GI upset
Avoid standing
rapidly
Potassium Administratino of Monitor for GI disturbances,
salts dose (soluble+non swallowing difficulty, diaarhea,
soluble) tiredness, limb weakness
Disorder of the
Cardiovascular
System
Hypertension

• High blood pressure


• It is not a disease but an important risk factor
for cardiovascular complications.
• It can be defined as a condition where blood
pressure is elevated to an extent where
clinical benefit is obtained from blood
pressure.
Blood
pressure
categories
Hypertension Complications
Complications of hypertension includes the
following:
• Myocardial infarction
• Stroke
• Hypertensive encephalopathy / malignant
hypertension
• Dissecting aortic aneurysm
• Hypertensive nephrosclerosis
• Peripheral vascular disease
Causes of Hypertension
Primary hypertension Secondary Hypertension
❑90-95% ❑5-10%
❑Essential hypertension ❑Renal disease
❑Etiology is unknown ❑Endocrine disease
❑Genetic factors clearly play a part ❑Vascular causes
❑Drugs (sympathomimetic amines,
estrogens, cyclosporine, erythropoietin,
NSAIDS, steroids)
REGULATION OF BLOOD PRESSURE
• The mean blood pressure is the product of
cardiac output and total peripheral
resistance.
• BP = CO ✕ TPR
• In hypertensive individuals, cardiac output is
not increased and high blood pressure arises
as a result of increased total peripheral
resistance caused by constriction of small
arterioles.
• Blood pressure is measured using a
Sphygmomanometer, a device
composed of an inflatable cuff to
restrict the blood flow, and a mercury
or mechanical manometer to measure
the pressure
• Manual (mercury &Aneroid
• Digital
• Hypertension itself causes no
symptoms
CLINICAL • Although headache may be present,
it is usually unclear if this is caused
PRESENTATION by hypertension or is an incidental
finding.
• Hypertension may also come to light
for first time when the individual
suffers a hypertension-related
complication such as MI or stroke.
HYPERTENSION
• Contributing factors
• Obesity
• Excess alcohol
• Salt intake
• Lack of exercise
• Other risk factors
• Smoking
• Diabetes
• Hyperlipidemia
Hypertensive Crisis ≥180 / ≥120 mmHg

Hypertensive Urgency Hypertensive Emergency

A hypertensive emergency, also known


A hypertensive urgency is a clinical
as malignant hypertension, is high
situation in which blood pressure is very
blood pressure with potentially life-
high with minimal or no symptoms,
threatening symptoms and signs
and NO signs or symptoms indicating
indicative of acute impairment of one or
acute organ damage.
more organ systems.
Also known as accelerated,
malignant hypertension,
hypertensive emergency

Uncommon condition characterized


MALIGNANT by greatly elevated blood pressure
associated with evidence of ongoing
HYPERTENSION small vessel damage.

Evident in the optic fundus, where


papilloedema, hemorrhages and/or
exudates may be present.
Renal damage, including
hematuria, proteinuria and
impaired renal function is also
MALIGNANT characteristic.
HYPERTENSION
Confusion
Clinical features are:
Headache
Visual loss
Coma
MALIGNANT HYPERTENSION
• Medical emergency that
requires hospital
admission and rapid
control of BP over 12-24
hours towards normal
levels.
• Fatal in the absence of
treatment, with a 1 year
survival of less than
20%.
Management of
hypertension

• Diagnosis
• BP is measured using well maintained
sphygmomanometer
• If BP is increased, the measurement
should be repeated several times
over several weeks.
• Home or ambulatory blood pressure
measurements is recommended to
prevent “white coat hypertension”.
TREATMENT OF
HYPERTENSION

• Non-pharmacologic approaches
• Weight loss results in reduction in BP of
about
2.5/1.5 mmHg per kg (for overweight
patients).
• Reduce salt intake (aim is <100 mmol daily
sodium intake)
• Diet high in fruit and vegetables, legumes
and whole grain cereal improves
cardiovascular risk
TREATMENT OF
HYPERTENSION

• Non-pharmacologic approaches
• Regular dynamic exercise for at least 30
minutes on most days
• Alcohol intake should be restricted
• Quit smoking
TREATMENT OF HYPERTENSION

• ACE Inhibitors
A• Pharmacologic approaches
• Angiotensin Receptor Blockers

B • Beta-blockers

C • Calcium channel blockers

D • Diuretics (Thiazides)
TREATMENT OF HYPERTENSION

Pharmacologic approaches

AB/CD algorithm (Williams et al 2004)

Treatment AB
• Initial choice of ACE inhibitor or angiotensin receptor
blocker and β blocker as first line therapy in younger non-
black patients (<55 years)
• Rationale: Said patients often have hypertension
associated with high concentration of renin. It is
therefore logical to treat them with drugs that antagonize
the renin-angiotensin system.
TREATMENT OF HYPERTENSION
• Pharmacologic approaches
• AB/CD algorithm (Williams et al 2004)
• Treatment CD
• For elderly and black patients, who tend to have hypertension associated with
low renin concentration, calcium channel blockers and thiazide diuretics are
recommended.
• If initial drug therapy fails, A or B is combined with C or D.
• Further therapies should be added as necessary
(α-blockers, spironolactone).
PHARMACOLOGIC APPROACH
CLASS EXAMPLES MAJOR ADR’S NOTES
Diuretics Thiazide: Hyperlipidemia Cheap, effective. Efficacy
Bendroflumethiazide Impotence proven in clinical trials.
Hydrochlorothiazide Uremia
Dehydration Concerns about long term
Loops: Hyperkalemia metabolic effects.
Furosemide Gynecomastia More appropriate in older
patients.
K-Sparing:
Spironolactone Especially for patients
with cardiac failure.
Especially for resistant
hypertension.
PHARMACOLOGIC APPROACH
CLASS EXAMPLES MAJOR ADR’S NOTES
Beta-blockers Atenolol Tiredness Cheap
Propranolol Reduced exercise Adverse effects common.
Metoprolol tolerance Possible less effective in
Labetalol Bradycardia preventing cardiovascular
Celiprolol Cold peripheries events.
Claudication Especially for patients
Wheezing with ischemic heart
Cardiac failure disease
Impotence
PHARMACOLOGIC APPROACH
CLASS EXAMPLES MAJOR ADR’S NOTES
Calcium Channel Blocker Nifedipine Flushing Not well tolerated
(Dihydropyridine) Amlodipine Edema (especially early in
Postural hypotension treatment). Recent trials
Headache confirm reductions in
stroke and myocardial
infarction.
Similar efficacy to
thiazides.
Especially for elderly
patients and those with
ischemic heart disease or
diabetes.
PHARMACOLOGIC APPROACH
CLASS EXAMPLES MAJOR ADR’S NOTES
Calcium Antagonist Verapamil Constipation (verapamil Well tolerated. Suitable
(Rate-limiting) only) for patients with ischemic
heart disease who are
Diltiazem Bradycardia unable to tolerate β-
Heart block blocker.
Caution needed when
used in combination with
β-blockers.
Angiotensin Converting Captopril Cough More expensive. Cough
Enzyme (ACE) Inhibitor Enalapril Rash, taste disturbance very common.
Lisinopril Renal failure Appropriate for use in
Perindopril Angio-edema younger patients and
Ramipril those with cardiac failure
or diabetes.
PHARMACOLOGIC APPROACH
CLASS EXAMPLES MAJOR ADR’S NOTES
ɑ-blocker Prazosin Edema More expensive. Adverse effects
Doxazosin Postural hypotension common. No evidence to date of long-
Terazosin term efficacy. Less effective than
thiazides at preventing heart failure
and combined cardiovasvular
outcomes (ALLHAT study)
Second-line
Angiotensin receptor Losartan Renal failure More expensive.
blockers (ARBs) Valsartan Edema Especially for patients in whom ACE
Irbesartan Headache inhibitor indicated but not tolerated
due to cough.
More effective in preventing vascular
events than atenolol in patients with
left ventricular hypertrophy (LVH).
PHARMACOLOGIC APPROACH
CLASS EXAMPLES MAJOR ADR’S NOTES
Centrally acting Methyldopa Tiredness Poorly tolerated. Only use in severe
vasodilators Moxonidine Depression hypertension or hypertension of
pregnancy.
Third line

Direct-acting Diazoxide Edema Poorly tolerated. Only use in severe


vasodilators Minoxidil Postural hypotension hypertension.
Nitroprusside Headache
CORONARY ARTERY DISEASE (CAD)
• Coronary Heart
Disease (CHD)
• Ischemic Heart
Disease (IHD)
• Atherosclerotic
Heart Disease
(ASHD)
CORONARY ARTERY DISEASE
• It is the end result of the accumulation of atheromatous plaques
within the walls of the arteries that supply the myocardium with
oxygen and nutrients.
CORONARY ARTERY DISEASE
• It refers to a variety of pathologic conditions that cause narrowing or
obstruction of the coronary arteries, resulting in decreased blood
supply to the myocardium.
• Occurs most often between ages 30 and 50; men affected more often
than women; nonwhites have higher mortality rates.
• May manifest as angina pectoris or MI.
• Causative factor is atherosclerosis (deposits of cholesterol and lipids
within the walls of the artery).
Signs and Symptoms
RISK FACTORS

• Family history of CAD


• Elevated serum lipoproteins (LDL)
• Smoking
• Diabetes mellitus
• Hypertension
• Obesity
• Sedentary and/or stressful lifestyle
• Elevated serum uric acid levels
Management
• Screening and Diagnosis
• Electrocardiogram (ECG)
• Echocardiogram
• Electron Beam Computerized Tomography (EBCT)
• Magnetic Resonance Angiography (MRA)
• Stress test
• Angiogram
Management
• Treatment
• Lifestyle Changes
• Medicines
• Cholesterol-lowering agents
• Anticoagulants, Aspirin, ACE inhibitors, β blockers
• Calcium channel blockers
• Nitroglycerin and nitrates
• Glycoprotein IIb-IIIa inhibitors
• Thrombolytic agents
• Special Procedures
• Angioplasty
• Coronary Artery Bypass Graft
Percutaneous Transluminal Coronary Angioplasty (PTCA)

• Referred to as a
"balloon treatment”
• Special balloons are
used to open up the
arteries. involves the
use of stents to help
keep the arteries
open.
Coronary Artery Bypass Graft (CABG)
• Arteries or veins from
elsewhere in the
patient's body are
grafted from the aorta
to the coronary
arteries to bypass
atherosclerotic
narrowing and
improve the blood
supply to the coronary
circulation supplying
the myocardium.
COMMON THERAPEUTIC PROBLEMS IN CAD

Problem Comment

Used incorrectly, nitrated may cause hypotensive Advise to sit down when usingnitrate sprays or SL
episodes or collapse tablets

A daily nitrate-free period is required Avoid long acting preparations and prescribe
to maintain efficacy of nitrates asymmetrically (e.g. 8am and 2pm)

NSAIDS are associated with renal failure when given Warn patient to use paracetamol as their analgesic
with ACEinhibitors of choice

Speed is essential when patients need fibrinolytic Arrange emergency admission to hospital where
drugs after infarction fast-track systems should exist
COMMON THERAPEUTIC PROBLEMS IN CAD
Problem Comment
Aspirin may cause bleeding Advise on taking with food and water. Consider use of prophylactic
agents in high-riskpatient
β-blockers are often considered unpleasant Encourage patient to use regularly. Change the time of day.
to take Consider a vasodilator if cold extremities are a problem. Consider
verapamil or diltiazem.
β-blockers are contraindicated in Consider verapamil or diltiazem. Pay strict attention to other
respiratory and peripheral vascular disease treatmentsand removal of precipitating factors.
Patients often receive multiple drugs for Use once-daily preparations, dosing aids and intensive social and
prophylaxis and for treatment of co- educational support. Avoid all unnecessary drugs.
existing disorders
ACEinhibitors are contraindicated in Advice women of child-bearing years to avoid conception or seek
pregnancy, especially the first trimester specialist advice first.
SHOCK
CLINICAL PHARMACY &
PHARMACOTHERAPEUTICS 1
(PSMA411)
Shock may result
from trauma,
A critical condition
heatstroke, blood
brought on by the
loss, an allergic
sudden drop in blood
reaction, severe
flow through the
infection, poisoning,
body.
severe burns or other
causes.

SHOCK
When a person is in
If untreated, this can
shock, his or her
lead to permanent
organs aren't getting
organ damage or
enough blood or
even death.
oxygen.
Causes

• Heart conditions (heart attack, heart failure)


• Heavy internal or external bleeding, such as from a serious injury or
rupture of a blood vessel
• Dehydration, especially when severe or related to heat illness.
• Infection (septic shock)
• Severe allergic reaction (anaphylactic shock)
• Spinal injuries (neurogenic shock)
• Burns
• Persistent vomiting and diarrhea
Pathophysiology

• Shock results in failure of the


circulatory system to deliver
sufficient oxygen (O2) to body
tissues despite normal or
reduced O2 consumption.
General pathophysiologic
mechanisms of different forms
of shock are similar except for
initiating events.
Signs & Symptoms

• Cool, clammy skin


• Pale or ashen skin
• Bluish tinge to lips or fingernails (or gray in the case of dark complexions)
• Rapid pulse
• Rapid breathing
• Nausea or vomiting
• Enlarged pupils
• Weakness or fatigue
• Dizziness or fainting
• Changes in mental status or behavior, such as anxiousness or agitation
Types of shock

1. Obstructive shock
o Occurs when blood can’t get where it needs to go.
o A pulmonary embolism is one condition that may cause an interruption to
blood flow.
o Conditions that can cause a buildup of air or fluid in the chest cavity can also
lead to obstructive shock.
▪ These include:

▪ pneumothorax (collapsed lung)

▪ hemothorax (blood collects in the space between the chest wall and
lung)
▪ cardiac tamponade (blood or fluids fill the space between the sac that
surrounds the heart and the heart muscle)
Types of stock

2. Cardiogenic shock
o Damage to your heart can decrease the blood flow to your
body, leading to cardiogenic shock.
o Common causes of cardiogenic shock include:
▪ damage to your heart muscle

▪ irregular heart rhythm


• very slow heart rhythm
Types of stock

3. Distributive shock
o Conditions that cause the blood vessels to lose their tone can cause
distributive shock.
o When your blood vessels lose their tone, they can become so open
and floppy that not enough blood pressure supplies the organs.
o Drug toxicities and brain injuries can also lead to distributive shock.
o Distributive shock can result in symptoms including:
▪ flushing

▪ low blood pressure


▪ loss of consciousness
Types of stock
Types of distributive shock:
A. Septic shock
▪ results from bacteria multiplying in the blood and releasing toxins.
▪ Common causes:
▪ Pneumonia
▪ urinary tract infections
▪ skin infections (cellulitis)
▪ abdominal infections (such as a ruptured appendix)
▪ Meningitis
B. Anaphylactic shock
▪ A type of severe hypersensitivity or allergic reaction.
▪ Causes:
▪ allergy to insect stings
▪ medicines
▪ foods (nuts, berries, seafood)
C. Neurogenic shock
▪ Caused by damage to the central nervous system, usually a spinal cord injury.

▪ This causes blood vessels to dilate, and the skin may feel warm and flushed.
▪ The heart rate slows, and blood pressure drops very low.
Types of stock

4. Hypovolemic shock
o Caused by severe blood and fluid loss, such as from
traumatic bodily injury, which makes the heart unable
to pump enough blood to the body, or
severe anemia where there is not enough blood to
carry oxygen through the body.
DIAGNOSIS • First responders and doctors often recognize
shock by its external symptoms. They may also
check for:
o low blood pressure
o weak pulse
o rapid heartbeat
• Once diagnosed with shock, the priority is to
provide lifesaving treatment to get blood
circulating through the body as quickly as
possible.

▪ This can be done by giving fluid, drugs,


blood products, and supportive care.

• Once stable, the doctor can try to diagnose the


cause of shock. To do so, they may order one or
more tests, such as imaging or blood tests.
TREATMENT

• Provide first aid treatment


o If they’re unconscious, check to
see if they’re still breathing and
have a heartbeat.
o If you don’t detect breathing or a
heartbeat, begin CPR.
TREATMENT
o If they’re breathing:
▪ Lay them down on their back.
▪ Elevate their feet at least 12 inches above
the ground. This position, known as the
shock position, helps direct blood to their
vital organs where it’s most needed.
▪ Cover them with a blanket or extra clothing
to help keep them warm.
▪ Check their breathing and heart rate
regularly for changes.
▪ If you suspect the person has injured their
head, neck, or back, avoid moving them.
TREATMENT

Pharmacological
• Epinephrine and other drugs to treat
anaphylactic shock
• Blood transfusion to replace lost
blood and treat hypovolemic shock
• Medications, heart surgery, or other
interventions to treat cardiogenic
shock
• Antibiotics to treat septic shock
STROKE
CLINICAL PHARMACY &
PHARMACOTHERAPEUTICS 1
(PSMA411)
Risk Factors
Signs and Symptoms
Types of stroke
• Laboratory test
• Computed tomography (CT) head
scan
• Magnetic resonance imaging of the
Diagnosis head
• Carotid Doppler studies
• Electrocardiogram
• Transthoracic echocardiogram
• Transesophageal echocardiogram
• Transcranial Doppler
The goals of treatment for acute stroke
are to:
(1) reduce the ongoing neurologic injury
Desired and decrease mortality and long-
term disability;
outcome (2) Prevent complications secondary to
immobility and neurologic
dysfunction; and
(3) prevent stroke recurrence
• Respiratory and cardiac support
• Ischemic stroke patients presenting within
hours of symptom onset should be
evaluated for reperfusion therapy.
• Elevated blood pressure should remain
untreated in the acute period (first 7 days)
Treatment after ischemic stroke because of the risk of
decreasing cerebral blood flow and
worsening symptoms.
• If blood pressure is treated in the acute
phase, short-acting parenteral agents (e.g.,
labetalol, nicardipine, nitroprusside) are
preferred.
• Alteplase
• Aspirin
• Antiplatelet (ex. Clopidogrel)
Pharmacological • Warfarin
treatment • ACE inhibitor
(ischemic stroke) • Diuretic
• Low-molecular-weight heparin or low-dose
subcutaneous unfractionated heparin
• There are currently no standard
pharmacologic strategies for treating
intracerebral hemorrhage.
• Medical guidelines for managing blood
Pharmacological pressure, increased intracranial pressure,
treatment and other medical complications in acutely
(hemorrhagic ill patients in neurointensive care units
stroke) should be followed.
• The calcium channel blocker nimodipine is
recommended to reduce the incidence and
severity of neurologic deficits resulting from
delayed ischemia.
Non pharmacologic treatment

• Surgical decompression can be lifesaving in cases of significant swelling


associated with cerebral infarction.
• Rehabilitation is very effective in reducing longterm disability.
• Carotid endarterectomy is effective in reducing stroke incidence and
recurrence .
• Carotid stenting may be effective in reducing recurrent stroke risk.
• In subarachnoid hemorrhage due to a ruptured intracranial aneurysm or
arteriovenous malformation, surgical intervention to clip or ablate the
vascular abnormality substantially reduces mortality from rebleeding.
Venous
Thromboembolism
Clinical Pharmacy &
Pharmacotherapeutics 1

(PSMA411)
Venous
thromboembolism
(VTE)

• Refers to a blood clot that starts in a vein. It is


the third leading vascular diagnosis after heart
attack and stroke.
2 Types:
• Deep vein thrombosis (DVT) Deep vein thrombosis is a clot in a deep
vein, usually in the leg. DVT sometimes affects the arm or other veins.
• Pulmonary embolism (PE) A pulmonary embolism occurs when a DVT
clot breaks free from a vein wall, travels to the lungs and then blocks
some or all of the blood supply. Blood clots originating in the thigh
are more likely to break off and travel to the lungs than blood clots in
the lower leg or other parts of the body.
Etiology

Venous thromboembolism is
associated with Virchow’s
triad: three conditions that
predispose to thrombus
formation.
• Hypercoagulability
• Stasis
• Endothelial damage
Major general surgery

Major orthopedic surgery

Lower-extremity paralysis due to spinal cord injury

Risk factors
Fracture of the pelvis, hip or long bones

Multiple trauma

Cancer
DVT mainly affects the large veins in the lower leg and thigh,
almost always on one side of the body at a time. The clot
can block blood flow and cause:

Leg pain or tenderness of the thigh or calf

Signs & Leg swelling (edema)


Symptoms
Skin that feels warm to the touch

Reddish discoloration or red streaks


PE, or pulmonary embolism, can be fatal and occurs when the DVT
breaks free from a vein wall and blocks some or all of the blood supply
to the lungs, causing:

Unexplained shortness of breath

Rapid breathing
Signs &
Symptoms Chest pain anywhere under the rib cage (may be worse with deep
breathing)

Fast heart rate

Light headedness or passing out


Blood work may be done initially, including a test
called D-dimer, which detects clotting activity.

Diagnosis For DVT: ultrasound of the leg is most often used

For PE: Computed tomography, or CT scan, or CAT


scan is most often used. Sometimes ventilation-
perfusion lung scan is used. Both tests are able to see
intravenous dyes in the arteries of the lung, looking
for blockages by clots.
• Be proactive. Ask for a risk assessment for VTE when if you are admitted to
the hospital or are having surgery.
• Those at risk may take anti-clotting, or blood-thinning, medications or use
Prevention mechanical devices such as compression stockings or compression devices.
Getting out of bed quickly after surgery is also advised if possible.
• Anticoagulants (heparin or low molecular weight heparin) or tablets such as
apixaban, dabigatran, rivaroxaban, edoxaban and warfarin.
• Thrombolytic therapy, which includes drugs such as a tissue plasminogen
activator (tPA), — a clot-dissolving enzyme.
Treatment • Surgical procedures may also be used. This can involve placing a filter in the
body's largest vein, the inferior vena cava, to prevent blood clots from
traveling to the lungs. It can also involve removing a large blood clot from
the vein or injecting clot-busting medicines into the vein or lung artery.
Pop Quiz
1. It is the end result of the accumulation of atheromatous
plaques within the walls of the arteries that supply the
myocardium with oxygen and nutrients.
2. This is caused by severe blood and fluid loss, such as from
traumatic bodily injury, which makes the heart unable to
pump enough blood to the body, or severe anemia where
there is not enough blood to carry oxygen through the body.
3. This antihypertensive treatment is for elderly and black
patients, who tend to have hypertension associated with low
renin concentration.
Pop Quiz
4. These agents are effective in lowering LDL levels while
increasing HDL levels and lowering triglyceride levels.

5. A disorder characterized by deviations from the normal


heartbeat pattern.
OUR LADY OF FATIMA
UNIVERSITY
College of Pharmacy

Pharmacotherapy of
Renal Disorders

CLINICAL PHARMACY &


PHARMACOTHERAPEUTICS 1
(PHMA 411)
ACUTE KIDNEY FAILURE

• Kidneys suddenly lose the ability to


eliminate excess salts, fluids, and
waste materials from the blood.
• Body fluids can rise to dangerous
levels when kidneys lose their
filtering ability.
• Also called acute kidney injury or
acute kidney failure.
• Acute kidney failure can be life-
threatening and requires intensive
treatment.
CAUSES

• Acute kidney failure can occur for


many reasons. Among the most
common reasons are:
o acute tubular necrosis (ATN)
o severe or sudden dehydration
o toxic kidney injury from poisons or
certain medications
o autoimmune kidney diseases, such
as acute nephritic syndrome
and interstitial nephritis
o urinary tract obstruction
CAUSES

• Reduced blood flow can damage your


kidneys. The following conditions can
lead to decreased blood flow to your
kidneys:
o low blood pressure
o burns
o dehydration
o hemorrhage
o injury
o septic shock
o serious illness
o surgery
CAUSES

• Certain disorders can cause clotting


within your kidney’s blood vessels,
and this can lead to acute kidney
failure. These conditions include:
o hemolytic uremic syndrome
o idiopathic thrombocytopenic
thrombotic purpura (ITTP)
o malignant hypertension
o transfusion reaction
o scleroderma
RISK FACTORS

• kidney disease
• liver disease
• diabetes, especially if it’s not well
controlled
• high blood pressure
• heart failure
• morbid obesity
• bloody stools
• breath odor
SIGNS AND SYPTOMS •

slow, sluggish movements
generalized swelling or fluid retention
• fatigue
• pain between ribs and hips
• hand tremor
• bruising easily
• changes in mental status or mood, especially in older
adults
• decreased appetite
• decreased sensation, especially in your hands or feet
• prolonged bleeding
• seizures
• nausea
• vomiting
• high blood pressure
• a metallic taste in your mouth
DIAGNOSIS • Using a stethoscope
• Results of laboratory tests
o blood urea nitrogen (BUN)
o serum potassium
o serum sodium
o estimated glomerular filtration rate (eGFR)
o urinalysis
o creatinine clearance
o serum creatinine
• Ultrasound
• Abdominal X-ray
• Abdominal CT scan
• Abdominal MRI
• Blood tests may also reveal underlying causes
of acute kidney failure.
TREATMENT

• Pharmacological
o Diuretics may help your
kidneys eliminate fluid.
o Calcium and insulin can help
you avoid dangerous
increases in your blood
potassium levels.
TREATMENT

• Dialysis
o Dialysis involves diverting
blood out of the body into a
machine that filters out
waste.
o The clean blood then returns
to the body.
TREATMENT

• Non-pharmacological
o Diet restrictions
▪ A diet high in carbohydrates
and low in protein, salt, and
potassium is usually
recommended.
CHRONIC
KIDNEY
DISEASE
CLINICAL PHARMACY &
PHARMACOTHERAPEUTICS 1
(PSMA411)
A reduction in the glomerular
filtration rate (GFR) and/or urinary
abnormalities or structural
abnormalities of the renal tract.
Chronic kidney
disease
The severity of CKD is classified
from 1 to 5 depending upon the
level of GFR.
MORE PREVALENT IN THE ELDERLY
POPULATION.

9TH LEADING CAUSE OF DEATH


CHRONIC
KIDNEY
DISEASE
KIDNEY SHRINKS AS CKD PROGRESSES.

KIDNEY DAMAGE OR DECREASED GFR OF


<60ML/MIN FOR 3 MORE OR MONTHS.
CAUSES

Glomerulonephritis (gloe- Interstitial nephritis (in-


mer-u-low-nuh-FRY-tis), tur-STISH-ul nuh-FRY-tis),
Type 1 or type 2
High blood pressure an inflammation of the an inflammation of the
diabetes kidney's filtering units kidney's tubules and
(glomeruli) surrounding structures

Prolonged obstruction of Vesicoureteral (ves-ih-koe-


Recurrent kidney
the urinary tract, from yoo-REE-tur-ul) reflux, a
Polycystic kidney infection, also called
conditions such as condition that causes
disease pyelonephritis (pie-uh-
enlarged prostate, kidney urine to back up into your
low-nuh-FRY-tis)
stones and some cancers kidneys
Diabetes

High blood pressure

Heart and blood vessel (cardiovascular) disease

Smoking

Risk Factors Obesity

Being African-American, Native American or Asian-American

Family history of kidney disease

Abnormal kidney structure

Older age
PATHOPHYSIOLOGY OF CKD
Signs & symptoms
STAGES OF KIDNEY DISEASES
DIAGNOSIS (LABORATORY STUDIES)

1.COMPLETE BLOOD COUNT


2.URINALYSIS
3.SERUM ALBUMIN LEVELS
4.LIPID PROFILE
5.SERUM CALCIUM AND PHOSPHAT
6.ALKALINE PHOSPHATASE
7.INTACT PARATHYROID HORMONE (PTH LEVELS)
DIAGNOSIS (IMAGING STUDIES AND BIOPSY)

1.RENAL ULTRASONOGRAPHY
2.RETROGRADE PYELOGRAPHY
3.COMPUTED SCANNING
4.MAGNETIC IMAGING RESONANCE
5.RENAL RADIONUCLIDE SCANNING
6.PERCUTANEOUS RENAL BIOPSY
Aims of the treatment of CKD
• Reverse or arrest the process causing the renal damage
• Avoid conditions that might worsen renal failure
• Treat the secondary complications of CKD (renal anaemia and bone
disease)
• Relieve symptoms
• Implement regular dialysis treatment and/or transplantation at the
most appropriate time.
TREATMENT (Pharmacological)
HYPERTENSION
- ACE inhibitors
- Angiotensin II receptor blockers
- Diuretics
- Non-dihydropyridine Calcium channel blockers
HYPERGLYCEMIA
• Intensive therapy in patients
with type 1 and type 2 diabetes
TREATMENT reduces microvascular
(Pharmacological) complications, including
nephropathy. Intensive therapy
can include insulin or oral drugs
and involves blood sugar testing
at least three times daily.
TREATMENT (Pharmacological)

• CALCIUM SALTS (CALCIUM CARBONATE, CALCIUM ACETATE)


- TREATMENT OF HYPERPHOSPHATEMIA
• VITAMIN D ANALAOGUES (CALCITRIOL)
- PREVENTION AND TREATMENT OF SECONDARY HYPERPARATHYROIDISM
• PHOSPHATE SCAVENGERS (SEVELAMER)
- REDUCES SERUM PHOSPHORUS LEVELS IN THE BODY
• HEMATOPOIETIC GROWTH FACTORS (EPOETIN, DARBOPOIETIN)
- STIMULATES ERYTHOPOETIN PRODUCTION
• IRON PRODUCTS (FERROUS SULFATE, IRON DEXTRAN IV)
- FOR TREATMENT OF ANEMIA
SUPPORTIVE THERAPIES
• The primary goal of lipid-lowering therapies in CKD is to decrease the risk for
progressive atherosclerotic cardiovascular disease.
• A secondary goal is to reduce proteinuria and renal function decline seen with
administration of statins (3-hydroxy-3-methylglutaryl coenzyme A reductase
inhibitors).
NON-PHARMACOLOGIC TREATMENT

• A low-protein diet (0.6 to 0.75 g/kg/day) can delay progression of CKD in


patients with or without diabetes, although the benefit is relatively
small.
• Low salt diet
• Smoking cessation
DRUG DOSING
IN RENAL
INSUFFICIENCY
• Chronic kidney disease affects renal drug elimination
and other pharmacokinetic processes involved in drug
disposition (e.g., absorption, drug distribution, nonrenal
clearance [metabolism]).
• Drug dosing errors are common in patients with renal
impairment and can cause adverse effects and poor
Drug outcomes.
DOSING • Dosages of drugs cleared renally should be adjusted
according to creatinine clearance or glomerular
filtration rate.
• Recommended methods for maintenance dosing
adjustments are dose reductions, lengthening the
dosing interval, or both.
Effect on drug absorption

Factors that theoretically affect


There is little quantitative bioavailability include alterations
information regarding influence in GI transit time, gastric pH,
of impaired renal function on edema of the GI tract, vomiting
drug absorption and and diarrhea, and concomitant
bioavailability. drug therapy, especially antacid
or H2-antagonist administration.
Effect on drug distribution
• The volume of distribution of many drugs is significantly increased or decreased in patients with CKD. Changes
result from altered protein or tissue binding, or pathophysiologic alterations in body composition (e.g., fractional
contribution of total body water to total body weight).
• Plasma protein binding of acidic drugs (e.g., warfarin, phenytoin) is decreased in CKD, whereas binding of basic
drugs (e.g., quinidine, lidocaine) is usually normal or slightly decreased or increased.
• Ideally, unbound (versus total) drug concentrations should be monitored, especially for drugs that have a narrow
therapeutic range, are highly protein bound (free fraction less than 20%), and have marked variability in the free
fraction (e.g., phenytoin, disopyramide).
• Methods for calculating volume of distribution (VD) can be influenced by renal disease.
Effect on
metabolism
• Effect of End-Stage Renal
Disease on Nonrenal
Clearance of Selected Drugs
• Altered renal filtration, secretion, and/or
Effect on absorption can have dramatic effects on the
pharmacokinetics of a drug.
renal • The impact depends on the fraction of drug
excretion normally eliminated unchanged by the kidney
and on the degree of renal insufficiency.
Drug dosage regimen design

• The optimal dosage regimen for patients with renal insufficiency requires an individualized assessment.
• The optimal regimen depends on an accurate characterization of the relationship between the drug’s
pharmacokinetic parameters and renal function and on an accurate assessment of the patient’s residual renal
function.
• If the relationship between CLcr and the kinetic parameters of a drug (i.e., total body clearance [CL] and
elimination rate constant [k]) is known, these data should be used to individualize drug therapy.
• If the relationship between CLcr and the kinetic parameters is unknown, then the patient’s kinetic parameters can
be based on the fraction of drug eliminated renally unchanged (fe) in subjects with normal renal function.
Men: 97 to 137 ml/min
Normal values for
the creatinine
clearance
Women: 88 to 128 ml/min
Stepwise Approach to Adjust Drug Dosages
for Patients with Renal Insufficiency
Antihypertensive agents
• Thiazide diuretics are first-line agents for treating uncomplicated hypertension, but
they are not recommended if the serum creatinine level is higher than 2.5 mg per dL
(220 μmol per L) or if the creatinine clearance is lower than 30 mL per minute.
• Loop diuretics are most commonly used to treat uncomplicated hypertension in
patients with chronic kidney disease.
• The addition of aldosterone blockers (e.g., spironolactone , eplerenone) has been
shown to reduce mortality in patients with severe heart failure.
• Potassium-sparing diuretics and aldosterone blockers should be avoided in patients
with severe chronic kidney disease because of the rise in serum potassium that
typically accompanies renal dysfunction.
Antihypertensive agents
• Angiotensin-converting enzyme (ACE) inhibitors and angiotensin receptor blockers
(ARBs) are first-line hypertensive agents for patients with type 1 or 2 diabetes
mellitus and proteinuria or early chronic kidney disease.6 These agents reduce blood
pressure and proteinuria, slow the progression of kidney disease, and provide long-
term cardiovascular protection.
• Hydrophilic beta blockers (e.g., atenolol, bisoprolol, nadolol, acebutolol ) are
eliminated renally and dosing adjustments are needed in patients with chronic kidney
failure.
• Metoprolol tartrate, metoprolol succinate, propranolol, and labetalol are
metabolized by the liver and adjustments are not required.
• Other antihypertensive agents that do not require dosing adjustments include
calcium channel blockers, clonidine and alpha blockers.
Hypoglycemic agents
• Metformin is 90 to 100 percent renally excreted,18 its use is not recommended
when the serum creatinine level is higher than 1.5 mg per dL (130 μmol per L) in
men or higher than 1.4 mg per dL (120 μmol per L) in women, in patients older
than 80 years, or in patients with chronic heart failure.
• Sulfonylureas (e.g., chlorpropamide, glyburide ) should be avoided in patients
with stages 3 to 5 chronic kidney disease. The half-life of chlorpropamide is
significantly increased in these patients, which can cause severe hypoglycemia.
• Glyburide has an active metabolite that is eliminated renally, and accumulation of
this metabolite can cause prolonged hypoglycemia in patients with chronic kidney
disease.
• Glipizide does not have an active metabolite and is safe in these patients.
Antimicrobials

• Many antimicrobial agents are eliminated renally and require dosing adjustments in patients
with chronic kidney disease; however, several commonly used agents do not require
adjustments.
• Excessive serum levels of injectable penicillin G or carbenicillin may be associated with
neuromuscular toxicity, myoclonus, seizures, or coma.
• Imipenem/ cilastatin can accumulate in patients with chronic kidney disease, causing seizures
if doses are not reduced.
• Tetracyclines, with the exception of doxycycline have an antianabolic effect that may
significantly worsen the uremic state in patients with severe disease.
• Nitrofurantoin has a toxic metabolite that can accumulate in patients with chronic kidney
disease, causing peripheral neuritis.
• Aminoglycosides should be avoided in patients with chronic kidney disease when possible. If
used, initial doses should be based on an accurate GFR estimate. Renal function and drug
concentrations should be monitored and dosages adjusted accordingly.
Analgesics
• Metabolites of meperidine, dextropropoxyphene , morphine, tramadol, and
codeine can accumulate in patients with chronic kidney disease, causing central
nervous system and respiratory adverse effects. Not recommended in patients
with stage 4 or 5 disease.
• A 50 to 75 percent dose reduction for morphine and codeine is recommended in
patients with a creatinine clearance less than 50 mL per minute (0.83 mL per
second).
• Extended-release tramadol should be avoided in patients with chronic kidney
disease. The dosing interval of tramadol (regular release) may need to be
increased to every 12 hours in patients with a creatinine clearance less than 30
mL per minute (0.5 mL per second).
• Acetaminophen can be used safely in patients with renal impairment.
NSAIDs
• Short-term use of NSAIDs is generally safe in patients who are well hydrated; who
have good renal function; and who do not have heart failure, diabetes, or
hypertension.
• Long-term use and high daily dosages of COX-2 inhibitors and other NSAIDs
should be avoided if possible.
• Patients at high risk of NSAID-induced kidney disease should receive serum
creatinine measurements every two to four weeks for several weeks after
initiation of therapy because renal insufficiency may occur early in the course of
therapy.
Classification of Acid-Base Disorder
• RESPIRATORY ACIDOSIS
• CO2 partial pressure – >40mmHg (Hypercapnia)
• Resulted from:
• Decreased in minute ventilation.
• Also known Hypoventilation
Classification of Acid-Base Disorder
• METABOLIC ACIDOSIS
• Serum HCO3 - <24 mEq/L or < 24mmol/L
• Resulted from:
• Increased acid production
• Acid ingestion
• Decreased renal acid excreation
• GI or renal HCO3- loss
Classification of Acid-Base Disorder
• RESPIRATORY ALKALOSIS
• CO2 partial pressure – <38mmHg (Hypocapnia)
• Resulted from:
• Inreased in minute ventilation.
• Also known Hyperventilation
Classification of Acid-Base Disorder
• METABOLIC ALKALOSIS
• Serum HCO3 - >28 mEq/L or >28mmol/L
• Resulted from:
• Acid loss
• HCO3- retention
RESPIRATORY ACIDOSIS
• The body is unable to remove excess carbon dioxide. It remained to
your blood and other fluids.
• SYPMTOMS:
• Fatigue
• SOB
• Confusion
CAUSES:
• Chest deformities
• Chest injuries
• Chronic lung and airway diseases
• Overuse of sedatives
• ObesitY
TYPES OF RESPIRATORY ACIDOSIS
• CHRONIC RESPIRATORY ACIDOSIS
• Blood slowly become acidic
• No noticeable symptoms
• ACUTE RESPIRATORY ACIDOSIS
• Suddenly
• Leaving kidney no time to adjust
DIAGNOSIS
• Arterial Blood gas tets
• Metabolic panel
• Pulmonary function test
• CXR
TREATMENT
• Bronchodilators
• Non-invasive positive pressure ventilation
• Antibiotics
COMPLICATIONS
• Respiratory Failure
• Organ failure
• Shock
PREVENTION
• Maintain healthy weight
• Take sedatives under strict doctor’s supervision, DON”T COMBINE IT
WITH ALCOHOL.
• Don’t smoke
METABOLIC ACIDOSIS
• CAUSES:
• Liver failure
• Cancer
• Kidney disease
• Severe dehydration
• ASA, Ethylene glycol and methanol poisoning
DIAGNOSIS
• Serum electrolytes
• Urine pH
• Arterial blood gas
TREATMENT
• Sodium bicarbonate until the blood pH returned to its normal value.
ALKALOSIS
• High levels of Carbon Dioxide or increase bicarbonate.

• SYPMTOMS:
• Muscle twitch
• Numbness and tingling
• Nausea and Vomiting
• Lightheadedness
• Confusion
METABOLIC ACIDOSIS
• CAUSES:
• Lack of oxygen
• High altitude
• Fever
• Lung and liver disease
• Salicylate poisoning
• Too much Bicarbonate consumption
DIAGNOSIS
• Metabolic panel
• Blood gas analysis
• Urine pH
TREATMENT
• Chloride and potassium containing medications
Pop Quiz
1. It helps regulate myocardial and neurological function, fluid
balance, oxygen delivery, acid-base balance, and other
biological processes.
2. A reduction in the glomerular filtration rate (GFR) and/or
urinary abnormalities or structural abnormalities of the renal
tract.
3. This is known as hypoventilation.
Pop Quiz
4. T/F: IV calcium is indicated to antagonize the neuromuscular
and cardiovascular effects of magnesium.

5. Potassium should be administered in saline because dextrose


can stimulate insulin secretion and worsen intracellular shifting of
potassium
OUR LADY OF FATIMA
UNIVERSITY
College of Pharmacy

PHARMACOTHERAPY
OF AUTONOMIC
NERVOUS SYSTEM
DISORDERS

CLINICAL PHARMACY &


PHARMACOTHERAPEUTICS
(PHMA 411)
MYASTHENIA GRAVIS

• A neuromuscular disorder that causes


weakness in the skeletal muscles,
which are the muscles your body uses
for movement.
• It occurs when communication
between nerve cells and muscles
becomes impaired. This impairment
prevents crucial muscle contractions
from occurring, resulting in muscle
weakness.
• The most common primary disorder of
neuromuscular transmission.
MYASTHENIA • The name myasthenia gravis,
which is Latin and Greek in origin,
GRAVIS means “grave, or serious, muscle
weakness.”
• There is no known cure.
• Available treatments can control
symptoms and often allow people
to have a relatively high quality of
life.
• trouble talking
SIGNS AND • problems walking up stairs or lifting
objects
SYMPTOMS • facial paralysis
• difficulty breathing due to muscle
weakness
• difficulty swallowing or chewing
• fatigue
• hoarse voice
• drooping of eyelids
• double vision
CAUSES
Caused by an autoimmune problem.
o Autoimmune disorders occur when your immune system mistakenly attacks
healthy tissue.
o In this condition, antibodies, which are proteins that normally attack foreign,
harmful substances in the body, attack the neuromuscular junction.
o Damage to the neuromuscular membrane reduces the effect of the
neurotransmitter substance acetylcholine, which is a crucial substance for
communication between nerve cells and muscles.
o This results in muscle weakness.

• The exact cause of this autoimmune reaction is unclear to scientists.


• According to the Muscular Dystrophy Association, one theory is that certain viral or
bacterial proteins may prompt the body to attack acetylcholine.
DIAGNOSIS
• Neurological exam
o checking your reflexes
o looking for muscle weakness
o checking for muscle tone
o making certain your eyes move
properly
o testing sensation in different
areas of your body
o testing motor functions, like
touching your finger to your nose
DIAGNOSIS
• repetitive nerve stimulation test
• blood testing for antibodies
associated with MG
• edrophonium (Tensilon) test: a drug
called Tensilon (or a placebo) is
administered intravenously, and
you’re asked to perform muscle
movements under doctor observation
• imaging of the chest using CT
scans or MRI to rule out a tumor
TREATMENT

1. Pharmacological
o Corticosteroids
and immunosuppressants
▪ To suppress the immune system.
▪ These medications help
minimize the abnormal immune
response that occurs in MG.
o Cholinesterase inhibitors, such as
pyridostigmine (Mestinon)
▪ To increase communication
between nerves and muscles
TREATMENT

o Plasmapheresis
▪ Also known as a plasma exchange.
▪ This process removes harmful
antibodies from the blood, which may
result in an improvement in muscle
strength.
▪ This is a short-term treatment.
▪ The body continues to produce the
harmful antibodies and weakness may
recur. Plasma exchange is helpful
before surgery or during times of
extreme MG weakness.
TREATMENT

o Intravenous immune globulin (IVIG)


▪ A blood product that comes from
donors. It’s used to treat
autoimmune MG.
▪ Although it’s not entirely known
how IVIG works, it affects the
creation and function of
antibodies.
TREATMENT
2. Thymus gland removal
o Removal of the thymus gland,
which is part of the immune
system, may be appropriate for
many patients with MG.
o Once the thymus is removed,
patients typically show less muscle
weakness..
3. Non-pharmacological
o Get plenty of rest to help minimize
muscle weakness.
o Avoid stress and heat exposure, as
both can worsen symptoms
GLAUCOMA
CLINICAL PHARMACY &
PHARMACOTHERAPEUTICS 1
(PHMA 411)
GLAUCOMA

• An eye disease that can damage the optic


nerve. The optic nerve supplies visual
information to the brain from the eyes.
• Glaucoma is usually, but not always, the
result of abnormally high pressure inside the
eye. Over time, the increased pressure can
erode the optic nerve tissue, which may
lead to vision loss or even blindness.
CAUSES
• The back of your eye continuously makes a clear fluid called aqueous humor.
• As this fluid is made, it fills the front part of your eye.
• Then, it leaves your eye through channels in your cornea and iris.
• If these channels are blocked or partially obstructed, the natural pressure in your eye, which is called the
intraocular pressure (IOP), may increase. As the IOP increases, the optic nerve may become damaged.
As damage to the nerve progresses, one may begin losing sight in the eye.

• These factors may play a role in increasing the pressure:


o dilating eye drops
o blocked or restricted drainage in your eye
o medications, such as corticosteroids
o poor or reduced blood flow to your optic nerve
o high or elevated blood pressure
SIGNS AND SYMPTOMS

• severe eye pain


• nausea
• vomiting
• redness in your eye
• sudden vision disturbances
• seeing colored rings around
lights
• sudden blurred vision
1. Open-Angle (Chronic)
Glaucoma
o Has no signs or symptoms
except gradual vision loss.
o This loss may be so slow
that vision can suffer
irreparable damage before
any other signs become
apparent.
o According the National Eye
Institute (NEI), this is the
most common type of
glaucoma.

TYPES OF GLAUCOMA
2. Angle-Closure (Acute)
Glaucoma
o If the flow of the aqueous
humor fluid is suddenly
blocked, the rapid buildup of
fluid may cause a severe,
quick, and painful increase
in pressure.
o This is an emergency
situation.

TYPES OF GLAUCOMA
3. Congenital Glaucoma
o Children born with congenital
glaucoma have a defect in the
angle of their eye, which slows
or prevents normal fluid
drainage.
o Usually presents with
symptoms, such as cloudy
eyes, excessive tearing, or
sensitivity to light. Congenital
glaucoma can run in families.

TYPES OF GLAUCOMA
4. Secondary Glaucoma
o This is often a side effect of
injury or another eye condition,
such as cataracts or eye
tumors.
o Medicines, such as
corticosteroids, may also
cause this type of glaucoma.

TYPES OF GLAUCOMA
5. Normal Tension Glaucoma
o In some cases, people
without increased eye
pressure develop damage to
their optic nerve. The cause
of this isn’t known.
o However, extreme sensitivity
or a lack of blood flow to the
optic nerve may be a factor
in this type of glaucoma.

TYPES OF GLAUCOMA
RISK FACTORS
• African American, Irish, Russian, Japanese, Hispanic, Inuit, or Scandinavian
descent
• Over 40 years old
• Have a family history of glaucoma
• Have poor vision
• Have diabetes
• Taking certain steroid medications such as prednisone
• Have had an injury to your eye or eyes
• Have corneas that are thinner than usual
• Have high blood pressure, heart disease, diabetes, or sickle cell anemia
• Have high eye pressure
• Are nearsighted or farsighted
DIAGNOSIS

1. Tonometry Test
o This class of tests measures the eye’s
internal pressure.
2. Pachymetry Test
o People with thin corneas have an
increased risk of developing glaucoma. A
pachymetry test can tell the doctor if
corneas are thinner than average.
3. Perimetry Test
o This test, also known as a visual field
test, can tell your doctor if glaucoma is
affecting the vision by measuring your
peripheral, or side, vision and your
central vision.
TREATMENT

• Eye drops.
o These either lower the creation of
fluid in your eye or increase its
flow out, lowering eye pressure.
o Side effects include allergies,
redness, stinging, blurred vision,
and irritated eyes. Some
glaucoma drugs may affect
your heart and lungs.
TREATMENT
• Oral medications
o beta-blocker or a carbonic anhydrase inhibitor
▪ These drugs can improve drainage or slow the creation of fluid in the
eye.
TREATMENT

• Laser surgery
o This procedure can slightly raise the
flow of fluid from the eye for an open-
angle glaucoma.
o Procedures include:
▪ Trabeculoplasty
• This opens the drainage area.
TREATMENT

▪ Iridotomy
• This makes a tiny hole in the iris to let fluid flow more freely.
▪ Cyclophotocoagulation
• This treats areas of the middle layer of the eye to lower fluid production.
▪ Microsurgery
• In a procedure called a trabeculectomy, doctor creates a new channel to drain the fluid
and ease eye pressure.
• This form of surgery may need to be done more than once.
• It involves the implantation of a tube to help drain fluid.
• This surgery can cause temporary or permanent vision loss, as well as bleeding or
infection.
TREATMENT

• Open-angle glaucoma is most often treated with


combinations of eye drops, laser trabeculoplasty, and
microsurgery.
• Infant or congenital glaucoma - is usually treated with
surgery because the cause is a problem with the
drainage system.
Pop Quiz
1. The test used to diagnose myasthenia gravis.

2. In most forms of glaucoma, what damages the optic nerve?

3. This is often a side effect of injury or another eye condition,


such as cataracts or eye tumors.
REFERENCES
• Walker, Roger & Whittlesea, Cate. (2012). Clinical Pharmacy & Therapeutics.
5th ed. Elsevier Ltd.
• American Pharmacists Association, National Association of Chain DrugStores
Foundation. Medication therapy management in community pharmacy practice:
core elements of an MTM service (version 1.0).J Am Pharm Assoc.
2005;45:573-9.
• Cipolle RJ. Strand LM, Morley PC. Pharmaceutical Care Practice: The
Clinician’s Guide. New York: McGraw Hill; 2004.
• The American Pharmacists Association and the National Association of Chain
Drug Stores Medication Therapy Management in Pharmacy practice Core
elements of an MTM service model.
• https://www.globalpremeds.com/blog/2015/01/02/understanding-soap-format-
for-clinical-rounds/

REFERENCES


• Boyette LC, Manna B. Physiology, Myocardial Oxygen Demand. [Updated 2019 Apr
3]. In: StatPearls [Internet]. Treasure Island (FL): StatPearls Publishing; 2020 Jan-.
Available from: https://www.ncbi.nlm.nih.gov/books/NBK499897/
• Kumar, A., & Cannon, C. P. (2009). Acute coronary syndromes: diagnosis and
management, part I. Mayo Clinic proceedings, 84(10), 917–938.
https://doi.org/10.1016/S0025-6196(11)60509-0
• Walker, Roger & Whittlesea, Cate. (2012). Clinical Pharmacy & Therapeutics. 5th
ed. Elsevier Ltd.
REFERENCES
• 1. National Kidney Foundation. K/DOQI clinical practice guidelines for chronic kidney disease:
evaluation, classification, and stratification. Am J Kidney Dis. 2002;39(2 suppl 1):S1–266.
• 2. Poggio ED, Wang X, Greene T, Van Lente F, Hall PM. Performance of the modification of diet in renal
disease and Cockcroft-Gault equations in the estimation of GFR in health and in chronic kidney
disease. J Am Soc Nephrol. 2005;16:459–66.
• 3. Burkhardt H, Hahn T, Gretz N, Gladisch R. Bedside estimation of the glomerular filtration rate in
hospitalized elderly patients. Nephron Clin Pract. 2005;101:c1–8.
• 4. Aronoff GR. Drug Prescribing in Renal Failure: Dosing Guidelines for Adults. 4th ed. Philadelphia, Pa.:
American College of Physicians, 1999.
• 5. Saseen JJ, Carter BL. Hypertension. In: DiPiro JT, Talbert RL, Yee GC, Matzke GR, Wells BG, Posey LM,
eds. Pharmacotherapy. 6th ed. New York, N.Y.: McGraw-Hill, 2005:185–215.
• 6. Chobanian AV, Bakris GL, Black HR, Cushman WC, Green LA, Izzo JL Jr, et al. The seventh report of the
Joint National Committee on Prevention, Detection, Evaluation, and Treatment of High Blood Pressure:
the JNC 7 report [Published correction appears in JAMA 2003;290:197] JAMA. 2003;289:2560–72.
• 7. Carter BL. Dosing of antihypertensive medications in patients with renal insufficiency. J Clin
Pharmacol. 1995;35:81–6.
• 8. Brater DC. Diuretic therapy. N Engl J Med. 1998;339:387–95.
REFERENCES
• 9. Pitt B, Zannad F, Remme WJ, Cody R, Castaigne A, Perez A, et al., for the Randomized Aldactone Evaluation Study Investigators. The
effect of spironolactone on morbidity and mortality in patients with severe heart failure. N Engl J Med. 1999;341:709–17.
• 10. Pitt B, Remme W, Zannad F, Neaton J, Martinez F, Roniker B, et al., for the Eplerenone Post-Acute Myocardial Infarction Heart Failure
Efficacy and Survival Study Investigators. Eplerenone, a selective aldosterone blocker, in patients with left ventricular dysfunction after
myocardial infarction [Published correction appears in N Engl J Med 2003;348:2271] N Engl J Med. 2003;348:1309–21.
• 11. Juurlink DN, Mamdani MM, Lee DS, Kopp A, Austin PC, Laupacis A, et al. Rates of hyperkalemia after publication of the Randomized
Aldactone Evaluation Study. N Engl J Med. 2004;351:543–51.
• 12. Palmer BF. Managing hyperkalemia caused by inhibitors of the renin-angiotensin-aldosterone system. N Engl J Med. 2004;351:585–
92.
• 13. Hu Y, Carpenter JP, Cheung AT. Life-threatening hyperkalemia: a complication of spironolactone for heart failure in a patient with renal
insufficiency. Anesth Analg. 2002;95:39–41.
• 14. Bakris GL, Weir MR. Angiotensin-converting enzyme inhibitor–associated elevations in serum creatinine: is this a cause for
concern? Arch Intern Med. 2000;160:685–93.
• 15. Ahmed A. Use of angiotensin-converting enzyme inhibitors in patients with heart failure and renal insufficiency: how concerned
should we be by the rise in serum creatinine? J Am Geriatr Soc. 2002;50:1297–300.
• 16. Palmer BF. Angiotensin-converting enzyme inhibitors and angiotensin receptor blockers: what to do if the serum creatinine and/or
serum potassium concentration rises. Nephrol Dial Transplant. 2003;18:1973–5.
• 17. Kappel J, Calissi P. Nephrology: 3. Safe drug prescribing for patients with renal insufficiency. CMAJ. 2002;166:473–7.
• 18. Snyder RW, Berns JS. Use of insulin and oral hypoglycemic medications in patients with diabetes mellitus and advanced kidney
disease. Semin Dial. 2004;17:365–70.
REFERENCES
• https://www.webmd.com/eye-health/glaucoma-eyes#1

• https://www.healthline.com/health/glaucoma#treatments

• https://www.healthline.com/health/myasthenia-gravis#treatments

• https://www.ninds.nih.gov/Disorders/Patient-Caregiver-Education/Fact-Sheets/Myasthenia-Gravis-Fact-Sheet

• https://www.kidneyfund.org/kidney-disease/chronic-kidney-disease-ckd/#symptoms-of-kidney-failure

• https://www.mayoclinic.org/diseases-conditions/chronic-kidney-disease/symptoms-causes/syc-20354521

• National Kidney Foundation. K/DOQI clinical practice guidelines for chronic kidney disease: evaluation, classification, and stratification. Am J Kidney Dis. 2002;39(2 suppl 1):S1–266.

• 2. Poggio ED, Wang X, Greene T, Van Lente F, Hall PM. Performance of the modification of diet in renal disease and Cockcroft-Gault equations in the estimation of GFR in health and in chronic kidney disease. J Am Soc Nephrol.
2005;16:459–66.

• Burkhardt H, Hahn T, Gretz N, Gladisch R. Bedside estimation of the glomerular filtration rate in hospitalized elderly patients. Nephron Clin Pract. 2005;101:c1–8.

• https://www.medicalnewstoday.com/articles/324627

• https://www.healthline.com/health/acute-kidney-failure#treatment

• https://www.cdc.gov/ncbddd/dvt/facts.html

• https://www.heart.org/en/health-topics/venous-thromboembolism/what-is-venous-thromboembolism-vte

• https://www.healthline.com/health/stroke#stroke-medication

• https://www.healthline.com/health/cerebrovascular-accident

• https://www.healthline.com/health/shock#treatment

• https://medlineplus.gov/ency/article/000039.htm

• https://www.mayoclinic.org/first-aid/first-aid-shock/basics/art-20056620

• https://www.mayoclinic.org/diseases-conditions/coronary-artery-disease/symptoms-causes/syc-20350613

• https://www.healthline.com/health/coronary-artery-disease

• https://www.who.int/news-room/fact-sheets/detail/hypertension

• https://www.heart.org/en/health-topics/high-blood-pressure/the-facts-about-high-blood-pressure/what-is-high-blood-pressure

• William et al. Guidelines for management of hypertension: report of the fourth working party of the British Hypertension Society, 2004-BHS IV. doi: 10.1038/sj.jhh.1001683.
phcp notes:*go guys kaya natin 'to

pathophysiology

since the causative orgs are microog - bact, viruses, para, fungi, prions -> these dif causative
agents will be colonizing -> after colonizing they will be going to the mucosal surfaces in the
CNS -> then it would go to the blood -> leading to the CNS through BBB = cns infections

-types:

meningitis - inflam of fluid and membranes (meninges)

= (meninges + inflam (itis) = meningitis

encephalitis - inflam of brain due to viral infection

spinal and cranial abcesses -

- abcesses (collection of pus within tissues located at the spinal and cranial

discitis - inflam between the intervertebral spaces of the spine

epilepsy - basically a cns infection but there is infections on it that could lead to seizures

- abnormal brain activity causing seizures

other severe complications that are spreading in the cns and they can be tx and dx by biopsy,
debridement, decom, or recons

---------------

meningitis -*picture na may layers

-there could be infected cerebral spinal fluid whenever there is infection by meningitis

-----------

picture ni brain

-bacterial meningitis complications:

1. septicemia - infection of the blood

-for encephalopathy: another brain disease that could affect the fxn or structure of the brain

-reasons: prions

- prions - are abnormal pathogenic transmissible to induce abnormal foldings of specific normal
proteins
- ex.

CJD - creutzfeld jakob disease: abnormal isoform of glycoproteins (a type of prion


glycoproteins)

if variant form: vCJD

Kuru - prion has loss of coordination and control over the muscle movements

-tuberculoma: masses fr. mycobacterium tuberculosis infection

-poliovirus: can affect spinal cord and cause paralysis

----------------

risk factors

-dif diseases -> y? -> whenever there are concommitant diseases the immune system is
affected when there are dif. disease along with microorg(immunosuppression) -> there are
pathogen/microorg that could easily infect the cns

------------

fxn of meninges (membrane = barrier/protection)

-------------

patho

-entering of bacteria -> will go to the cerebral spinal fluid -> bacterial growth -> inflam within cfs
and brain tissue

-----------

diagram

- iatrogenic: reason - bcs of dif intervention that has been occurred

- BBB (bebeq) - serves as protection

- immune system could be producing subs like cytokines

- meningitis: distrupted BBB - so higher BBB permeability,

-inc. matrix metalloproteinaeses (MMP) (can degrade matrix & non-matrix proteins) inc. MMP =
inc. degradation

-NO - nitric oxide

- ischemia: restriction in the blood supply to tissues so there would be shortage of O2 which is
needed for cellular metabolism
-leukocyte: since we are talking about immune system activation

- inc. ROS (reactive oxygen species) - leading to oxidation

-ICP - intracranial pressure - high ICP = brain disease/disorder such as meningitis

-------------------

s and s

-Kernig sign: severe stiffness of the hamstring of the legs causing the inability to straighten the
leg

-Brudzinski sign: severe neck stiffness causes px's hips and knees to flex

-Diastasis: separation of parts

-opisthotonus: abnormal posture

-papilledema: optic nerve at the back of the eye that becomes swollen

----------------

definitive test'

-csf purpose: protect the brain's spinal cord fr. injury

-csf culture: lab test for bact, fungi, and viruses in fluid that moves around the space of the
spinal cord

---------------

-CRP - C reactive protein: a blood test marker for inflam in the body

-PCR - polymerase chain reaction: that could defect a genetic material from specific org

-Latex agglutination: a sample containing specific antigen mixed w/ antibodies coated on


surface of latex part

-"agglutination" = agglutination of antigen and antibody

-procalcitonin: a subs produce by many c cells in the body in response to bacterial infection

------------

brain abscess

-brain parenchyma: (role) -> tissue need of the brain cell in neurons -> if damaged: could result
to loss cognitive ability or lead to death
----------

pathogenesis

reasons of occurence:

-contiguous foci -> connected or touching the side

-crypto -> unknown

-------------

changes in epi

-otogenic -> regions are the EARS

----------

hematogenous spread

-AVM -> arteriovenous malformation (abnormal tangle of blood vessels in brain/spine)

-------------

pathophysio

-cerebritis -> inflam of brain in setting of infection before development of cerebral abcess -> so
there would be an infection first then it will lead to cerebral abcess then it would evolve on
collection of pus

----------

location and clinical features

-AMS: altered mental status

-ipsilateral -> same side of the body

-------------

Lab test

-ESR -> Erythrocyte sedimentation rate

-BC - blood count

-LP - lumbar puncture

--------------------------

semisolid
-one of the most common example: ointments/unguents/chrisma/occulentum

-EXTERNAL -> but there ointments for opthalmic use

-2 types of ointments:

-medicated

-unmedicated -> as protectant, emollient or lubricant (lubricant bcs of its oily characteristic)

-----------

-blank ->surfactants

-therapeutoc agents/ medicated agents

-types of ointment bases

----------

-pliable: make the skin moisturized

-drug delivery -> ointment = DF -> for the delivery of the drug in the skin or mucous membrane

--------

in the manuf pharmacy/compounding area these are the methods used in the prep of semi
solids:

-most common lab method for ointment -> fusion method

-incorporation -> ointmetn roller mills/pill tiles

-purpose: so that the components are mix until the uniform preparation is attain

-pulverization by intervention -> -purpose: used as comminution process with the used of
volatile solvents (e.g. alcohol, ether)

-also used in gummy subs like camphor

-spatulation -> form of blending

-example of ointments undergoing this method: sulfur ointment

-fusion method -> porcelain dish/evap dish, stirring rod, large steam jacketted kettles/ water
bath (since +heat to make ointmetn)

-combine all the components by melting -> heating (H2O bath) -> cooled

------------------
mixing by fusion

- for mixing: used stirring rod

-------

1. evaporating basin on a water bath -> monitor the temperature so as to not OVERHEAT (will
lead to instability)

2. waxy (e.g. ointment bases) -> warm, heat then become liq

3. GENTLY -> if overstirring = affect the final px

-blank - to avoid bubbles/ lump formation

-eutectic mixtures: subs taht liquefy when mixed, rubbed or triturated together -> so meaning if u
have eutectic mixture the MP of that type of mixtures are below the RT (e.g. menthol, camphor,
thymol)

-----------

notes during

-it is always necessary to make an excess of 10% (deliverable vol)-> to still achieve the
prescribe amount -> if sakto baka hindi mag enough (e.g. syrups, suspensions)

--------

part of incorporation -> trituration

2. ointmetn slab/pile tile

3. blank -> doubling up technique

- it is necessary to have 2-3 times the volume of base to powder otherwise it will crumble
(tendency of the ointment/semi solid prep to break)

---------

4. mortar with a flat base and a flat head of pestle but usually reserve in larger scale batches

----------

ointment bases

1. oleaginous base/HC bases

-incorporation with water is w/ some degree of difficulty since insoluble in h2o


-levigating agent (e.g. mineral oil/liq petrolatum)

-----------

DISORDERS OF JOINTS AND BONES

Gout

-how does it happen?

inc uric acid/urate - if it precipitate/become super saturated = it will become monosodium urate
cyrstals - deposited in the avascular tissues (cartilages, tendons, and ligaments)

- reasons of inc. uric acid

1. dec renal secretion of uric acid bcs hereditary, diuretics, dec GFR (problem in the kidney),
ethanol (it can block urine secretion, stimulate liver urine synthesize)

2. inc prod of uric acid

- inc nucleoprotein turnover in hematologic conditions (associated diseases: lymphoma (CA of


lymphatic system), leukemia (CA of blood and tissues), hemolytic anemia (breaking down of
RBC)

-conditions with cell death (psoriasis, cytotoxic cancer therapy, radiation therapy)

-hereditary

3. inc. purine rich food

-liver, kidney, asparagus, meat broths, mushrooms, mussels, sardines, sweetbreads, drinks rich
in fructose

-ACUTE GOAT - pain in one joint only

- CHRONIC - repeated episodes of pain and inflam of more than one joints

-----------------------

GOUT

- hyperuricemia accumulation of MSU crystal around the tissues - tophi (stone like crystals),
interstitial renal disease (dec. renal secretion), uric a nephrolithiasis (kidney stones)

----

patho (read)
uric acid - blood and tissue - would react with na = MSU

-----

Dx

- JFT - to see if there are uric fluid in the joints

- blood test - uric acid can be seen in blood and tissues

- xray, ultrasound, dual - image if the joint is having inflam

----

s and s

-PODAGRA - main ma'am trish glitching herself out--

--

picture (read)

-------

Tx

All nsaids but Celecoxib is an selective COX 2 inhibitor

---

osteo

PROBLEM: cartilage in the joints are worn out - friction between the bones = pAiN

-cartilage can be worn out due to strenuous activites

-even though there will be unavoidable circumstances that could possible worn out the cartilage
the body have REPAIR MECHANISM - can restore normal fxn - and will lead cell mediated
remodelling

- so why the cartilages still worn out/osteo happens? -bcs of the rate of damage is greater than
the rate of repair = joint degeneration (if there is joint degeneration there would be painful and
tender inflam)

- what is happening during the damage in the joint?

1. proteolytic breakdown of cartilage matrix: meaning there would be breakdown of protein into
smaller peptide and that would be release in synovial fluid

2. fibrillation and erosion of cartilage surface and release of breakdown px to synovial fluid
-fibrillation - defective adhesion of chondrocytes to fibronectin - the purpose of fibronectin: binds
to collagen , fibrin, and otjher matrix proteins

3. synovial inflam - in the synovial cells it could ingest a breakdown px to phagocytosis and
produce proteases and pro-inflammatory cytokines

-proteins breakdown - immune system attack using phagocytosis, proteases, and pro-inflam
cytokines

-(2) types of synovial cells:

- type a: occuring for phago

- type b: secrete hyaluronic acid and proteins of synovial fluid

-purpose of synovial fluid: to reduce friction

-------------------------

Diagnosis:

- erythrocyte sedimentation rate (ESR) - settling of red blood cells

- osteophytes - deteriorated cartilage

-----

crepitus - cracking or grinding

----

-analysis of synovial fluid bcs of inflam happening in SF

----

Pharmacological - best choice: acetaminophen

----

medications - usually for pain bcs the body can repair the cartilage

------

Rheumatoid arthritis - there is also inflam but it is bcs of AUTO IMMUNE DISEASE

- in osteo inflam is cause by the broken down of proteins - phagocytosis process

- dyregulated inflammatory process in the synovium of joints - will lead to destruction of


cartilaginous and bony elements of joint - resulting to pain and disability
-since auto-immune: there are triggers such as: smoking, obesity and family hx = if present the
body would be producing autoantibodies - if there would be autoantibodies there would be
rheumatoid factors and anticyclic citrullinated peptide antibody (present in joint linings)

-there are also others inflam subs involved such as TNF alpha or tumor necrosis alpha,
Interleukin 1,6, 8, and transforming growth factor (TGF-b) beta, FGF (fibroblast growth factor),
and Platelet derived growth factor = these subs would lead in the inflam of joint - would lead to
joint and tissue destruction sicne the body cannot recognize what to destroy

------------

picture body

- RA: symmetrical

-----

OSTEOPOROSIS

-involved in skeletal fragility meanign there would be reduced bone mass (osteopenia),
deterioration of bone tissue, inc. risk of frature (bcs of low bone mass)

- what is happening in the body?

- resorption = daily removal of the mineral in body

- but eventhough there would be daily mineral removal happening in the body - the body can
still balance it through deposition - but the problem in osteoporosis:

THERE WOULD BE NO BALANCE = excessive resorption = bones weaken and over time
bones will become brittle and prone to fracture

-osteoARTHRITIS and RA = inflam

- osteoporosis = FRAGILITY OF THE BONES

------

-Vit D important in the absorption of calcium - ca abs low and ca excre high

-Parathyroid hormone inc = PTH activity could stimulate RESORPTION

- IGF 1 = can help in bone formation

- OSTEOBLASTS are the cells that form new bone.

----------

causes:

-over age = bone weakening


-----

diagnosis

-kyphosis- misalignment/curve alignment of spinal cord - changes in posture = loss of height

-----

Diagnosis

- Bone mineral density (BMD)

hyperlipidemia

-inc. amount of lipids or FA

- "dys" - there would be abnormalities

-inc. amnt of cholesterol (HYPERlipidemia)

-(enough amnt of cholesterol- helpful in the body since it is considered as building block of Vit D,
digestive bile, sex hormones, outermembrane cells)

-cholesterol is a type of lipid and high amount of it would be found in the blood = so there would
be excess cholesterol which are not used in this disease = so leads to higher risk of CV and
stoke

- y higher risk of CV or stroke - bcs built up of cholesterol/excess of cholesterol in the artery -


will be develop as plaque - there would be difficulty in the BF = leading to high BP = dif kinds of
CV disease or even stroke (stroke - o2 in the blood would be able to pass, hence, will not supply
the brain = abnormalities in the flow of blood)

-----

hyperlipidemia

-LDL (LuhDiLovengmamaniya) - called bad cholesterol - bcs it could form plaque that could clog
the arteries.

-triglycerol - this would be converted to used calories (ata hindi ko sure)

-triglyceride -store used calories and helpful bcs this provide the body with energy

-HDL (high) - good cholesterol - prevent the cholesterol plaque formation in the arteries - bcs
HDL brings back the cholesterol to the liver for excretion and excreted through the bile (HDL
also excreted in the bile)

-----------
-lipoproteins - tiny particles that contain choletesterol esters and triglycerine surrounded by them
bcs they are needed to transport this dif. types of lipids

- chylomicrons - transport specifically lipids absorb from intestine to adipose, cardiac, and
skeletal muscle cells

-----

Physiology

- cholesterol and trigly will not be dissolve in water (since body is consist of water)

------

-chylomicron remnants- eventually the chylomicrons remove by the liver then digested by
packing in very low density lipoprotein particles (hatdog) - what is remaining after being remove
by the liver in VLDL particles

-Lp a - if it is present in the dx tests this would really represent high risk of CV diseases

------

-atherogenesis - means formation of plaque

-pro-atherogenic - they make cause atherosclerosis

-anti (HDL) - prevents atherogenesis/formation of plaque (good cholesterol)

----

exogenous

-absorbed dietary lipids = trigly are form from the small intestine cell (FA, glycerol) by the
process of esterification

- lipids - esterification - trigly and cholesterol formed in the SI cells or bind to form chylomicron
(combining Apo BEC)

- and then chylomicrons enter the circulation (capillaries) - travels to peripheral parts of body

- FFA (free fatty acids) are release fr chylomicrons to be used as energy and convert into
glyceride to be stored in adipose and muscle tissues

-remnants are used in the formation of HDL

-LPL (lipprotein lipase) - would be found in tiny BV within the muscle and fatty tissues so they
break down the fats in the form of triglyceride (when FFA release they would be converted as
triglyceride using LPL and used as a source of energy)
-others wiuld be just stored in the liver (HDL that has been formed it would be going back to the
liver and excreted in via the bile)

-summary: dietary lipids - forming trigly in the small intestine - fomring chylomicrons - chylo
would be entering the circulation - releasing FFA that would be converted to triglyceride and be
stored in the muscle and adipose using LPL - and the remnants would be forming HDL (stored
in the liver then excreted through the bile)

endogenous

-VLDL (very low) is formed in the liver from the trigly and cholesterol ester - would be hydrolyze
by LPL in the capillaries (LPL is found in the capillaries)- so it would be forming Intermediate
(IDL) and VLDL remnants

-VLDL remnants are cleared in the circulation or they could be incorporated with LDL and then
LDL particles contain the core of cholesterol ester and smaller amount of trigly and they would
be internalized by the hepatic tissues (in the LIVER: the LDLs are converted into bile acid and
secreted in the intestine)

-summary: there would be trigly and cholesterol that would be forming from VLDL hydrolyze by
LPL - they would become IDL/VLDL remnants - will be incorporated with the LDL - internalized
by teh heaptic tissues producing or converting into bile acid and then secreted in the intestine -
if there would be NON-hepatic tissue (peripheral tissue) so LDL would be use in the hormone
prod, cell membrane synthesis - LDL are also taken up by macrophages and other cells which
can lead to excess accumulation and the formation of foam cells which are important in the
plaque formation

---------

patho

-atherosclerotic lesions - there would be damage (bcs of inflammatory response)

---------

stages of atherosclerosis

- there would be endothelial injury (1) and then lipoprotein deposition is also starting (2), there
would inflam reaction (3), lastly smooth muscle cell cap formation (4)

- for the endothelial injury this would be the initial factor that bein the plaque formation bcs the
endothelium is constantly exposed esp if there would be risk factors like tobacco use, diabetes,
and dyslipidemia

- lipoprotein deposition: there would be already lipoprotein molecules that being empty - they
are being modified by oxidation and glycatio n= there would be inflammatory reaction in the
arterial walls so that why the arteries would be damage (3), smooth muscle cells then would
migrate in the surface to the plaque forming fibrous cap (4) = THERE WOULD BE
COMPLICATED PLAQUES

------

Etiology

-genetic - there would be high amount of lipids/LDL

-lifestyle - sedentary lifestyle (kain-tulog-gege)

----

Signs and Symptoms

- cutaneous xanthomas - fatty growth develop underneath the skin

-peripheral neuropathy - multiple peripheral nerves are being damage

--------

intervention

-ASCVD - atherosclerotic cardiovascular disease

-statins DOC

------

SKIN

Eczema/atopic (hyperimmune system response) dermatitis (skin inflam = that's why there are
eruptions)

-considered inflammatory, chronically, relapsing, non-contagious, and extremely pruritic disease

-reasons/because of complex interaction of genetic predispositions, envi triggers, and immune


disregulation

-pathophysiology: there would be inc. levels of IgE

- "erythematous": there would be skin rashes by injured or blood capillaries

-patches: are the large area of color change with smooth surface

-papules: elevated solid or pleasant or equal to one 1 cm in diameter

-lichenified skin: thickened and leathery bcs of the continuous rubbing/scratching of the skin

------------

clinical types
1. atophy - bcs of immune response/during childhood

2. most of atophy ezcema cause: high amount of IgE

---------

how

1. chronic

a. inc. history of PLEURAL involvement

-duration of time would be the determinant of the atopic eczema

--------

exacerbating/triggers

how does it happen in the body? since it is bcs of immune response: there would be activated
antigenic presenting cells migrating at lymph nodes - there would be prime naive T cells into
TH2 cells (Helper T cells - for the production of IL (interleukin) 9,10,25, important in immune
response)- there would be inc. level of cytokines - also with TNF a (Tumor necrosis factor alpha
- important in inflammatory fxn) and IFN-y (interferon gamma - secreted by activated T cells -
promote macrophage activation, mediate antiviral and antibacterial immunity) - together with all
of them: TH2 cells, cytokines, TNF a, and IFN-y - DAMAGE THE SKIN BARRIER BY
APOPTOSIS OF THE KERATINOCYTES (cells found in epidermis), ALSO INFERE THE FXN
OF TYPE JUNCTIONS, PROMOTE TH2 RESPONSE BY ENHANCING THE TSLP function
(Thymic Stronal Lymphopoeitin - expressed mainly of epithelial cells and there would be
colonization pathogen e.g. Staph au that could impare the barriers fxn through the virulence
factors to induce keratinocyte death and to boost TH2 inflam)

-so together with genetic and immunonological factors contribute to skin barrier dysfunction and
that would manifesting as atopic dermatitis or eczema

- NO CURE but just being relieved by the use of moisturizer, apply antiage cream, taking oral
allergy or anti itch meds, not scratching them, taking a warm bath, usign mild soaps w/o dyes
and perfumes, using humidifier, waering cool and smooth clothing, and avoid stress

---------

2. Contact dermatitis

a. allergic contact dermatitis - common condiiton of skin irritation and this would be manifested
by itchy, dry skin, and rash

-dealyed type 4 hypersensitivity reax to envi allergy


-has 2 phases: the sensitization to allergen, and allergic response after exposure

-sensitized allergens captured by langerhans cells (dendritic epidermal cells) - activated -


migrate to lymph nodes - process and present antigen specific T cell (the antigen presenting
cells present to sensitize the T-cell which can expand and trigger inflam reax resulting to the s
and s of contact dermatitis) = itchiness, dry skin, and rashes

--------------

Irritant contact dermatitis

-more painful than pruritic

-non immunologically mediated to toxic subs. upon contact to skin

-only irritant and not allergic reax

-----------

Seborrheic Dermatitis

-types:

1. discoid eczema - coin shaped

2. stasis - found in the varicose veins so found lower legs

3. asteatotic - craking of skin

----------

1st line tx - NOT REALLY CURING BUT RELIEVING THE PAIN AND ITCHINESS

-systemic tx - target the inflam only

-------------

Acne vulgaris

- hyperseborrhea (hypersection of sebum and hyperactivity of sebaceous gland)

- there will also be abnormal follicular keratinization

-proliferation of propionibacterium acnes

-pilosebaceous follicle - is the hair pili and sebaceous gland

-----

-ostium follicular plug - clog in the hair follicle

-inc keratin filled microcomodones - that would be the acnes appearing


--------

-Propionibacterium acnes

-----

Retinoids (all retinoids need Rx - Adapalene

-prescription by dermatologist

-Pregnancy category C (caution)

---------

psoriasis

- Psoriasis vulgaris is a common skin disorder characterised by focal formation of inflamed,


raised plaques (red patches with whitetish built up of dead skin cells)

that constantly shed scales derived from excessive growth of skin epithelial cell - so not
contagious - but inflam disorders bcs mainly of genes

-what happened to psoriasis? the t lymphocyte mediated autoimmune disease and new
biological therapy that could target tcells have enter to it

----

Patho

-excessive growth of epithelial cells so it would produce more and more -> thus, poorly adhere
in stratum corneum -> scaly flaking -> like a silver scales

---------

types

1. chronic

2. plaque

3. guttate

4. inverse

5. pustular

6. erythrodermic

7. scalp

8. nail
9. oral

10. psoriatic arthritis

-------------

CONTRACEPTION

-day 1: follicular phase - time between day 1 and ovulation

- estrogen rises as it prepares to be release -day 14

endocrine

adreno (insufficiency of ACTH)

-production:

1. ACTH - from anterior pituitary gland

- tropic hormone (affects the target cell by stimulating other endocrine gland)

- hormone that stimulate APG to produce ACTH: CRH (corticotropin-releasing hormone) - CRH
made from the hypothalamus

- ACTH - main fxns: glucose metab adn immune system

- acts on the target organ (Adrenal cortex)

- In the AC there are dif zones (or zonas). These zones can produce dif. subs which are very
important in the functioning of the body

- AC:

1. zona fasiculata - produces glucocorticoids (including cortisol: influences the body utility to
convert food to energy, response to immune system's inflam and stress)

- glucocorticoid important in the negative feedback loop to inhibit excessive ACTH and CRH

- too much STRESS = inc. stimulation of ACTH and CRH = negative feedback (balancing)

- x CRH = x stimulation of APG = x ACTH = bodily fxns mentioned will be affected

2. zona reticularis - produces androgen/estrogen (male/female hormones)

3. zona glomerulosa - produces mineralcorticoids (includes aldosterone: maintain the body's


balance of Na adn K to have normal BP)

----

chronic - addison's
insufficiency - hypofxn

- destruction of the adrenal cortex = all the fxn of the zonas will be diminish

---

patho

- no acth = mineralocorticoed deficiency

---

glucocoritcoid def

- since related to glucose metabolism -so: it could cause severe insulin sensitivity

- excess glucose in the liver - so it is associated here

- low/dec cortisol - (negative feedback) so inc. ACTH

---

etio

- inc compensatory manner (negative feedback)

- secondary adrenal insuf: "EXO(outside)genous - not really fr the body, supplied by meds)

---

lab

-HYPOcortosolism (low cortisol)

-plasma acth levels:

-usually in the morning the normal range for acth level: 10-5- pg/ml

- explanation: having higher the normal acth = adrenoglands not producing enough
cortisol/hormones. so that is why they overeacted. have insuf but overeacting beyond normal -
so the levels in the plasma would inc

- metyrapone stimulation test: principle - based upon the principle that its administration results
in dec cortisol concentrations, which should be followed by an increase in corticotropin (ACTH)
secretion due to the dec. glucocorticoid negative feedback. this is test performed primarily to
detect partial defects in pituitary ACTH secretion

----

BUN - blood urine nitrogen


- nitrogen waste px of urea

- when proteins are brokendown = produces urea

--------------------------

clinical manifestation

waterhouse - an infection

----------------------

diabetes (paborito)

how diabetes happen in general:

-food (containing sugar and carbs) - sugar brokendown in the body and transformed into
glucose - for energy source

-normally there would be available hormonal insulin (produced by beta cells of pancreas) -
purpose: regulates the glucose in the body

- high amnt of glucose - the insulin stimulate the cells to absorb enough glucose fr blood to
convert to energy or stimulate the liver to absorb and store excess glucose

- definition of diabetes: having high glucose conc in the blood

- high amount blood sugar in the body

- type 1: immune system attacks and destroy the B cells

- bcs of dif auto immune disease associated with (causes civil war lol)

- B cells insufficiency = insulin insufficiency

- usually given with insulin therapy but not all insulin stimulating oral drugs

- type 2: body does not produce enough insulin = insuf of insulin

- combination of B cell def + peripheral insulin resistance

- reasons of insulin resistance: 1. change in insulin receptor

2. obesity

---

etiology

-type 1: auto immune disorder


-2: b cell def + peripheral resistance

age of onset

1: younger

2: older

obesity

1: rare

2: common

family history

1: rare

2: common

genetic association

1: common (auto immune disease related to genetics)

2: not common

insulin resistance

1: none

2: yes

presence of body insulin

1: nOpE

2: it has its own insulin but resistant

oral agents

1: not responding

2: kinda yes

----

gamma cells - inh. both alpha and beta secretion for the sake of balance

---
picture:

1. as the glucose enters the GLUT-2 in the B cell pof the pancreas - beta cell take up the
glucose - sugar transformed in ATP through mitochodria- so a small energy carrying hindi ko
gets anong kinarry monosomething? could close K channel - resulting to membrane
DEpolarization - causes massive influx of calcium inside the absorption - which in turn allows
the vesicles to release insulin outside the cell = insulin regulate blood glucose transport yay

incretin effect - a phenomenon wherein an oral glucose elicits higher insulin secretory
responses than IV glucose despite inducing similar levels of glycemia

- 2 gut-derived incretin hormones: glucose-dependent insulinotropic polypeptide (GIP) and


glucagon-like peptide-1 (GLP-1)- so they secreted fr. intestine or ingestine of glucose or
nutrients to stimulate insulin secretion fr beta cells

---

lab and dx tests

-all exc. RP/BG in px with classic hyperglycemic signs need to be repeated and confirmed on a
separate day.

-many acute stresses such as severe infection, burns, trauma can lead transient hyperglycemia
due to secretion of hormones like catecholamines, and cortisol that could oppose the insulin -
the dx of diabetes requires persistence of hyperglycemia colomer of that solution of acute illness
(hindi ko nagets). so they need to be repeated exc. RP/BG bcs it might get false results just bcs
of other factors

-OGTT - oral glucose tolerance test (sa preggy hi pcol)

---

clin manif

type 1

3Ps - polyuria (excessive urination), polydipsia (excessive thirst), polyphagia (excessive hunger)

-not only in pancreas:

1. associated with adipose tissue and skeletal muscle- polyphagia

2. cant store insulin bcs the insulin stimulation is in the liver = ketoacidosis

3. kidneys - polyuria - excessive vol depletion = polydipsia


---

-nephropathy - kidneys affected; diabetes x with kidney problem

-retinopathy - blood vessels of the eyes are being damage

- vegf - Vascular endothelial growth factor - -s a signalling protein that promotes the growth of
new blood vessels

- neuropathy - problem in the nervous system x diabetes

---

insulin prep:

- injected in the belly (adipose tissue) part of the stomach (middle part)

---basa nalang si ma'am


1.The blood clotting mechanism require this metallic ion as a 5. Contraindicated drug in patients with allergic bronchial
co-factor asthma
A. Barium A. Atropine
B. Sodium B. Methacholine
C. Calcium C. D-tubocurarine
D. Magnesium D. Propranolol
E. Potassium E. Phentotamine

Calcium should be present in order to activate Astham is treated as the symptoms arise. For acute
factors IX to IXa, factors X to Xa and factors II to IIa in the asthma, the drug of choice is a B-agonist because of their
intrinsic and extrinsic systems of blood coagulation greater potency and fast-acting property. Bronchodilators
such as theophylline and atropine are also used, therefore B-
blockers is contraindicated
2. The metabolism of this compound/substance leads to gout
A. Calcium
B. Purines 6. Bone marrow depression is most likely caused by this
C. Cholesterol antibiotic
D. Carbohydrate A. Streptomycin
B. Tetracycline
The end product of purine metabolism is uric acid in C. Amphotericin B
gout, there is hyperuricemia, acute or chronic recurrent D. Penicillin G
arthritis and deposits of monosodium urates E. Chloramphenicol

Chloramphenicol can exert three major types of


3. Mottling of teeth in children due to tetracycline therapy adverse effects
can occur between the ages of 1. Irreversible bone marrow aplasia unrelated to dose
A. One month in utero and 3 years 2. Reversible bone marrow suppression associated with
B. Six months in utero and 8 years chloramphenicol serum concentrations consistently
C. Birth and ten years exceeding 25mg/L
D. One and ten years 3. Gray baby syndrome associated with serum
concentrations of 40 to 200mg/L
Neonates and babies prior to the first dentition is of
greatest risk of untoward effect of tetracycline on teeth.
However, pigmentation of the permanent dentition is likely 7. The responses of the drug should be monitored when
to develop if the drug is administered between the ages 2 concurrently administered with propranolol
months and 5 years, during which these tooth are building A. Digoxin
calcium deposits B. Quinidine
C. Chlorothiazide
D. Insulin
4. Acromegaly can be manifested by a/an
A. Crowded teeth Propranolol inhibits the rebound of glucose levels in
B. Large tongue non-diabetic individuals following insulin administration due
C. Hypoglycemia to propranolol-induced suppression of lipolysis. Furthermore,
D. Edema propranolol has been shown to exhibit skeletal muscle
glycogenolysis which may contribute indirectly to
The condition of acromegaly is the over production hypoglycaemia through the continuous utilization of glucose
of growth hormones thus leading to increases size of some
parts of the body
8. Rheumatoid arthritis is associated with the following
except
A. Fatigue
B. Anorexia
C. Morning stiffness
D. Joing swelling and pain
E. Bleeding
The early symptoms of rheumatoid arthritis may be Patients with pre-existing hearing loss, however, may fail to
non-specific and consist of malaise, fatigue, diffuse experience tinnitus despite potentially toxic serum salicylate
musculoskeletal pain and morning stiffness. After sometime, concentration of greater than 40mg/dL
the noon-specific musculoskeletal pain localized to the joints
bilaterally. The hands, wrists and feet are the ones first
involved 12. Drug which is used in Parkinsonism
A. Reserpine
B. Trihexyphenydyl
9. The condition manifested by a patient having a white cell C. Physostigmine
count of 3000 cells per cubic millimetre D. Chlorpromazine
A. Leukemia E. Pyridoxine
B. Leukolysis
C. Leukopenia The anticholinergic drugs of choice for Parkinson’s
D. Leukocytosis disease are the synthetic agents because they act more
selectively than atropine and scopolamine. The most
Leukopenia is a reduction in the number of white commonly used anticholinergics are trihexyphenidyl and
cells; occurs occasionally in viral disease benztropine. The antihistamines, diphenhydramine and
Leukocytosis is an increase in the number of white orphenadrine and the phenothiazine, ethoprozapine, have
cells generally indication an acute infection significant anticholinergic properties and are useful in
Leukemia is a condition when total white cell Parkinsonism
reaches as high as 500,000 per cubic millimetre

13. Hypothyrodism is treated with


10. Which of the following is considered to the drug of choice A. Perchlorate ion
for ringworm infection? B. Liothyronine
A. Nystatin C. Lugol’s solution
B. Neomycin D. Propylthiouracil
C. Amphtorecin B E. I 131
D. Griseofulvin
Liothyronine (T3), levothyroxine, liotrix,
Griseofulvin is usually employed systematically for thyroglobulin and dessicated thyroid are commercially
control of some dermatophytes belonging to the genera available thyroid preparations that can proved and maintain
Epidermophyton, Microsporium and Trichophyton a euthyroid state – the principal goal of thyroid hormone
Nystatin is indicated for the treatment of cutaneous, therapy. The first three preparations are synthetic while the
intestinal, and vaginal infections of Candida latter two are from natural sources
Neomycin, an aminoglycoside, indicated for
infectious diarrhea cause by enteropathogenic E. coli
Amphotericin – intravenous administration for the 14. Normal haemoglobin valued for females
treatment of potentially life-threatening disseminated A. 10-14 g/dL
mycotic infections such as blastomycosis, systemic B. 12-16 g/dL
candidiasis, coccidioidomycosis, cryptococcosis, C. 14-18 g/dL
histoplasmosis and moniliasis D. 16-20 g/dL
E. 18-22 g/dL

11. Salicylate administered for arthritis is decreased due to Hemoglobin is the oxygen carrying compound
this most frequent symptom of salicylate overdosage contained in the red blood cells. The normal values for male
A. Tinnitus is 14.0 to 18.0 g/dL and 11.5 to 15.5 g/dL for females. The
B. Metabolic acidosis total haemoglobin concentration depends primarily upon the
C. Skin rash number of red blood cells in the blood sample, although, it is
D. Fever also slightly influenced by the amount of haemoglobin in
E. Respiratory alkalosis each red cell

Tinnitus can be a useful therapeutic endpoint. In


those without hearing defects because it usually is apparent
when the serum salicylate concentration is about 30mg/dL.
15.Which of the following is not a sign and symptoms of (mean 1%=50 hours) it can be given once a day. Piroxicam is
hypothyroidism? indicated for arthritis or osteoarthritis
A. Forgetfulness
B. Sensitivity to cold
C. Unexplained weight gain 19. Pancuronium does not exert the following action, except:
D. Irritability A. Increase muscle relaxation
E. Lethargy B. Increase duration of activity
C. Enhance rapidity of action
Irritability is a symptom of hyperthyroidism. Along
with nervousness and insomnia, these symptoms are Pancuronium is approximately 5 times as potent as
manifested as an increase in alkaline phosphatise level, an tubocurarine as a competitive neuromuscular blocker. Its
increase in calcium and an increase in AST (aspartate) advantages are:
 Minimal cardiovascular effect
 Little histamine-releasing or hormonal actions
16. The penicillin which is least bound in blood serum Pancuronium was synthesized in 1964
A. Ampicillin
B. Cephalosphorin
C. Penicillin 20. What will happen to serum phenytoin level when the
chlormaphenicol/phenytoin combination is used?
Ampicillin is about 20% bound to plasma proteins A. Increase
B. Decrease
C. No effect
17. Which antibiotic is known to cause a Fanconi-like
syndrome upon ingestion of expired and degraded from this The increase in the amount of phenytoin in serum is
drug? due to the significant inhibition of metabolism of phenytoin
A. Ampicillin caused by chloramphenicol. Half-life of phenytoin in
B. Doxycycline increased 2 to 3 times normal, thus there is a necessity to
C. Tetracycline decrease phenytoin dosage
D. Cephalosphorin
E. Clindamycin
21. Which of the following does not show elevation of plasma
A Fanconi-like syndrome is characterized by: level when given simultaneously with cimetidine?
 Polyuria A. Metoprolol
 Polydypsia B. Propranolol
 Proteinuria C. Atenolol
 Acidosis
 Glycosuria Cimetidine interacts with the following drugs:
 Gross aminoacidosis  Warfarin
These toxic effects are due o the damage brought  Phenytoin
about by expired and degraded tetracycline on the proximal  Theophylline
renal tubule  Phenobarbital
 Benzodiazepines
 Propranolol
18. Piroxicam is used over other NSAIDS because it  Nifedipine
A. Relatively expensive  Digitoxin
B. Acts by a different mechanism that may be additive to  Quinidine
other NSAIDS  Mexiletine
C. May be given on a once a day schedule  TCA eg imipramine
D. Can be taken with meals Because cimetidine shows the metabolism by
E. Has essentially no gastrointestinal effects inhibiting cytochrom P-450 and this prolong drug half-life

Piroxicam, an oxicam derivative, belongs to a class


of enolic acids that has anti-inflammatory, analgesic and
antipyretic activities. Because piroxicam has a long half-life
22. Prednisone is a/an 26. Drug most likely to cause a lupus-like syndrome?
A. Adrenergic stimulant A. Captopril
B. Anti-inflammatory drug B. Hydralazine
C. Bronchodilator C. Propranolol
D. Minoxidil
The corticosteroids like prednisone inhibit both the
early manifestations of the inflammatory process: The lupus-like syndrome associated with
 Edema hydralazine occurs in less than 10% of patients with the
 Fibrin disposition following symptoms
 Migraton of leukocytes into the inflamed area  Myalgia
 Phagocytic activity and the later manifestations  Arthralgia
 Proliferation of capillaries and fibroblasts  Fever
 Deposition of collagen  Antinuclear antibodies
 Cicatrization  Rash
 Lymphadenopathy
 Chest pain
23. Patients taking which of the following drugs should their  Asthenia
CBC be monitored regularly?  Hepatosplenomegaly
A. Phenytoin Patients develop this syndrome usually after more
B. Bisacodyl than 2 months of therapy, slow acetylators of hydralazine,
C. Acetylcysteine during summer season and dosage of more than 200mg per
D. Isoproterenol day
E. Minocycline

Phenytoin has been implicated to cause aplastic 27. The diuretic of choice for poisoning in order to induce
anemia due to alteration in folate metabolism. Other diuresis
haematological reactions include neutropenia and leukemia A. HCTZ
B. Triamterene
C. Chlorthalidone
24. This laxative is recommended when chronic use is needed D. Mannitol
A. Bulk-forming
B. Stimulant Mannitol prevents the accumulation of high
C. Saline concentration of the toxic agent within the tubular fluid by
D. Lubricant exerting an osmotic effect within the tubular fluid, inhibiting
the reabsorption of water and maintaining the rate of urine
Examples of bulk laxatives flow
 Hydrophilic colloids – form gels within the large
intestine, distending the intestine and stimulating
its peristaltic activity 28. The vitamin used for hyperlipidemia
 Saline cathartics (magnesium hydroxide and A. Ascorbic acid
magnesium citrate) – also distend the bowel and B. Thiamine
stimulate its contraction C. Niacin
D. Cyanocobalamin
E. Pyridoxine
25. Which is not a pro-drug?
A. Bacampicillin Niacin is a rational choice for initial drug therapy for
B. Methampicillin hypercholesterolemic and hypertriglyceridemic patient at
C. Methicillin the same time because this drug can lower both cholesterol
D. None and triglyceride, but a better drug of choice would be
gemfibrozil
Bacampicillin is the 1-ethoxy-carbonyloxyethyl ester
of ampicilin. This inactive form is well absorbed after oral
administration and becomes active when it is hydrolyzed to
ampicillin during absorption from the GIT
29. Which anti-neoplastic causes congestive heart failure intermediate between those of regular insulin and protamine
(CHF)? zinc insulin suspension
A. Doxorubicin
B. Tamoxifen
C. Methotrexate 33. This drug characteristically produce throbbing headache
D. Vincristine A. Procainamide
E. Cisplatin B. Propranolol
C. Verapamil
There are two ways by when CHF induced by long- D. Glyceryltrinitrate
term doxorubicin therapy is manifested E. Nifedipine
 Acute, transient and usually benign
electrocardiographic changes Headache is a common and expected side effect of
 Chronic, cumulative, dose-dependent and the nitrates and patients usually develop tolerance to this
potentially fatal cardiomyopathy effect in several weeks of beginning therapy. Mild analgesics
help correct this inconvenience. Occasional patients
experience continuing headache and discontinue their
30. Penicillin with the best gram-negative spectrum therapy
A. Nafcillin
B. Methicillin
C. Penicillin V 34. Hyperkinesis in children is treated with which of the
D. Phenethicillin following mild CNS stimulant?
E. Ampicillin A. Caffeine
B. Doxapram
Ampicillin and the related aminopenicillins are C. Methylphenidate
bactericidal for both gram-positive and gram-negative D. Theophylline
bacteria. They are somewhat less active than pen G against
gram-positive cocci but sensitive to the latter Methylphenidate, dextroamphetamine and
microorganisms. The meningococcus, pneumococcus, pemoline are all effective in the management of hyperactive
gonococcus and L. monocytogenes are sensitive to the drug children. The first two are both available in standard and
sustained-released preparations

31. Cyclophosphamide is a/an


A. Alkylating agent 35. This drug would have an equal effect on both early
B. Antimetabolite asthmatic and late asthmatic response
C. Natural product A. Corticosteroids
B. Albuterol
Cyclophosphamide is a modified nitrogen mustard. C. Theophylline
Although itself is inactive, it undergoes metabolism in the D. Sodium cromolyn
liver where it changes to the active form. The cleavage of the
phosphamide ring is postulated as the step that produces the Cromolyn sodium (a furanochromone) is thought to
active drug within the tumor cells. The drug is useful clinically stabilize mast cell membranes and prevent the release of
although non-selective activity is observed chemical mediators of inflammation. It is therefore useful
since mast cell mediator and important role in the
pathogenesis of bronchial hyperactivity
32. Isophane insulin suspension is a
A. Fast acting insulin
B. Intermediate-acting insulin 36. Tamoxifen is indicated for the treatment of
C. Long activity insulin A. Hodgkin’s disease
B. Bladder cancer
Isophane insulin suspension is also known as NPH C. Breast cancer
insulin: N for neutral solution, P for protamine zinc insulin D. Colorectal cancer
content, H for its origin in Hagedorn’s laboratory. The
concentrations of insulin, protamine and zinc are so arrange Tamoxifen is generally considered first-line
that the preparation has an onset and a duration of action endocrine therapy because of its relatively low toxicity and
widespread experience with the use in breast cancer
37. Herpes simplex infection of the eyelids and conjunctiva is 40. Long term therapy is not recommended for
treated best by phenylbutazone since it has potential for producing
A. Thiabendazole A. Blood dyscrasias
B. Idoxuridine B. Damage to the optic nerve
C. Amphotericin C. Damage to the auditory nerve
D. Carbenicillin D. AOTA

Herpes simplex virus of the eye and conjunctiva Phenylbutazone and its analog, oxyphenbutazone,
causes corneal blindness. There are three approved drugs are well known to cause aplastic anemia and the risk when
available to manage this condition the drug is taken regularly and for a prolong period
 Trifluridine
 Idoxuridine
 Vidarabine 41. Drug of choice in the treatment of status epilepticus
Trifluridine is considered the drug of choice A. Phenobarbital
B. Amobarbital
C. Phenytoin
38. One of the following is a white blood cell except D. Paraldehyde
A Basophil E. Diazepam
B. Eosinophil
C. Monocyte Diazepam – 10mg is extremely effective for rapid
D. Reticulocyte but temporary, termination of seizure activity in status
E. Lymphocyte epilepticus. This is because high serum/CNS concentrations
is achieved rapidly
Reticulocyte is a young erythrocyte that is not yet Phenobarbital may be useful for the treatment of
completely mature status epilepticus in patients who cannot tolerate phenytoin
Basophil – an increase of which accompanies or in whom phenytoin has previously proven effective
chronic myeloid leukemia, myelofibrosis and polycythemia Phenytoin is considered the long-acting anti-
vera convulsant of choice for most patients with tonic-clonic
Eosinophil - a granular leukocyte that have surface status epilepticus
receptors that bing IgG and IgE
Monocyte – a non-granular WBC acting as
phagocytes, becoming transformed into macrophages after 42. Guanethidine’s antihypertensive activity is inhibited by
invading infected sites, where their number reach a peak in A. Diazepam
48 hours B. Amitryptyline
Lymphocyte – a non-granular leukocyte important C. HCTZ
in the process of immunity, producing antibodies and other D. Probenecid
agents involved in the immune process E. Nitrofurantoin

Desipramine and related compounds (amitryptyline


39. A tricyclic amine used to treat viral infections and imipramine) inhibit the uptake of guanathidine into the
A. Desipramine neuron terminal, thus preventing its accumulation from
B. Amantadine these sites
C. 5-fluouracil
D. Cytosine arabinoside
E. Idoxuridine 43. Emergency cases of CHF is treated by
A. Digitalis glycoside
Amantadine prevents influenza A viruses to B. Bed rest
penetrate the host cell by preventing the uncoating of the C. Inotropic agents
virus once it is attached to the cell membrane D. Vasodilators
E. Diuretics

A large number of inotropic agents are being


investigated for the treatment of CHF. The bipyridine
derivatives, amrinone and milrinone, are widely being used
and studied. These drugs selectively inhibit phosphodiesteras 47. All are characteristics of Parkinson’s disease except
F-III, the cyclic AMP specific cardiac phosphodiesterase A. Muscle rigidity
In patients with severe heart failure or cardiogenic B. Ambivalence
shock, digitalis is not the initial drug choice because of the C. Bradykinesia
delay in its time to peak action and its possible deleterious D. Tremor
effect of the peri-infarction area E. Postural instability

Tremor and rigidity is manifested early in the course


44. The mainstay for the treatment of Parkinsonism of the disease when dopamine depletion is not substantial.
A. Dopamine precursor Bradykinesia and postural instability appear later in the
B. Anticholinergic course of the disease and are the most disabling symptoms
C. Direct acting dopamine agonist
D. Indirect acting dopamine agonist
E. Physical therapy 48. In schizophrenia, there is primarily
A. Increased cholinergic activity
Anticholinergic block the excitatory B. Decreased cholinergic activity
neurotransmitter acetylcholine in the substantia nigra. These C. Decreased dopaminergic activity
agents improve tremor and rigidity more than bradykinesia, D. Increased dopaminergic activity
postural imbalance or gait disturbance
Commonly encountered symptoms of schizophrenia
include Bleuter’s 4 A’s
45. The following drugs are useful in the treatment of cardiac  Autism – preoccupation with internal stimuli
failure except  inappropriate effect – external manifestations of
A. Digoxin mood
B. Esmolol  Associtational disturbances - illogical or
C. Dopamine idiosyncratic thought processes
D. Amrinone  Ambivalence – Simultaneous, contradictory thinking
E. Dobutamine

Digitalis (digoxin, digitoxin0 increases the force of 49. Factor most directly associated to angina attacks
contraction of both the normal and abnormal hear (positive A. Stress
inotropic effect), and decreases the conduction of velocity B. Myocardial ischemia
and prolong the refractory period of AV mode C. Heavy smoking
Dopamine and dobutamine (inotropic agents) are D. Obesity
frequently used in acute cardiac emergencies, but their used
is limited by the need for IV administration In men, not in women, a positive family history of
Amrinone, a non symphatomimeetic inotrope, is heart attack is independently predictive of death from all
associated with an unacceptably high incidence of side causes and from cardiovascular and ischemic heart attacks.
effects when given by mouth, but parenteral form Hypertension, obesity, hypercholesterolemia, smoking and
stress are risk factors which can be altered

46. One of these is not useful in the treatment of cough


A. Noscapine 50. In unipolar depression
B. KI A. Lithium is more effective that tricyclic anti-depressants
C. Syrup of ipecac B. Tricyclic antidepressants are more effective than lithium
D. Dextromethorphan C. Neither lithium nor tricyclic antidepressants are effective
E. Iodoquinol D. Lithium and tricyclic antidepressants are equally effective

Iodoquinol, or diiodohydroxyquin, a halogenated Tricyclic antidepressants are used in unipolar


hydroxyquinoline, is one of the earlist drugs used for depression while lithium for bipolar depression
amoebiasis
51.What condition of the liver characterized by the presence 55. Patients should not take this drug with streptomycin
of nodule and fibrous tissue? A. Acetazolamide
A. Hepatitis B. Ethacrynic acid
B. Cirrhosis of the liver C. Triamterene
C. AOTA D. Chlorthalidone
E. Mannitol
Cirrhosis is a chronic disease of the liver where there
is widespread hepatic parenchymal cell destruction. There is Drugs that are ototoxic by itself can increase its
the formation of connective tissue and nodular regeneration. ototoxic effect by concurrent administration, especially in
The liver nodules change the anatomical location and renally compromised patients. Aminoglycosides (kanamycin,
function of the blood vessels and bile ducts, causing gentamicin, neomycin, streptomycin and tobramycin) and
obstruction and resulting in portal hypertension ascites, ethacrynic acid and is such a combination. Permanent or
jaundice and esophageal varices transient hearing loss results, thus, the concurrent use of
these drugs should be avoided

52. Asthma is treated with the anticholinergic drug


A. Ipratropium bromide 56. What disease is characterized by a wide variation over a
B. Benztropine short period of time in resistance to flow in intrapulmonary
C. Biperiden airways?
A. Chronic bronchitis
The anticholinergics, atropine, ipratropium B. Emphysema
bromide, glycopyrrolate, are used alone or in combination C. Asthma
with traditional bronchodilators in relieving asthma. These
are administered by metered-dose inhaler or nebulisation Chronic bronchitis is an inflammation of the
tracheobronchial tree in which cough and overproduction of
sputum are the prominent features
53. Ranitidine works effectively in the treatment of duodenal Pulmonary emphysema is a condition of the lungs
ulcers because it where an abnormal, permanent enlargement of airspaces
A. Strengthens the protective coating of intestinal wall distal to the terminal bronchiole is present
B. Is an agonist at H1 receptors and therefore reduces pepsin
secretion
C. Blocks histamine stimulation of gastric acid secretion by 57. This is not an alkylating anticancer agent
B2 blocker A. Busulfan
B. Chlorambucil
H2 antagonista are exemplified by cimetidine, C. Cisplatin
raniditine, famotidine and nizatidine. Of all the four, D. Vincristine
cimetidine is associated with more side effects and drug
interactions. Thus the use of ranitidine is recommended Vincristine acts by reversible inhibition of mitosis. It
binds to microtubule protein, tubulin, ultimately inhibiting
formation of mitotic spindles
54. The condition where there is an inflamed inner
membranous living of the heart caused by bacteria and fungi
A. Infective endocarditis 58. Proloid is indicated in
B. Angina pectoris A. Plummer’s disease
C. Congestive heart failure B. Myxedema
C.Graves’s disease
Infective endocarditis is a microbial infection of the
heart valves or other endocardial tissue. Its pathogenesis Proloid or thyroglobulin is a purified extract of hog,
involves a complex series of events which ends in the standardized biologically to give a T4-T3 ratio of 2.5:1. It is
formation of an infected platelet-fibrin thrombus on the used for myxedema coma which is the end stage of long-
valve surface standing, uncorrected hypothyroidism
Graves’s disease (toxic diffuse goiter) and
Plummer’s disease (toxic uninodular goiter) are
abnormalities that can cause hyperthyroidism, the former
being the most common
59. Methotrexate is an antineoplastic drug and is used in Lithium toxicity is reported as a consequence of the
A. Hyperuricemia presence of NSAIDs that increase renal tubular reabsorption
B. Arthritis of sodium by inhibition of prostaglandin E
C. Goiter

In the treatment of rheumatoid arthritis, 64. Pseudomembraneous colitis is best treated by


methotrexate may be given either parenterally or orally. A. Cephalexin
Approximately 57% of a low dose methotrexate is absorbed B. Erythromycin
C. Vancomycin

60. Which of the following reduces the effectiveness of oral The anticlostridal antibiotics are used for severe
contraceptives? cases of pseudomembraneous colitis (PMC). An example of
A. Rifampicin which is vancomycin (125-500mg orally 4x a day) for 7 days to
B. Phenytoin 10 days is widely used a first line therapy
C. Barbiturates
D. AOTA
65. Which is not used in mania?
Rifampicin significantly induces liver enzymes and A. Carbamazepine
increase metabolism of corticosteroids, oral contraceptives, B. Lithium
quinidine, diazepam, ketoconazole, propranolol, metoprolol, C. Acetazolamide
sulfonylureas and warfarin
Carbamazepine, a dibenzazepine tricyclic
compound which is now being used increasingly in the
61. One of the following is not a sedative-antihistamine treatment and prophylaxis of refractory bipolar disorders of
A. Astemizole acute mania or depression
B. Chlorpheniramine Litium therapy is used in a patient with a prior
C. Promethazine history of mood disorder in the patient himself or in the
D. NOTA family or if there is a continuous manic behaviour

Terfenadine and astemizole


 Penetrate the CNS poorly
 Have a decreased affinity for CNS H1 receptors
compared to classical H1 antagonists
 Lack of anticholinergic activity

62. Ferrous sulfate can make doxycycline ____________.


A. Lower the serum concentration
B. Shorten its serum half-life
C. Both
D. NOTA

Ferrous sulfate given orally (200-600mg) inhibits the


absorption of tetracycline from the GIT, thus resulting in a
decreased serum tetracycline level. The converse is true for
this interaction. When the concurrent administration of these
2 drugs is necessary, the antibiotic should be taken 3 hours
after or 2 hours before iron administration

63. Lithium salts should be__________ when diclofenac is


being used
A. Increased
B. Decreased
C. Dose retained
PRACTICE SET E. Panthothenic acid

1. All of the following therapies are used for metastatic 8. Prominent characteristic of angina pectoris
prostate cancer, except A. Diuresis
A. Estrogen B. Abnormal pulse rate
B. Leuprolide C. Squeezing chest pressure
C. Orchrectomy D. Skin pallor
D. Corticosteroids E. Weakness
E. Somatostatin analogues
9. Which analgesics is useful in patients with myocardial
2. Differential white blood cell counts in the laboratory are infaction?
useful in the diagnosis of A. Mefenamic acid
A. Anemia B. Acetaminophen
B. Sphetocytosis C. Morphine
C. Vitamin deficiency D. Aspirin
D. Eosinophilia
E. Thrombocytopenia 10. Which is not a clinical manifestation of gastroenteritis?
A. Fever
3. Which of the following barbiturates in the therapeutic B. Vomiting
doses is the longest acting? C. Abdominal pain
A. Amytal D. Night sweats
B. Nembutal
C. Secobarbital 11. All of the following are complication of untreated
D. Phenobarbital diabetes mellitus except
E. Pentothal A. Gangrene
B. Increased perception of pain and temperature
4. Which of the following classes of drugs undergoes the C. Atherosclerosis
least change in the body? D. Hypertension
A. Alcohols
B. Salicylates 12. Which of the following is not a diabetic complication?
C. Local anesthetics A. Retinopathy
D. Volatile anesthetics B. Cardiovascular complication
C. Nephropathy
5. Inflammation of the bladder D. Anemia
A. Urethritis
B. Cystitis 13. Phenobarbital is used for viral hepatitis to
C. Prostatitis A. Promote fatty acid synthesis
D. Pyelonephritis B. Activate the enzyme prolyl hydroxylase
E. Urinitis C. Induce the production of glucoronyl transferase

6. Overdosage with certain steroids leads to the 14. Shaking palsy


manifestations of Cushing’s syndrome. Which of the A. Frequent urination
following steroids produces most of the above effects? B. Stooped pressure
A. Estradiol C. Painful inflammation of the joint
B. Prednisolone
C. Diethylstilbestrol 15. Compounds whose absorption may be reduced by food
D. Testosterone A. Cefalexin
E. Progesterone B. Tetracycline
C. Isoniazid
7. A child shows evidence of poor growth, night blindness, D. Metronidazole
dryness of the conjunctiva, keratinisation of the cornea and a
dry scaly skin. Which preparation is indicated for therapy? 16. Compound whose absorption may be increased by food
A. Calciferol A. Griseofulvin
B. Dried yeast B. Nitrofurantoin
C. Vitamin A C. Rifampicin
D. Thiamine D. NOTA
17. Enhances the effect of barbiturates 26. The reversal of anticoagulant-induced
A. Alcohol hypoprothrombinemia is most rapidly accomplished by the
B. MAOI’s administration of
C. Pethidine A. Phytonadione
D. AOTA B. Menadione
C. Menadione Na bisulfite
18. A cardioselective beta blocker D. Fresh blood or plasma
A. Alprenolol
B. Betaxolol 27. The clinical investigation of a new drug consists of 4
C. Metoprolol phases. Phase 1 of the clinical testing involves
D. AOTA A. To animals to determine the side effects of the drug
B. To animals for toxicity studies
19. The following are calcium antagonists except C. By select clinicians to healthy volunteers
A. Nifedipine D. To select clinicians to patients suffering from the diseases
B. Felodipine
C. Nicardipine 28. Which of the following phrases best defined the clinical
D. NOTA disorder known as hemochromatosis?
A. Lack of circulating antibodies
20. Affected by food in the stomach so should be given with B. Excessive storage of iron by the body
food one hour before meals C. Abnormally shaped blood cells
A. Captopril D. Diminished circulating blood volume
B. Enalapril
C. Cilazapril 29. A white cell differential count is a laboratory procedure
that
21. Comes only as an oral preparation A. Determines the relative proportions of the various white
A. Ranitidine blood cells
B. Famotidine B. Determines the relative proportion of white cells to red
C. Nizatidine cells
D. Cimetidine C. Differentiates between iron-deficient and folic acid
deficient anemia
22. Treatment of acute diarrhea includes D. Differentiates immature from mature white blood cells
A. Stool binding agents
B. Anti-vomiting drugs 30. Patients on lithium carbonate therapy should be advised
C. Oral rehydration salts A. To drink plenty of water to avoid crystalluria
B. To stop taking the drug if they experience mild side effects
23. The following drugs lose some of their potency due to C. Not to restrict then normal dietary salt intake
sorption to plastic materials except D. Not to take the drug during their manic phase of the cycle
A. Diazepam E. Not to take the drug with food
B. Chloroquine
C. Isosorbide dinitrate 31. The intentional administration of intravenous fluid into
D. NOTA subcutaneous tissue is called
A. Infiltration
24. The primary mechanism of ibuprofen B. Venoclysis
A. Inhibits prostaglandin synthesis C. Hypodermoclysis
B. Stimulates platelet aggregation D. Hemodialysis
C. AOTA E. Hemolysis
D. NOTA
32. Kernicterus is a drug-induced disorder that may occur in
25. Diazoxide is administered the neonate following therapy with which of the following
A. Orally drugs?
B. IM A. Isoniazid
C. By rapid IV injection B. Sulfisoxazole
D. By IV infusion C. Phenytoin
D. Gentamicin
E. Promethazine
33. Which of the following enzymes would be elevated in 41. To be recorded in the recorded book except
acute pancreatitis? A. Pethidine
A. Alkaline phosphatise B. Fentanyl
B. Lactic dehydrogenase C. Codeine phosphate
C. Creatinine phosphokinase D. Obimin
D. Amylase E. Bromazepam

34. Fatigue, lethargy, and weakness are caused by the loss of 42. The following are true except
which ion? A. PT and C should be composed of at least 5 physicians
A. Sodium B. The pharmacists acts as the secretary of the committee
B. Lithium C. A mandatory committee under the conditions of
C. Calcium participation of the hospital insurance program
D. Potassium
43. IV admixture
35. All of the factors listed below may play a role in the A. Adding an additive to an IV fluid
development of duodenal ulcer disease, except: B. Incorporating an antibiotic into an IV drop
A. Spicy food ingestion C. Both A and B
B. Tobacco smoking D. Hyperalimentation
C. Genetic factors
D. Helicobacter pylori infection 44. TPN is administered through
A. Subclavian vein
36. What type of leukocyte appears first and is present in B. Femoral artery
greatest numbers in an inflammatory focus soon after tissue C. Nose
injury has occurred?
A. Plasma cell 45. Medication orders (chart orders) differ from prescriptions
B. Eosinophil in that they
C. Neutrophil A. Are intended for ambulatory use
D. Lymphocyte B. Contain only the generic name of the drug
C. May contain non-medication instructions from the
37. Most effective in acute treatment of migraine headache practitioner
A. Propranolol D. Contain quantity of medication to be dispensed
B. Methylsergide
C. Ergotamine sulfate 46. A package that contains one discrete pharmaceutical
dosage form
38. When used to treat susceptible urinary tract infection, A. Single unit package
which of the following anti-infective drugs will have its B. Unit dose dispensing system
activity increased in an alkaline urine? C. Drug distribution system
A. Methenamine D. NOTA
B. Nitrofurantoin
C. Tetracycline 47. A drug distribution system in which all doses are in unit
D. Gentamicin dose form at the time they are dispensed
A. Unit dose
39. The metabolism of which of the following compounds is B. Unit dose package
altered in patients taking anticonvulsants? C. Unit dose dispensing system
A. Pyridoxine D. AOTA
B. Folic acid
C. Riboflavin 48. The advantages of the individual prescription order
D. Tyrosine system of drug distribution
A. Increased potential for medication error
40. To be recorded in the additional opium book, except B. Increased drug inventory
A. Pentothal sodium C. Delay in drug administration
B. Nalbuphine HCl D. AOTA
C. Codeine phosphate, phenitamine maleate, guiafenesin E. NOTA
D. Diazepam
49. A standard dosing schedule of “q6h” dictates that doses ANSWER KEY
be given at
A. 10 AM and 10 PM 1. E 26. D
B. 9 AM, 1PM, 5PM 2. D 27. C
C. 9AM, 3PM, 9PM, 3AM 3. D 28. B
D. 6AM, 12NN, 6PM 4. D 29. A
5. B 30. A
50. A sign at patients door stating “NPO” means 6. B 31. C
A. Patient should be left undisturbed 7. C 32. B
B. Patient should not be given anything by mouth 8. B 33. D
C. Patient should be given drugs before meals 9. C 34. D
D. Patients should be given drugs after meals 10. D 35. A
11. B 36. C
12. D 37. C
13. C 38. D
14. B 39. B
15. B 40. A
16. A 41. E
17. D 42. A
18. D 43. C
19. D 44. A
20. A 45. C
21. C 46. A
22. C 47. C
23. B 48. C
24. A 49. C
25. C 50. B

You might also like